Surgery

Màu nền
Font chữ
Font size
Chiều cao dòng

Surgery

PreTest® Self-Assessment and Review

PRE TEST®

NOTICE

Medicine is an ever-changing science. As new research and clinical experience

broaden our knowledge, changes in treatment and drug therapy are required. The

authors and the publisher of this work have checked with sources believed to be

reliable in their efforts to provide information that is complete and generally in

accord with the standards accepted at the time of publication. However, in view of

the possibility of human error or changes in medical sciences, neither the authors

nor the publisher nor any other party who has been involved in the preparation or

publication of this work warrants that the information contained herein is in every

respect accurate or complete, and they disclaim all responsibility for any errors or

omissions or for the results obtained from use of the information contained in this

work. Readers are encouraged to confirm the information contained herein with

other sources. For example and in particular, readers are advised to check the product

information sheet included in the package of each drug they plan to administer

to be certain that the information contained in this work is accurate and that

changes have not been made in the recommended dose or in the contraindications

for administration. This recommendation is of particular importance in connection

with new or infrequently used drugs.

McGraw-Hill

Medical Publishing Division

PreTest® Series

NEW YORK ST. LOUIS SAN FRANCISCO AUCKLAND

BOGATÁ CARACAS LISBON LONDON MADRID

MEXICO CITY MILAN MONTREAL NEW DELHI

SAN JUAN SINGAPORE SYDNEY TOKYO TORONTO

PRE TEST®

Surgery

PreTest® Self-Assessment and Review

Ninth Edition

PETER L.GELLER, M.D.

Associate Professor of Clinical Surgery

Columbia University College of Physicians & Surgeons

New York, New York

Copyright © 2001, 1998, 1995, 1992, 1989, 1987, 1985, 1982, 1978 by The McGraw-Hill Companies,

Inc. All rights reserved. Manufactured in the United States of America. Except as permitted under the

United States Copyright Act of 1976, no part of this publication may be reproduced or distributed in any

form or by any means, or stored in a database or retrieval system, without the prior written permission

of the publisher.

0-07-137638-0

The material in this eBook also appears in the print version of this title: 0-07-135954-0.

All trademarks are trademarks of their respective owners. Rather than put a trademark symbol after

every occurrence of a trademarked name, we use names in an editorial fashion only, and to the benefit

of the trademark owner, with no intention of infringement of the trademark. Where such designations

appear in this book, they have been printed with initial caps.

McGraw-Hill eBooks are available at special quantity discounts to use as premiums and sales promotions,

or for use in corporate training programs. For more information, please contact George

Hoare, Special Sales, at [email protected] or (212) 904-4069.

TERMS OF USE

This is a copyrighted work and The McGraw-Hill Companies, Inc. ("McGraw-Hill") and its licensors

reserve all rights in and to the work. Use of this work is subject to these terms. Except as permitted

under the Copyright Act of 1976 and the right to store and retrieve one copy of the work, you may not

decompile, disassemble, reverse engineer, reproduce, modify, create derivative works based upon,

transmit, distribute, disseminate, sell, publish or sublicense the work or any part of it without

McGraw-Hill's prior consent. You may use the work for your own noncommercial and personal use;

any other use of the work is strictly prohibited. Your right to use the work may be terminated if you

fail to comply with these terms.

THE WORK IS PROVIDED "AS IS". McGRAW-HILL AND ITS LICENSORS MAKE NO GUARANTEES

OR WARRANTIES AS TO THE ACCURACY, ADEQUACY OR COMPLETENESS OF

OR RESULTS TO BE OBTAINED FROM USING THE WORK, INCLUDING ANY INFORMATION

THAT CAN BE ACCESSED THROUGH THE WORK VIA HYPERLINK OR OTHERWISE,

AND EXPRESSLY DISCLAIM ANY WARRANTY, EXPRESS OR IMPLIED, INCLUDING BUT

NOT LIMITED TO IMPLIED WARRANTIES OF MERCHANTABILITY OR FITNESS FOR A

PARTICULAR PURPOSE. McGraw-Hill and its licensors do not warrant or guarantee that the functions

contained in the work will meet your requirements or that its operation will be uninterrupted or

error free. Neither McGraw-Hill nor its licensors shall be liable to you or anyone else for any inaccuracy,

error or omission, regardless of cause, in the work or for any damages resulting therefrom.

McGraw-Hill has no responsibility for the content of any information accessed through the work.

Under no circumstances shall McGraw-Hill and/or its licensors be liable for any indirect, incidental,

special, punitive, consequential or similar damages that result from the use of or inability to use the

work, even if any of them has been advised of the possibility of such damages. This limitation of liability

shall apply to any claim or cause whatsoever whether such claim or cause arises in contract, tort

or otherwise.

DOI: 10.1036/0071376380

Terms of Use

v

CONTENTS

Preface . . . . . . . . . . . . . . . . . . . . . . . . . . . . . . . . . . . . . . . . . . . . . . . vii

Introduction . . . . . . . . . . . . . . . . . . . . . . . . . . . . . . . . . . . . . . . . . . . . ix

PRE- AND POSTOPERATIVE CARE

Questions . . . . . . . . . . . . . . . . . . . . . . . . . . . . . . . . . . . . . . . . . . . . . . 1

Answers, Explanations, and References . . . . . . . . . . . . . . . . . . . . . . 16

CRITICAL CARE:ANESTHESIOLOGY, BLOOD GASES,

RESPIRATORY CARE

Questions . . . . . . . . . . . . . . . . . . . . . . . . . . . . . . . . . . . . . . . . . . . . . 33

Answers, Explanations, and References . . . . . . . . . . . . . . . . . . . . . . 47

SKIN:WOUNDS, INFECTIONS, BURNS;HANDS;

PLASTIC SURGERY

Questions . . . . . . . . . . . . . . . . . . . . . . . . . . . . . . . . . . . . . . . . . . . . . 67

Answers, Explanations, and References . . . . . . . . . . . . . . . . . . . . . . 73

TRAUMA AND SHOCK

Questions . . . . . . . . . . . . . . . . . . . . . . . . . . . . . . . . . . . . . . . . . . . . . 83

Answers, Explanations, and References . . . . . . . . . . . . . . . . . . . . . . 98

TRANSPLANTS, IMMUNOLOGY,AND ONCOLOGY

Questions . . . . . . . . . . . . . . . . . . . . . . . . . . . . . . . . . . . . . . . . . . . . 121

Answers, Explanations, and References . . . . . . . . . . . . . . . . . . . . . 135

ENDOCRINE PROBLEMS AND BREAST

Questions . . . . . . . . . . . . . . . . . . . . . . . . . . . . . . . . . . . . . . . . . . . . 153

Answers, Explanations, and References . . . . . . . . . . . . . . . . . . . . . 165

GASTROINTESTINALTRACT, LIVER,AND PANCREAS

Questions . . . . . . . . . . . . . . . . . . . . . . . . . . . . . . . . . . . . . . . . . . . . 181

Answers, Explanations, and References . . . . . . . . . . . . . . . . . . . . . 208

Terms of Use

CARDIOTHORACIC PROBLEMS

Questions . . . . . . . . . . . . . . . . . . . . . . . . . . . . . . . . . . . . . . . . . . . . 239

Answers, Explanations, and References . . . . . . . . . . . . . . . . . . . . . 252

PERIPHERALVASCULAR PROBLEMS

Questions . . . . . . . . . . . . . . . . . . . . . . . . . . . . . . . . . . . . . . . . . . . . 265

Answers, Explanations, and References . . . . . . . . . . . . . . . . . . . . . 277

UROLOGY

Questions . . . . . . . . . . . . . . . . . . . . . . . . . . . . . . . . . . . . . . . . . . . . 287

Answers, Explanations, and References . . . . . . . . . . . . . . . . . . . . . 291

ORTHOPEDICS

Questions . . . . . . . . . . . . . . . . . . . . . . . . . . . . . . . . . . . . . . . . . . . . 297

Answers, Explanations, and References . . . . . . . . . . . . . . . . . . . . . 301

NEUROSURGERY

Questions . . . . . . . . . . . . . . . . . . . . . . . . . . . . . . . . . . . . . . . . . . . . 307

Answers, Explanations, and References . . . . . . . . . . . . . . . . . . . . . 312

OTOLARYNGOLOGY

Questions . . . . . . . . . . . . . . . . . . . . . . . . . . . . . . . . . . . . . . . . . . . . 319

Answers, Explanations, and References . . . . . . . . . . . . . . . . . . . . . 322

Bibliography . . . . . . . . . . . . . . . . . . . . . . . . . . . . . . . . . . . . . . . . . . 325

vi Contents

Terms of Use

PREFACE

No longer can students assume that this kind of continuing education ends

with the completion of formal training and the successful completion of

licensing or certifying examinations. As of October 1979, all 22 member

boards of the American Board of Medical Specialties committed themselves

to the principle of periodic recertification of their members. Despite the

Board's recognition that the cognitive skills measured in the objective

examination do not assure clinical competence, recertification efforts-

insofar as they involve examinations-are based on the assumption that

knowledge of current information on which good clinical decisions should

be made is worth cultivating; that, while such information does not guarantee

competent practice, lack of it probably impedes competent practice,

that this knowledge, unlike technical skills, is reasonably easy to assess;

and that it can be acquired by well-motivated physicians. These assumptions

all seem reasonable.

The questions presented in this book deal with issues of relative

importance to medical students; other problem-oriented materials are

becoming available that are aimed at more sophisticated audiences-

groups that, within a very few years, will include the present generation of

students. Regular review of such material is a habit worth developing. We

hope that this edition of Surgery: PreTest® Self-Assessment and Review will

justify your efforts in working through the problems by providing guidance

for further study and by helping you to develop enduring learning habits.

PETER L.GELLER, M.D.

vii

INTRODUCTION

Each question in Surgery: PreTest® Self-Assessment and Review, Ninth Edition,

is accompanied by an answer, a paragraph explanation, and a specific page

reference to either a current journal article, a textbook, or both. A bibliography,

which lists all the sources used in the book, follows the last chapter.

Perhaps the most effective way to use this book is to allow yourself one

minute to answer each question in a given chapter; as you proceed, indicate

your answer beside each question. By following this suggestion, you

will be approximating the time limits imposed by the board examinations.

When you have finished answering the questions in a chapter, you

should then spend as much time as you need verifying your answers and

carefully reading the explanations. Although you should pay special attention

to the explanations for the questions you answered incorrectly, you

should read every explanation. The authors of this book have designed the

explanations to reinforce and supplement the information tested by the

questions. If, after reading the explanations for a given chapter, you feel

you need still more information about the material covered, you should

consult and study the references indicated.

STUDENT REVIEWER

Jeffrey J. Anderegg

The University of Iowa College of Medicine

Iowa City, Iowa

viii

PRE- AND POSTOPERATIVE

CARE

Questions

DIRECTIONS: Each item below contains a question or incomplete

statement followed by suggested responses. Select the one best response to

each question.

1

1. A pregnant woman in her 32nd

wk of gestation is given magnesium

sulfate for pre-eclampsia. The earliest

clinical indication of hypermagnesemia

a. Loss of deep tendon reflexes

b. Flaccid paralysis

c. Respiratory arrest

d. Hypotension

e. Stupor

2. Five days after an uneventful

cholecystectomy, an asymptomatic

middle-aged woman is found to

have a serum sodium level of 120

meq/L. Proper management would

a. Administration of hypertonic saline

solution

b. Restriction of free water

c. Plasma ultrafiltration

d. Hemodialysis

e. Aggressive diuresis with furosemide

3. A 50-year-old patient presents

with symptomatic nephrolithiasis.

He reports that he underwent a

jejunoileal bypass for morbid obesity

when he was 39. One would

expect to find

a. Pseudohyperparathyroidism

b. Hyperuric aciduria

c. "Hungry bone" syndrome

d. Hyperoxaluria

e. Sporadic unicameral bone cysts

4. Following surgery, a patient

develops oliguria. You believe the

patient is hypovolemic, but before

increasing intravenous fluids you

seek corroborative data. This

would include

a. Urine sodium of 28 meq/L

b. Urine chloride of 15 meq/L

c. Fractional excretion of sodium less

than 1

d. Urine/serum creatinine ratio of 20

e. Urine osmolality of 350 mOsm/kg

Terms of Use

5. A 45-year-old woman with

Crohn's disease and a small intestinal

fistula develops tetany during

the 2nd wk of parenteral nutrition.

The laboratory findings include Ca

8.2 meq/L; Na 135 meq/L; K 3.2

meq/L; C1 103 meq/L; PO4 2.4

meq/L; albumin 2.4; pH 7.48; 38

kPa; P 84 kPa; bicarbonate 25

meq/L. The most likely cause of the

patient's tetany is

a. Hyperventilation

b. Hypocalcemia

c. Hypomagnesemia

d. Essential fatty acid deficiency

e. Focal seizure

6. A patient with a nonobstructing

carcinoma of the sigmoid colon is

being prepared for elective resection.

To minimize the risk of postoperative

infectious complications,

your planning should include

a. A single preoperative parenteral

dose of antibiotic effective against

aerobes and anaerobes

b. Avoidance of oral antibiotics to prevent

emergence of Clostridium difficile

c. Postoperative administration for

2-4 days of parenteral antibiotics

effective against aerobes and anaerobes

d. Postoperative administration for

5-7 days of parenteral antibiotics

effective against aerobes and anaerobes

e. Operative time less than 5 h

7. A 70-year-old man with aortic

and mitral valvular regurgitation

undergoes an emergency sigmoid

colectomy and end colostomy for

perforated diverticulitis. His postoperative

course is complicated by

a myocardial infarction and atrial

fibrillation. Four weeks later, he

has improved and requests elective

colostomy closure. You would recommend

a. Discontinuation of antiarrhythmic

and antihypertensive medications

on the morning of surgery

b. Discontinuation of beta-blocking

medications on the day prior to

surgery

c. Control of congestive heart failure

with diuretics and digitalis in

severe cases

d. Administration of prophylactic

antibiotics, other than ampicillin

and gentamicin, for patients with

valvular heart disease who are

undergoing gastrointestinal procedures

e. Postponement of elective surgery

for 6-8 wk after a subendocardial

myocardial infarction

2 Surgery

Items 8-9

A previously healthy 55-yearold

man undergoes elective right

hemicolectomy for a Dukes A cancer

of the cecum. His postoperative

ileus is somewhat prolonged, and

on the fifth postoperative day his

nasogastric tube is still in place.

Physical examination reveals

diminished skin turgor, dry

mucous membranes, and orthostatic

hypotension. Pertinent laboratory

values are as follows:

• Arterial blood gases: pH 7.56; PO2

85 kPa; PCO2 50 kPa

• Serum electrolytes (meq/L): Na+

132; K+ 3.1; C1− 80; HCO3− 42

• Urine electrolytes (meq/L): Na+

2; K− 5; C1− 6

8. The values given above allow

the descriptive diagnosis of

a. Uncompensated metabolic alkalosis

b. Respiratory acidosis with metabolic

compensation

c. Combined metabolic and respiratory

alkalosis

d. Metabolic alkalosis with respiratory

compensation

e. "Paradoxical" metabolic respiratory

alkalosis

9. The most appropriate therapy

for the patient described would be

a. Infusion of 0.9% NaC1 with supplemental

KC1 until clinical signs

of volume depletion are eliminated

b. Infusion of isotonic (0.15 N) HC1

via a central venous catheter

c. Clamping the nasogastric tube to

prevent further acid losses

d. Administration of acetazolamide to

promote renal excretion of bicarbonate

e. Intubation and controlled

hypoventilation on a volumecycled

ventilator to further increase

PCO2

Pre- and Postoperative Care 3

Items 10-11

A 23-year-old woman is

brought to the emergency room

from a halfway house, where she

apparently swallowed a handful of

pills. The patient complains of

shortness of breath and tinnitus,

but refuses to identify the pills she

ingested. Pertinent laboratory values

are as follows:

• Arterial blood gases: pH 7.45; PO2

126 kPa; PCO2 12 kPa

• Serum electrolytes (meq/L): Na+

138; K+ 4.8; C1− 102; HCO3− 8

10. The patient's acid-base disturbance

is best characterized by which

of the following descriptions?

a. Acute respiratory alkalosis, compensated

b. Chronic respiratory alkalosis, compensated

c. Metabolic acids, compensated

d. Mixed metabolic acidosis and respiratory

alkalosis

e. Mixed metabolic acidosis and respiratory

acidosis

11. The most likely cause of the

disturbance in this patient is an

overdose of

a. Phenformin

b. Aspirin

c. Barbiturates

d. Methanol

e. Diazepam (Valium)

12. A 65-year-old man undergoes

a technically difficult abdominoperineal

resection for a rectal cancer

during which he receives three

units of packed red blood cells.

Four hours later in the intensive

care unit he is bleeding heavily from

his perineal wound. Emergency

coagulation studies reveal normal

prothrombin, partial thromboplastin,

and bleeding times. The fibrin

degradation products are not

elevated but the serum fibrinogen

content is depressed and the

platelet count is 70,000/μL. The

most likely cause of the bleeding is

a. Delayed blood transfusion reaction

b. Autoimmune fibrinolysis

c. A bleeding blood vessel in the surgical

field

d. Factor VIII deficiency

e. Hypothermic coagulopathy

13. A 78-year-old man with a

history of coronary artery disease

and an asymptomatic reducible

inguinal hernia requests an elective

hernia repair. You explain to him

that valid reasons for delaying the

proposed surgery include

a. Coronary artery bypass surgery 3

mo earlier

b. A history of cigarette smoking

c. Jugular venous distension

d. Hypertension

e. Hyperlipidemia

4 Surgery

14. A 68-year-old man is admitted

to the coronary care unit with an

acute myocardial infarction. His

postinfarction course is marked by

congestive heart failure and intermittent

hypotension. On the fourth

hospital day, he develops severe

midabdominal pain. On physical

examination, blood pressure is

90/60 mm Hg and pulse is 110

beats/min and regular; the abdomen

is soft with mild generalized tenderness

and distention. Bowel sounds

are hypoactive; stool hematest is

positive. The next step in this

patient's management should be

which of the following?

a. Barium enema

b. Upper gastrointestinal series

c. Angiography

d. Ultrasonography

e. Celiotomy

15. A 30-year-old woman in the

last trimester of pregnancy suddenly

develops massive swelling of

the left lower extremity from the

inguinal ligament to the ankle. The

correct sequence of workup and

treatment should be

a. Venogram, bed rest, heparin

b. Impedance plethysmography, bed

rest, heparin

c. Impedance plethysmography, bed

rest, vena caval filter

d. Impedance plethysmography, bed

rest, heparin, warfarin (Coumadin)

e. Clinical evaluation, bed rest, warfarin

16. A 20-year-old woman is found

to have an activated partial thromboplastin

time (APTT) of 78/32 on

routine testing prior to cholecystectomy.

Further investigation reveals

a prothrombin time (PT) of 13/12

(patient/control), a template bleeding

time of 13 min, and a platelet

count of 350 × 100/μL. Which one

of the following characteristics of

this woman's coagulopathy is true?

a. Infusion of purified factor VIII is

usually required to normalize its

concentration prior to surgery

b. Infusion of cryoprecipitate will not

be followed by an improvement in

coagulation

c. Most of these patients are, or

become, seropositive for HIV

d. Epistaxis or menorrhagia is uncommon

e. Lack of platelet aggregation in

response to ristocetin is a common

feature of this disease

17. The chief surgical risk to

which patients with polycythemia

vera are exposed is that due to

a. Anemic disturbances

b. Hemorrhage

c. Infection

d. Renal dysfunction

e. Cardiopulmonary complications

Pre- and Postoperative Care 5

18. A victim of blunt abdominal

trauma requires a partial hepatectomy.

He is rapidly transfused with

8 units of appropriately crossmatched

packed red blood cells

from the blood bank. He is noted in

the recovery room to be bleeding

from intravenous puncture sites

and the surgical incision. His coagulopathy

is likely due to thrombocytopenia

and deficiencies of which

clotting factors?

a. II only

b. II and VII

c. V and VIII

d. IX and X

e. XI and XII

19. Following celiotomy, normal

bowel motility can ordinarily be

presumed to have returned

a. In the stomach in 4 h, the small

bowel in 24 h, and the colon after

the first oral intake

b. In the stomach in 24 h, the small

bowel in 4 h, and the colon in 3

days

c. In the stomach in 3 days, the small

bowel in 3 days, and the colon in 3

days

d. In the stomach in 24 h, the small

bowel in 24 h, and the colon in

24 h

e. In the stomach in 4 h, the small

bowel immediately, and the colon

in 24 h

20. A 65-year-old woman has a

life-threatening pulmonary embolus

5 days following removal of a

uterine malignancy. She is immediately

heparinized and maintained

in good therapeutic range for the

next 3 days, then passes gross

blood from her vagina and develops

tachycardia, hypotension, and

oliguria. Following resuscitation,

an abdominal CT scan reveals a

major retroperitoneal hematoma.

You should now

a. Immediately reverse heparin by a

calculated dose of protamine and

place a vena cava filter (e.g., a

Greenfield filter)

b. Reverse heparin with protamine,

explore and evacuate the

hematoma, and ligate the vena cava

below the renal veins

c. Switch to low-dose heparin

d. Stop heparin and observe closely

e. Stop heparin, give fresh frozen

plasma (FFP), and begin warfarin

therapy

6 Surgery

21. Which of the following surgical

interventions is least likely to

provide acceptable prolongation of

life for patients with AIDS?

a. Splenectomy for AIDS-related idiopathic

thrombocytopenic purpura

b. Colonic resection for perforation

secondary to cytomegalovirus infection

c. Cholecystectomy for acalculous

cholecystitis

d. Tracheostomy for ventilatordependent

patients with respiratory

failure

e. Gastric resection for a bleeding gastric

lymphoma or Kaposi's sarcoma

22. An elderly diabetic woman

with chronic steroid-dependent

bronchospasm has an ileocolectomy

for a perforated cecum. She is

taken to the ICU intubated and is

maintained on broad-spectrum

antibiotics, renal-dose dopamine,

and a rapid steroid taper. On postoperative

day 2 she develops a fever

of 39.2°C (102.5°F), hypotension,

lethargy, and laboratory values

remarkable for hypoglycemia and

hyperkalemia. The most likely

diagnosis of this acute event is

a. Sepsis

b. Hypovolemia

c. Adrenal insufficiency

d. Acute tubular necrosis

e. Diabetic ketoacidosis

23. A cirrhotic patient with abnormal

coagulation studies due to

hepatic synthetic dysfunction

requires an urgent cholecystectomy.

A transfusion of fresh frozen

plasma is planned to minimize the

risk of bleeding due to surgery. The

optimal timing of this transfusion

would be

a. The day before surgery

b. The night before surgery

c. On call to surgery

d. Intraoperatively

e. In the recovery room

24. On postoperative day 3, an

otherwise healthy 55-year-old man

recovering from a partial hepatectomy

is noted to have scant

serosanguineous drainage from his

abdominal incision. His skin staples

are removed, revealing a 1.0-

cm dehiscence of the upper

midline abdominal fascia. Which of

the following actions is most

appropriate?

a. Removing all suture material and

packing the wound with moist sterile

gauze

b. Starting intravenous antibiotics

c. Placing an abdominal (Scultetus)

binder

d. Prompt resuturing of the fascia in

the operating room

e. Bed rest

Pre- and Postoperative Care 7

25. Five days after a sigmoid colectomy

for cancer, a patient's skin staples

are removed and a large gush of

serosanguineous fluid emerges.

Examination of the wound reveals

an extensive fascial dehiscence. The

most appropriate management is

a. Wide opening of the wound to

assure adequate drainage

b. Smear and culture of the fluid and

appropriate antibiotics after the

smear is reviewed

c. Careful reapproximation of the

wound edges with tape

d. Immediate return to the operating

e. Application of a Scultetus binder

26. Signs and symptoms of hemolytic

transfusion reactions include

a. Hypothermia

b. Hypertension

c. Polyuria

d. Abnormal bleeding

e. Hypesthesia at the transfusion site

27. A patient suspected of having

a hemolytic transfusion reaction

should be managed with

a. Removal of nonessential foreign

body irritants, e.g., Foley catheter

b. Fluid restriction

c. 0.1 M HC1 infusion

d. Steroids

e. Fluids and mannitol

28. The surgeon should be particularly

concerned about which coagulation

function in patients receiving

anti-inflammatory or analgesic medications?

a. APTT

b. PT

c. Reptilase time

d. Bleeding time

e. Thrombin time

29. The substrate depleted earliest

in the postoperative period is

a. Branched-chain amino acids

b. Non-branched-chain amino acids

c. Ketone

d. Glycogen

e. Glucose

30. Diagnostic abdominal laparoscopy

is contraindicated in which

of the following patients?

a. A patient with rebound tenderness

following a tangential gunshot

wound to the abdomen

b. A stable patient with a stab wound

to the lower chest wall

c. A patient with a mass in the head of

the pancreas

d. A young female with pelvic pain

and fever

e. An elderly patient in the intensive

care unit suspected of having

intestinal ischemia

8 Surgery

31. A 23-year-old woman undergoes

total thyroidectomy for carcinoma

of the thyroid gland. On the

second postoperative day, she

begins to complain of tingling sensation

in her hands. She appears

quite anxious and later complains

of muscle cramps. Initial therapy

should consist of

a. 10 mL of 10% magnesium sulfate

intravenously

b. Oral vitamin D

c. 100 μg of oral Synthroid

d. Continuous infusion of calcium

gluconate

e. Oral calcium gluconate

32. Hypocalcemia is associated

with

a. Acidosis

b. Shortened QT interval

c. Hypomagnesemia

d. Myocardial irritability

e. Hyperproteinemia

33. The enteric fluid with an electrolyte

(Na+, K+, C1−) content similar

to that of Ringer's lactate is

a. Saliva

b. Contents of small intestine

c. Contents of right colon

d. Pancreatic secretions

e. Gastric juice

34. Which of the following medications

administered for hyperkalemia

counteracts the myocardial

effects of potassium without reducing

the serum potassium level?

a. Sodium polystyrene sulfonate

(Kayexalate)

b. Sodium bicarbonate

c. 50% dextrose

d. Calcium gluconate

e. Insulin

Pre- and Postoperative Care 9

Items 35-37

An in-hospital workup of a 78-

year-old, hypertensive, mildly asthmatic

man who is receiving

chemotherapy for colon cancer

reveals symptomatic gallstones.

Preoperative laboratory results are

notable for a hematocrit of 24%

and a urinalysis with 18-25 WBCs

and gram-negative bacteria. On call

to the operating room he receives

intravenous penicillin. His abdomen

is shaved in the operating

room. An open cholecystectomy is

performed and, despite a lack of

indications, the common bile duct

is explored. The wound is closed

primarily with a Penrose drain exiting

a separate stab wound. On

postoperative day 3 the patient

develops a wound infection.

35. Which of the following changes

could make this wound a less favorable

environment for infection?

a. Decreasing the operative time and

wound contamination by omitting

the common bile duct exploration

b. Placing a Penrose drain exiting

directly through the lateral corner

of the wound

c. Using oral rather than intravenous

penicillin perioperatively

d. Leaving a seroma in the wound to

prevent desiccation of the tissues

e. Reinforcing the wound closure

with a sheet of prosthetic polypropylene

mesh

36. Which of the following characteristics

of this patient might

increase the risk of a wound infection?

a. History of colon surgery

b. Hypertension

c. Male sex

d. Receipt of chemotherapy

e. Asthma

37. Which of the following

changes in the care of this patient

could decrease the chance of a

postoperative wound infection?

a. Increasing the length of the preoperative

hospital stay to prophylactically

treat the asthma with steroids

b. Treating the urinary infection prior

to surgery

c. Shaving the abdomen the night

prior to surgery

d. Continuing the prophylactic antibiotics

for three postoperative days

e. Use of a closed drainage system

brought out through the operative

incision

10 Surgery

Items 38-39

The two solutions most commonly

used to maintain fluid and

electrolyte balance in the postoperative

management of patients are

5% dextrose in 0.9% sodium chloride

and lactated Ringer's solution.

38. A correct statement regarding

5% dextrose in 0.9% saline is

which of the following?

a. It contains the same concentration

of sodium ions as does plasma

b. It can be given in large quantities

without seriously affecting acidbase

balance

c. It is isosmotic with plasma

d. It has a pH of 7.4

e. It may cause a dilutional acidosis

39. Correct statements regarding

lactated Ringer's solution include

which of the following?

a. It contains a higher concentration

of sodium ions than does plasma

b. It is most appropriate for replacement

of nasogastric tube losses

c. It is isosmotic with plasma

d. It has a pH of less than 7.0

e. It may induce a significant metabolic

acidosis

40. Four days after surgical evacuation

of an acute subdural

hematoma, a 44-year-old man

becomes mildly lethargic and

develops asterixis. He has received

2400 mL of 5% dextrose in water

intravenously each day since

surgery, and he appears well

hydrated. Pertinent laboratory values

are as follows:

• Serum electrolytes (meq/L): Na+

118; K+ 3.4; C1− 82; HCO3− 24

• Serum osmolality: 242 mOsm/L

• Urine sodium: 47 meq/L

• Urine osmolality: 486 mOsm/L

A correct statement about this

patient's fluid and electrolyte status

is which of the following?

a. His low serum sodium indicates

sodium deficiency, which should

be treated with 3% saline infusion

b. He probably has the syndrome of

inappropriate secretion of antidiuretic

hormone

c. His blood glucose level should be

checked because the hyponatremia

may be artifactual

d. Water restriction is rarely effective

in severe cases of hyponatremia

e. The underlying problem is the

inappropriate excretion of sodium

(renal sodium wasting)

Pre- and Postoperative Care 11

41. A 43-year-old woman develops acute renal failure following an emergency

resection of a leaking abdominal aortic aneurysm. Three days after

surgery, the following laboratory values are obtained:

• Serum electrolytes (meq/L): Na+ 127; K+ 5.9; C1− 92; HCO3− 15

• Blood urea nitrogen: 82 mg/dL

• Serum creatinine: 6.7 mg/dL

The patient has gained 4 kg since surgery and is mildly dyspneic at

rest. Eight hours after these values are reported, the electrocardiogram

shown below is obtained. The initial treatment for this patient should be

12 Surgery

a. 10% calcium gluconate, 10 mL

b. Digoxin, 0.25 mg every 3 h for three doses

c. Oral Kayexalate

d. Lidocaine, 100 mg

e. Emergent hemodialysis

42. Prophylactic regimens of documented benefit in decreasing the risk of

postoperative thromboembolism include

a. Early ambulation

b. External pneumatic compression devices placed on the upper extremities

c. Elastic stockings

d. Leg elevation for 24 h postoperatively

e. Dipyridamole therapy for 48 h postoperatively

43. Signs and symptoms associated

with early sepsis include

a. Respiratory acidosis

b. Decreased cardiac output

c. Hypoglycemia

d. Increased arteriovenous oxygen difference

e. Cutaneous vasodilation

Pre- and Postoperative Care 13

DIRECTIONS: Each group of questions below consists of lettered

options followed by numbered items. For each numbered item, select the

appropriate lettered option(s). Each lettered option may be used once,

more than once, or not at all. Choose exactly the number of options

indicated following each item.

Items 44-46

Match the gastrointestinal content at each site with its appropriate

ionic composition (meq/L).

Na K C1 HCO3

a. 140 5 104 30

b. 140 5 75 115

c. 60 10 130 0

d. 10 26 10 30

e. 60 30 40 50

44. Salivary (SELECT 1 COMPOSITION)

45. Stomach (SELECT 1 COMPOSITION)

46. Small bowel (SELECT 1 COMPOSITION)

Items 47-50

A 42-year-old man has a calculated resting energy expenditure of 1800

kcal/day (basal energy expenditure plus 10%). Match the following clinical

situations with the appropriate daily energy requirement.

a. 1600

b. 2300

c. 2800

d. 3600

e. 4500

14 Surgery

47. Sepsis (SELECT 1 EXPENDITURE)

48. Skeletal trauma (SELECT 1

EXPENDITURE)

49. Third-degree burns of 60% of

body surface area (BSA) (SELECT

1 EXPENDITURE)

50. Prolonged starvation (SELECT

1 EXPENDITURE)

Pre- and Postoperative Care 15

PRE- AND POSTOPERATIVE

CARE

Answers

1. The answer is a. (Schwartz, 7/e, pp 65-66.) States of magnesium

excess are characterized by generalized neuromuscular depression. Clinically,

severe hypermagnesemia is rarely seen except in those patients with

advanced renal failure treated with magnesium-containing antacids.

Hypermagnesemia is produced intentionally, however, by obstetricians

who use parenteral magnesium sulfate (MgSO4) to treat preeclampsia.

MgSO4 is administered until depression of the deep tendon reflexes is

observed, a deficit that occurs with modest hypermagnesemia (over 4

meq/L). Greater elevations of magnesium produce progressive weakness,

which culminates in flaccid quadriplegia and in some cases respiratory

arrest from paralysis of the chest bellows mechanism. Hypotension may

occur because of the direct arteriolar relaxing effect of magnesium.

Changes in mental status occur in the late stages of the syndrome and are

characterized by somnolence that progresses to coma.

2. The answer is b. (Schwartz, 7/e, pp 57-63.) Acute severe hyponatremia

sometimes occurs following elective surgical procedures. It is usually

the result of the combination of appropriate postoperative stimulation

of antidiuretic hormone and injudicious administration of excess free

water in the first few postoperative days. Totally sodium-free intravenous

fluids (e.g., dextrose and water) should be given with great caution postoperatively,

since occasionally the resulting hyponatremia can be associated

with sudden death from a flaccid heart or with severe permanent

brain damage. The condition is usually best treated by withholding free

water and allowing the patient to reequilibrate spontaneously. At levels

below 115 meq/L, seizures or mental obtundation may mandate treatment

with hypertonic sodium solutions. This must be done with extreme care

because the risk of fluid overload with acute pulmonary or cerebral edema

is high.

16

3. The answer is d. (Sabiston, 15/e, p 931.) Any patient who has lost

much of the ileum (whether from injury, disease, or elective surgery) is at

high risk of developing enteric hyperoxaluria if the colon remains intact.

Calcium oxalate stones will develop in at least 10% of these patients. The

condition results from excessive absorption of oxalate from the colon

through two related synergistic mechanisms: unabsorbed fatty acids combine

with calcium, which prevents the formation of insoluble calcium

oxalate and allows oxalate to remain available for colonic absorption; and

unabsorbed fatty acids and bile acids also increase the permeability of the

colon to the oxalate.

4. The answer is c. (Schwartz, 7/e, pp 452-455.) When oliguria occurs

postoperatively, it is important to differentiate between low output caused

by the physiologic response to intravascular hypovolemia and that caused

by acute tubular necrosis. The fractional excretion of sodium (FENa) is an

especially useful test to aid in this differentiation. Values of FE < 1% in an

oliguric setting indicate aggressive sodium reclamation in the tubules; values

above this suggest tubular injury. The fractional excretion is a simple

calculation: (urine Na × serum creatinine) ÷ (serum sodium × urinary creatinine).

In the setting of postoperative hypovolemia, all findings would

reflect the kidney's efforts to retain volume: the urine sodium would be

below 20 meq/L, the urine chloride would not be helpful except in the

metabolically alkalotic patient, the serum osmolality would be over 500

mOsm/kg, and the urine/serum creatinine ratio would be above 40.

5. The answer is c. (Schwartz, 7/e, pp 64-66.) Magnesium deficiency is

common in malnourished patients and patients with large gastrointestinal

fluid losses. The neuromuscular effects resemble those of calcium deficiency-

namely, paresthesia, hyperreflexia, muscle spasm, and ultimately

tetany. The cardiac effects are more like those of hypercalcemia. An electrocardiogram

therefore provides a rapid means of differentiating between

hypocalcemia and hypomagnesemia. Hypomagnesemia also causes potassium

wasting by the kidney. Many hospital patients with refractory hypocalcemia

will be found to be magnesium deficient. Often this deficiency

becomes manifest during the response to parenteral nutrition when normal

cellular ionic gradients are restored. A normal blood pH and arterial PCO2

rule out hyperventilation. The serum calcium in this patient is normal when

Pre- and Postoperative Care Answers 17

adjusted for the low albumin. Hypomagnesemia causes functional hypoparathyroidism,

which can lower serum calcium and thus result in a combined

defect.

6. The answer is c. (Schwartz, 7/e, pp 143-149.) Many clinical and experimental

studies have looked at the optimum bowel preparation and preoperative

regimen for elective colonic surgery to reduce the postoperative

infectious complications of wound infection, intraabdominal abscess, and

anastomotic leakage. Currently, a postoperative rate of wound infection of

only 5% can be attained by combining mechanical cleansing, oral antibiotics,

and perioperative parenteral antibiotics. The type of mechanical cleansing

does not matter as long as it is effective. Preoperative oral antibiotics may be

administered one or more days prior to surgery and should cover aerobes

and anaerobes (e.g., neomycin-erythromycin). Parenteral antibiotics effective

against aerobes and anaerobes (e.g., cefoxitin) should be administered on call

to the operating room as a single dose and no more than 24 h postoperatively.

Both antibiotic regimens yield maximum prophylaxis without fostering resistant

transformation of microbes. Procedures that require operative time

greater than 3 h or that involve the extraperitoneal rectum are associated with

an increased risk of infectious complications.

7. The answer is c. (Schwartz, 7/e, pp 462-465.) There are several recommended

interventions in cardiac patients who are undergoing noncardiac

surgery. The two factors that correlate best with postoperative lifethreatening

or fatal cardiac complications are myocardial infarction (transmural

or subendocardial) and uncontrolled congestive heart failure.

Hence, delay of elective surgery for 6 mo after myocardial infarction and

preoperative control of congestive heart failure with diuretics and digitalis,

in severe cases, will have the greatest effect in decreasing the risks of

surgery. A patient's cardiac medications should be continued preoperatively,

including during the morning of surgery, to maintain adequate therapeutic

levels. This is especially true for beta blockers, which can manifest

withdrawal rebound hypertension and tachycardia approximately 24 h

after discontinuation. Patients with prosthetic valves or valvular heart disease

should be given prophylactic antibiotics to prevent seeding of their

valves during episodes of significant bacteremia. This most commonly

occurs during gastrointestinal or genitourinary procedures. Ampicillin and

18 Surgery

gentamicin cover the flora frequently encountered, including enterococci

and gram-negative organisms.

8. The answer is d. (Greenfield, 2/e, pp 259-266.) Both the arterial pH

and the PCO2 are elevated in the patient presented in the question; the disturbance

is alkalosis with hypoventilation. The PCO2 typically increases by

0.5-1.0 pKa for each meq/L increase in serum bicarbonate. These findings

suggest that the hypoventilation is compensatory rather than a primary

phenomenon. This assumption is further supported by the absence of clinical

lung disease.

9. The answer is a. (Greenfield, 2/e, pp 259-266.) The development of a

clinically significant metabolic alkalosis in a patient requires not only the

loss of acid or addition of alkali, but renal responses that maintain the alkalosis.

The normal kidney can tremendously augment its excretion of acid or

alkali in response to changes in ingested load. However, in the presence of

significant volume depletion and consequent excessive salt and water

retention, the tubular maximum for bicarbonate reabsorption is increased.

Correction of volume depletion alone is usually sufficient to correct the

alkalosis, since the kidney will then excrete the excess bicarbonate. HCl

infusion is usually unnecessary and can be dangerous. Acetazolamide is

unlikely to be effective in the face of distal Na+ reabsorption (in exchange

for H+ secretion). Moreover, to the extent that acetazolamide causes natriuresis,

it will exacerbate the volume depletion.

10. The answer is d. (Greenfield, 2/e, pp 260-266.) The patient presented

in the question is in a state of metabolic acidosis as shown by a markedly

increased anion gap of 28 meq unmeasured anions per liter of plasma.

However, the respiratory response is greater than can be explained by a

compensatory response, since the patient is mildly alkalemic. The disturbance

cannot be pure respiratory alkalosis, since the serum bicarbonate

does not drop below 15 meq/L as a result of renal compensation and the

anion gap does not vary by more than 1-2 meq/L from its normal value of

12 in response to a respiratory disturbance. The renal response to hyperventilation

involves wasting of bicarbonate and compensatory retention of

chloride; it does not involve a change in the concentration of "unmeasured"

anions, such as albumin and organic acids.

Pre- and Postoperative Care Answers 19

11. The answer is b. (Anderson, Ann Intern Med 85:745-748, 1976.) The

acid-base disturbance in the patient described in the previous question

demonstrates the value of extracting all available information from a small

amount of rapidly retrievable data, e.g., arterial blood gases. Salicylates

directly stimulate the respiratory center and produce respiratory alkalosis.

By building up an accumulation of organic acids, salicylates also produce a

concomitant metabolic acidosis. Characteristically both disturbances exist

simultaneously following massive ingestion of salicylates. If sedative agents

have been taken as well, the respiratory alkalosis (and even the respiratory

compensation) may be absent. Phenformin and methanol overdoses also

produce "high-anion-gap" metabolic acidosis, but without the simultaneous

respiratory disturbance. In the case presented, the patient's history of

tinnitus in conjunction with her mixed metabolic acidosis-respiratory

alkalosis is essentially pathognomonic of salicylate intoxication.

12. The answer is c. (Sabiston, 15/e, pp 131-133.) Whenever significant

bleeding is noted in the early postoperative period, the presumption

should always be that it is due to an error in surgical control of blood vessels

in the operative field. Hematologic disorders that are not apparent during

the long operation are most unlikely to surface as problems

postoperatively. Blood transfusion reactions can cause diffuse loss of clot

integrity; the sudden appearance of diffuse bleeding during an operation

may be the only evidence of an intraoperative transfusion reaction. In the

postoperative period, transfusion reactions usually present as unexplained

fever, apprehension, and headache-all symptoms difficult to interpret in

the early postoperative period. Factor VIII deficiency (hemophilia) would

almost certainly be known by history in a 65-year-old man, but if not,

intraoperative bleeding would have been a problem earlier in this long

operation. Severely hypothermic patients will not be able to form clots

effectively, but clot dissolution does not occur. Care should be taken to prevent

the development of hypothermia during long operations through the

use of warmed intravenous fluid, gas humidifiers, and insulated skin barriers.

13. The answer is c. (Goldman, J Cardiothorac Anesth 1:237, 1987.) The

work of Goldman and others has served to identify risk factors for perioperative

myocardial infarction. The highest likelihood is associated with

recent myocardial infarction: the more recent the event, the higher the risk

20 Surgery

up to 6 mo. It should be noted, however, that the risk never returns to normal.

A non-Q-wave infarction may not have destroyed much myocardium,

but it leaves the surrounding area with borderline perfusion; hence the particularly

high risk of subsequent perioperative infarction. Evidence of congestive

heart failure, such as jugular venous distention, or S3 gallop also

carries a high risk, as does the frequent occurrence of ectopic beats. Old

age and emergency surgery are risk factors independent of these others.

Coronary revascularization by coronary artery bypass graft (CABG) tends

to protect against myocardial infarction. Smoking, diabetes, hypertension,

and hyperlipidemia (all of which predispose to coronary artery disease) are

surprisingly not independent risk factors, although they may increase the

death rate should an infarct occur. The value of this information and data

derived from further testing is that it identifies the patient who needs to be

monitored invasively with a systemic arterial catheter and pulmonary arterial

catheter. Most perioperative infarcts occur postoperatively when the

"third-space" fluids return to the circulation, which increases the preload

and the myocardial oxygen consumption. This generally occurs around the

third postoperative day.

14. The answer is c. (Schwartz, 7/e, pp 966-967.) Acute mesenteric

ischemia may be difficult to diagnose. The condition should be suspected in

patients with either systemic manifestations of arteriosclerotic vascular disease

or low cardiac output states associated with a sudden development of

abdominal pain that is out of proportion to the physical findings. Lactic acidosis

and an elevated hematocrit reflecting hemoconcentration are common

laboratory findings. Abdominal films show a nonspecific ileus pattern. The

cause may be embolic occlusion or thrombosis of the superior mesenteric

artery, primary mesenteric venous occlusion, or nonocclusive mesenteric

ischemia secondary to low cardiac output states. A mortality of 65-100% is

reported. The majority of affected patients are at high operative risk, but

since early diagnosis followed by revascularization or resectional surgery or

both is the only hope for survival, celiotomy must be performed once the

diagnosis of arterial occlusion or bowel infarction has been made. Initial

treatment of nonocclusive mesenteric ischemia includes measures to

increase cardiac output and blood pressure and the direct intraarterial infusion

of vasodilators such as papaverine into the superior mesenteric system.

The patient presented in the question is at risk for both occlusive and

nonocclusive mesenteric ischemic disease. If his clinical status permits,

Pre- and Postoperative Care Answers 21

angiographic studies should be performed before the operation to establish

the diagnosis and to determine whether embolectomy, revascularization, or

nonsurgical management is indicated as initial treatment.

15. The answer is b. (Schwartz, 7/e, pp 1007-1014.) This patient has a

left iliofemoral vein thrombosis, as evidenced by sudden massive swelling

of her entire left lower extremity. Noninvasive venous testing should be

quite helpful as the venous obstruction extends above the knee; therefore,

venography and x-ray exposure are unnecessary. Heparin is the preferred

agent because it does not cross the placenta, while warfarin does. The vena

caval filter is not indicated because there is no contraindication to heparin

therapy and there has not been any evidence of pulmonary embolus.

16. The answer is e. (Sabiston, 15/e, pp 134-135.) von Willebrand disease

has an autosomal dominant pattern of inheritance that affects both

men and women. The deficiency of factor VIII activity is generally less

severe than in classic hemophilia and tends to fluctuate even in an

untreated patient. However, the bleeding tendency is compounded by

abnormal platelet function. This is responsible for the common occurrence

of epistaxis and menorrhagia. In 70% of patients, platelets fail to aggregate

in response to the diagnostic reagent ristocetin. Transfusion of cryoprecipitate

provides factor VIII R:WF (the von Willebrand factor), whereas infusions

of high-purity concentrates of factor VIII:C are not effective. These

patients do not generally require treatment unless they need surgery or are

severely injured; therefore, they have not usually received the contaminated

concentrates responsible for the 80% prevalence of HIV seropositivity

among hemophiliacs.

17. The answer is b. (Schwartz, 7/e, pp 85-87.) Intraoperative and postoperative

hemorrhage is a significant problem in the patient with polycythemia

vera. Despite thrombocytosis, these patients have a hemorrhagic

tendency generally ascribed to a qualitative deficiency of the platelets. Elective

surgery should be postponed until the hematocrit and platelet count

reach normal levels. Alkylating agents, such as busulfan or chlorambucil,

are effective in this regard. In the emergency situation, phlebectomy should

be performed prior to operation and also an especially careful hemostatic

technique should be employed. Infection is also a problem in patients with

22 Surgery

polycythemia vera, but hemorrhagic problems are the more frequently

encountered complications.

18. The answer is c. (Schwartz, 7/e, p 96.) When large amounts of banked

blood are transfused, the recipient becomes deficient in factors V and VIII

(the "labile" factors) and an acquired coagulopathy ensues. Since banked

blood is also deficient in platelets, thrombocytopenia may also develop.

19. The answer is b. (Schwartz, 7/e, p 467.) The misconception that the

entire bowel does not function in the early postoperative period is still

widely held. Intestinal motility and absorption studies have clarified the

patterns by which bowel activity resumes. The stomach remains uncoordinated

in its muscular activity and does not empty efficiently for about 24 h

after abdominal procedures. The small bowel functions normally within

hours of surgery and is able to accept nutrients promptly, either by nasoduodenal

or percutaneous jejunal feeding catheters or, after 24 h, by gastric

emptying. The colon is stimulated in large measure by the gastrocolic

reflex but ordinarily is relatively inactive for 3-4 days.

20. The answer is a. (Greenfield, 2/e, pp 96-97.) In a heparinized patient

with significant life-threatening hemorrhage, immediate reversal of heparin

anticoagulation is indicated. Protamine sulfate is a specific antidote to

heparin and should be given as 1 mg for each 100 U heparin if hemorrhage

begins shortly after a bolus of heparin. For a patient (such as this) in whom

heparin therapy is ongoing, the dose should be based on the half-life of

heparin (90 min). Since protamine is also an anticoagulant, only half the

calculated circulating heparin should be reversed. The protaminization

should be followed by placement of a percutaneous vena cava filter (Greenfield

filter). In this critically ill patient, exploration of the retroperitoneal

space would be surgically challenging and meddlesome.

21. The answer is d. (Diettrich, Arch Surg 126:860-865, 1991.) Patients

who have AIDS frequently present with problems that potentially require

surgical care. The involvement of surgeons with these patients will increase

as more effective treatments are developed and the AIDS patient's survival

is prolonged. AIDS patients not only suffer from common surgical illnesses,

they also develop problems especially associated with their altered immune

Pre- and Postoperative Care Answers 23

status, such as bleeding from gastrointestinal lymphomas or Kaposi's

lesions, bowel ischemia, perforation from parasitic or viral infection, acalculous

cholecystitis, and retroperitoneal and intraabdominal masses due to

massive lymphadenitis. With the exception of tracheostomy, experience

has demonstrated that surgery can be performed with acceptable morbidity

and mortality and that it seems to provide comfort and prolong quality

life. Though it may facilitate nursing care, tracheostomy does not reverse or

slow the pulmonary failure once the patient has become ventilator dependent.

22. The answer is c. (Schwartz, 7/e, pp 1639-1640.) Acute adrenal insufficiency

is classically manifested as changing mental status, increased temperature,

cardiovascular collapse, hypoglycemia, and hyperkalemia. The

diagnosis can be difficult to make and requires a high index of suspicion. Its

clinical presentation is similar to that of sepsis; however, sepsis is generally

associated with hyperglycemia and no significant change in potassium. The

treatment for adrenal crisis is hydrocortisone 100 mg intravenously, volume

resuscitation, and other supportive measures to treat any new or ongoing

stress. Then, 200-400 hydrocortisone mg is administered over the next 24

h, followed by a taper of the steroid as tolerated.

23. The answer is c. (Schwartz, 7/e, pp 95-96.) Transfusions with fresh

frozen plasma (FFP) are given to replenish clotting factors. The effectiveness

of the transfusion in maintaining hemostasis is dependent on the

quantity of each factor delivered and its half-life. The half-life of the most

stable clotting factor, factor VII, is 4-6 h. A reasonable transfusion scheme

would be to give FFP on call to the operating room. This way the transfusion

is complete prior to the incision with circulating factors to cover the

operative and immediate postoperative period.

24. The answer is c. (Sabiston, 15/e, pp 344-345.) Serosanguineous

drainage is classically associated with fascial dehiscence. A reasonable

approach to this problem is to remove several sutures and gently explore

the wound to determine the extent of the dehiscence. A small fascial dehiscence

(1-2 cm) can be treated conservatively with local wound care and an

abdominal binder to support the fascia. A larger dehiscence requires reoperation

for formal reclosure of the fascia. High-risk patients with a large fascial

dehiscence may be treated with an abdominal binder and modified bed

24 Surgery

rest, which allows both intraabdominal adhesion formation and local granulation.

Although fascial dehiscence can occur from local infection, it is

usually not an infectious process and does not require parenteral antibiotic

therapy.

25. The answer is d. (Sabiston, 15/e, pp 344-345.) The appearance of a

gush of serosanguineous fluid from an abdominal incision is pathognomonic

of a disruption of the deep fascia. The source of large amounts of

serous fluid is the peritoneum. The temptation to avoid direct reclosure of

these wounds when the fascial defect is larger than 1-2 cm should be

resisted because delayed resumption of normal ambulation and activity

with a late ventral hernia is the best outcome to be hoped for. Evisceration,

wound infection, or protracted convalescence is far more likely. Recurrence

of eviscerations following reclosure of these wounds is extremely rare,

though 10-20% will later develop incisional hernias. The Scultetus binder

is a corsetlike cloth wrap that was once a favored support to reduce likelihood

of evisceration in those wounds in which the fascia was left unrepaired

after dehiscence.

26. The answer is d. (Sabiston, 15/e, p 124.) Allergic and febrile reactions

occur in about 1% of all transfusions. Hemolytic transfusion reactions are

much less common (0.2%) with fatal reactions in 1:100,000 transfusions.

Hemolytic transfusion reactions are due to the reaction of recipient antibodies

against transfused antigens. These reactions can be both immediate

and delayed. Symptoms of a hemolytic transfusion reaction include fever,

chills, and pain and heat at the infusion site, as well as respiratory distress,

anxiety, hypotension, and oliguria. During surgery a hemolytic transfusion

reaction can manifest as abnormal bleeding.

27. The answer is e. (Sabiston, 15/e, p 124.) Hemolytic transfusion reactions

lead to hypotension and oliguria. The increased hemoglobin in the

plasma will be cleared via the kidneys, which leads to hemoglobinuria.

Placement of an indwelling Foley catheter with subsequent demonstration

of oliguria and hemoglobinuria not only confirms the diagnosis of a

hemolytic transfusion reaction but is useful in monitoring corrective therapy.

Treatment begins with discontinuation of the transfusion, followed by

aggressive fluid resuscitation to support the hypotensive episode and

increase urine output. Inducing a diuresis through aggressive fluid resusci-

Pre- and Postoperative Care Answers 25

tation and osmotic diuretics is important to clear the hemolyzed red cell

membranes, which can otherwise collect in glomeruli and cause renal

damage. Alkalinization of the urine (pH > 7) helps prevent hemoglobin

clumping and renal damage. Steroids do not have a role in the treatment of

hemolytic transfusion reactions.

28. The answer is d. (Sabiston, 15/e, p 133.) Platelet dysfunction, measured

by bleeding time, has been associated with a long list of drugs.

Among nonsteroidal anti-inflammatory and analgesic medications, aspirin,

indomethacin, phenylbutazone, acetominophen, and phenacetin have

been implicated, along with aminopyrine and codeine. Ibuprofen, however,

has not. In addition, many antibiotics, anticonvulsants, and sedatives

have been associated with thrombasthenia. Any time platelet abnormalities

are suspected, a careful review of the drugs the patient is receiving should

be undertaken, and a measurement should be made of the platelet count

and bleeding time. Platelet dysfunction does not affect APTT, PT, reptilase,

or thrombin times.

29. The answer is d. (Sabiston, 15/e, pp 60-62.) The metabolic response

to surgery (and other trauma) is a result of neuroendocrine stimulation that

sharply accelerates protein breakdown, stimulates gluconeogenesis, and

produces glucose intolerance. The glycogen stores are rapidly depleted

because of a fall in insulin and a rise in glucagon levels in the plasma. The

peripheral effects of the neuroendocrine secretion result in an increase in

plasma levels of amino acids, free fatty acids, lactate, glucose, and glycerol.

In the liver, the cortisol and glucagon stimulate glycogenolysis, gluconeogenesis,

and increased substrate uptake.

30. The answer is a. (Berci, Am J Surg 161:332-335, 1991.) The indications

for diagnostic laparoscopic exploration are increasing rapidly as the

tools and techniques for such intervention improve. In the stable trauma

patient with a tangential gunshot wound or with a stab wound to the lower

chest wall or abdomen, laparoscopy may show no actual peritoneal penetration

and might make a laparotomy unnecessary. If the peritoneum or

diaphragm is injured, subsequent laparotomy and exploration are generally

indicated to exclude other possible injuries and to facilitate repair of

the diaphragm. All unstable patients or those with signs of peritoneal irritation

(e.g., rebound tenderness) should undergo prompt celiotomy.

26 Surgery

Laparoscopic staging of malignancies allows improved preoperative assessment

of the resectability of intraabdominal malignancies. The procedure

has proved particularly useful in cases with pancreatic carcinoma. Laparoscopic

evaluations may expedite differentiation of competing etiologies of

right lower quadrant pain; this would allow appendectomy for appendicitis

or appropriate therapy such as intravenous antibiotics for pelvic inflammatory

disease and preempt celiotomy. In critically ill patients, the

development of low flow or embolic ischemic insults to the bowel can be

fatal if not recognized and treated early. Many such patients are already

being ventilated in intensive care units; in this setting, bedside laparoscopy

can ascertain the need for early exploration for bowel revascularization or

resection.

31. The answer is d. (Schwartz, 7/e, p 1693.) Postthyroidectomy

hypocalcemia is usually due to transient ischemia of the parathyroid glands

and is self-limited. When it becomes symptomatic, it should be treated

with intravenous infusions of calcium. In most cases the problem is

resolved in several days. If hypocalcemia persists, oral therapy is then

added with calcium gluconate. Vitamin D preparations are only used if

hypocalcemia is prolonged and permanent hypocalcemia is suspected.

There is no role for thyroid hormone replacement or magnesium sulfate in

the treatment of hypocalcemia.

32. The answer is c. (Schwartz, 7/e, p 64.) Hypocalcemia is associated

with a prolonged QT interval and may be aggravated by both hypomagnesemia

and alkalosis. Serum calcium levels below 7.0 mg/dL, encountered

most frequently following parathyroid or thyroid surgery or in patients

with acute pancreatitis, should be treated with intravenous calcium gluconate

or lactate. The myocardium is very sensitive to calcium levels; therefore

calcium is considered a positive inotropic agent. Calcium increases the

contractile strength of cardiac muscle as well as the velocity of shortening.

In its absence the efficiency of the myocardium decreases. Hypocalcemia

often occurs with hypoproteinemia even though the ionized serum calcium

fraction remains normal.

33. The answer is b. (Schwartz, 7/e, p 56.) Bile and the fluids found in

the duodenum, jejunum, and ileum all have an electrolyte content similar

to that of Ringer's lactate. Saliva, gastric juice, and right colon fluids have

Pre- and Postoperative Care Answers 27

high K+ and low Na+ content. Pancreatic secretions are high in bicarbonate.

It is important to consider these variations in electrolyte patterns when calculating

replacement requirements following gastrointestinal losses.

34. The answer is d. (Schwartz, 7/e, p 63.) Reduction of an elevated

serum potassium level is important to avoid the cardiovascular complications

that ultimately culminate in diastolic cardiac arrest. Kayexalate is a

cation exchange resin that is instilled into the gastrointestinal tract and

exchanges sodium for potassium ions. Its use is limited to semiacute and

chronic potassium elevations. Sodium bicarbonate causes a rise in serum

pH and shifts potassium intracellularly. Administration of glucose initiates

glycogen synthesis and uptake of potassium. Insulin can be used in conjunction

with this to aid in the shift of potassium intracellularly. Calcium

gluconate does not affect the serum potassium level but rather counteracts

the myocardial effects of hyperkalemia.

35-37. The answers are 35-a, 36-d, 37-b. (Schwartz, 7/e, pp 448-452.)

The determinants of a postoperative wound infection include those related

to the bacteria, the environment (i.e., the wound), and the host's defense

mechanisms. Within this triad there are factors predetermined by the status

of the patient [e.g., age, obesity, steroid dependence, multiple diagnoses

(more than three), immunosuppression] and by the type of procedure

(e.g., contaminated versus clean, emergent versus elective). However, there

are several factors that can be optimized by the surgeon. Decreasing the

bacterial inoculum and virulence by limiting the patient's prehospital stay,

clipping the operative site in the operating room, administering perioperative

antibiotics (within a 24-h period surrounding operation) with an

appropriate antimicrobial spectrum, treating remote infections, avoiding

breaks in technique, using closed drainage systems (if needed at all) that

exit the skin away from the surgical incision, and minimizing the duration

of the operation have all been shown to decrease postoperative infection.

Making a wound less favorable to infection requires attention to basic halstedian

principles of hemostasis, anatomic dissection, and gentle handling

of tissues as well as limiting the amount of foreign body and necrotic tissue

in the wound. Although they are the most difficult factors to influence, host

defense mechanisms can be improved by optimizing nutritional status, tissue

perfusion, and oxygen delivery.

28 Surgery

38-39. The answers are 38-e, 39-d. (Schwartz, 7/e, pp 66-67.) Isotonic

saline solutions contain 154 meq/L of both sodium and chloride ions. Each

ion is in a substantially higher concentration than is found in the normal

serum (Na = 142 meq/L; C1 = 103 meq/L). When isotonic solutions are

given in large quantities, they overload the kidney's ability to excrete chloride

ion, which results in a dilutional acidosis. They also may intensify preexisting

acidosis by reducing the base bicarbonate:carbonic acid ratio in

the body. Isotonic saline solutions are particularly useful in hyponatremic

or hypochloremic states and whenever a tendency to metabolic alkalosis is

present, as occurs with significant nasogastric suction losses or vomiting.

Administration of lactated Ringer's solution is appropriate for replacing

gastrointestinal losses and correcting extracellular fluid deficits. Containing

130 meq/L sodium, lactated Ringer's is hyposmolar with respect to

sodium and provides approximately 150 mL of free water with each liter

given. Although this is ordinarily not a significant load, in some clinical situations

it can be. Lactated Ringer's is sufficiently "physiological" to enable

administration of large amounts without significantly affecting the body's

acid-base balance. It is worth noting that both isotonic saline and lactated

Ringer's are acidic with respect to the plasma: 0.9% NaC1/5% dextrose has

a pH of 4.5; lactated Ringer's has a pH of 6.5.

40. The answer is b. (Schwartz, 7/e, pp 473-474.) The patient presented

has the syndrome of inappropriate antidiuretic hormone secretion (SIADH).

Although this syndrome is primarily associated with diseases of the central

nervous system or of the chest (e.g., oat cell carcinoma of the lung), excessive

amounts of antidiuretic hormone are also present in most postoperative

patients. The pathophysiology of SIADH involves an inability to dilute the

urine; administered water is therefore retained, which produces dilutional

hyponatremia. Body sodium stores and fluid balance are normal, as evidenced

by the absence of the clinical findings suggestive of abnormalities of

extracellular fluid volume. While hypertonic saline infusions can transiently

improve hyponatremia, the appropriate therapy is to restrict water ingestion

to a level below the patient's ability to excrete water. Hypertonic saline may

be dangerous, since it can shift accumulated water into the extracellular

fluid and precipitate pulmonary edema in the patient who suffers from low

cardiac reserves. Hyperglycemia cannot account for the hyponatremia seen

in this patient because the serum osmolality, as well as the serum sodium, is

Pre- and Postoperative Care Answers 29

depressed. Hyponatremia resulting from hyperglycemia would be associated

with an elevated serum osmolality.

41. The answer is a. (Schwartz, 7/e, p 63.) The electrocardiogram exhibited

in the question demonstrates changes that are essentially diagnostic of

severe hyperkalemia. Correct treatment for the affected patient includes

administration of a source of calcium ions (which will immediately oppose

the neuromuscular effect of potassium) and administration of sodium ions

(which, by producing a mild alkalosis, will shift potassium into cells); each

will temporarily reduce serum potassium concentration. Infusion of glucose

and insulin would also effect a temporary transcellular shift of potassium.

However, these maneuvers are only temporarily effective; definitive

treatment calls for removal of potassium from the body. The sodiumpotassium

exchange resin sodium polystyrene sulfonate (Kayexalate)

would accomplish this removal, but over a period of hours and at the price

of adding a sodium ion for each potassium ion that is removed. Hemodialysis

or peritoneal dialysis is probably required for this patient, since these

procedures also rectify the other consequences of acute renal failure, but

they would not be the first line of therapy given the acute need to reduce

the potassium level. Both lidocaine and digoxin would not only be ineffective

but contraindicated, since they would further depress the myocardial

conduction system.

42. The answer is b. (Sabiston 15/e, pp 1594-1616.) The problem of deep

vein thrombosis and pulmonary embolism is significant in general surgery.

There are approximately 2.5 million episodes of deep vein thrombosis and

600,000 pulmonary embolic events that result in 200,000 deaths annually.

The problem is exacerbated by the disorder's frequent unheralded progression-

only 20-25% of fatal pulmonary emboli are suspected clinically by

the physician or manifest by classic signs or symptoms. The fact that most

deaths due to pulmonary embolism occur before effective therapy can be

started highlights the importance of preventive measures. Several documented

factors help identify those at increased risk, including age greater

than 40, obesity, malignancy, venous disease, congestive heart failure and

atrial fibrillation, and prolonged bed rest. Virchow initially attributed

venous thrombosis to the combination of venous stasis, hypercoagulability,

and endothelial injury. The first two conditions are exacerbated by operative

positioning and stress such that 25% of patients at moderate risk will

30 Surgery

develop venous thromboembolism, 50% within 24 h and 80% within 72 h

postoperatively. The recommendation for prophylaxis in those at high risk

is preoperative anticoagulation with warfarin. No prophylaxis is recommended

for those at low risk (e.g., those less than age 40 with normal

weight and no venous disease). Prophylactic regimens for those at moderate

risk are basically chemical or mechanical, and the best two, which have

equivalent effectiveness, are representative of each type. First, low-dose

heparin (5000 U) started 2 h preoperatively and continued every 12 h

postoperatively will decrease the risk of deep vein thrombosis from 25 to

7% and of major pulmonary embolus from 6 to 0.6%. External pneumatic

compression devices not only obviate venous stasis, but they also have a

systemic effect on coagulation, such that use on the arms also significantly

reduces venous thromboembolism of the lower extremities. Early ambulation,

elastic stockings, leg elevation, and dipyridamole (Persantine) alone

have not been documented to be effective.

43. The answer is e. (Schwartz, 7/e, pp 115-120.) It is important to identify

and treat occult or early sepsis before it progresses to septic shock and

the associated complications of multiple organ failure. An immunocompromised

host may not manifest some of the more typical signs and symptoms

of infection, such as elevated temperature and white cell count; this

forces the clinician to focus on more subtle signs and symptoms. Early sepsis

is a physiologically hyperdynamic, hypermetabolic state representing a

surge of catecholamines, cortisol, and other stress-related hormones. A

changing mental status, tachypnea that leads to respiratory alkalosis, and

flushed skin are often the earliest manifestations of sepsis. Intermittent

hypotension requiring increased fluid resuscitation to maintain adequate

urine output is characteristic of occult sepsis. Hyperglycemia and insulin

resistance during sepsis are typical in diabetic as well as nondiabetic

patients. This relates to the gluconeogenic state of the stress response. The

cardiovascular response to early sepsis is characterized by an increased cardiac

output, decreased systemic vascular resistance, and decreased peripheral

utilization of oxygen, which yields a decreased arteriovenous oxygen

difference.

44-46. The answers are 44-d, 45-c, 46-a. (Schwartz, 7/e, p 56.) One of

the most common causes of dehydration and metabolic disarray in surgical

patients is the failure to replace gastrointestinal losses. External losses can

Pre- and Postoperative Care Answers 31

often be collected for measurement of volume and ionic composition.

Accurate replacement of these measured losses is clearly the best method of

avoiding imbalance. However, a knowledge of the ionic composition of the

intestinal contents at various sites permits an accurate estimate for early

replacement. Most of these secretions start as extracellular fluid (with a

composition similar to that of plasma) and are modified by intestinal

glands. The stomach substitutes hydrogen ions for sodium and thus eliminates

all but a tiny fraction of bicarbonate. The glands of the small intestine

secrete various amounts of bicarbonate; the chloride content is depressed

to an equivalent degree (to maintain ionic balance). Colonic contents

(stool) and saliva are most notable for their potassium content. Stool also

has a high bicarbonate content. Severe diarrhea can therefore cause potassium

depletion and a metabolic acidosis.

47-50. The answers are 47-c, 48-b, 49-d, 50-a. (Schwartz, 7/e, pp

33-40.) Resting energy expenditure in the nonstressed patient is approximately

10% greater than basal energy expenditure. The resting energy

expenditure increases directly proportional to the degree of stress. Studies

by Kinney and associates using indirect calorimetry have documented the

relative degree of increase in resting energy expenditure for a variety of

clinical situations. The following table summarizes these results:

32 Surgery

Change in Energy

Clinical Situation Expenditure

Prolonged starvation Decreased 10-30%

Skeletal trauma Increased 10-30%

Sepsis Increased 30-60%

Third-degree burns 20% BSA Increased 50-100%

CRITICAL CARE:

ANESTHESIOLOGY, BLOOD

GASES, RESPIRATORY CARE

Questions

DIRECTIONS: Each item below contains a question or incomplete

statement followed by suggested responses. Select the one best response to

each question.

33

51. The most common physiologic

cause of hypoxemia is

a. Hypoventilation

b. Incomplete alveolar oxygen diffusion

c. Ventilation-perfusion inequality

d. Pulmonary shunt flow

e. Elevated erythrocyte 2,3-diphosphoglycerate

level (2,3-DPT)

52. Generally accepted indications

for mechanical ventilatory support

include

a. PaO2 of less than 70 kPa and PaCO2 of

greater than 50 kPa while breathing

room air

b. Alveolar-arterial oxygen tension

difference of 150 kPa while breathing

100% O2

c. Vital capacity of 40-60 mL/kg

d. Respiratory rate greater than 35

breaths/min

e. A dead space:tidal volume ratio

(VD/VT) less than 0.6

53. In a hemolytic reaction caused

by an incompatible blood transfusion,

the treatment that is most

likely to be helpful is

a. Promoting a diuresis with 250 ml

of 50% mannitol

b. Treating anuria with fluid and

potassium replacement

c. Acidifying the urine to prevent

hemoglobin precipitation in the

renal tubules

d. Removing foreign bodies, such as

Foley catheters, which may cause

hemorrhagic complications

e. Stopping the transfusion immediately

54. Which of the following inhalation

anesthetics accumulates in

air-filled cavities during general

anesthesia?

a. Diethyl ether

b. Nitrous oxide

c. Halothane

d. Methoxyflurane

e. Trichloroethylene

Terms of Use

55. Major alterations in pulmonary

function associated with

adult respiratory distress syndrome

(ARDS) include

a. Hypoxemia

b. Increased pulmonary compliance

c. Increased resting lung volume

d. Increased functional residual

capacity

e. Decreased dead space ventilation

56. The curve depicted below

plots the normal relationship of

arterial PO2 and percentage of

hemoglobin saturation with other

variables controlled at pH 7.4,

PaCO2 40 kPa, temperature 37°C

(98.6°F), and hemoglobin 15 g/dL.

Which of the following statements

regarding this oxygen dissociation

relationship is true?

34 Surgery

50 100

PO2 torr

% Hb Saturation

100

50

a. Modest decrements of arterial PO2

have a major effect on alveolar oxygen

uptake

b. Modest decrements of hemoglobin

saturation have a major effect on

tissue oxygen uptake

c. The curve shifts to the left with acidosis

d. The curve shifts to the left following

banked blood transfusion

e. The curve is unaffected by chronic

lung disease

57. A 64-year-old man afflicted

with severe emphysema, who receives

oxygen therapy at home, is

admitted to the hospital because of

upper gastrointestinal bleeding.

The bleeding ceases soon after

admission, and the patient

becomes agitated and then disoriented;

he is given intramuscular

diazepam (Valium), 5 mg. Twenty

minutes later he is unresponsive.

Physical examination reveals a stuporous

but arousable man who has

papilledema and asterixis. Arterial

blood gases are pH 7.17; PO2 42

kPa; PCO2 95 kPa. The best immediate

therapy would be to

a. Correct hypoxemia with high-flow

nasal oxygen

b. Correct acidosis with sodium bicarbonate

c. Administer intravenous dexamethasone,

10 mg

d. Intubate the patient

e. Call for neurosurgical consultation

58. Dopamine is a frequently used

drug in critically ill patients because

a. At high doses it increases splanchnic

flow

b. At high doses it increases coronary

flow

c. At low doses it decreases heart rate

d. At low doses it lowers peripheral

resistance

e. It inhibits catecholamine release

59. Which statement regarding

transmission of viral illness through

homologous blood transfusion is

true?

a. The most common viral agent

transmitted via blood transfusion

in the United States is human

immune deficiency virus (HIV)

b. Blood is routinely tested for cytomegalovirus

(CMV) because CMV

infection is often fatal

c. The most frequent infectious complication

of blood transfusion continues

to be viral meningitis

d. Up to 10% of those who develop

posttransfusion hepatitis will develop

cirrhosis or hepatoma or both

e. The etiologic agent in posttransfusion

hepatitis remains undiscovered

Critical Care:Anesthesiology, Blood Gases, Respiratory Care 35

Items 60-61

A 68-year-old hypertensive

man undergoes successful repair

of a ruptured abdominal aortic

aneurysm. He receives 9 L Ringer's

lactate solution and 4 units of

whole blood during the operation.

Two hours after transfer to the surgical

intensive care unit, the following

hemodynamic parameters are

obtained:

• Systemic blood pressure (BP):

90/60 mm Hg

• Pulse rate: 110 beats/min

• Central venous pressure (CVP):

7 mm Hg

• Pulmonary artery pressure:

28/10 mm Hg

• Pulmonary capillary wedge pressure:

8 mm Hg

• Cardiac output: 1.9 L/min

• Systemic vascular resistance: 35

Woods units (normal is 24-30

Woods units)

• PaO2: 140 kPa (FiO2: 0.45)

• Urine output: 15 mL/h (specific

gravity: 1.029)

• Hematocrit: 35%

60. Propermanagementwouldnow

call for

a. Administration of a diuretic to

increase urine output

b. Administration of a vasopressor

agent to increase systemic blood

pressure

c. Administration of a fluid challenge

to increase urine output

d. Administration of a vasodiluting

agent to decrease elevated systemic

vascular resistance

e. A period of observation to obtain

more data

61. The patient then has an

improvement in all hemodynamic

parameters. However, 6 h later he

develops ST segment depression,

and a 12-lead cardiogram shows

anterolateral ischemia. New hemodynamic

parameters are obtained:

• Systemic BP: 70/40 mm Hg

• Pulse rate: 100 beats/min

• Central venous pressure (CVP):

18 cm H2O

• Pulmonary capillary wedge pressure

(PCWP): 25 mm Hg

• Cardiac output: 1.5 L/min

• Systemic vascular resistance: 25

Woods units

The single best pharmacologic

intervention would be

a. Sublingual nitroglycerin

b. Intravenous nitroglycerin

c. A short-acting beta blocker

d. Sodium nitroprusside

e. Dobutamine

36 Surgery

62. A 56-year-old man undergoes

a left upper lobectomy. An epidural

catheter is inserted for postoperative

pain relief. Ninety minutes

after the first dose of epidural morphine,

the patient complains of

itching and becomes increasingly

somnolent. Blood gas measurement

reveals the following: pH 7.24;

PaCO2 58; PaO2 100; HCO3− 28. Initial

therapy should include

a. Endotracheal intubation

b. Intramuscular diphenhydramine

(Benadryl)

c. Epidural naloxone

d. Intravenous naloxone

e. Alternative analgesia

63. If end-diastolic pressure is

held constant, increasing which of

the following will increase the cardiac

index?

a. Peripheral vascular resistance

b. Pulmonary wedge pressure

c. Heart rate

d. Systemic diastolic pressure

e. Viscosity of the blood

64. A 73-year-old woman with a

long history of heavy smoking

undergoes femoral artery-popliteal

artery bypass for resting pain in her

left leg. Because of serious underlying

respiratory insufficiency, she

continues to require ventilatory

support for 4 days after her operation.

As soon as her endotracheal

tube is removed, she begins complaining

of vague upper abdominal

pain. She has daily fever spikes to

39°C (102.2°F) and a leukocyte

count of 18,000/μL. An upper

abdominal ultrasonogram reveals a

dilated gallbladder, but no stones

are seen. A presumptive diagnosis

of acalculous cholecystitis is made.

You would recommend

a. Nasogastric suction and broadspectrum

antibiotics

b. Immediate cholecystectomy with

operative cholangiogram

c. Percutaneous drainage of the gallbladder

d. Endoscopic retrograde cholangiopancreatography

(ERCP) to

visualize and drain the common

bile duct

e. Provocation of cholecystokinin release

by cautious feeding of the

patient

Critical Care:Anesthesiology, Blood Gases, Respiratory Care 37

Items 65-67

A 32-year-old man undergoes a distal pancreatectomy, splenectomy,

and partial colectomy for a gunshot wound to the left upper quadrant of

the abdomen. One week later he develops a shaking chill in conjunction

with a temperature spike to 39.44°C (103°F). His blood pressure is 70/40

mm Hg with a pulse of 140 beats/min and his respiratory rate is 45

breaths/min. He is transferred to the ICU where he is intubated and a

Swan-Ganz catheter is placed.

65. Which of the following would be most consistent with this patient's

preintubation arterial blood gas measurement?

38 Surgery

pH PaCO2 PaO2

a. 7.31 48 61

b. 7.52 28 76

c. 7.45 40 77

d. 7.40 30 72

e. 7.40 48 94

66. Which of the following is consistent with the expected initial Swan-

Ganz catheter readings?

a. Cardiac output: 7.0 L/min

b. Peripheral vascular resistance: 1660 dynes

c. Pulmonary artery pressure: 50/20 mm Hg

d. Pulmonary capillary wedge pressure: 16 mm Hg

e. Central venous pressure: 18 mm Hg

67. Initial therapy for this patient would include

a. Furosemide

b. Propranolol

c. Sodium nitroprusside

d. Broad-spectrum antibiotics

e. Laparotomy

68. The preoperative characteristics

of patients likely to experience

postoperative ischemia after noncardiac

surgery include

a. Angina

b. More than three premature ventricular

contractions (PVCs) per minute

c. Dyspnea on exertion

d. Tricuspid regurgitation

e. Age greater than 60 years

69. Which statement regarding

local anesthetics is true?

a. When used for infiltration anesthesia,

the maximal safe total dose of

lidocaine is 3.0 mg per kilogram of

body weight

b. Addition of epinephrine (1:200,000)

to the solution of lidocaine, procaine,

or bupivacaine does not increase the

maximal safe total dose but increases

the duration of the block

c. Numerous individuals are hypersensitive

to local anesthetics

d. A local anesthetic in contact with a

nerve trunk will cause sensory loss

but not motor paralysis in the area

innervated

e. Rapid systemic administration of

local anesthetics may produce death

without signs of CNS stimulation

70. Compensatory mechanisms

during acute hemorrhage include

a. Decreased cerebral and coronary

blood flow

b. Decreased myocardial contractility

c. Renal and splanchnic vasodilation

d. Increased respiratory rate

e. Decreased renal sodium resorption

71. The correlation between pulmonary

capillary wedge pressure

(PCWP) and left ventricular enddiastolic

pressure (LVEDP) as measured

by pulmonary artery

catheterization may be adversely

affected by

a. Aortic stenosis

b. Aortic regurgitation

c. Coronary artery disease

d. Positive-pressure ventilation with

positive end-expiratory pressure/

continuous positive airway pressure

(PEEP/CPAP)

e. Bronchospasm

72. Which statement regarding

perioperative risk of stroke in

patients with a past history of

stroke is true?

a. The mortality after postoperative

stroke is high

b. Most postoperative strokes occur

directly after surgery and appear

related to operative events

c. The risk of stroke correlates with

the length of time since previous

stroke

d. General state of health and severity

of illness as measured by ASA classification

are significant predictors

of recurrent stroke

e. The risk of stroke correlates with a

history of multiple strokes or poststroke

transient ischemic attacks

(TIAs)

Critical Care:Anesthesiology, Blood Gases, Respiratory Care 39

73. An 18-year-old woman develops

urticaria and wheezing after an

injection of penicillin. Her blood

pressure is 120/60 mm Hg, heart

rate is 155 beats/min, and respiratory

rate is 30 breaths/min. Immediate

therapy should include

a. Intubation

b. Epinephrine

c. Beta blockers

d. Iodine

e. Fluid challenge

74. During blood transfusion,

clotting of transfused blood is associated

with

a. ABO incompatibility

b. Minor blood group incompatibility

c. Rh incompatibility

d. Transfusion through Ringer's lactate

e. Transfusion through 5% dextrose

and water

75. When an arterial blood gas

determination of PCO2 40 kPa is

obtained

a. There is probably a paradoxical

aciduria

b. Alveolar ventilation is adequate

c. Arterial PO2 will indicate the adequacy

of alveolar ventilation

d. Arterial PO2 will indicate the degree

of ventilation-perfusion mismatch

e. Arterial PO2 can be safely predicted

to exceed 90 kPa on room air

76. An obese 50-year-old woman

undergoes a laparoscopic cholecystectomy.

In the recovery room she

is found to be hypotensive and

tachycardic. Her arterial blood

gases reveal a pH of 7.29, partial

pressure of oxygen of 60 kPa, and

partial pressure of CO2 of 54 kPa.

The most likely cause of this

woman's problem is

a. Acute pulmonary embolism

b. CO2 absorption from induced

pneumoperitoneum

c. Alveolar hypoventilation

d. Pulmonary edema

e. Atelectasis from high diaphragm

77. Among patients who require

nutritional resuscitation in an

intensive care unit, the best evidence

that nutritional support is

adequate is

a. Urinary nitrogen excretion levels

b. Total serum protein level

c. Serum albumin level

d. Serum transferrin levels

e. Respiratory quotient

78. Paradoxical aciduria (the

excretion of acid urine in the presence

of metabolic alkalosis) may

occur in the presence of

a. Release of inappropriate antidiuretic

hormone

b. Severe crush injury

c. Acute tubular necrosis

d. Gastric outlet obstruction

e. An eosinophilic pituitary adenoma

40 Surgery

79. If a patient suffered a pulmonary

arterial air embolism during

an open thoracotomy, the

anesthesiologist's most likely observation

would be

a. Unexpected systemic hypertension

b. Rising right atrial filling pressures

c. Reduced systemic arterial oxygen

saturation

d. Rising systemic CO2 partial pressures

e. Falling end-tidal CO2

80. A 72-year-old man undergoes

resection of an abdominal aneurysm.

He arrives in the ICU with

a core temperature of 33°C

(91.4°F) and shivering. The physiologic

consequence of the shivering

a. Rising mixed venous oxygen saturation

b. Increased production of carbon

dioxide

c. Decreased consumption of oxygen

d. Rising base excess

e. Decreased minute ventilation

81. To prepare for operating on a

patient with a bleeding history

diagnosed as von Willebrand's disease

(recessive), you would give

a. High-purity factor VIII:C concentrates

b. Low-molecular-weight dextran

c. Fresh frozen plasma (FFP)

d. Cryoprecipitate

e. Whole blood

82. Which of the following clinical

situations is an indication for treatment

with extracorporeal membrane

oxygenation (ECMO)?

a. A 1-day-old, full-term, anencephalic

4-kg boy suffering from

meconium aspiration syndrome

and hypoxia

b. A 75-year-old man with Alzheimer's

disease, severe pneumonia,

and elevated pulmonary

arterial pressure

c. A neonate with a diagnosis of

severe pulmonary hypoplasia who

is in respiratory failure

d. A 5-year-old girl with rhabdomyosarcoma

metastatic to the

lungs

e. Preoperatively in a 3-day-old boy

with a congenital diaphragmatic

hernia

83. The accidental aspiration of

gastric contents into the tracheobronchial

tree should be initially

treated by

a. Tracheal intubation and suctioning

b. Steroids

c. Intravenous fluid bolus

d. Cricothyroidotomy

e. High positive end-expiratory pressure

Critical Care:Anesthesiology, Blood Gases, Respiratory Care 41

84. In performing a tracheostomy,

authorities agree that

a. The strap muscles should be

divided

b. The thyroid isthmus should be preserved

c. The trachea should be entered at

the second or third cartilaginous

d. Only horizontal incisions should

be used

e. Formal tracheostomy is preferable

to cricothyroidotomy as an emergency

procedure

85. If malignant hyperthermia is

suspected intraoperatively

a. Complete the procedure but pretreat

with dantrolene prior to future

elective surgery

b. Administer inhalational anesthetic

agents

c. Administer succinylcholine

d. Hyperventilate with 100% oxygen

e. Acidify the urine to prevent myoglobin

precipitation in the renal

tubules

86. Central venous pressure (CVP)

may be decreased by

a. Pulmonary embolism

b. Hypervolemia

c. Positive-pressure ventilation

d. Pneumothorax

e. Gram-negative sepsis

87. Characteristics of continuous

arteriovenous hemofiltration

(CAVH) in the treatment of surgical

patients with acute renal failure

include

a. CAVH is useful only in hemodynamically

stable patients

b. CAVH requires placement of largebore

(8 French) arterial and venous

catheters, usually in the femoral

vessels

c. CAVH is not effective in treating

hypervolemia

d. Continuous heparinization of the

patient who undergoes CAVH is

unnecessary

e. During CAVH, blood flow is maintained

by a mechanical extracorporeal

pump-oxygenator

88. Signs and symptoms of unsuspected

Addison's disease include

a. Hypothermia

b. Hypokalemia

c. Hyperglycemia

d. Hyponatremia

e. Hypervolemia

89. The etiologic factor implicated

in the development of pulmonary

insufficiency following major nonthoracic

trauma is

a. Aspiration

b. Atelectasis

c. Fat embolism syndrome

d. Fluid overload

e. Pneumonia

42 Surgery

90. For the severely traumatized

patient requiring airway management

a. Awake endotracheal intubation is

indicated in patients with penetrating

ocular injury

b. Steroids have been shown to be of

value in the treatment of aspiration

of acidic gastric secretions

c. The stomach may be assumed to be

empty only if a history is obtained

indicating no ingestion of food or

liquid during the prior 8 h

d. Intubation should be performed in

the emergency room if the patient

is unstable

e. Cricothyroidotomy is contraindicated

in the presence of maxillofacial

injuries

91. Treatment for clostridial myonecrosis

(gas gangrene) includes

which of the following measures?

a. Administration of an antifungal

agent

b. Administration of antitoxin

c. Wide debridement

d. Administration of hyperbaric oxygen

e. Early closure of tissue defects

92. An abnormal ventilationperfusion

ratio (Qs/Qr) in the postoperative

patient has been associated

with

a. Pulmonary thromboembolism

b. Lower abdominal surgery

c. Starvation

d. The upright position

e. Increased cardiac output

93. Correct statements concerning

drowning or near-drowning

include which of the following?

a. The prognosis for recovery of cerebral

function in affected persons is

better if submersion occurs in

warm water rather than extremely

cold water

b. A majority of victims will demonstrate

a severe metabolic alkalosis

c. Prompt administration of corticosteroids

to affected persons has

been shown to decrease the extent

of pulmonary membrane damage

d. Renal damage may occur in

affected persons as a result of

hemoglobinuria

e. The most important initial treatment

of drowning victims is emptying

the stomach of swallowed water

94. Spontaneous retroperitoneal

hemorrhage during anticoagulant

therapy

a. Is best confirmed by bleeding scan

b. Is equally likely with parenteral

and oral anticoagulants

c. May mimic an acute surgical

abdomen

d. Frequently requires laparotomy for

ligation of the bleeding site

e. Is seen in over 30% of patients

receiving long-term anticoagulation

Critical Care:Anesthesiology, Blood Gases, Respiratory Care 43

95. Correct statements concerning

smoke inhalation ("smoke poisoning")

include which of the following?

a. Smoke poisoning is a thermal

rather than chemical injury

b. Carbon monoxide levels are not

likely to be elevated unless there is

evidence of skin or oropharyngeal

burns

c. Chest x-rays during the early

postinhalation period show a characteristic

"ground glass" appearance

d. Damage to the upper respiratory

tract is common and is usually

found on laryngoscopy

e. Patients with elevated carboxyhemoglobin

levels should be hospitalized

for a minimum of 24 h

96. Indications for surgical intervention

to remove smuggled drug

packets that have been ingested

include

a. Refusal to take high doses of laxatives

b. Refusal to allow endoscopic

retrieval

c. Refusal to allow digital rectal disimpaction

d. Intraintestinal drug packets evident

on abdominal x-ray in an asymptomatic

smuggler

e. Signs of toxicity from leaking drug

packets

44 Surgery

DIRECTIONS: Each group of questions below consists of lettered

options followed by numbered items. For each numbered item, select the

appropriate lettered option(s). Each lettered option may be used once,

more than once, or not at all. Choose exactly the number of options

indicated following each item.

Items 97-99

Match the side effects below

with the appropriate anesthetic.

a. Nitrous oxide (N2O)

b. Halothane

c. Methoxyflurane

d. Enflurane

e. Morphine

97. Seizures (SELECT 1 AGENT)

98. Decreased peripheral resistance

(SELECT 1 AGENT)

99. Possible worsening of distention

in bowel obstruction (SELECT

1 AGENT)

Items 100-102

For each clinical problem

described below, select the appropriate

methods of physiologic monitoring.

a. Arterial catheterization

b. Central venous catheterization

c. Pulmonary artery catheterization

d. Ventilation monitoring

e. Blood gas monitoring

f. Intracranial pressure monitoring

g. Metabolic monitoring

h. Continuous ECG monitoring

100. A 74-year-old man has a 5-h

elective operation for repair of an

abdominal aortic aneurysm. He

had a small myocardial infarction 3

years earlier. In the ICU on the first

postoperative day, he may be ready

for extubation and is receiving

dobutamine by continuous infusion.

(SELECT 5 METHODS)

101. A 22-year-old rugby player is

rushed to the operating room

because of abdominal tenderness,

tachycardia, and hypotension following

a collision with another

player. He is otherwise healthy. At

exploration a significant hemoperitoneum

is found due to a ruptured

spleen. (SELECT 3 METHODS)

102. A comatose 28-year-old

woman who sustained a depressed

skull fracture in an automobile collision

receives enteral nutrition via

a nasoenteric feeding tube. She has

been unconscious for 6 wk. Her

vital signs are stable and she

breathes room air. Following her

initial decompressive craniotomy,

she has returned to the operating

room twice for intracranial bleeding.

(SELECT 3 METHODS)

Critical Care:Anesthesiology, Blood Gases, Respiratory Care 45

46 Surgery

Items 103-105

For each test listed below, select the coagulation factors whose functions

are measured.

a. Factor II

b. Factor V

c. Factor VII

d. Factor VIII

e. Factor IX

f. Factor X

g. Factor XI

h. Factor XII

i. Platelets

j. Fibrinogen

103. Prothrombin time (SELECT 5 FACTORS)

104. Partial thromboplastin time (SELECT 8 FACTORS)

105. Bleeding time (SELECT 1 FACTOR)

CRITICAL CARE:

ANESTHESIOLOGY, BLOOD

GASES, RESPIRATORY CARE

Answers

51. The answer is c. (Greenfield, 2/e, p 1984.) Although hypoventilation,

incomplete oxygen diffusion, and pulmonary shunts all are causes of

hypoxemia, the most common cause is ventilation-perfusion inequality.

The mismatch of ventilation and blood flow occurs to some degree in the

normal upright lung but may become extreme in the diseased lung. The

three indices used to measure ventilation-perfusion inequality are alveolararterial

PO2 difference, physiologic shunt (venous admixture), and alveolar

dead space. Elevated 2,3-diphosphoglycerate (2,3-DPG) levels shift the

oxygen dissociation curve to the right and thereby augment tissue oxygenation.

This elevation does not result in hypoxemia.

52. The answer is d. (Greenfield, 2/e, pp 221-225.) Anticipation and early

aggressive treatment of pulmonary insufficiency by mechanical ventilatory

support are critical in managing the seriously ill patient. Readily measured

changes that can be used to determine either the need for intubation or the

appropriate time for weaning from mechanical respiratory support include

arterial blood gas levels, dead space-tidal volume ratio (VD/VT), alveolararterial

oxygen tension difference [(A-a)DO2], vital capacity, and respiratory

rate. Indications for mechanical ventilation include a respiratory rate over

35 breaths/min, vital capacity less than 15 mL/kg, (A-a)DO2 greater than

350 kPa after 15 min on 100% oxygen, VD/VT greater than 0.6, PaO2 less

than 60 kPa, and PaCO2 greater than 60 kPa.

53. The answer is e. (Schwartz, 7/e, pp 97-98.) Whenever a hemolytic

reaction caused by an incompatible blood transfusion is suspected, the

transfusion should be stopped immediately. A Foley catheter should be

inserted, and hourly urine output should be monitored. Renal damage

caused by precipitation of hemoglobin in the renal tubules is the major

47

serious consequence of hemolysis. This precipitation is inhibited in an

alkaline environment and is promoted in an acid environment. Stimulating

diuresis with 100 mL of 20% mannitol and alkalinizing the urine with 45

meq sodium bicarbonate intravenously are indicated procedures. Fluid

and potassium intake should be restricted in the presence of severe oliguria

or anuria.

54. The answer is b. (Greenfield, 2/e, p 439.) Nitrous oxide (N2O) has a

low solubility compared with other inhalation anesthetics. Its blood:gas

partition coefficient is 0.47, and it is 30 times more soluble in blood than

is nitrogen (N2). N2O is also the only anesthetic gas less dense than air. As

a result of these properties, N2O may cause progressive distention of airfilled

spaces during prolonged anesthesia. This can lead to undesirable situations

whenever there is a pneumothorax or intestinal obstruction or

when procedures like pneumoventriculography (in which the intracranial

air space is not free to expand in response to the diffusion of gas into the

ventricles) are performed. In each of these cases the N2O diffuses into the

gas-filled compartment faster than N2 can diffuse out. Since the typical

mixture of ingested air (or pneumothorax air) is 80% N2 and the usual mixture

of nitrous oxide anesthetic gas is 80% N2O, rapid increase in the size

of gas-filled chambers with potentially serious consequences may occur.

55. The answer is a. (Schwartz, 7/e, pp 693-694.) Adult respiratory distress

syndrome (ARDS) has been called "shock lung" or "traumatic wet

lung" and occurs under a variety of circumstances. Clinically, its manifestations

can range from minimal dysfunction to unrelenting pulmonary failure.

Three major physiologic alterations include (1) hypoxemia usually

unresponsive to elevations of inspired oxygen concentration; (2) decreased

pulmonary compliance, as the lungs become progressively "stiffer" and

harder to ventilate; and (3) decreased functional residual capacity. Progressive

alveolar collapse occurs owing to leakage of protein-rich fluid into the

interstitium and the alveolar spaces with the subsequent radiologic picture

of diffuse, fluffy infiltrates bilaterally. Ventilatory abnormalities develop

that result in shunt formation, decreased resting lung volume, and

increased dead space ventilation.

56. The answer is d. (Schwartz, 7/e, pp 496-497.) The shape of the oxygen

dissociation curve translates into several physiologic advantages. The

48 Surgery

relatively flat slope above a PO2 of 50 pKa means that, in this region of the

curve, hemoglobin saturation decreases slightly with decrements in PO2;

loading of oxygen at the alveolar level is therefore affected minimally with

mild to moderate degrees of hypoxemia. The steeper slope at the lower end

of the curve means that, as the hemoglobin becomes desaturated, arterial

PO2 drops only minimally, and a gradient that favors oxygen diffusion into

tissue cells is maintained. Acidosis, a rise in PaCO2, and elevation of temperature

all shift the curve to the right, which enhances tissue oxygen uptake.

Red blood cell organic phosphates, particularly 2,3-diphosphoglycerate

(2,3-DPG), also affect the dissociation curve. Banked blood, being low in

2,3-DPG, shifts the curve to the left and therefore decreases tissue oxygen

uptake. 2,3-DPG levels increase with chronic hypoxia. Chronic lung disease,

therefore, results in a shift of the curve to the right, which enhances

oxygen delivery to peripheral tissues.

57. The answer is d. (Schwartz, 7/e, pp 59-61.) The patient presented in

the question is suffering from acute, life-threatening respiratory acidosis

that has been compounded, if not produced, by the injudicious administration

of a central nervous system depressant. While hypoxemia must also

be corrected, the immediate task is to correct the acidosis caused by carbon

dioxide accumulation. Both disturbances can be resolved by skillful endotracheal

intubation and by ventilatory support. Sodium bicarbonate and

high-flow nasal oxygen would both be inappropriate. Bicarbonate should

not be administered because buffer reserves are already adequate (serum

bicarbonate is still 34 meq/L based on the Henderson-Hasselbalch equation).

Nasal oxygen administration is not warranted because both acidemia

and hypoxemia are themselves potent stimulants to spontaneous ventilation.

Headache, confusion, and papilledema are all signs of acute carbon

dioxide retention and do not imply the presence of a structural intracranial

lesion.

58. The answer is b. (Schwartz, 7/e, pp 454-455.) Dopamine has a variety

of pharmacologic characteristics that make it useful in critically ill

patients. In low doses (1-5 mg/kg/min), dopamine affects primarily the

dopaminergic receptors. Activation of these receptors causes vasodilation

of the renal and mesenteric vasculature and mild vasoconstriction of the

peripheral bed, which thereby redirects blood flow to kidneys and bowel.

At these low doses the net effect on the overall vascular resistance may be

Critical Care Answers 49

slight. As the dose rises (2-10 mg/kg/min), β1-receptor activity predominates

and the inotropic effect on the myocardium leads to increased cardiac

output and blood pressure. Above 10 mg/kg/min, α-receptor stimulation

causes peripheral vasoconstriction, shifting of blood from extremities to

organs, decreased kidney function, and hypertension. At all doses, the

diastolic blood pressure can be expected to rise; since coronary perfusion

is largely a result of the head of pressure at the coronary ostia, coronary

blood flow should be increased.

59. The answer is d. (Goodnough, Am J Surg 159:602-609, 1990.)

Cytomegalovirus (CMV) is harbored in blood leukocytes. CMV infection is

endemic in the United States, and its prevalence increases steadily with

age. While acute CMV infection may cause transient fever, jaundice, and

hepatosplenomegaly in cases of large blood donor exposures, posttransfusion

CMV infection (seroconversion) is not a significant clinical problem in

immunocompetent recipients, and therefore blood is not routinely tested

for the presence of CMV. Posttransfusion non-A, non-B hepatitis, however,

not only represents the most frequent infectious complication of transfusion,

but is associated with an incidence of chronic active hepatitis up to

16% and an 8-10% incidence of cirrhosis or hepatoma or both. The etiologic

agent in over 90% of cases of posttransfusion hepatitis has been identified

as hepatitis C.

60. The answer is c. (Sabiston, 15/e, pp 81-84.) A ruptured abdominal

aneurysm is a surgical emergency often accompanied by serious hypotension

and vascular collapse before surgery and massive fluid shifts with

renal failure after surgery. In this case, all the hemodynamic parameters

indicate inadequate intravascular volume, and the patient is therefore suffering

from hypovolemic hypotension. The low urine output indicates

poor renal perfusion, while the high urine specific gravity indicates adequate

renal function with compensatory free water conservation. The

administration of a vasopressor agent would certainly raise the blood pressure,

but it would do so by increasing peripheral vascular resistance and

thereby further decrease tissue perfusion. The deleterious effects of shock

would be increased. A vasodilating agent to lower the systemic vascular

resistance would lead to profound hypotension and possibly complete vascular

collapse because of pooling of an already depleted vascular volume.

This patient's blood pressure is critically dependent on an elevated systemic

50 Surgery

vascular resistance. To properly treat this patient, rapid fluid infusion and

expansion of the intravascular volume must be undertaken. This can be

easily done with lactated Ringer's solution or blood (or both) until

improvements in such parameters as the pulmonary capillary wedge pressure,

urine output, and blood pressure are noted.

61. The answer is e. (Sabiston, 15/e, pp 84-86.) This patient has developed

pump failure due to a combination of preexisting coronary artery

occlusive disease and high preload following a fluid challenge; afterload

remains moderately high as well because of systemic vasoconstriction in

the presence of cardiogenic shock. Poor myocardial performance is

reflected in the low cardiac output and high pulmonary capillary wedge

pressure. Therapy must be directed at increasing cardiac output without

creating too high a myocardial oxygen demand on the already failing heart.

Administration of nitroglycerin could be expected to reduce both preload

and afterload, but if it is given without an inotrope it would create unacceptable

hypotension. Nitroprusside similarly would achieve afterload

reduction but would result in hypotension if not accompanied by an

inotropic agent. A beta blocker would act deleteriously by reducing cardiac

contractility and slowing the heart rate in a setting in which cardiac output

is likely to be rate dependent. Dobutamine is a synthetic catecholamine

that is becoming the inotropic agent of choice in cardiogenic shock. As a

β1-adrenergic agonist, it improves cardiac performance in pump failure

both by positive inotropy and peripheral vasodilation. With minimal

chronotropic effect, dobutamine only marginally increases myocardial oxygen

demand.

62. The answer is d. (Thoren, Anesth Analg 67:687, 1988.) Thoracic

epidural narcotics have become an increasingly popular means of postoperative

pain relief in thoracic and upper abdominal surgery. Local action on

gamma opiate receptors ensures pain relief and consequent improvement

in respiration without vasodilation or paralysis. The less lipid-soluble opiates

are effective for long periods. Their slow absorption into the circulation

also ensures a low incidence of centrally mediated side effects, such as

respiratory depression or generalized itching. When these do occur, the

intravenous injection of an opiate antagonist is an effective antidote. The

locally mediated analgesia is not affected. One poorly understood side

effect, which is apparently unrelated to systemic levels, is a profound

Critical Care Answers 51

reduction in gastric activity. This may be an important consideration after

thoracic surgery when an early resumption of oral intake is anticipated.

63. The answer is c. (Schwartz, 7/e, p 849.) The cardiac index is computed

by dividing the cardiac output by the body surface area; the cardiac

output is the product of the stroke volume and the heart rate [CI =

CO/BSA; CO = SV × HR; therefore, CI = (SV × HR)/BSA]. An increased

heart rate will directly increase the cardiac output and cardiac index. The

remaining choices in the question will either decrease or not affect the

stroke volume and consequently will not increase the cardiac index.

64. The answer is c. (Schwartz, 7/e, pp 1452-1454.) The development of

acute postoperative cholecystitis is an increasingly recognized complication

of the severe illnesses that precipitate admissions to the intensive care

unit. The causes are obscure but probably lead to a common final pathway

of gallbladder ischemia. The diagnosis is often extremely difficult because

the signs and symptoms may be those of occult sepsis. Moreover, the

patients are often intubated, sedated, or confused as a consequence of the

other therapeutic or medical factors. Biochemical tests, though frequently

revealing abnormal liver function, are nonspecific and nondiagnostic. Bedside

ultrasonography is usually strongly suggestive of the diagnosis when a

thickened gallbladder wall or pericholecystic fluid is present, but radiologic

findings may also be nondiagnostic. If the diagnosis is delayed, mortality

and morbidity are very high. Percutaneous drainage of the gallbladder

is usually curative of acalculous cholecystitis and affords stabilizing palliation

if calculous cholecystitis is present. Some authors have recommended

prophylactic percutaneous drainage of the gallbladder under CT guidance

in any ICU patient who is failing to thrive or has other signs of low-grade

sepsis after appropriate therapy for the primary illness has been provided.

The distractor items in the question are all either too aggressive to be safely

done in critically ill patients or too cautious for a patient with a potentially

fatal complication.

65-67. The answers are 65-b, 66-b, 67-d. (Schwartz, 7/e, pp 115-120.)

The case presented is most consistent with septic shock from a postoperative

intraabdominal abscess. In the early phase of septic shock the respiratory

profile is characterized by mild hypoxia with a compensatory

hyperventilation and respiratory alkalosis. Hemodynamically, a hyperdy-

52 Surgery

namic state is seen with an increase in cardiac output and a decrease in

peripheral vascular resistance in the face of relatively normal central pressures.

Initial therapy is aimed at resuscitation and stabilization. This

includes fluid replacement and vasopressors as well as antibiotic therapy

aimed particularly at gram-negative rods and anaerobes for patients with

presumed intraabdominal collections, especially after bowel surgery.

Laparotomy and drainage of a collection is the definitive therapy but

should await stabilization of the patient and confirmation of the presence

and location of such a collection.

68. The answer is c. (Charlson, Ann Surg 210:637-648, 1989.) The landmark

study by Goldman in 1978 identified cardiac risk factors in noncardiac

surgical patients that included previous infarction (particularly

infarction within 6 mo, but with increased risk continuing for life), functional

impairment such as dyspnea on exertion, age over 70 years, mitral

regurgitation, more than five premature ventricular contractions (PVCs)

per minute, and a tortuous or calcified aorta. Angina alone was not a risk

factor. Subsequent studies by others have differed regarding the importance

of several of these factors, which probably reflects different comorbid

characteristics in the study populations (e.g., diabetes and hypertension).

Additional predictors of perioperative cardiac risk that achieved significance

in some studies but not in others include cardiomegaly, upper

abdominal or intrathoracic surgery, and intraoperative hypotension.

69. The answer is e. (Schwartz, 7/e, p 681.) The maximal safe total dose

of lidocaine administered to a 70-kg man is 4.5 mg/kg, or approximately

30-35 mL of a 1% solution. The addition of epinephrine to lidocaine,

procaine, or bupivacaine not only doubles the duration of infiltration

anesthesia, but increases by one-third the maximal safe total dose by

decreasing the rate of absorption of drug into the bloodstream.

Epinephrine-containing solutions should not, however, be injected into

tissues supplied by end arteries (e.g., fingers, toes, ears, nose, penis).

Hypersensitivity to local anesthetics is uncommon and occurs most

prominently with anesthetics of the ester type (procaine, tetracaine).

While small nerve fibers seem to be most susceptible to the action of local

anesthetics, these agents act on any part of the nervous system and on

every type of nerve fiber. CNS toxicity usually appears as stimulation

followed by depression, probably because of an early selective depression

Critical Care Answers 53

of inhibitory neurons; with a massive overdose, all neurons may be

depressed simultaneously.

70. The answer is d. (Schwartz, 7/e, pp 103-113.) Acute hemorrhage

triggers the potent vasopressor activity of both angiotensin and vasopressin

to increase blood flow to the heart and brain via selective vasoconstriction

of the skin, kidneys, and splanchnic organs. Adrenergic discharge also

results in selective vasoconstriction of skin, renal, and splanchnic vessels.

Myocardial contractility and heart rate are increased, with a resultant

increased cardiac output. Hyperventilation is the typical response to the

metabolic (lactic) acidosis associated with hemorrhagic shock and hypoperfusion.

Aldosterone release, with subsequent increased renal sodium

resorption, is mediated by angiotensin II and ACTH, which prevents further

intravascular depletion.

71. The answer is e. (Greenfield, 2/e, pp 195-197.) When a Swan-Ganz

pulmonary artery catheter is in the wedge position, i.e., isolating the pulmonary

arterial system from the pulmonary capillaries, the measured pulmonary

capillary wedge pressure (PCWP) is usually equivalent to both the

left atrial pressure (LAP) and the left ventricular end-diastolic pressure

(LVEDP). Pathologic processes in the pulmonary vasculature and heart

valves, however, may alter this relationship. Pulmonary vasoocclusive disease

may elevate the PCWP independently of the LAP or LVEDP. Bronchospasm

affecting the airway but not the pulmonary vasculature should

not affect the validity of Swan-Ganz catheter readings. Mitral stenosis and

regurgitation cause increased LAP and PCWP, which result in an overestimated

LVEDP. However, aortic stenosis and regurgitation elevate the PCWP,

LAP, and LVEDP equally. Accurate measurement of PCWP by a Swan-Ganz

catheter may not be possible in the presence of positive airway pressure

with PEEP/CPAP; transmission of the positive airway pressure to the pulmonary

microvasculature via the alveoli, especially in the upper lung

zones, results in measurement of alveolar pressure rather than LAP or

LVEDP. Coronary artery disease does not affect the relationship between

PCWP, LAP, and LVEDP.

72. The answer is a. (Landercasper, Arch Surg 125:986-989, 1990.) In an

8-year, retrospective study of 173 consecutive patients with a documented

medical history of stroke who underwent subsequent general anesthesia

54 Surgery

and surgery (excluding cardiac, cerebrovascular, and neurological surgery),

5 patients (2.9%) had documented postoperative strokes from 3 to 21 days

(mean 12.2 days) after surgery. The risk of stroke did not correlate with

age, sex, history of multiple strokes or poststroke transient ischemic attacks

(TIAs), ASA classification, aspirin use, coronary artery disease, peripheral

vascular disease, intraoperative blood pressure, time since previous stroke,

or cause of previous stroke. The risk of recurrent stroke appears to be comparable

with that of surgical patients who do not have a history of prior

stroke and are undergoing cardiac and peripheral vascular surgery. Most

recurrent strokes occur many hours to days following surgery and do not

appear to be directly related to operative events. The mortality after postoperative

stroke is high.

73. The answer is b. (Schwartz, 7/e, pp 211-212.) This patient is having

an anaphylactoid reaction with destabilization of the cardiovascular and

respiratory systems. Anaphylactoid reactions are most commonly caused

by iodinated contrast media, β-lactam antibiotics (e.g., penicillin), and

Hymenoptera stings. Manifestations of anaphylactoid reactions include

both the lethal (bronchospasm, laryngospasm, hypotension, dysrhythmia)

and the nonlethal (pruritus, urticaria, syncope, weakness, and seizure).

Epinephrine is the initial treatment for laryngeal obstruction and bronchospasm,

followed by histamine antagonists (H1 and H2 blockers), aminophylline,

and hydrocortisone. Vasopressors and fluid challenges may be

given for shock. Conscious patients are usually stabilized with injected or

inhaled epinephrine, while unconscious patients and those with refractory

hypotension or hypoxia should be intubated.

74. The answer is d. (Schwartz, 7/e, pp 97-98.) Most transfusion reactions

are hemolytic and are due to clerical errors that result in administration

of blood with major (ABO) and minor antigen incompatibility.

Interestingly, Rh incompatibility is not associated with intravascular

hemolysis. Administration of blood through hypotonic solutions such as

5% dextrose and water results in swelling of the erythrocytes and hemolysis.

Calcium-containing solutions such as Ringer's lactate cause clotting

within the intravenous line rather than hemolysis and may lead to pulmonary

embolism. Delayed transfusion reactions, caused by a presumed

anamnestic immune response that occurs 3-21 days after blood is infused,

result in a hemolytic anemia.

Critical Care Answers 55

75. The answer is b. (Schwartz, 7/e, pp 494-496.) Because of the highly

efficient diffusion characteristics of the gas carbon dioxide, PaCO2 levels are

reliable indicators of adequacy of alveolar ventilation. A PaCO2 of 40 kPa is

the normal value. Paradoxical aciduria occurs when hypokalemic metabolic

alkalosis is present as the kidney excretes hydrogen ion in an effort to

conserve potassium ion. Though a PaCO2 of 40 kPa is not incompatible with

metabolic alkalosis, it would ordinarily be higher as the patient tries to conserve

carbolic acid by hypoventilating to compensate. PaO2 levels are influenced

by so many other variables (e.g., age, concentration of inspired O2,

altitude) that no inferences can be made about adequacy of alveolar ventilation

from PaO2 alone, nor can PaO2 be safely predicted by the presence of

normocarbia. The ventilation-perfusion mismatch is a reflection of the gradient

between alveolar and arterial oxygen tension in relationship to percentage

of inspired O2.

76. The answer is c. (Schwartz, 7/e, pp 494-496.) Because of the ease

with which carbon dioxide diffuses across the alveolar membranes, the

PaCO2 is a highly reliable indicator of alveolar ventilation. In this postoperative

patient with respiratory acidosis and hypoxemia, the hypercarbia is

diagnostic of alveolar hypoventilation. Acute hypoxemia can occur with

pulmonary embolism, pulmonary edema, and significant atelectasis, but in

all those situations the CO2 partial pressures should be normal or reduced

as the patient hyperventilates to improve oxygenation. The absorption of

gas from the peritoneal cavity may affect transiently the PaCO2, but should

have no effect on oxygenation.

77. The answer is c. (Schwartz, 7/e, pp 36-46.) The serum albumin level

provides a rough estimate of protein nutritional adequacy. The accuracy of

this estimate is affected by the long half-life of albumin (3 wk) and vagaries

of hemodilution. The acute-phase serum proteins have a very short half-life

(hours) and may also provide good short-term indications of nutritional

status. Transferrin is one of these acute-phase proteins, but unfortunately

its levels too are influenced by changes in intravascular volume and, along

with the other acute-phase reactants, rise nonspecifically during acute illness.

All the listed responses provide some useful information about nutrition

and adequacy of replacement.

78. The answer is d. (Schwartz, 7/e, pp 60-62.) The body has elaborate

mechanisms to compensate for metabolic acidosis. Not only do most body

56 Surgery

functions work better in an acidotic state, the patient is able to move

toward correction of the pH by excreting acid urine and by hyperventilating

to "blow off" carbonic acid. On the other hand, we are poorly equipped

to deal with metabolic alkalosis. We cannot hold our breath to save acid

since the respiratory center overrides our efforts as the PaCO2 rises and the

PaO2 falls. The kidney cannot make urine under any circumstance that is

very far above normal pH. In the subtraction alkalosis that accompanies

gastric outlet obstruction with loss of gastric acid by vomiting or suction,

the potassium depletion and volume deficits provoke exchange of sodium

for hydrogen ion in the distal tubule with resultant exacerbation of the

metabolic alkalosis. All the other conditions listed would be expected to

produce acidosis; consequently, acid urine would not be paradoxical.

79. The answer is e. (Schwartz, 7/e, pp 159-161.) Air carried into the

pulmonary arterial vasculature creates an abnormal blood-air interface that

leads to denaturing of plasma proteins and creates amorphous proteinaceous

and cellular debris and endothelial injury. The ensuing increased

capillary permeability results in alveolar flooding. The occlusion of pulmonary

vessels increases the proportion of ventilated but underperfused

alveoli. The increment in dead space results in a drop in end-tidal carbon

dioxide.

80. The answer is b. (Schwartz, 7/e, p 447.) Shivering is the physiologic

effort of the body to generate heat to maintain the core temperature. In

healthy persons, shivering increases the metabolic rate by 3-5 times and

results in increased oxygen consumption and carbon dioxide production.

In critically ill patients these metabolic consequences are almost always

counterproductive and should be prevented with other means employed to

correct systemic hypothermia. In the presence of vigorous shivering, oxygen

debt in the muscles and lactic acidemia develop.

81. The answer is d. (Schwartz, 7/e, pp 78-84.) von Willebrand disease is

similar to true hemophilia in frequency of occurrence. It is being diagnosed

more commonly today because of more reliable assays for factor VIII. This

autosomal dominant disorder (recessive transmission can occur) is characterized

by a diminution in factor VIII:C (procoagulant) activity. The reduction

in activity is not as great as in classic hemophilia, and the clinical

manifestations are more subtle. These manifestations are often overlooked

until an episode of trauma or surgery makes them apparent. Treatment

Critical Care Answers 57

requires correcting the bleeding time and providing factor VIII R:WF (the

von Willebrand factor). Only cryoprecipitate is reliably effective. Highpurity

factor VIII:C concentrates, effective in hemophilia, lack the von

Willebrand factor and are, consequently, undependable.

82. The answer is e. (Schwartz, 7/e, 1720-1721.) Extracorporeal membrane

oxygenation (ECMO) is a form of cardiopulmonary support that is

useful in the setting of potentially reversible pulmonary or cardiac disease.

Treatment of meconium aspiration syndrome, sepsis, pneumonia, and congenital

diaphragmatic hernia (pre- or postoperatively) are thus appropriate

uses. The technique is also applicable in some circumstances as a bridge to

cardiac or lung transplantation since the outlook for survival is quite good

if the child can be maintained in a good physiological state until donor

organs are available. Hypoplastic lungs do not have enough surface area to

perform adequate gas exchange and are unlikely to mature to a point where

they can sustain life. Babies with hypoplastic lungs will be bypass dependent

for life and consequently are not candidates for institution of ECMO

therapy.

83. The answer is a. (Schwartz, 7/e, p 458.) Gastric aspiration is best

treated by tracheal suctioning, oxygen, and positive-pressure ventilation.

Bronchoscopy is helpful if particulate matter is causing bronchial obstruction

or if the vomitus is found to contain particulate material. Bronchial

lavage is no longer recommended, and steroids have not been shown to be

of value. Fluids should be given sparingly because hypervolemia will

worsen the risk of pulmonary edema following aspiration. Tracheostomy

may be indicated for long-term airway management in obtunded or otherwise

severely debilitated patients; however, initial control of the airway

should be by orotracheal intubation whenever possible. High positive endexpiratory

pressure is not required unless respiratory failure develops.

84. The answer is c. (Schwartz, 7/e, p 156.) Although tracheostomy is

occasionally an emergency procedure, it can be more effectively performed

in an operating room where hemostasis and antisepsis are readily achieved.

Most authorities recommend a horizontal incision; however, limited direct

midline incisions have the advantage of not opening any unnecessary tissue

planes and perhaps reducing the incidence of bleeding complications.

Both approaches have advocates. In either case, the skin incision is made

58 Surgery

just below the cricoid cartilage, the strap muscles are spared and retracted,

the thyroid isthmus is divided if necessary, and the trachea is entered at the

second tracheal ring. The second and third tracheal rings are incised vertically,

allowing placement of the tracheostomy tube. The first tracheal ring

and the cricoid cartilage must be left intact.

85. The answer is d. (Schwartz, 7/e, pp 448, 499.) The cause of malignant

hyperthermia is unknown, but it is associated with inhalational anesthetic

agents and succinylcholine. It may develop in an otherwise healthy person

who has tolerated previous surgery without incident. It should be suspected

in the presence of a history of unexplained fever, muscle or connective

tissue disorder, or a positive family history (evidence suggests an

autosomal dominant inheritance pattern). In addition to fever during anesthesia,

the syndrome includes tachycardia, increased O2 consumption,

increased CO2 production, increased serum K+, myoglobinuria, and acidosis.

Rigidity rather than relaxation following succinylcholine injection may

be the first clue to its presence. Treatment of malignant hyperthermia

should include prompt conclusion of the operative procedure and cessation

of anesthesia, hyperventilation with 100% oxygen, and administration

of intravenous dantrolene. The urine should be alkalinized to protect the

kidneys from myoglobin precipitation. If reoperation is necessary, one

should premedicate heavily, alkalinize the urine, and avoid depolarizing

agents such as succinylcholine. Pretreatment for 24 h with dantrolene is

helpful; it is thought to act directly on muscle fiber to attenuate calcium

release.

86. The answer is e. (Schwartz, 7/e, pp 487-493.) Determination of CVP

is an integral part of the overall hemodynamic assessment of the patient.

This pressure can be affected by a variety of factors including those of cardiac,

noncardiac, and artifactual origin. Venous tone, right ventricular

compliance, intrathoracic pressure, and blood volume all influence CVP.

Vasoconstrictor drugs, positive pressure, ventilation (with and without

PEEP), mediastinal compression, and hypervolemia all increase CVP. Acute

pulmonary embolism, when clinically significant, elevates CVP by causing

right ventricular overload and increased right atrial pressure. Sepsis, on the

other hand, decreases CVP through both the release of vasodilatory mediators

and the loss of intravascular plasma volume due to increased capillary

permeability.

Critical Care Answers 59

87. The answer is b. (Greenfield, 2/e, pp 238-239.) Continuous arteriovenous

hemofiltration (CAVH) is a relatively new method of therapy for

acute renal failure in the intensive care unit. Continuous blood flow is

maintained by the hydrostatic pressure gradient between an inflowing arterial

cannula and the venous cannula that returns blood to the patient. The

blood passes through an extracorporeal membrane, which clears an ultrafiltrate

up to 12 L per day. This volume is replaced with an intravenous

solution at a rate that achieves the desired fluid balance. CAVH in the surgical

patient with acute renal failure allows a slow and continuous removal

of fluid and is particularly advantageous in the volume-overloaded patient.

Unlike traditional hemodialysis, it can be used over a wide range of blood

pressures in the unstable patient. Solutes (such as urea nitrogen and potassium)

that are not in the replacement intravenous fluid are also cleared.

The main complications associated with CAVH relate to vascular access

problems: arterial thrombosis, aneurysm, fistula formation, and infection.

Anticoagulation, with the concomitant bleeding risks, must be maintained

to prevent thrombosis of the filter and cannulae. The potential for electrolyte

imbalance during long-term CAVH requires careful monitoring.

88. The answer is d. (Greenfield, 2/e, pp 204-205.) Clinical manifestations

of adrenocortical insufficiency include hyperkalemia, hyponatremia,

hypoglycemia, fever, weight loss, and dehydration. There is excessive

sodium loss in the urine, contraction of the plasma volume, and perhaps

hypotension or shock. Classic hyperpigmentation is present in chronic

Addison's disease only. Addison's disease may present in newborns as a

congenital atrophy, as an insidious chronic state often due to tuberculosis,

as an acute dysfunction secondary to trauma or adrenal hemorrhage, or as

a semiacute adrenal insufficiency seen during stress or surgery. In this last

instance, signs and symptoms include nausea, lassitude, vomiting, fever,

progressive salt wasting, hyperkalemia, and hypoglycemia. It may be confirmed

by measurements of urinary Na+ loss and absence of response to

ACTH.

89. The answer is c. (Schwartz, 7/e, pp 693-694.) Posttraumatic pulmonary

insufficiency in the absence of significant thoracic trauma has been

attributed to a wide variety of etiologic agents, including aspiration, simple

atelectasis, lung contusion, fat embolism, pneumonia, pneumothorax, pulmonary

edema, and pulmonary thromboembolism. In a landmark mono-

60 Surgery

graph entitled Respiratory Distress Syndrome of Shock and Trauma, Blaisdell

and Lewis identified fat embolism syndrome as the etiologic factor. The

mechanism of this condition appears to be pulmonary alveolar injury due

to the mobilization of free fatty acids in the blood as an adrenergic response

to trauma, rather than pulmonary injury from embolization of fat globules

from fractured bones, as was originally thought.

90. The answer is d. (Sabiston, 15/e, p 296.) Securing a stable airway is

one of the most fundamental and important aspects of the management of

the severely injured patient. The level of control required will vary from a

simple oropharyngeal airway to tracheostomy, depending on the clinical

situation. Full control of the airway should be secured in the emergency

room if the patient is unstable. Endotracheal intubation will usually be the

method chosen, but one should be prepared to do a tracheotomy if

attempts at peroral or pernasal intubation are failing or are impractical

because of maxillofacial injuries. The most dangerous period is just prior to

and during the initial attempts to get control of the airway. Manipulation of

the oronasopharynx may provoke combative behavior or vomiting in a

patient already confused by drugs, alcohol, hypoxia, or cerebral trauma.

The risk of aspiration is high during these initial attempts, and one should

make no assumptions about the state of the contents of the patient's stomach.

Antacids are recommended just prior to the intubation attempt, if feasible.

Although steroids have been recommended in the past, they are no

longer considered of value in the management of aspiration of acidic gastric

juice. The best management requires prevention of the complication of

aspiration. In a reasonably cooperative patient, awake intubation with topical

anesthesia may help to avoid some of the risks of hypotension, arrhythmia,

and aspiration associated with the induction of anesthesia. If awake

intubation is inappropriate, then an alternative is rapid-sequence induction

with a thiobarbiturate followed by muscle paralysis with succinylcholine. If

elevated intracranial pressure is suspected, or if a penetrating eye injury

exists, awake intubation is contraindicated.

91. The answer is c. (Sabiston, 15/e, pp 269-270.) Necrotizing skin and

soft tissue infections may produce insoluble gases (hydrogen, nitrogen,

methane) through anaerobic bacterial metabolism. While the term "gas

gangrene" has come to imply clostridial infection, gas in tissues is more

likely not to be due to Clostridium species but rather to other facultative and

Critical Care Answers 61

obligate anaerobes, particularly streptococci. Though fungi have also been

implicated, they are less often associated with rapidly progressive infections.

Treatment for necrotizing soft tissue infections includes repeated

wide debridement, with wound reconstruction delayed until a stable,

viable wound surface has been established. The use of hyperbaric oxygen

in the treatment of gas gangrene remains controversial, due to lack of

proven benefit, difficulty in transporting critically ill patients to hyperbaric

facilities, and the risk of complications. Antitoxin has neither a prophylactic

nor a therapeutic role in the treatment of myonecrosis.

92. The answer is a. (Sabiston, 15/e, pp 1798-1799.) Abnormalities of

ventilation-perfusion ratio result from the shunting of blood to a hypoventilated

lung or from the ventilation of hypoperfused regions of lung tissue.

When this imbalance is extreme, as following massive pulmonary

thromboembolism, the effect is life-threatening hypoxemia. Other common

predisposing factors in the postoperative patient that contribute to

this maldistribution include the assumption of a supine position, thoracic

and upper abdominal incisions, obesity, atelectasis, and reduced cardiac

output.

93. The answer is d. (Shoemaker, 2/e, pp 39-41.) The metabolic and

physiologic effects of drowning and near-drowning depend upon variables

that include fluid temperature, extent of aspiration, and whether the aspirate

is fresh water or sea water. Cold-water submersion decreases oxygen

consumption and results in preferential shunting of blood flow to the heart

and brain. This shunting prolongs the period of submersion that can be

endured without irreversible cerebral damage. Return of normal cerebral

function after as long as 40 min of submersion in extremely cold water has

been reported. One should also remember that cooling below 30°C (86°F)

often causes cardiac arrhythmias. Ten percent of affected patients do not

aspirate fluid but succumb to asphyxia because of breath holding or

laryngospasm. Seventy percent have a significant metabolic acidosis requiring

administration of sodium bicarbonate. Significant electrolyte and blood

volume changes may or may not be present, depending on the degree of

aspiration and toxicity of the fluid medium. Renal damage may occur as a

result of hemoglobinuria (from hemolysis), acidosis, hypoxia, or changes

in renal blood flow. The most important initial treatment of drowning victims

is ventilation. Mouth-to-mouth or mouth-to-nose ventilation should

62 Surgery

be begun as soon as possible. Corticosteroids and prophylactic antibiotics

are not recommended for the prevention of pulmonary complications.

However, some workers feel that steroids may be of value in managing the

complication of cerebral edema.

94. The answer is c. (Greenfield, 2/e, pp 95-98.) Major hemorrhage that

requires the termination of anticoagulant therapy occurs in up to 15% of

anticoagulated patients. Spontaneous retroperitoneal hemorrhage constitutes

a small subset of such cases and can be a fatal complication. Heparin

is much more frequently associated with spontaneous retroperitoneal hemorrhage

than are oral agents. Advanced patient age and poor regulation of

coagulation times also increase the likelihood of bleeding complications.

Most cases of retroperitoneal hemorrhage present with flank pain and signs

of peritoneal irritations suggestive of an acute intraabdominal process. CT

scans are most useful in confirming the diagnosis and following the course

of the bleeding. Successful management is usually nonoperative and consists

of the discontinuation of anticoagulants, administration of vitamin K

or protamine, possible transfusion of clotting factors, and repletion of

intravascular volume with intravenous fluids.

95. The answer is e. (Pruitt, J Trauma 30:363-368, 1990.) Smoke inhalation

injuries ("smoke poisoning") and asphyxia account for almost onethird

of all fire fatalities. As opposed to respiratory burns, which are

thermal injuries of the upper respiratory tract, smoke inhalation is a chemical

injury to the distal tracheobronchial tree and alveoli. Most patients

admitted for this injury have elevated carbon monoxide levels, but a

minority will have physical evidence of skin burns (20%) or of oropharyngeal

burns (25%). Visible damage to the respiratory tract is not a frequent

finding. Chest films initially are often negative even in those patients who

subsequently develop respiratory failure from pulmonary edema or pneumonitis.

Patients with elevated carboxyhemoglobin levels or evidence of

smoke inhalation should be hospitalized for a minimum of 24 h for observation

regardless of normal arterial blood gases and chest x-ray.

96. The answer is e. (Robinson, Surgery 113:709-711, 1993.) Some drug

smugglers, often called "body packers" or "mules," ingest cocaine- or

heroin-filled packets and retrieve them at a later date from their stools. The

drugs are usually contained in latex or plastic packets. Rupture or leakage

Critical Care Answers 63

of even one bag carries the risk of severe toxicity and death. Although conservative

medical management with moderate doses of laxatives is usually

safe in stable body packers, close physiologic monitoring is necessary until

all packets are passed. High doses of laxatives, digital rectal disimpaction,

or endoscopic removal create a high risk of rupture of the bags and therefore

are generally discouraged. Emergency surgery is indicated when complications

develop.

97-99. The answers are 97-d, 98-e, 99-a. (Greenfield, 2/e, pp 438-443.)

Nitrous oxide (N2O) is a frequently used inhalation analgesic. However,

because the minimum alveolar anesthetic concentration (MAC) is so high

(over 100), true anesthesia at 1 atm pressure cannot be obtained without

compromising oxygen delivery to the patient. Since nitrous oxide is 30

times more soluble than nitrogen in blood, it enters a collection of trapped

air at a rate faster than that at which nitrogen leaves the collection. Thus,

the trapped air will increase in volume. If the trapped air is a result of

bowel obstruction, intestinal distention will increase.

Halothane is a very potent anesthetic with an MAC of 0.75. Cardiovascular

depression results from a number of different mechanisms.

Hypotension and decreased cardiac output have been associated with a

direct depression of myocardial muscle fibers and peripheral vascular

smooth muscle fibers. An effect on the medullary vasomotor centers as well

as on sympathetic ganglionic transmissions to the heart has been reported.

Enflurane is a halogenated inhalation anesthetic with an MAC of 1.2.

It is similar to halothane in its anesthetic characteristics. However, in a

small number of normal patients it may induce electroencephalographic

changes similar to those seen in epilepsy.

Methoxyflurane is the most potent and least volatile halogenated

inhalation anesthetic, with an MAC of 0.16. Its clinical use has been curtailed

because of the high risk of nephrotoxicity of the free fluoride ions

released during its biodegradation.

Morphine is a potent narcotic agent. Its use during general anesthesia

can potentiate the analgesic effects of the inhalation agents. It causes histamine

release with the risk of hypotension if given in a large bolus dose.

100-102. The answers are 100-a, c, d, e, h; 101-b, d, h; 102-f, g, h.

(Schwartz, 7/e, pp 485-507.) The decision to extubate an elderly patient

after major abdominal vascular surgery depends on accurate assessment of

hemodynamic and respiratory factors. Adding to the complexity of this

64 Surgery

particular patient is his past history of cardiac disease and his need for

inotropic support. Ventilatory and blood gas monitoring will determine

whether the patient can be weaned from the respirator. Continuous blood

pressure monitoring via arterial catheterization and pulmonary artery

catheter readings will enable the responsible physicians to assess volume

status and the ongoing need for inotropic support, both critical in the fluid

management of a patient about to be extubated. Continuous ECG monitoring

is essential for this patient because of the high incidence of perioperative

cardiac arrhythmias, particularly atrial fibrillation, following

surgery in which large fluid shifts are anticipated.

Conversely, an otherwise healthy young patient who is discovered to

have a ruptured spleen during emergency laparotomy is unlikely to require

prolonged intubation beyond the time of surgery. Nor is he likely to require

hemodynamic support in the form of pressors or inotropes, since his problem

is one of pure volume loss (blood), which can be met with rapid colloid

and crystalloid administration in the operating room. Consequently,

this patient may be managed with a central venous catheter (or even a

large-bore peripheral catheter, if one can be placed quickly) and ECG and

ventilatory monitoring during his operative procedure. If his hemorrhage is

successfully controlled, early discontinuation of physiologic monitoring

can be anticipated.

Chronically ill patients, such as brain-injured patients in vegetative

states, require nutritional monitoring insofar as they are unable to articulate

their nutritional needs. Caloric expenditure may be calculated to a

modest degree of accuracy using noninvasive methods based on body surface

area, age, and sex (Harris-Benedict equation). More accurate assessment,

particularly appropriate in disease states where metabolic activity is

accelerated, can be made using measurements of oxygen consumption and

carbon dioxide production. A patient who has suffered blunt head trauma

requiring repeated surgeries for intracranial bleeding will likely be monitored

with an intracranial pressure device. Other indications for intracranial

pressure monitoring include subarachnoid hemorrhage, hydrocephalus,

postcraniotomy, and Reye syndrome. ECG monitoring may also be helpful

in this setting, as increasing intracranial pressure may be presaged by

bradycardia.

103-105. The answers are 103-a, b, c, f, j; 104-a, b, c, d, e, f, g, h;

105-i. (Schwartz, 7/e, pp 88-90.) Prothrombin time measures the speed of

coagulation in the extrinsic pathway. A tissue source of procoagulant

Critical Care Answers 65

(thromboplastin) with calcium is added to plasma. The test will detect deficiencies

in factors II, V, VII, X, and fibrinogen and is used to monitor

patients receiving coumarin derivatives. However, even small amounts of

heparin will artificially prolong the clotting time, so that accurate prothrombin

times can only be obtained when the patient has not received

heparin for at least 5 h.

The intrinsic pathway is measured by the partial thromboplastin time.

This test is sensitive for defects in factors VIII, IX, XI, XII, and all the factors

of the extrinsic pathway and is used to monitor the status of patients

on heparin.

The bleeding time assesses the interaction of platelets and the formation

of the platelet plug. Therefore it will pick up deficiencies in both qualitative

and quantitative platelet function. Ingestion of aspirin within 1 wk

of the test will alter the result.

The thrombin time assesses qualitative abnormalities in fibrinogen and

the presence of inhibitors to fibrin polymerization. A standard amount of

fibrin is added to a fixed volume of plasma and clotting time is measured.

66 Surgery

SKIN:WOUNDS,

INFECTIONS, BURNS;

HANDS; PLASTIC SURGERY

Questions

DIRECTIONS: Each item below contains a question or incomplete

statement followed by suggested responses. Select the one best response to

each question.

67

106. Wasting of the intrinsic muscles

of the hand can be expected to

follow injury of the

a. Ulnar nerve

b. Radial nerve

c. Brachial nerve

d. Axillary nerve

e. Thenar and hypothenar nerves

107. Although wide surgical excision

is the traditional treatment for

malignant melanoma, narrow excision

of thin (less than 1 mm deep)

stage I melanomas has been found

to be equally safe and effective

when the margin of resection is as

small as

a. 3 mm

b. 5 mm

c. 1 cm

d. 3 cm

e. 5 cm

108. With regard to wound healing,

which one of the following

statements is correct?

a. Collagen content reaches a maximum

at approximately 1 wk after

injury

b. Monocytes are essential for normal

wound healing

c. Fibroblasts appear in the wound

within 24-36 h after the injury

d. The function of the monocyte in

wound healing is limited to phagocytosis

of bacteria and debris

e. Early in wound healing, type I collagen

is predominant

Terms of Use

Items 109-110

109. While you are on duty in the

emergency room, a 12-year-old

boy arrives with pain and inflammation

over the ball of his left foot

and red streaks extending up the

inner aspect of his leg. He remembers

removing a wood splinter

from the sole of his foot on the previous

day. The most likely infecting

organism is

a. Clostridium perfingens

b. Clostridium tetani

c. Staphylococcus

d. Escherichia coli

e. Streptococcus

110. The appropriate antibiotic to

prescribe while awaiting specific

culture verification is

a. Penicillin

b. Erythromycin

c. Tetracycline

d. Azathioprine

e. Cloxacillin

111. Proper treatment for frostbite

consists of

a. Debridement of the affected part

followed by silver sulfadiazine

dressings

b. Administration of corticosteroids

c. Administration of vasodilators

d. Immersion of the affected part in

water at 40-44°C (104-111.2°F)

e. Rewarming of the affected part at

room temperature

112. The true statement regarding

tendon injuries in the hand is

a. Flexor digitorum superficialis

inserts on the distal phalanx

b. Flexor digitorum profundus inserts

on the middle phalanx

c. The tendons of flexor digitorum

superficialis arise from a common

muscle belly

d. The best results for repair of a

flexor tendon are obtained with

injuries in the fibro-osseous tunnel

(zone 2)

e. The process of healing a tendon

injury involves formation of a

tenoma

113. Which one of the following

cases is considered a cleancontaminated

wound?

a. Open cholecystectomy for cholelithiasis

b. Herniorrhaphy with mesh repair

c. Lumpectomy with axillary node

dissection

d. Appendectomy with walled-off abscess

e. Gunshot wound to the abdomen

with injuries to the small bowel and

sigmoid colon

68 Surgery

114. A 45-year-old woman undergoes

an uneventful laparoscopic

cholecystectomy for which she

receives one dose of cephalosporin.

One week later, she returns to the

emergency room with fever, nausea,

and copious diarrhea and is

subsequently diagnosed with

pseudomembranous colitis. With

respect to this disease, which one of

the following statements is correct?

a. Surgical intervention is frequently

required

b. After appropriate antibiotic therapy,

the relapse rate is less than 5%

c. Tissue culture assay for Clostridium

difficile toxin B is neither sensitive

nor specific; therefore diagnosis

should be based on clinical findings

d. If surgery is performed, a left hemicolectomy

is usually adequate to

treat pseudo-membranous colitis

e. Indications for surgical treatment

include intractable disease, failure

of medical therapy, toxic megacolon,

and colonic perforation

115. A 60-year-old woman presents

with the skin lesion shown

below, which had been present for

10 years. She reported a history of

radiation treatments to that hand

for "eczema." Correct statements

concerning this lesion include

a. It is more malignant than basal cell

carcinoma

b. It occurs more frequently in

brunettes

c. It rarely metastasizes to regional

lymph nodes

d. It should be treated by radiation

therapy

e. It is rarely associated with chronic

sun exposure

Skin:Wounds, Infections, Burns;Hands;Plastic Surgery 69

Items 116-117

116. A 25-year-old man is brought

to the emergency room after sustaining

burns during a fire in his

apartment. He has blistering and

erythema of his face, left upper

extremity, and chest with frank

charring of his right upper extremity.

He is agitated, hypotensive, and

tachycardiac. Which one of the following

statements concerning this

patient's initial wound management

is correct?

a. Topical antibiotics should not be

used, as they will encourage growth

of resistant organisms

b. Early excision of facial and hand

burns is especially important

c. Escharotomy should only be performed

if neurologic impairment is

d. Excision of areas of third-degree or

of deep second-degree burns usually

takes place 3-7 days after

injury

e. Split-thickness skin grafts over the

eschar of third-degree burns should

be performed immediately in order

to prevent fluid loss

117. Which one of the following

statements regarding the above

burn patient is correct?

a. High-dose penicillin should be

administered prophylactically

b. Tetanus prophylaxis is not necessary

if the patient has been immunized

in the previous 3 years

c. This burn can be estimated at 60%

total body surface area using the

"rule of nines"

d. The most sensitive indicator of adequacy

of fluid resuscitation is heart

rate

e. This patient should undergo immediate

intubation for airway protection

and oxygen administration

118. True statements regarding

squamous cell carcinoma of the lip

include

a. The lesion often arises in areas of

persistent hyperkeratosis

b. More than 90% of cases occur on

the upper lip

c. The lesion constitutes 30% of all

cancers of the oral cavity

d. Radiotherapy is considered inappropriate

treatment for these lesions

e. Initially metastases are to the ipsilateral

posterior cervical lymph

nodes

70 Surgery

119. Which of the following statements

regarding carpal tunnel syndrome

is correct?

a. It is rarely secondary to trauma

b. It may be associated with pregnancy

c. It most often causes dysesthesia during

waking hours

d. It is often associated with vascular

compromise

e. Surgical treatment involves release

of the extensor retinaculum

120. Which of the following is

true with regard to wound contraction?

a. It is the primary process affecting

closure of a sutured or stapled surgical

wound

b. Bacterial colonization significantly

slows the process of contraction

c. It may account for a maximum of

50% decrease in the size of a wound

d. It is based on specialized fibroblasts

that contain actin myofilaments

e. The percentage reduction of wound

size is increased with increased

adherency of skin to underlying

tissue

121. Management of leukoplakia

of the oral cavity includes

a. Excisional biopsy of all lesions

b. Application of topical antibiotics

c. Low-dose radiation therapy

d. Ascertaining that dentures fit

properly

e. Application of topical chemotherapeutic

agents

122. An 8-lb infant, born following

uncomplicated labor and delivery,

is noted to have a unilateral

cleft lip and palate. The parents

should be advised that

a. The child almost certainly has

other congenital anomalies

b. Rehabilitation requires adjunctive

speech therapy

c. Lip repair is indicated at 1 year of

age

d. Palate repair is indicated prior to 6

mo of age

e. Cosmetic revisions to the nose

should be performed at the same

time as cleft lip repair

123. A 40-year-old woman undergoes

wide excision of a pigmented

lesion of her thigh. Pathologic

examination reveals malignant

melanoma that is Clark's level IV.

Findings on examination of the

groin are normal. The patient

should be advised that

a. Radiotherapy will be an important

part of subsequent therapy

b. The likelihood of groin node

metastases is remote

c. Immunotherapy is an effective

form of adjunctive treatment for

metastatic malignant melanoma

d. Groin dissection is not indicated

unless and until groin nodes

become palpable

e. Intralesional bacille Calmette-

Guérin (BCG) administration has

been found to aid in local control in

the majority of patients

Skin:Wounds, Infections, Burns;Hands;Plastic Surgery 71

DIRECTIONS: Each group of questions below consists of lettered

options followed by numbered items. For each numbered item, select the

appropriate lettered option(s). Each lettered option may be used once,

more than once, or not at all. Choose exactly the number of options

indicated following each item.

Items 124-127

Match each description with

the correct skin or subcutaneous

lesions.

a. Cystic hygroma

b. Basal cell carcinoma

c. Port-wine stain

d. Strawberry hemangioma

e. Malignant melanoma

f. Squamous cell carcinoma

124. A 56-year-old woman presents

with a small, pigmented

lesion on her forearm, which has

been growing over the last 2 mo.

She is a fair-complected woman

with a history of sun exposure.

(SELECT 3 LESIONS)

125. A 6-mo-old child presents

with a red lesion on the face.

(SELECT 2 LESIONS)

126. Surgical excision is the first

line of therapy. (SELECT 5

LESIONS)

127. Radiation may be useful as

adjuvant therapy. (SELECT 3

LESIONS)

Items 128-131

Match each description with

the correct growth factors or

cytokines.

a. Platelet-derived growth factor

(PDGF)

b. Transforming growth factor

c. Tumor necrosis factor (TNF)

d. Fibroblast growth factor

e. Interleukin 1 (IL-1)

f. Thromboxane A2

128. Platelets are the cell of origin.

(SELECT 3 CHOICES)

129. Macrophages are the cell of

origin. (SELECT 5 CHOICES)

130. They stimulate fibroblast proliferation.

(SELECT 4 CHOICES)

131. They stimulate collagen synthesis.

(SELECT 3 CHOICES)

72 Surgery

SKIN:WOUNDS,

INFECTIONS, BURNS;

HANDS; PLASTIC SURGERY

Answers

106. The answer is a. (Sabiston, 15/e, pp 1479-1485.) The ulnar nerve

innervates 15 of the 20 intrinsic muscles of the hand. The musculocutaneous,

radial, ulnar, and median nerves are all important to hand function.

The musculocutaneous and radial nerves allow forearm supination; the

radial nerve alone innervates the extensor muscles. The median nerve is the

"eye of the hand" because of its extensive contribution to sensory perception;

it also maintains most of the long flexors, the pronators of the forearm,

and the thenar muscles.

107. The answer is c. (Schwartz, 7/e, pp 333, 523-527.) Wide excision of

melanomas, with margins of 3-5 cm beyond the lateral edges of tumor, has

traditionally been considered mandatory. A 5-year prospective multicenter

study of over 600 randomly assigned patients with thin stage I melanomas,

however, showed that local recurrence rates, as well as the subsequent

development of metastatic disease, were not different when margins of 1 cm

or 3 cm were taken, provided that tumor thickness did not exceed 1 mm.

108. The answer is b. (Greenfield 2/e, pp 67-83.) Wound healing is an

overlapping sequence of inflammation, proliferation, and remodeling. The

inflammatory phase is characterized by a rapid influx of neutrophils, followed

in about 2 days by an influx of mononuclear cells. These monocytes

act not only by phagocytosing debris and bacteria, but also by secreting

numerous growth factors including tumor necrosis factor (TNF), transforming

growth factor, platelet-derived growth factor (PDGF), and fibroblast

growth factor, which are essential to normal wound healing.

Angiogenesis and collagen formation take place during the proliferative

phase of wound healing. Fibroblasts, which enter the wound at about day 3,

continue to proliferate with increasing collagen deposition. Throughout the

73

proliferative phase, type III collagen predominates. Collagen content is

maximum at 2-3 wk, at which time the remodeling phase begins. Type III

collagen, which is elastic fibrils, is gradually replaced by rigid fibrils, or type

I collagen, at this time. During remodeling, collagen deposition and degradation

reach a steady state, which may continue for up to 1 year.

109-110. The answers are 109-e, 110-a. (Greenfield, 2/e, p 1970.) The

significant observation in this question is the description of lymphangitic

inflammatory streaking up the inner aspect of the patient's leg. This is

highly suggestive of a streptococcal infection and the presumptive therapy

should be high doses of a bactericidal antibiotic. Penicillin remains the

mainstay of therapy against presumed streptococcal infections. Most streptococcal

cellulitis is adequately treated by penicillin, elevation of the

infected extremity, and attention to the local wound to ascertain adequate

local drainage and absence of any persisting foreign body. However, the

clinician must be alert to the possibility of a more fulminant and life- or

limb-threatening infection by clostridia, microaerophilic streptococcus, or

other potentially synergistic organisms that can produce rapidly progressive

deep infections in fascia of muscle. Smears and cultures of drainage

fluid or aspirates should be taken. Close observation of the wound is essential,

and aggressive debridement in the operating room is mandatory at the

slightest suggestion that fasciitis or myonecrosis may be ensuing.

111. The answer is d. (Greenfield, 2/e, pp 412-414.) Many methods of

treating frostbite have been tried throughout the years. These include massage,

warm-water immersion, or covering the affected area. Rapid warming

by immersion in water slightly above normal body temperature (40-448°C)

is the most effective method; however, because the frostbitten region is

numb and especially vulnerable, it should be protected from trauma or

excessive heat during treatment. Further treatment may include elevation to

minimize edema, administration of antibiotics and tetanus toxoid, and

debridement of necrotic skin as needed.

112. The answer is e. (Schwartz, 7/e, pp 2025-2056.) Each digit has two

long flexors, named superficial and deep according to the relative position of

the muscle bellies. In the fingers each superficial flexor tendon divides

around the corresponding deep tendon to reach its insertion on the base of

74 Surgery

the middle phalanx. The deep flexor tendon continues to its insertion on

the base of the distal phalanx. Only the deep flexors can flex the distal

interphalangeal joint. Since the tendons of the deep flexors share a common

muscle belly, only the superficial flexors can move a finger when the

adjacent fingers are immobilized. These tendons are prevented from bowstringing

across the joints by the flexor retinaculum of the wrists and the

fibroosseous tunnels, which extend from the distal palmar crease to the

middle phalanx. They run within synovial sheaths and are nourished by

vincula tendinum (short mesenteries). The process of healing a tendon

injury involves the formation of a tenoma, which tends to become adherent

to the surrounding sheath. A difficult balance has to be struck between

the desire to prevent adhesions by early mobilization and the risk of rupturing

an unhealed tendon. Verdan has divided the hand into six regions

according to the anatomy surrounding the tendons. Zone 2, sometimes

referred to as "no-man's land," refers to the fibroosseous tunnels. Repair in

this region is fraught with difficulty.

113. The answer is a. (Schwartz, 7/e, pp 130-131.) Surgical wounds can

be divided into three categories based on the amount of bacterial contamination.

Clean wounds are those in which no part of the respiratory, gastrointestinal,

or genitourinary tract is entered. Examples include

herniorrhaphy and breast surgery. Clean-contaminated wounds encompass

those cases in which the above systems are entered, but without evidence

of active infection or gross spillage. Examples include elective cholecystectomy

or elective colon resection with adequate bowel preparation. Contaminated

wounds are those in which there is active infection (perforated

appendicitis with abscess) or gross spillage (gunshot wound with large

or small bowel injuries). While contaminated and clean-contaminated

wounds require perioperative antibiotics, clean wounds need not be

treated with prophylactic antibiotics.

114. The answer is e. (Lipsett, Surgery 116:491-496, 1994.) Pseudomembranous

colitis is a common nosocomial infection most often caused by

Clostridium difficile toxins A and B. Antibiotic use allows overgrowth of C. difficile,

leading to abdominal pain, fever, diarrhea, and increased WBCs. Diagnosis

is confirmed by isolation of C. difficile toxin B via tissue culture assay.

Sensitivity and specificity are quite high (greater than 90%), but may require

Skin:Wounds, Infections, Burns;Hands;Plastic Surgery Answers 75

24-48 h to complete. The vast majority of patients will respond to oral vancomycin

or metronidazole, although 20-30% of patients may relapse.

Because response to antibiotic therapy is high, surgical intervention is infrequently

required (<1%). Indications for surgery include intractable or fulminant

disease, failure of medical therapy, colonic perforation, and toxic

megacolon. Pseudomembranous colitis often involves the entire colon,

despite normal-appearing serosa. Therefore, the procedure of choice is a

subtotal colectomy with ileostomy. Overall mortality of 35-40% is described,

with <20% mortality for those patients undergoing subtotal colectomy.

115. The answer is a. (Schwartz, 7/e, pp 257, 522, 527, 617-621.) Squamous

cell carcinoma occurs in people who have had chronic sun exposure,

chronic ulcers or sinus tracts (draining osteomyelitis), and a history of radiation

or thermal injury (Margolin's ulcer). It is more malignant than basal

cell carcinoma, grows more rapidly, and metastasizes. It occurs more frequently

in blondes and fair-skinned people. A radiation-induced carcinoma,

or one arising in a burn scar, should not be treated with radiation

therapy for fear of further damage.

116. The answer is d. (Schwartz, 7/e, pp 242-244.) Early wound management

is characterized by early excision of areas of devitalized tissue,

with the exception of deep wounds of the palms, soles, genitals, and face.

Staged excision of deep partial-thickness or full-thickness burns occurs

between 3 and 7 days after the injury. There are several proven advantages

to early excision including decreased hospital stay and lower cost. This is

especially true of burns encompassing >30-40% total body surface area. In

conjunction with early excision, topical antimicrobials such as silver sulfadiazine

are extremely important in delaying colonization of the newly

excised or fresh burn wounds. Permanent coverage through split-thickness

skin grafting usually occurs more than 1 wk after injury. Skin autograft

requires a vascular bed and therefore cannot be placed over eschar. Meticulous

attention to deep circumferential burns is crucial in the management

of burn patients. Progressive tissue edema may lead to progressive vascular

and neurologic compromise. Because the blood supply is the initial system

affected, frequent assessment of flow is vital, with longitudinal escharotomy

performed at the first sign of vascular compromise. A low threshold

should be maintained in performing an escharotomy in the setting of

severely burned limbs.

76 Surgery

117. The answer is e. (Schwartz, 7/e, pp 228-232, 234-238.) Aggressive

evaluation and treatment of burn victims has led to increased survival of

the 2 million patients treated for burns each year in the United States. A

systematic approach to the patient with attention to airway/vascular access

and aggressive fluid resuscitation has proved essential. In the patient with

obvious facial burns who is hemodynamically unstable, airway access is the

first priority. Fluid resuscitation is initiated using the Parkland formula,

with urine output of 0.5-1.0 mL/kg/h being the most sensitive indicator of

the adequacy of resuscitation. The extent of the burn can be roughly estimated

using the "rule of nines," in which the head and the upper extremities

are each 9% of the total body surface area (TBSA) and the anterior

trunk, posterior trunk, and the lower extremities are each 18% of the

TBSA. The neck encompasses 1% of the TBSA. This patient has burns of

roughly 40% TBSA (face 4.5%, upper extremities 18%, and anterior trunk

18%). Tetanus prophylaxis is indicated in all patients who have not been

immunized within 1 year; prophylactic intravenous antibiotics are not

indicated because they have not been shown to be of benefit in decreasing

early cellulitic infections. Conversely, they have been shown to lead to

increased complications secondary to resistant gram-negative organisms.

118. The answer is a. (Schwartz, 7/e, pp 629, 631.) Squamous cell carcinoma

of the lip is the most common malignant tumor of the lip and constitutes

15% of all malignancies of the oral cavity. Basal cell carcinomas do

occur on the lip, but much less frequently. There is a strong association

between squamous cell tumors of the lip and sun exposure. Therefore,

these lesions are more common in the southern United States and in occupational

groups who work out of doors. Because of its greater sun exposure,

the lower lip is the site of more than 90% of such lesions. Persistent

hyperkeratosis precedes 35-40% of these lesions. The incidence of metastases

increases with the size of the lesion, and spread is usually via lymphatics

to the ipsilateral submental node. Contralateral nodal metastases

are rare unless the lesion crosses the midline. Approximately 10-15% of all

patients have metastases at the time of diagnosis. These lip tumors are very

responsive to radiotherapy, which works well for small to medium-sized

lesions. Large lesions treated with radiotherapy usually require surgical

reconstruction. Radiotherapy should not be used in patients who will have

ongoing sun exposure to the area because radiation therapy sensitizes the

tissues to solar trauma.

Skin:Wounds, Infections, Burns;Hands;Plastic Surgery Answers 77

119. The answer is b. (Schwartz, 7/e, pp 2063-2070.) Signs and symptoms

of carpal tunnel syndrome are related to the distribution of the

median nerve. This nerve, which passes through the carpal tunnel is the

wrist with the finger flexor tendons, may suffer compression from fibrous

scarring or malalignment following a fracture of the wrist. Nerve compression

may also occur in patients with rheumatoid arthritis who develop

flexor tenosynovitis. In women, the syndrome frequently first appears during

pregnancy and recurs during the premenstrual phase of subsequent

menstrual cycles. In these cases, symptoms are presumably the result of the

effects of fluid retention and pressure on the median nerve owing to tissue

swelling. In many instances, symptoms are limited to nocturnal pain and

paresthesias. If conservative treatment of carpal tunnel syndrome is unsuccessful,

surgical treatment may be required. Open and endoscopic techniques

have been employed, both of which release adhesions of the median

nerve and divide the transverse carpal ligament. The extensor retinaculum

is located on the dorsal aspect of the wrist and contains the six compartments

of extensor tendons.

120. The answer is d. (Schwartz, 7/e, pp 270-277.) While epithelialization

is responsible for the healing of a closed incision, wound contraction is the

primary method of closure in open wounds. During this process, the skin

surrounding the wound is pulled over the wound surface and may account

for up to a 90% reduction in the size of an open wound. In areas of greater

adherence of skin to underlying tissue, the ability of contraction to close the

wound is hindered due to the decreased mobility of the skin. Therefore, in

areas of tight skin adherence such as the leg, contraction may only account

for 30-40% reduction in wound size. Fibroblasts in the open wound, which

predominate during the proliferative phase, contain increasing numbers of

actin microfilaments, thereby becoming myofibroblasts. These specialized

fibroblasts are felt to be responsible for wound contraction either through

intrinsic cellular contraction or attachment to collagen strands. Bacterial colonization

does not harm the process of wound contraction and surgical

wound healing. While wound infection is often difficult to diagnose in open

wounds, it is generally accepted that bacterial counts of 1 million bacteria per

gram of tissue are deleterious to wound closure.

121. The answer is d. (Schwartz, 7/e, pp 604, 610.) White patches in the

oral cavity (leukoplakia) sometimes are incorrectly interpreted as a pre-

78 Surgery

malignant condition. Microscopic examination of leukoplakia may in fact

reveal hyperplasia, keratosis, or dyskeratosis, of which the last finding is the

most serious because of its association with malignancy. Only about 5% of

patients with leukoplakia develop cancer. A suggested treatment protocol

for patients with thin lesions advocates a program of strict oral hygiene and

avoidance of alcohol and tobacco. Biopsy is reserved only for those with

thick lesions (since carcinoma in situ may be present). Radiation therapy is

contraindicated. Approximately 50% of all oral cancers occur in patients

who have associated areas of hyperkeratosis and dyskeratosis. Chronic irritation,

as may occur with poorly fitting dentures, may result in leukoplakia.

122. The answer is b. (Schwartz, 7/e, pp 2106-2110.) Clefts of the lip

and palate occur relatively frequently (1 in 750 live births); they may be

unilateral or bilateral and can vary from a small notch to a complete cleft of

the lip and palate. Most clefts occur as isolated anomalies, but occasionally

they are associated with neurologic, orthopedic, or cardiac anomalies. A

frequently recommended protocol for management is lip repair in the first

3 mo of life and palate repair at 12 to 18 mo. Other cosmetic procedures

can be performed late in childhood and adolescence. Palate repair after 2

years of age is associated with a high incidence of speech impairment, often

requiring speech therapy; repair in the early months of life can lead to a

hazardous loss of blood that is poorly tolerated by the infant. Repair of the

lip usually should be accomplished as soon as the infant is sufficiently stabilized

to tolerate anesthesia with reasonable safety. Ten to twelve weeks is

often recommended as the time for lip repair. At this age, the affected baby

usually can be converted to dropper or cup feedings in the postoperative

period, which thereby facilitates healing of the lip by reducing the need for

suckling with the freshly wounded tissues.

123. The answer is d. (Greenfield, 2/e, pp 2231-2242.) The survival of

patients with malignant melanoma correlates with the depth of invasion

(Clark) and the thickness of the lesion (Breslow). It is widely held that

patients with thin lesions (<0.76 mm) and Clark level I and II lesions are

adequately managed by wide local excision. The incidence of nodal metastases

rises with increasing Clark level of invasion such that a level IV lesion

has a 30-50% incidence of nodal metastases. The assumption that removal

of microscopic foci of disease is beneficial, in conjunction with retrospective

data indicating improved survival in patients who have undergone

Skin:Wounds, Infections, Burns;Hands;Plastic Surgery Answers 79

removal of clinically negative but pathologically positive nodes, has led to

the widely held belief that prophylactic node dissections are indicated for

melanoma. Prospective data has challenged this concept. Veronesi and Sim

have found that patients undergoing prophylactic node dissections survived

no longer than those who were followed closely and underwent node

dissections only after nodes became palpable. The subject remains controversial,

and further study and follow-up are necessary. Immunotherapy has

not been successful in controlling widespread metastatic melanoma even

when added to chemotherapy. Intralesional administration of BCG has

been demonstrated to control local skin lesions in only 20% of patients.

Dinitrochlorobenzene (DNCB) can also be used.

124-127. The answers are 124-b, e, f; 125-c, d; 126-a, b, c, e, f;

127-b, e, f. (Greenfield, 2/e, pp 2231-2245.) Cutaneous neoplasms are

extremely prevalent in the United States, with basal cell and squamous cell

carcinoma being the most common. Patients who are at particular risk for

malignant neoplasms are those with fair complexion and frequent sun

exposure. Other risk factors for basal and squamous cell carcinomas

include radiation damage, chronic wounds, and scar tissue. Surgical excision

is the treatment of choice of all malignant cutaneous neoplasms. Radiation

therapy may be useful for palliation of metastatic melanoma and may

be considered as adjuvant therapy for squamous cell carcinoma and aggressive

or invasive basal cell carcinoma. The pediatric population may suffer

from multiple cutaneous lesions, including port-wine stains and strawberry

hemangiomas. Both are capillary hemangiomas, but with very different

clinical courses. Port-wine stains are present from birth and do not

regress; therefore, surgical excision is a treatment option in small lesions.

Other treatment options include laser cauterization or tattooing. Strawberry

hemangiomas typically grow rapidly over 6-12 mo, but 90% regress

spontaneously; therefore, commonly no intervention is required. For particularly

large or rapidly growing lesions, excision, laser cauterization, or

steroids may be considered. Cystic hygromas are masses of lymphatic vessels

typically present in the head and neck region, usually apparent at

birth. They are easily diagnosed with ultrasonography and are treated with

surgical excision.

128-131. The answers are 128-a, b, f; 129-a, b, c, d, e; 130-b, c, d,

e; 131-b, c, e. (Greenfield, 2/e, pp 108-126.) Numerous cytokines or

80 Surgery

growth factors are liberated from various cells at the time of injury. The

process of wound healing requires that these factors act in an orchestrated

manner. Platelets release ADP, thromboxane A2, transforming growth factor,

and platelet-derived growth factor within 1 h of injury. When

macrophages become the predominant cell (at 2-3 days), numerous additional

cytokines are released, including IL-1, fibroblast growth factor, TNF,

transforming growth factor, and PDGF, among others. Integral to adequate

wound healing is fibroblast proliferation and collagen synthesis. Stimulants

of fibroblast proliferation include TNF, IL-1, fibroblast growth factor, transforming

growth factor, epithelial growth factor, and plasminogen activator

inhibitor. Collagen synthesis is then initiated and progresses during the

proliferative phase of wound healing upon stimulation by IL-1, TNF, and

transforming growth factor. Excessive or unopposed release of cytokines is

thought to be responsible in various pathologic states of wound healing,

such as pulmonary fibrosis or hepatic cirrhosis, and can lead to failure of

multiple organ systems.

Skin:Wounds, Infections, Burns;Hands;Plastic Surgery Answers 81

This page intentionally left blank.

TRAUMA AND SHOCK

Questions

DIRECTIONS: Each item below contains a question or incomplete

statement followed by suggested responses. Select the one best response to

each question.

83

132. A teenage boy falls from his

bicycle and is run over by a truck.

On arrival in the emergency room,

he is awake and alert and appears

frightened but in no distress. The

chest radiograph suggests an airfluid

level in the left lower lung

field and the nasogastric tube

seems to coil upward into the left

chest. The next best step in management

a. Placement of a left chest tube

b. Immediate thoracotomy

c. Immediate celiotomy

d. Esophagogastroscopy

e. Removal and replacement of the

nasogastric tube; diagnostic peritoneal

lavage

133. Which of the following conditions

is most likely to follow

a compression-type abdominal injury?

a. Renal vascular injury

b. Superior mesenteric thrombosis

c. Mesenteric vascular injury

d. Avulsion of the splenic pedicle

e. Diaphragmatic hernia

134. A 65-year-old man who

smokes cigarettes and has chronic

obstructive pulmonary disease falls

and fractures the 7th, 8th, and 9th

ribs in the left anterolateral chest.

Chest x-ray is otherwise normal.

Appropriate treatment might include

a. Strapping the chest with adhesive

tape

b. Immobilization with sandbags

c. Tube thoracostomy

d. Peritoneal lavage

e. Surgical fixation of the fractured

135. Blunt trauma to the abdomen

most commonly injures which of

the following organs?

a. Liver

b. Kidney

c. Spleen

d. Intestine

e. Pancreas

Terms of Use

136. Ligation of injured major

peripheral veins is rarely preferable

to repair, but may be justified for

which reason?

a. In severe popliteal vascular injuries,

venous ligation leads to a decreased

amputation rate following successful

arterial reconstruction when

compared with combined arterial

and venous repair

b. Venous ligation leads to a decreased

incidence of chronic venous insufficiency

when compared with venous

repair

c. Venous ligation leads to a decreased

operative time in patients with

multiple injuries or severe trauma

when compared with venous repair

d. In the presence of extensive associated

soft tissue injury, venous return

is already sufficiently impaired to

render venous repair pointless

e. Even though ligated veins thrombose,

they often recanalize

137. A 27-year-old man sustains a

single gunshot wound to the left

thigh. In the emergency room he is

noted to have a large hematoma of

his medial thigh. He complains of

paresthesias in his foot. On examination

there are weak pulses palpable

distal to the injury and the

patient is unable to move his foot.

The appropriate initial management

of this patient would be

a. Angiography

b. Immediate exploration and repair

c. Fasciotomy of anterior compartment

d. Observation for resolution of spasm

e. Local wound exploration

Items 138-139

A 25-year-old woman arrives

in the emergency room following

an automobile accident. She is

acutely dyspneic with a respiratory

rate of 60 breaths/min. Breath

sounds are markedly diminished

on the right side.

138. The first step in managing

the patient should be to

a. Take a chest x-ray

b. Draw arterial blood for blood gas

determination

c. Decompress the right pleural space

d. Perform pericardiocentesis

e. Administer intravenous fluids

84 Surgery

139. A chest x-ray of this woman

beforetherapywouldprobablyreveal

a. Air in the right pleural space

b. Shifting of the mediastinum toward

the right

c. Shifting of the trachea toward the

right

d. Dilation of the intrathoracic vena

cava

e. Hyperinflation of the left lung

140. Among the physiologic responses

to acute injury is

a. Increased secretion of insulin

b. Increased secretion of thyroxine

c. Decreased secretion of vasopressin

(ADH)

d. Decreased secretion of glucagon

e. Decreased secretion of aldosterone

141. In a stable patient, the management

of a complete transection

of the common bile duct distal to

the insertion of the cystic duct

would be optimally performed

with a

a. Choledochoduodenostomy

b. Loop choledochojejunostomy

c. Primary end-to-end anastomosis of

the transected bile duct

d. Roux-en-Y choledochojejunostomy

e. Bridging of the injury with a T tube

142. Nonoperative management of

penetrating neck injuries has been

advocatedasanalternative tomandatory

exploration in asymptomatic

patients. Which of the following

findings would constitute a relative,

rather than an absolute, indication

for formal neck exploration?

a. Expanding hematoma

b. Dysphagia

c. Dysphonia

d. Pneumothorax

e. Hemoptysis

143. Following blunt abdominal

trauma, a 12-year-old girl develops

upper abdominal pain, nausea, and

vomiting. An upper gastrointestinal

series reveals a total obstruction

of the duodenum with a "coiled

spring" appearance in the second

and third portions. Appropriate

management is

a. Gastrojejunostomy

b. Nasogastric suction and observation

c. Duodenal resection

d. TPN to increase the size of the

retroperitoneal fat pad

e. Duodenojejunostomy

144. Following traumatic peripheral

nerve transection, regrowth

usually occurs at which of the following

rates?

a. 0.1 mm per day

b. 1 mm per day

c. 5 mm per day

d. 1 cm per day

e. None of the above

Trauma and Shock 85

Items 145-147

A 28-year-old man is brought

to the emergency room for a severe

head injury after a fall. Initially

lethargic, he becomes comatose and

does not move his right side. His

left pupil is dilated and responds

only sluggishly.

145. The most common initial

manifestation of increasing intracranial

pressure in the victim of

head trauma is

a. Change in level of consciousness

b. Ipsilateral (side of hemorrhage)

pupillary dilation

c. Contralateral pupillary dilation

d. Hemiparesis

e. Hypertension

146. Initial emergency reduction

of intracranial pressure is most

rapidly accomplished by

a. Saline-furosemide (Lasix) infusion

b. Urea infusion

c. Mannitol infusion

d. Intravenous dexamethasone (Decadron)

e. Hyperventilation

147. In the patient described,

compression of the affected nerve is

produced by

a. Infection within the cavernous sinus

b. Herniation of the uncal process of

the temporal lobe

c. Laceration of the corpus callosum

by the falx cerebri

d. Occult damage to the superior cervical

ganglion

e. Cerebellar hypoxia

148. A 31-year-old man is brought

to the emergency room following

an automobile accident in which

his chest struck the steering wheel.

Examination reveals stable vital

signs, but the patient exhibits multiple

palpable rib fractures and

paradoxical movement of the right

side of the chest. Chest x-ray shows

no evidence of pneumothorax or

hemothorax, but a large pulmonary

contusion is developing. Proper

treatment would consist of which

of the following?

a. Tracheostomy, mechanical ventilation,

and positive end-expiratory

pressure

b. Stabilization of the chest wall with

sandbags

c. Stabilization with towel clips

d. Immediate operative stabilization

e. No treatment unless signs of respiratory

distress develop

86 Surgery

149. A 30-year-old man is stabbed

in the arm. There is no evidence of

vascular injury, but he cannot flex

his three radial digits. He has injured

the

a. Flexor pollicis longus and flexor

digitus medius tendons

b. Radial nerve

c. Median nerve

d. Thenar and digital nerves at the

wrist

e. Ulnar nerve

150. Following a 2-h fire-fighting

episode, a 36-year-old fireman begins

complaining of a throbbing

headache, nausea, dizziness, and

visual disturbances. He is taken to

the emergency room where his

carboxyhemoglobin (COHb) level

is found to be 31%. Appropriate

treatment would be to

a. Begin an immediate exchange

transfusion

b. Transfer the patient to a hyperbaric

oxygen chamber

c. Begin bicarbonate infusion and give

250 mg acetazolamide (Diamox)

intravenously

d. Administer 100% oxygen by mask

e. Perform flexible bronchoscopy

with further therapy determined by

findings

151. An elderly pedestrian collides

with a bicycle-riding pizza

delivery man and suffers a unilateral

fracture of his pelvis through

the obturator foramen. You would

manage this injury by

a. External pelvic fixation

b. Angiographic visualization of the

obturator artery with surgical

exploration if the artery is injured

or constricted

c. Direct surgical approach with internal

fixation of the ischial ramus

d. Short-term bed rest with gradual

ambulation as pain allows after 3

days

e. Hip spica

152. Regarding high-voltage electrical

burns to an extremity

a. Injuries are generally more superficial

than those of thermal burns

b. Intravenous fluid replacement is

based on the percentage of body

surface area burned

c. Antibiotic prophylaxis is not required

d. Evaluation for fracture of the other

extremities and visceral injury is

indicated

e. Cardiac conduction abnormalities

are unlikely

Trauma and Shock 87

153. Which of the following fractures

or dislocations of the extremities

induced by blunt trauma is

associated with significant vascular

injuries?

a. Knee dislocation

b. Closed posterior elbow dislocation

c. Midclavicular fracture

d. Supracondylar femur fracture

e. Tibial plateau fracture

154. A 23-year-old previously

healthy man presents to the emergency

room after sustaining a single

gunshot wound to the left chest.

The entrance wound is 3 cm inferior

to the nipple and the exit

wound is just below the scapula. A

chest tube is placed that drains 400

mL of blood and continues to drain

50-75 mL/h during the initial

resuscitation. Initial blood pressure

of 70/0 mm Hg responds to 2 L

crystalloid and is now 100/70 mm

Hg. Abdominal examination is

unremarkable. Chest x-ray reveals

a reexpanded lung and no free air

under the diaphragm. The next

management step should be

a. Admission and observation

b. Peritoneal lavage

c. Exploratory thoracotomy

d. Exploratory celiotomy

e. Local wound exploration

155. A patient is brought to the

emergency room after a motor

vehicle accident. He is unconscious

and has a deep scalp laceration and

one dilated pupil. His heart rate is

120 beats/min, blood pressure is

80/40 mm Hg, and respiratory rate

is 35 breaths/min. Despite rapid

administration of 2 L normal

saline, the patient's vital signs do

not change significantly. The injury

likely to explain this patient's

hypotension is

a. Epidural hematoma

b. Subdural hematoma

c. Intraparenchymal brain hemorrhage

d. Basilar skull fracture

e. None of the above

156. When operating to repair

civilian colon injuries

a. A colostomy should be performed

for colonic injury in the presence of

gross fecal contamination

b. The presence of shock on admission

or more than two associated

intraabdominal injuries is an

absolute contraindication to primary

colonic repair

c. Distal sigmoidal injuries should not

be repaired primarily

d. Right-sided colonic wounds should

not be repaired primarily

e. Administration of intravenous

antibiotics with aerobic and anaerobic

coverage has not been shown

to decrease the incidence of wound

infections after repair of colonic

injuries

88 Surgery

157. A 34-year-old prostitute with a history of long-term intravenous drug

use is admitted with a 48-h history of pain in her left arm. Physical examination

is remarkable for crepitus surrounding needle track marks in the

antecubital space with a serous exudate. The plain x-ray of the arm is

shown below. Which of the following organisms is most likely to be

responsible for this condition?

Trauma and Shock 89

a. Anaerobic streptococcus

b. Staphylococcus aureus

c. Pseudomonas aeruginosa

d. Clostridium perfringens

e. Escherichia coli

158. Regarding myocardial contusion

from blunt chest trauma,

which of the following statements

is correct?

a. Elevated cardiac isoenzyme levels

sensitively identify patients at risk

for life-threatening arrhythmias

b. The majority of patients have

abnormalities on the initial ECG

post injury

c. First-pass radionuclide angiography

(RNA) and echocardiography

are considered the "gold standard"

for diagnosis

d. RNA and echocardiography are

good predictors of subsequent cardiac

complications such as arrhythmias

and pump failure

e. All patients diagnosed with myocardial

contusion should be monitored

in an intensive care unit

setting for 72 h

159. Protein metabolism after

trauma is characterized by

a. Decreased liver gluconeogenesis

b. Inhibition of skeletal muscle breakdown

by interleukin 1 and tumor

necrosis factor (TNF, cachectin)

c. Decreased urinary nitrogen loss

d. Hepatic synthesis of acute-phase

reactants

e. Decreased glutamine consumption

by fibroblasts, lymphocytes, and

intestinal epithelial cells

160. A 36-year-old man sustains a

gunshot wound to the left buttock.

He is hemodynamically stable.

There is no exit wound, and an xray

of the abdomen shows the bullet

to be located in the right lower

quadrant. Correct management of a

suspected rectal injury would

include

a. Barium studies of the colon and

rectum

b. Barium studies of the bullet track

c. Endoscopy of the bullet track

d. Angiography

e. Sigmoidoscopy in the emergency

161. Correct statements regarding

blunt trauma to the liver include

which of the following?

a. Hepatic artery ligation for control

of bleeding is associated with

decreased morbidity and mortality

b. The incidence of intraabdominal

infections is significantly lower in

patients with abdominal drains

c. Intracaval shunting has dramatically

improved survival among

patients with hepatic vein injuries

d. Nonanatomic hepatic debridement,

with removal of the injured

fragments only, is preferable to

resection along anatomic planes

e. Major hepatic lacerations that are

sutured closed will result in intrahepatic

hematomas, hemobilia, and

bile fistulas

90 Surgery

162. If injury to a major artery in

an extremity is suspected, surgical

exploration should be carried out

regardless of the presence of palpable

pulses distal to the injury. The

rationale is that the presence of palpable

distal pulses does not reliably

exclude

a. Significant arterial injury

b. Significant injury to adjacent motor

nerve trunks

c. Significant injury to adjacent long

d. Significant injury to adjacent veins

e. Subsequent development of a compartment

syndrome and the need

for fasciotomy

163. The response to shock

includes which of the following

metabolic effects?

a. Increase in sodium and water

excretion

b. Increase in renal perfusion

c. Decrease in cortisol levels

d. Hyperkalemia

e. Hypoglycemia

164. Appropriate treatment for an

acute stable hematoma of the pinna

of the ear includes which of the following

measures?

a. Ice packs and prophylactic antibiotics

b. Excision of the hematoma

c. Needle aspiration

d. Incision, drainage, and pressure

bandage

e. Observation alone

165. Animal and clinical studies

have shown that administration of

lactated Ringer's solution to patients

with hypovolemic shock may

a. Increase serum lactate concentration

b. Impair liver function

c. Improve hemodynamics by alleviating

the deficit in the interstitial

fluid compartment

d. Increase metabolic acidosis

e. Increase the need for blood transfusion

Items 166-167

An 18-year-old high school

football player is kicked in the left

flank. Three hours later he develops

hematuria. His vital signs are

stable.

166. The diagnostic tests performed

reveal extravasation of contrast

into the renal parenchyma.

Treatment should consist of

a. Resumption of normal daily activity

excluding sports

b. Exploration and suture of the laceration

c. Exploration and wedge resection of

the left kidney

d. Nephrostomy

e. Antibiotics and serial monitoring of

blood count and vital signs

Trauma and Shock 91

167. Initial diagnostic tests in the

emergency room should include

which of the following?

a. Retrograde urethrography

b. Retrograde cystography

c. Arteriography

d. Intravenous pyelogram

e. Diagnostic peritoneal lavage

168. True statements concerning

penetrating pancreatic trauma

include

a. Most injuries do not involve adjacent

organs

b. Management of a ductal injury to

the left of the mesenteric vessels is

Roux-en-Y pancreaticojejunostomy

c. Management of a ductal injury in

the head of the pancreas is pancreaticoduodenectomy

d. Small peripancreatic hematomas

need not be explored to search for

pancreatic injury

e. The major cause of death is exsanguination

from associated vascular

injuries

169. Rapid fluid resuscitation of

the hypovolemic patient after abdominal

trauma is significantly enhanced

by which of the following?

a. Placement of long 18-gauge subclavian

vein catheters

b. Placement of percutaneous femoral

vein catheters

c. Bilateral saphenous vein cutdowns

d. Placement of short, large-bore percutaneous

peripheral intravenous

catheters

e. Infusion of cold whole blood

170. Use of the pneumatic antishock

garment (PASG)

a. Elevates blood pressure by an

"autotransfusion" effect, with augmentation

of venous return and

cardiac output

b. Is not recommended for control of

persistent bleeding in the setting of

severe pelvic fracture

c. Increases peripheral vascular resistance

d. Expedites assessment of lower

body injuries in the trauma patient

e. Should be terminated by means of

prompt deflation as soon as the

trauma patient reaches the emergency

department

92 Surgery

171. Which of the following situations

would be an indication for

performance of a thoracotomy in

the emergency room?

a. Massive hemothorax following

blunt trauma to the chest

b. Blunt trauma to multiple organ systems

with obtainable vital signs in

the field but none on arrival in the

emergency room

c. Rapidly deteriorating patient with

cardiac tamponade from penetrating

thoracic trauma

d. Penetrating thoracic trauma and no

signs of life in the field

e. Penetrating abdominal trauma and

no signs of life in the field

172. A 22-year-old man sustains a

gunshot wound to the abdomen. At

exploration, an apparently solitary

distal small-bowel injury is treated

with resection and primary anastomosis.

On postoperative day 7,

small-bowel fluid drains through

the operative incision. The fascia

remains intact. The fistula output is

300 mL/day and there is no evidence

of intraabdominal sepsis.

Correct treatment includes

a. Early reoperation to close the fistula

tract

b. Broad-spectrum antibiotics

c. Total parenteral nutrition

d. Somatostatin to lower fistula output

e. Loperamide to inhibit gut motility

Trauma and Shock 93

173. A 26-year-old man sustains a

gunshot wound to the left thigh.

Exploration reveals that a 5-cm

portion of superficial femoral artery

is destroyed. Appropriate management

includes

a. Debridement and end-to-end anastomosis

b. Debridement and repair with an

interposition prosthetic graft

c. Debridement and repair with an

interposition arterial graft

d. Debridement and repair with an

interposition vein graft

e. Ligation and observation

174. The patient illustrated on the

chest x-ray film and contrast study

on the following page was hospitalized

after a car collision in which

he suffered blunt trauma to the

abdomen. He sustained several left

rib fractures, but was hemodynamically

stable. True statements about

the injury demonstrated in the

films include

a. The injury depicted is the most frequent

organ injury in the setting of

blunt trauma to the abdomen

b. Delayed operative repair is indicated

after the patient's rib fractures

are allowed to stabilize

c. Surgical treatment of this injury is

indicated during this hospitalization

d. Early repair of this injury is preferably

accomplished through a left

posterolateral thoracotomy

e. If this injury is incidentally discovered

during a surgical exploration,

it should not be repaired

94 Surgery

95

DIRECTIONS: Each group of questions below consists of lettered

options followed by numbered items. For each numbered item, select the

appropriate lettered option(s). Each lettered option may be used once,

more than once, or not at all. Choose exactly the number of options

indicated following each item.

Items 175-177

For each diagnostic technique

listed below, select the injuries it

reliably identifies.

a. Significant intraperitoneal bleeding

b. Injury of a retroperitoneal organ

c. Retroperitoneal (pelvic and visceral)

vascular extravasation

d. Minor lacerations of liver and

spleen

e. Subcapsular hematomas of liver

and spleen

f. Injury of the small intestine

g. Diaphragmatic injury

175. Diagnostic peritoneal lavage

(SELECT 2 INJURIES)

176. Abdominal computed tomography

(SELECT 3 INJURIES)

177. Visceral angiography

(SELECT 1 INJURY)

Items 178-180

For each scenario listed below,

choose the abdominal organs most

likely to be injured.

a. Diaphragm

b. Liver

c. Spleen

d. Small intestine

e. Large intestine

f. Kidneys

g. Stomach

h. Pancreas

i. Bladder

j. Great vessels (aorta/vena cava)

178. A motorist decelerates

rapidly after striking a stalled vehicle.

He is traveling at 55 mi/h at the

time of impact. He is wearing a seat

belt and his car is equipped with an

air bag. (SELECT 3 ORGANS)

179. A man is shot with a highvelocity

bullet that traverses his

mid-abdomen at the level of the

umbilicus. (SELECT 3 ORGANS)

180. An unsuspecting victim is

struck forcefully in the upper

abdomen by a mugger with a baseball

bat. (SELECT 4 ORGANS)

96 Surgery

Items 181-185

For each of the immediately

life-threatening injuries of the chest

listed below, select the proper intervention.

a. Endotracheal intubation

b. Cricothyroidotomy

c. Subxiphoid window

d. Tube thoracostomy

e. Occlusive dressing

181. Laryngeal obstruction (SELECT

1 INTERVENTION)

182. Open pneumothorax (SELECT

1 INTERVENTION)

183. Flail chest (SELECT 1 INTERVENTION)

184. Tension pneumothorax (SELECT

1 INTERVENTION)

185. Pericardial tamponade (SELECT

1 INTERVENTION)

Trauma and Shock 97

98

TRAUMA AND SHOCK

Answers

132. The answer is c. (Greenfield, 2/e, pp 284, 337.) The finding of an airfluid

level in the left lower chest with a nasogastric tube entering it after

blunt trauma to the abdomen is diagnostic of diaphragmatic rupture with

gastric herniation into the chest. This lesion needs to be fixed immediately.

With continuing negative pressure in the chest, each breath sucks more of

the abdominal contents into the chest and increases the likelihood of vascular

compromise of the herniated viscera. While the diaphragm is easily

fixed from the left chest, this injury should be approached from the

abdomen. The possibility of injury below the diaphragm after sufficient

blunt injury to rupture the diaphragm mandates examination of the

intraabdominal solid and hollow viscera; adequate exposure of the

diaphragm to allow secure repair is possible from this approach.

133. The answer is e. (Sabiston, 15/e, pp 308-312.) In the rapiddeceleration

injury associated with automobile crashes, the abdominal viscera

tend to continue moving anteriorly after the body wall has been

stopped. These organs exert great stress upon the structures anchoring

them to the retroperitoneum. Intestinal loops stretch and may tear their

mesenteric attachments, injuring and thrombosing the superior mesenteric

artery; kidneys and spleen may similarly shear their vascular pedicles. In

these injuries, however, ordinarily the intraabdominal pressure does not

rise excessively and diaphragmatic hernia is not likely. Diaphragmatic hernia

is primarily associated with compression-type abdominal or thoracic

injuries that increase intraabdominal or intrathoracic pressure sufficiently

to tear the central portion of the diaphragm.

134. The answer is d. (Sabiston, 15/e, pp 307-309.) The preeminent concern

in treatment of rib fractures is the prevention of pulmonary complications

(atelectasis and pneumonia), particularly for patients with preexisting

pulmonary disease, who are in danger of progressing to respiratory failure.

Attempts to relieve pain by immobilization or splinting, such as strapping

the chest, merely compound the problem of inadequate ventilation. Tube

thoracostomy is indicated only if pneumothorax is diagnosed. Mild pain

may be controlled with oral analgesics, and patients with minor fracture

injuries, if they can be closely monitored, may be managed at home with

appropriate instructions for coughing and deep breathing. Patients with

significant fractures or severe pain should be hospitalized. Rib fractures

in the elderly are particularly treacherous. Intercostal nerve blocks often

provide prolonged periods of pain relief and, together with appropriate

pulmonary physiotherapy, will inhibit development of respiratory complications.

Rib fractures are often associated with either intrathoracic or

intraabdominal injuries. In particular, fractures of the left chest wall should

arouse suspicion of splenic trauma. In equivocal cases, peritoneal lavage

will often be diagnostic. Rib fractures heal spontaneously, without need for

surgical fixation.

135. The answer is c. (Sabiston, 15/e, pp 312-325.) The diagnosis of

injuries resulting from blunt abdominal trauma is difficult; injuries are

often masked by associated injuries. Thus, trauma to the head or chest,

together with fractures, frequently conceals intraabdominal injury. Apparently

trivial injuries may rupture abdominal viscera in spite of the protection

offered by the rib cage. The structures most likely to be damaged in

blunt abdominal trauma are, in order of frequency, the spleen, kidney,

intestine, liver, abdominal wall, mesentery, pancreas, and diaphragm.

Abdominal paracentesis is a rapid, sensitive diagnostic test for patients

with suspected intraabdominal injury and may be extremely helpful in the

management of patients with associated head, thoracic, or pelvic trauma in

whom signs and symptoms of the abdominal injuries may be masked or

overlooked. Abdominal CT scans, which should be done promptly and

rapidly, are being used more frequently to evaluate these injuries.

136. The answer is c. (Sabiston, 15/e, p 333.) In the past, ligation rather

than repair of large veins in the extremities has been advocated in patients

with multiple injuries or severe trauma. Venous repair adds to the operative

time, often results in thrombosis and occlusion, and was thought to lead to

an increased incidence of pulmonary embolization. Recent studies, including

reviews of the Viet Nam Vascular Registry, indicate that the risk of pulmonary

embolization is not increased with repair and that vein repair, in

conjunction with arterial repair, increases limb salvage, particularly in

popliteal injuries. Venous repair may also be necessary in the presence of

Trauma and Shock Answers 99

extensive soft tissue trauma and an already severely compromised venous

return. Long-term follow-up reveals that the sequelae of chronic venous

insufficiency develop with increasing frequency in those patients who have

had ligations of lower-extremity veins. Morbidity from chronic deep

venous occlusion may be diminished even in those patients who develop

thrombosis following repair, because recanalization often occurs. Ligated

veins do not recanalize. For these reasons, it is currently recommended

that large veins be repaired whenever clinically feasible.

137. The answer is b. (Sabiston, 15/e, pp 332-333.) The five P's of arterial

injury include Pain, Paresthesias, Pallor, Pulselessness, and Paralysis.

In the extremities the tissues most sensitive to anoxia are the peripheral

nerves and striated muscle. The early developments of paresthesias and

paralysis are signals that there is significant ischemia present and immediate

exploration and repair are warranted. The presence of palpable pulses

does not exclude an arterial injury because this presence may represent a

transmitted pulsation through a blood clot. When severe ischemia is

present, the repair must be completed within 6-8 h to prevent irreversible

muscle ischemia and loss of limb function. Delay to obtain an angiogram or

to observe for change needlessly prolongs the ischemic time. Fasciotomy

may be required but should be done in conjunction with and after reestablishment

of arterial flow. Local wound exploration is not recommended

because brisk hemorrhage may be encountered without the securing of

prior vascular control.

138-139. The answers are 138-c, 139-a. (Sabiston, 15/e, p 308.) Tension

pneumothorax is a life-threatening problem requiring immediate

treatment. A lung wound that behaves as a ball or flap valve allows escaped

air to build up pressure in the intrapleural space. This causes collapse of

the ipsilateral lung and shifting of the mediastinum and trachea to the contralateral

side, in addition to compression of the vena cava and contralateral

lung. Sudden death may ensue because of a decrease in cardiac output;

hypoxemia; and ventricular arrhythmias. To accomplish rapid decompression

of the pleural space, a large-gauge needle should be passed into the

intrapleural cavity through the second intercostal space at the midclavicular

line. This may be attached temporarily to an underwater seal with subsequent

insertion of a chest tube after the life-threatening urgency has been

relieved.

100 Surgery

Tension pneumothorax produces characteristic x-ray findings of ipsilateral

lung collapse, mediastinal and tracheal shift, and compression of the

contralateral lung. Occasionally, adhesions prevent complete lung collapse,

but the tension pneumothorax is evident because of the mediastinal displacement.

A pleural effusion would not be expected acutely in the absence

of associated intrapleural blood.

140. The answer is a. (Schwartz, 7/e, pp 103-105.) Though the immediate

release of catecholamines causes a transient drop in the insulin levels,

shortly thereafter there is a significant rise in plasma insulin levels in

injured humans. Since injured patients are highly hypermetabolic, it might

be expected that the activity of the thyroid hormones would be increased

following injury. This is not the case, however, and increased levels of the

thyroid hormones are not seen. Vasopressin (ADH) is regulated by the

serum osmolality. In the postinjury period many factors are at play that

provoke the excretion of vasopressin. Glucagon secretion is normal or

increased after injury; not only are aldosterone levels elevated, but the

diurnal fluctuations ordinarily seen are lost.

141. The answer is d. (Schwartz, 7/e, pp 192-193.) Traumatic injury to

the common bile duct must be considered in two separate categories. Complete

transection of the common bile duct can be handled in many ways. If

the patient is unstable and time is limited, simply placing a T tube in either

end of the open common bile duct and staging the repair is the treatment

of choice. In a stable patient a biliary enteric bypass is preferred. This can

be accomplished by Roux-en-Y choledochojejunostomy or cholecystojejunostomy.

The jejunum is favored over the duodenum because if the anastomosis

leaks, a lateral duodenal fistula is avoided. For similar reasons the

defunctionalizing of the jejunal limb is also preferable. This can be accomplished

by creating a Roux-en-Y limb of jejunum. Primary end-to-end

repair of a completely transected common bile duct is not recommended

because of the high incidence of stricture and need for reoperation and creation

of a biliary enteric bypass. However, primary repair is the procedure

of choice if the common bile duct is lacerated or only partially transected.

142. The answer is d. (Schwartz, 7/e, pp 163-166.) Reports of a more

than 50% incidence of negative explorations of the neck, iatrogenic complications,

and serious injuries overlooked at operation have caused a

Trauma and Shock Answers 101

reassessment of the dictum that all penetrating neck wounds that violate

the platysma must be explored. Stable patients with zone III (between the

angle of the mandible and the skull) or zone I (inferior to the cricoid cartilage)

injuries, or multiple neck wounds, should undergo initial angiography

irrespective of the ultimate treatment plan. Algorithms exist for

nonoperative management of asymptomatic patients that employ observation

alone or combinations of vascular and aerodigestive contrast studies

and endoscopy. Nevertheless, recognition of acute signs of airway distress

(stridor, hoarseness, dysphonia), visceral injury (subcutaneous air, hemoptysis,

dysphagia), hemorrhage (expanding hematoma, unchecked external

bleeding), or neurologic symptoms referable to carotid injury (stroke or

altered mental status) or lower cranial nerve or brachial plexus injury

requires formal neck exploration. Pneumothorax would mandate a chest

tube; the necessity for exploration would depend on clinical judgment and

institutional policy.

143. The answer is b. (Schwartz, 7/e, pp 194-195.) Duodenal

hematomas result from blunt abdominal trauma. They present as a high

bowel obstruction with abdominal pain and occasionally a palpable right

upper quadrant mass. An upper gastrointestinal series is almost always

diagnostic with the classic coiled spring appearance of the second and third

portions of the duodenum secondary to the crowding of the valvulae conniventes

(circular folds) by the hematoma. Nonsurgical management is the

mainstay of therapy because the vast majority of duodenal hematomas

resolve spontaneously. Simple evacuation of the hematoma is the operative

procedure of choice. However, bypass procedures and duodenal resection

have been performed for this problem. In patients with duodenal obstruction

from the superior mesenteric artery syndrome, the obstruction is usually

the result of a marked weight loss and, in conjunction with this, loss of

the retroperitoneal fat pad that elevates the superior mesenteric artery from

the third and fourth portions of the duodenum. Nutritional repletion and

replenishment of this fat pad will elevate the artery off the duodenum and

relieve the obstruction.

144. The answer is b. (Schwartz, 7/e, pp 1884-1885.) Transection of a

peripheral nerve results in hemorrhage and in retraction of the severed

nerve ends. Almost immediately, degeneration of the axon distal to the

injury begins. Degeneration also occurs in the proximal fragment back to

102 Surgery

the fist node of Ranvier. Phagocytosis of the degenerated axonal fragments

leaves a neurilemmal sheath with empty cylindrical spaces where the axons

were. Several days following the injury, axons from the proximal fragment

begin to regrow. If they make contact with the distal neurilemmal sheath,

regrowth occurs at about the rate of 1 mm per day. However, if associated

trauma, fracture, infection, or separation of neurilemmal sheath ends precludes

contact between axons, growth is haphazard and a traumatic neuroma

is formed. When neural transection is associated with widespread

soft tissue damage and hemorrhage (with increased probability of infection),

many surgeons choose to delay reapproximation of the severed nerve

end for 3-4 wk.

145-147. The answers are 145-a, 146-e, 147-b. (Schwartz, 7/e, pp

179-180.) Closed head injuries may result in cerebral concussion from

depression of the reticular formation of the brainstem. This type of injury

is usually reversible.

Local bleeding and swelling (intracranial or extracranial) produce an

increase in the intracranial pressure. A characteristic symptom pattern

occurs initiated by progressive depression of mental status. Increasing

intracranial pressure tends to displace brain tissue away from the source of

the pressure; if the pressure is sufficient, herniation of the uncal process

through the tentorium cerebri occurs.

Pupillary dilation is caused by compression of the ipsilateral oculomotor

nerve and its parasympathetic fibers. If the pressure is not relieved, the

contralateral oculomotor nerve will become involved and, ultimately, the

brainstem will herniate through the foramen magnum and cause death.

Hypertension and bradycardia are preterminal events.

Emergency measures to reduce intracranial pressure while preparing

for localization of the clot or for a craniotomy or both include hyperventilation,

dexamethasone (Decadron), and mannitol infusion. Of these,

hyperventilation produces the most rapid decrease in brain swelling.

148. The answer is a. (Schwartz, 7/e, pp 688-689.) Flail chest is diagnosed

in the presence of paradoxical respiratory movement in a portion of

the chest wall. At least two fractures in each of three adjacent rib or costal

cartilages are required to produce this condition. Complications of flail

chest include segmental pulmonary hypoventilation with subsequent

infection and ultimately respiratory failure. Management of flail chest

Trauma and Shock Answers 103

should be individualized. If adequate pain control and pulmonary toilet

can be provided, patients may be managed without stabilization of the flail.

Often, intercostal nerve blocks and tracheostomy aid in this form of management.

If stabilization is required, external methods such as sandbags or

towel clips are no longer used. Surgical stabilization with wires is used if

thoracotomy is to be performed for another indication. If this is not the

case, "internal" stabilization is performed by placing the patient on

mechanical ventilation with positive end-expiratory pressure. Tracheostomy

is recommended because these patients usually require 10-14 days

to stabilize their flail segment and postventilation pulmonary toilet is simplified

by tracheostomy. Indications for mechanical ventilation include significant

impedance to ventilation by the flail segment, large pulmonary

contusion, an uncooperative patient (e.g., owing to head injury), general

anesthesia for another indication, more than five ribs fractured, and the

development of respiratory failure.

149. The answer is c. (Schwartz, 7/e, pp 2048-2050.) The motor components

of the median nerve maintain the muscular function of most of the

long flexors of the hand as well as the pronators of the forearm and the

thenar muscles. The median nerve is also an extremely important sensory

innervator of the hand and is commonly described as the "eye of the hand"

because the palm, the thumb, and the index and middle fingers all receive

their sensation via the median nerve.

150. The answer is d. (Schwartz, 7/e, pp 238-239.) Carbon monoxide

(CO) is the leading cause of toxin-related death in the United States. It is

produced by the incomplete combustion of fossil fuels and is emitted by

virtually all gas-powered engines and appliances that burn fossil fuel, e.g.,

home furnaces, water heaters, stoves, pool heaters, kerosene heaters, and

charcoal fires. Tobacco smoke-particularly smoke released from the tip of

the cigarette, which has 2.5 times more CO than inhaled smoke-produces

a significant amount of the gas; nonsmokers working in closed quarters

with smokers may have carboxyhemoglobin (COHb) levels as high as 15%,

easily enough to cause headache and some impairment of judgment. Fire

fighters are at particularly high risk for CO intoxication. The pathophysiology

of CO poisoning is unclear. It is known to cause an adverse shift in the

oxygen-hemoglobin dissociation curve, to cause direct cardiovascular

depression, and to inhibit cytochrome A3. Tissue hypoxia is the result.

104 Surgery

Treatment is directed toward increasing the partial pressures of O2 to which

the transalveolar hemoglobin is exposed. In most cases, administering

100% oxygen through a tightly fitted face mask will result in a serum elimination

half-life of COHb of 80 min (compared with 520 min when one

breathes room air). In severe cases, where coma, seizures, or respiratory

failure are present, the partial pressure of O2 is increased by administering

it in a hyperbaric chamber with an atmospheric pressure of 2.8. In this situation

the serum elimination half-life is reduced to 23 min. In any case, the

oxygen therapy should continue until the COHb levels reach 10%.

151. The answer is d. (Schwartz, 7/e, pp 169, 204.) Most pelvic fractures

are the result of automobile-pedestrian accidents and these fractures are a

frequent cause of death. The pelvis is extremely vascular with a diffuse

blood supply that makes hemorrhage common and surgical control of

bleeding difficult. This patient has a type II fracture (single break in pelvic

ring) through a non-weight-bearing portion of the pelvis. These fractures

are best treated by bed rest until hemodynamic stability is assured and

thereafter by gentle ambulation as pain permits. The clinician must watch

carefully for associated injuries to bladder, urethra, and colon and be alert

to the many other possible concurrent injuries to an elderly patient who

has suffered a collision, even a low-velocity attack from a pizza man.

152. The answer is d. (Schwartz, 7/e, pp 250-252.) The treatment of

electrical injury should be modified from that of thermal burns because tissue

damage is much deeper than is apparent at first inspection. The heat

generated is proportional to the resistance to the flow of current. Bone, fat,

and tendons offer the greatest resistance. Therefore, the tissue deep within

the center of an extremity may be injured while more superficial tissues are

spared. For this reason, the quantification of fluid requirements cannot be

based on the percentage of body surface area involved, as in the Parkland,

Brooke, or Baxter formulas, which are used to calculate fluid replacement

after thermal burns. Massive fluid replacement is usually essential. A brisk

urine output is desirable because of the likelihood of myonecrosis with

consequent myoglobinuria and renal damage. As with deep thermal burns,

debridement, skin grafting, and amputation of extremities may be required

following electrical injury. However, fasciotomy is more frequently

required than escharotomy with electrical injury because deep myonecrosis

results in increased intracompartmental pressures and compromised

Trauma and Shock Answers 105

limb perfusion. In addition, distant fractures may result owing to vigorous

muscle contraction during the accident or if subsequent falls occur. Cardiac

or respiratory arrest may occur if the pathway of the current includes

the heart or brain. An electrical current can also damage the pulmonary

alveoli and capillaries and lead to respiratory infections, a major cause of

death in these victims. Owing to the deep myonecrosis that often accompanies

high-voltage injury, prophylaxis for clostridia with high-dose penicillin

may be considered. Mafenide acetate is preferred over other topical

antimicrobials because of its deeper penetration of eschar.

153. The answer is a. (Bunt, Am J Surg 160:226-228, 1990.) In a 4-year

retrospective study of 569 at-risk parajoint fractures or dislocations resulting

from blunt trauma, there was only a 1.5% incidence of associated vascular

injury. Angiograms and vascular surgical consultations were obtained

when vascular compromise was suspected owing to clinical examination or

Doppler confirmation of flow abnormalities. While vascular injuries due to

fractures on either side of a joint (e.g., supracondylar femur fracture or tibial

plateau fracture) were uncommon, major joint dislocations were more

commonly associated with vascular injury. An exception to this rule is the

type III supracondylar humerus fracture, where displacement of bone may

injure or entrap the tethered brachial artery. Clavicular fractures are rarely

associated with significant vascular injury. The highest rate of vascular

injury occurs with knee dislocations because of the extreme force required

to dislocate the joint. In open elbow dislocations, the brachial artery is

often disrupted by forcible hyperextension of the joint; closed elbow dislocations

are rarely associated with vascular injury unless the dislocation is

anterior.

154. The answer is d. (Greenfield, 2/e, pp 317-331.) Gunshot wounds to

the lower chest are often associated with intraabdominal injuries. The

diaphragm can rise to the level of T4 during maximal expiration. Therefore,

any patient with a gunshot wound below the level of T4 should be

subjected to abdominal exploration. Exploratory thoracotomy is not indicated

because most parenchymal lung injuries will stop bleeding and heal

spontaneously with the use of tube thoracostomy alone. Indication for thoracic

exploration for bleeding is usually in the range of 100-150 mL/h over

several hours. Peritoneal lavage is not indicated even though the abdominal

examination is unremarkable. As many as 25% of patients with nega-

106 Surgery

tive physical findings and negative peritoneal lavage will have significant

intraabdominal injuries in this setting. These injuries include damage to

the colon, kidney, pancreas, aorta, and diaphragm. Local wound exploration

is not recommended because the determination of diaphragmatic

injury with this technique is unreliable.

155. The answer is e. (Sabiston, 15/e, pp 297-298.) Loss of consciousness

following head trauma should be assumed to be due to intracranial hemorrhage

until proved otherwise. However, a thorough evaluation of the

head-injured patient includes assessment for other potentially lifethreatening

injuries. Rarely, a patient may have sufficient hemorrhage from

a scalp laceration to cause hypotension. In the patient described, hypotension

and tachycardia should not be uncritically attributed to the head

injury, since these findings in the setting of blunt trauma are suggestive of

serious thoracic, abdominal, or pelvic hemorrhage. When cardiovascular

collapse occurs as a result of rising intracranial pressure, it is generally

accompanied by hypertension, bradycardia, and respiratory depression.

156. The answer is b. (Sabiston, 15/e, pp 324-325.) Because of the colon's

poor blood supply and its fecal content, colon injuries are more difficult to

manage than small-bowel injuries. Recently the necessity of mandatory

colostomy for civilian colon injuries has been questioned. About 85% of

civilian colon injuries are small wounds from low- or medium-velocity gunshots

or stab wounds, which are less likely to produce gross fecal spillage.

These injuries can be repaired primarily in the absence of gross contamination,

regardless of the right- or left-sided location of injury. Shock on admission

and multiple associated injuries are not universally viewed as absolute

contraindications to primary repair in such cases. Gross contamination or

large amounts of hard intraluminal feces remain generally accepted contraindications

to primary repair. Alternatives include end colostomy with

mucous fistula or Hartmann's pouch, exteriorization of a primary repair,

and protection of a primary repair in the distal colon by formation of a proximal

colostomy. In all cases in which traumatic colon injury is suspected, the

early administration of broad-spectrum intravenous antibiotics seems to

reduce the incidence of postoperative infectious complications.

157. The answer is d. (Sabiston, 15/e, pp 269-270.) Because they are so

often malnourished and at high risk for other conditions that alter their

Trauma and Shock Answers 107

immunocompetence, drug addicts have an extraordinary susceptibility to

infections of the type that can quickly progress to threaten life and limb.

Among the most virulent are those that give rise to anaerobic cellulitis.

Terms sometimes used for these infections are gas abscess, gangrenous cellulitis,

localized gas gangrene, and epifascial gangrene. Suppuration and extensive

gas formation are common and usually localized, unlike the infections

associated with myonecrosis. These lesions may be clostridial or nonclostridial.

Clostridium perfringens is the most common culprit, but anaerobic

cellulitis and gas formation have been associated with a variety of

obligate anaerobes including Bacteroides species, Peptostreptococcus, and

Peptococcus, and the gram-negative enteric bacilli (E. coli, Klebsiella),

staphylococci, and streptococci. Pseudomonas aeruginosa is not implicated

in these aggressive infections. Since the progressive injury results from liberation

of bacterial exotoxins, antitoxin administration at this stage is futile.

Treatment is determined by immediate inspection of a Gram stain of the

thin, dark, malodorous wound drainage or a needle aspirate of the crepitant

area: if large, "boxcar-shaped" gram-positive bacilli are present, it is a

clostridial infection and high doses of parenteral penicillin G (20 million

U/day) are indicated; if a polymicrobial Gram stain is seen, clindamycinaminoglycoside

should be added until specific sensitivities are known.

Aggressive debridement is always indicated.

158. The answer is c. (Dubrow, Surgery 106:267-273, 1989. Miller, Arch

Surg 124:805-807, 1989.) The spectrum of blunt cardiac injuries includes

myocardial contusion, rupture, and internal (chamber and septal) disruptions

such as traumatic septal defects, papillary muscle tears, and valvular

tears. Myocardial contusions are by far the most common of these injuries.

They usually occur in persons who sustain a direct blow to the sternum, as

seen in a driver whose sternum is forcibly compressed by the steering column

in a deceleration injury. Over 50% of patients with myocardial contusion

demonstrate external signs of thoracic trauma, including sternal

tenderness, abrasions, ecchymosis, palpable crepitus, rib fractures, or flail

segments. Overall, fewer than 10% of patients have conduction abnormalities,

dysrhythmias, or ischemic patterns on initial ECG. Elevated cardiac

isoenzyme levels are specific for myocardial injury, but they lack clinical

significance in patients without ECG abnormalities or hemodynamic instability.

First-pass radionuclide angiography (RNA) and echocardiography

provide sensitive assessment of ventricular wall motion and ejection frac-

108 Surgery

tion after blunt chest trauma and are currently viewed as the "gold standard"

for the diagnosis of myocardial contusion. But while RNA and

echocardiography sensitively detect small abnormalities in myocardial

function, they are poor predictors of the significant cardiac complications

of pump failure and arrhythmia. Traditionally, management of patients

with myocardial contusion has included continuous ECG monitoring in an

intensive care unit for 48-72 h, even in hemodynamically stable patients

without other injuries. Because of the large number of patients with blunt

chest trauma from automobile accidents, however, this policy has been

scrutinized. Virtually all patients who develop cardiac complications display

ECG abnormalities on arrival in the emergency room or within the

first 24 h. Since an abnormal ECG is a good predictor of subsequent complications,

stable patients with possible myocardial contusions but with a

normal ECG tracing may be placed on telemetry for 24 h, rather than monitored

in an ICU.

159. The answer is d. (Weissman, Anesthesiology 73:308-327, 1990.)

Injury and sepsis result in accelerated protein breakdown with increased

urinary nitrogen loss and increased peripheral release of amino acids. The

negative nitrogen balance represents the net result of breakdown and synthesis

(with breakdown increased and synthesis increased or diminished).

Amino acids such as alanine are released by muscle and transported to the

liver for incorporation into acute-phase proteins including fibrinogen,

complement, haptoglobin, and ferritin. The amino acids also undergo gluconeogenesis

to glucose, which is utilized primarily by the brain and other

glycolytic tissues such as peripheral nerves, erythrocytes, and bone marrow.

Other tissues receive energy from fat in the form of fatty acids or

ketone bodies during starvation following major trauma; this helps to conserve

body protein. Glutamine is the most abundant amino acid in the

blood, and its levels in muscle and blood decrease following injury and

sepsis as it is consumed rapidly by replicating fibroblasts, lymphocytes,

and intestinal endothelial cells. The use of glutamine may decrease protein

catabolism in the intestine and may help prevent atrophy of the gastrointestinal

tract in starved and parenterally nourished patients. Along with the

counterregulatory hormones (glucagon, epinephrine, cortisol), interleukin

1 appears to mediate muscle breakdown. Recent studies have indicated

that TNF (also called cachectin because of the role it plays in muscle wasting

in septic or oncologic patients) also may be a principal catabolic

Trauma and Shock Answers 109

cytokine in the traumatized patient. This protein is secreted by

macrophages and further affects metabolism by inducing secretion of interleukin

1 and inhibiting synthesis and activity of lipogenic enzymes.

160. The answer is e. (Sabiston, 15/e, pp 324-325. Schwartz, 7/e, pp

199-200.) Penetrating injury to the intraperitoneal or extraperitoneal rectum

should be diagnosed by immediate sigmoidoscopy. Contrast studies of

the rectum, when sigmoidoscopy is inconclusive, should use a watersoluble

radiopaque medium such as Gastrografin. The use of barium is

contraindicated because its spillage in the peritoneal cavity mixed with

feces would increase the likelihood of subsequent intraabdominal

abscesses. Instrumentation of the bullet track is also contraindicated

because of the risk of injury to adjacent structures (e.g., bladder, ureters,

iliac vessels). Angiography is not a sensitive method for demonstrating

injury of the intestinal wall.

161. The answer is d. (Schwartz, 7/e, pp 188-192.) The overwhelming

majority of patients explored for blunt trauma to the liver sustain their

injuries in motor vehicle accidents. In a large consecutive series of 323

patients with blunt hepatic trauma who were explored for the finding of

hemoperitoneum on peritoneal lavage, the mortality was 31%. Forty-two

percent of the deaths, due primarily to liver injury, occurred intraoperatively

during the initial operation following admission. All operations were

performed at a regional trauma center by staff trauma surgeons. Their findings

included the following observations: (1) intraoperative deaths were

due to uncontrolled hemorrhage; (2) patients with major hepatic injuries

who survived operation but nevertheless died appeared to succumb either

to sepsis or to associated injuries, usually involving the head or chest; (3)

hepatic artery ligation for control of bleeding yielded dismal results-of the

3 surviving patients who underwent hepatic artery ligation (an additional

11 died), 2 required reoperation for continued bleeding; (4) the use of

drains (passive and active) was associated with a significantly greater incidence

of intraabdominal infectious complications; (5) intracaval shunting

was used in 7 severely injured patients without a survivor; (6) while minor

hepatic injuries required little or no treatment, major lacerations could

usually be controlled with simple absorbable sutures placed 2-3 cm from

the fracture edge, without occurrence of subsequent intrahepatic

hematoma, hemobilia, or bile fistulae; (7) hepatic fragmentation may be

110 Surgery

treated by nonanatomic debridement, with suture ligation of individual

bleeding points-of nine attempts at formal anatomic resection in stable

patients, all ended in uncontrollable hemorrhage and death.

162. The answer is a. (Schwartz, 7/e, pp 172-175, 204-206.) The presence

of ischemic changes following vascular trauma is an indication for

emergency exploration and repair. Nonsurgical management of arterial

trauma when distal pulses are palpable may lead to delayed sequelae of

embolization, occlusion, secondary hemorrhage, false aneurysm, and traumatic

arteriovenous fistula. The presence of palpable pulses does not reliably

exclude significant arterial injury. Injuries that may be missed if

exploration is not performed include lacerations and partial transections

containing hematomas, intramural or intraluminal thromboses, and intimal

disruptions or tears. Injury to motor nerves would be apparent on neurologic

examination. Injury to bone would be diagnosed by x-ray. Adjacent

venous injury, in the absence of an expanding hematoma, would not by

itself mandate exploration because there are numerous collateral venous

channels in the extremities. Prophylactic fasciotomy is not routinely performed

for all arterial injuries but is indicated in the presence of an

ischemic period exceeding 4-6 h, combined arterial and major venous

injury, prolonged periods of hypotension, massive associated soft tissue

trauma, and massive edema.

163. The answer is d. (Schwartz, 7/e, pp 102-105.) The biochemical

changes associated with shock result from tissue hypoperfusion, endocrine

response to stress, and specific organ system failure. During shock, the

sympathetic nervous system and adrenal medulla are stimulated to release

catecholamines. Renin, angiotensin, antidiuretic hormone, adrenocorticotropin,

and cortisol levels increase. Resultant changes include sodium

and water retention and an increase in potassium excretion, protein catabolism,

and gluconeogenesis. Potassium levels rise as a result of increased

tissue release, anaerobic metabolism, and decreased renal perfusion. If

renal function is maintained, potassium excretion is high and normal

plasma potassium levels are restored.

164. The answer is d. (Sabiston, 15/e, pp 1277-1278.) A subperichondrial

hematoma in the pinna of the ear may lead to avascular necrosis of the

cartilage with shriveling of the pinna and fibrosis and calcification of the

Trauma and Shock Answers 111

hematoma. The result is the deformity known as "cauliflower ear." Appropriate

treatment consists of evacuation of the hematoma by incision and

tight packing of the skin and perichondrium onto the cartilage with a pressure

dressing. Needle aspiration does not effect adequate drainage. Ice

packs may be helpful early, but are not sufficient to prevent the deformity;

antibiotics are not indicated for this lesion. Since the hematoma is subperichondrial,

excision of the hematoma would remove the perichondrium

and lead to cartilage deformities.

165. The answer is c. (Sabiston, 15/e, pp 83-84, 123.) Infusion of lactated

Ringer's solution is an effective immediate step, both clinically and experimentally,

in managing hypovolemic shock. Use of this balanced salt solution

helps correct the fluid deficit (in the extracellular, extravascular

compartment) resulting from hypovolemic shock. This procedure may

decrease requirements for whole blood in patients with hemorrhagic

shock. If blood loss has been minimal and is controlled, whole blood transfusion

may be avoided entirely. The theoretical objection to infusion of lactated

Ringer's solution is that it will increase lactate levels and compound

the problem of lactic acidosis. This has not been borne out in animal or

clinical studies. Along with the hemodynamic improvement that follows

volume restitution, liver function improves, lactate metabolism is

improved, excess lactate levels drop, and metabolic acidosis improves.

166-167. The answers are 166-e, 167-d. (Cass, Urol Clin North Am

16:213-220, 1989.) In stable patients with suspected genitourinary tract

injury, the first urologic study other than urinalysis should be the intravenous

urogram. The technique of high-dose drip infusion is desirable

because the high concentration of contrast achieved greatly facilitates interpretation

in an unprepared patient. Intravenous pyelography should be

performed before retrograde cystography to avoid obscuring visualization

of the lower ureteral tract. The study also may preclude the need for retrograde

urethrography in cases where, unlike the case presented, there is a

suspicion of urethral injury. Renal arteriography is not indicated routinely

but should be performed to rule out renal pedicle injury when no kidney

function is demonstrated by drip infusion urography. Peritoneal lavage is

not useful in the diagnosis of genitourinary injuries because the structures

are retroperitoneal. Seventy to eighty percent of patients with blunt renal

trauma are successfully treated nonsurgically. Bed rest may reduce the like-

112 Surgery

lihood of secondary hemorrhage; antibiotics may reduce the chance of

infection's developing in a perirenal hematoma. Failure of conservative

treatment is indicated by rising fever, increasing leukocytosis, evidence of

secondary hemorrhage, and persistent or increasing pain and tenderness in

the region of the kidney.

168. The answer is e. (Sabiston, 15/e, pp 315-317.) The majority of penetrating

pancreatic injuries can be managed with simple drainage. Injury to

the major pancreatic duct to the left of the mesenteric vessels is effectively

treated with a distal pancreatectomy. The high morbidity and mortality of

a pancreaticoduodenectomy for trauma limit its use to extensive blunt

injuries to both pancreatic head and duodenum. For ductal injury in the

region of the head of the pancreas, a Roux-en-Y limb of jejunum should be

brought up and used to drain the transected duct. The proximity of the

pancreas to many other major structures makes combined injuries frequent

(90%). Complications of pancreatic injury include fistula, pseudocyst, and

abscess, but the cause of death in patients with pancreatic injury is most

frequently exsanguination from associated injury to major vascular structures

such as the splenic vessels, mesenteric vessels, aorta, or inferior vena

cava. Finally, however small, all peripancreatic hematomas should be

explored to search for pancreatic injury. Simple drainage is usually adequate

treatment in such cases, but failure to recognize a pancreatic injury

can have catastrophic sequelae.

169. The answer is d. (Dutky, J Trauma 29:856-860, 1989.) Rapid fluid

administration is often the key to successful trauma resuscitation. Some of

the important factors affecting the rate of fluid resuscitation include the

diameter of the intravenous tubing, the size and length of the venous cannulae,

the fluid viscosity, and the site of administration. According to

Poiseuille's law, flow is proportional to the fourth power of the radius of a

catheter and inversely proportional to its length. Therefore, the shorter a

catheter and the larger its diameter, the faster one can infuse a solution

through it. Central venous placement alone does not assure rapid flow.

Importantly, the diameter of the intravenous tubing employed may be the

rate-determining factor in fluid delivery: blood-infusion tubing allows

twice the flow of standard intravenous tubing and should be used when

rapid fluid resuscitation is needed. Any patient who is suspected of having

a major abdominal injury should immediately have at least two short,

Trauma and Shock Answers 113

large-bore (16-gauge or larger) intravenous cannulae placed in peripheral

veins. Longer, smaller catheters, such as standard 18-gauge central venous

catheters, may take more time to place and will have lower flow rates. Once

fluid resuscitation is under way, one may elect to place an 8- or 9-French

pulmonary artery catheter-introducer via a central venous approach for

further volume administration, as well as for measurement of central

venous pressure or for Swan-Ganz catheter insertion. Lower-extremity

venous cannulae, placed by saphenous vein cutdown or percutaneously

into the femoral veins, are no longer advised as primary access for patients

with abdominal trauma, since possible disruption of iliac veins or the inferior

vena cava will render volume infusion ineffective. Studies have demonstrated

that the flow rate of cold whole blood is roughly two-thirds that of

whole blood at room temperature. Diluting and warming the blood by

"piggybacking" it into infusion lines that are delivering crystalloid will

decrease the blood's viscosity, enhance flow, and minimize hypothermia.

170. The answer is c. (Flint, Ann Surg 211:703-707, 1990. Trunkey, Can J

Surg 27:479-486, 1984.) The pneumatic antishock garment (PASG) is composed

of inflatable overalls with three compartments, two for the legs and

one for the abdomen. It has now been convincingly demonstrated that the

PASG elevates blood pressure by increasing peripheral vascular resistance

rather than by an "autotransfusion" effect on venous return and increased

cardiac output. The PASG is beneficial for controlling bleeding from pelvic

fractures by reduction of pelvic volume and immobilization to restrict fracture

movement. The suit pressure must be released very slowly because

rapid deflation can lead to sudden, irreversible hypotension. This is probably

due to a sudden decrease in peripheral vascular resistance and to the

effects of vasodilation and wash-out of accumulated metabolites of capillary

beds under the suit. Upon reperfusion of the lower body, a systemic

metabolic acidemia with hyperkalemia may result and must be closely

monitored. For these reasons satisfactory intravenous volume must be

attained prior to decompression of the PASG, a delay that may prevent adequate

early evaluation of concealed injuries to the lower body.

171. The answer is c. (Schwartz, 6/e, p 675.) Although indications for

thoracotomy in the emergency room are controversial, the procedure

appears to be most beneficial when it is employed to (1) release cardiac

tamponade in patients with penetrating thoracic trauma who are deterio-

114 Surgery

rating too rapidly for a subxiphoid pericardial window to be created; (2)

allow cross-clamping of the descending aorta in patients with intraabdominal

bleeding for whom other measures are not effective in maintaining

blood pressure; and (3) allow effective internal cardiac massage in patients

who arrive in the emergency room with faint or absent pulses and distant

heart sounds, and for whom other resuscitative efforts are unsuccessful. By

contrast, existing evidence suggests that patients who are unsalvageable

and do not benefit from emergency room thoracotomy include (1) those

with no vital signs (pulse, pupillary reaction, spontaneous respiration) in

the field and (2) those with blunt trauma to multiple organ systems and

absent vital signs upon arrival in the emergency room.

172. The answer is c. (Schwartz, 6/e, pp 1181-1182.) Most enterocutaneous

fistulas result from trauma sustained during surgical procedures.

Irradiated, obstructed, and inflamed intestine is prone to fistulization.

Complications of fistulas include fluid and electrolyte depletion, skin

necrosis, and malnutrition. Fistulas are classified according to their location

and the volume of output, because these factors influence prognosis

and treatment. When the patient is stable, a barium swallow is obtained to

determine (1) the location of the fistula, (2) the relation of the fistula to

other hollow intraabdominal organs, and (3) whether there is distal

obstruction. Proximal small-bowel fistulas tend to produce a high output

of intestinal fluid and are less likely to close with conservative management

than are distal, low-output fistulas. Small-bowel fistulas that communicate

with other organs, particularly the ureter and bladder, may need aggressive

surgical repair because of the risk of associated infections. The presence of

obstruction distal to the fistula (e.g., an anastomotic stricture) can be diagnosed

by barium contrast study and mandates correction of the obstruction.

When these poor prognostic factors for stabilization and spontaneous

closure are observed, early surgical intervention must be undertaken. The

patient in the question, however, appears to have a low-output, distal enterocutaneous

fistula. Control of the fistulous drainage should be provided by

percutaneous intubation of the tract with a soft catheter. This is usually

accomplished under fluoroscopic guidance. Antispasmodic drugs have not

been proved effective; somatostatin has been used with mixed success in

the setting of high-output (greater than 500 mL/day) fistulas. There is no

indication for antibiotics in the absence of sepsis. Total parenteral nutrition

(TPN) is given to maintain or restore the patient's nutritional balance while

Trauma and Shock Answers 115

minimizing the quantity of dietary fluids and endogenous secretions in the

gastrointestinal tract. A period of 4-6 wk of TPN therapy is warranted to

allow for spontaneous closure of a low-output, distal fistula. Should conservative

management fail, surgical closure of the fistula is performed.

173. The answer is d. (Schwartz, 6/e, pp 981-982.) Traumatic arterial

injuries can be handled with several techniques. The basic principles of

debridement of injured tissue and reestablishment of flow should be

observed. Primary end-to-end anastomosis is preferable if this can be

accomplished without tension. When 5 cm of artery has been destroyed, it

is impossible to perform a tension-free primary anastomosis, and a

reversed saphenous vein graft is the repair of choice. Ligation of the artery

is to be avoided in order to prevent gangrene and limb loss. The use of

prosthetic material (Gore-Tex) in a potentially infected field is also to be

avoided as infection at the suture line often leads to delayed hemorrhage.

Harvesting an arterial graft of similar diameter from elsewhere in the body

is hazardous and unnecessary when vein is available.

174. The answer is c. (Cameron, 4/e, pp 820-824.) Traumatic injuries to

the diaphragm are associated with both blunt and penetrating trauma. The

spleen, kidneys, intestines, and liver are the most frequently injured

abdominal organs in blunt trauma; the diaphragm is the least. Missed

injuries lead to problems with herniation and bowel strangulation with sufficient

frequency that repair should not be delayed. All such injuries

require repair once the diagnosis is made and the patient has been stabilized.

Most acute defects in the diaphragm can be repaired via an abdominal

approach, which allows exploration for coexisting injuries.

175-177. The answers are 175-a, d; 176-a, b, e; 177-c. (Davis, pp

2789-2790. Walters, Surg Gynecol Obstet 165:496-502, 1988.) Peritoneal

lavage is a diagnostic technique used to identify occult intraperitoneal injury

in patients with abdominal trauma. An abnormal lavage is obtained when the

lavage effluent exceeds allowable levels of blood, bile, or amylase; the presence

of vegetable matter also constitutes an abnormal result. Lavage has been

used most widely in the triage of hemodynamically stable victims of abdominal

trauma who are suspected of having significant injuries but who manifest

equivocal physical findings. Further indications for lavage are the

suspicion of abdominal injury in patients with altered sensoria, patients with

116 Surgery

unexplained blood loss, and patients who require general anesthesia to treat

other injuries. The technique is exquisitely sensitive to intraabdominal bleeding

and will detect as little as 20 mL of free blood in the peritoneal cavity.

Because stable retroperitoneal hematomas and minor lacerations of the liver

and spleen often shed sufficient blood to produce a positive lavage, some

authors have advocated abdominal CT as the preferred method of identifying

occult operable injuries of the abdomen. Also, CT with oral and intravenous

contrast can provide accurate images of the injured retroperitoneum and the

solid intraabdominal viscera (as lavage cannot). Neither CT nor lavage has

been a reliable indicator of small intestinal and diaphragmatic injuries, and

neither has been useful in obtaining hemostasis nonoperatively. Angiography,

however, may be employed to demonstrate visceral or pelvic arterial extravasation

and to control hemorrhage by selective embolization.

178-180. The answers are 178-d, e, f; 179-d, e, j; 180-b, c, f, h.

(Schwartz, 6/e, pp 193-219.) Deceleration injuries commonly result from

high-speed motor vehicle accidents and falls from considerable heights.

The mechanism of injury is the shearing of pedicled organs from their

points of attachment to the retroperitoneum. Because these pedicles are

usually vascular, the injury results in bleeding and ischemia of the affected

organ. Pedicled organs in the abdomen include the intestines (small and

large) and the kidneys. Deceleration injuries to the aorta occur in the mediastinum

and are usually fatal.

The small intestine and its mesentery is by far the most commonly

injured abdominal organ in penetrating trauma because of its sheer mass

and central location. A midline bullet at the level of the umbilicus is most

likely to strike small intestine, the transverse colon, and perhaps the aorta

or vena cava. The great vessels bifurcate just at the level of the umbilicus.

The diaphragm, stomach, and pancreas would be superior to this injury;

the bladder below; and the liver, spleen, and kidneys lateral.

The relative incidence of organ injury in blunt trauma is highest for

solid organs (spleen, liver, and kidneys). Although hollow viscera are less

likely to be injured by blunt trauma, this rule does not apply when the hollow

viscus is full; for example, rupture of a full urinary bladder is frequently

described when blunt force is applied to the lower abdomen. In

addition to the spleen, liver, and kidneys, extreme blunt force to the upper

abdomen may fracture the pancreas, which is susceptible to injury because

of its position overlying the rigid spinal column.

Trauma and Shock Answers 117

181-185. The answers are 181-b, 182-e, 183-a, 184-d, 185-c.

(Schwartz, 6/e, pp 672-684.) Flail chest describes the paradoxical motion of

the chest wall that occurs when consecutive ribs are broken in more than

one place, usually following blunt trauma to the thorax. Respiratory distress

may ensue when the noncompliant flail segment interferes with generation

of adequate positive and negative intrathoracic pressure needed to

move air through the trachea. In addition, a blow sufficiently violent to

cause a flail chest may also contuse the underlying pulmonary

parenchyma, which compounds the respiratory distress. Treatment consists

of stabilizing the chest wall. Although some temporary benefit may be

gained by external buttressing of the chest (e.g., with sandbags, or by turning

the patient onto the affected side), endotracheal intubation provides

rapid and safe control of the airway, as well as stabilization of the chest

internally by positive pressure ventilation.

Airway obstruction denotes partial or complete occlusion of the tracheobronchial

tree by foreign bodies, secretions, or crush injuries of the

upper respiratory tract. Patients may present with symptoms ranging from

cough and mild dyspnea to stridor and hypoxic cardiac arrest. An initial

effort should be made to digitally clear the airway and to suction visible

secretions; in selected, stable patients, fiberoptic endoscopy may be

employed to determine the cause of obstruction and to retrieve foreign

objects. Unstable patients whose airways cannot be quickly reestablished

by clearing the oropharynx must be intubated. An endotracheal intubation

may be attempted, but cricothyroidotomy is indicated in the presence of

proximal obstruction or severe maxillofacial trauma.

Blunt or penetrating trauma to the pericardium and heart will result in

pericardial tamponade when fluid pressure in the pericardial space exceeds

central venous pressure and thus prevents venous return to the heart. The

result is shock, despite adequate volume and myocardial function. The treatment

is pericardial decompression. A subxiphoid, supradiaphragmatic incision

and creation of a pericardial "window," ideally performed in the

operating room, provides a rapid, safe means of confirming the diagnosis of

tamponade and of relieving venous obstruction. If heavy bleeding is encountered

on opening the pericardial window, a sternotomy may be performed.

Tension pneumothorax occurs when a laceration of the visceral pulmonary

pleura acts as a one-way valve that allows air to enter the pleural

space from an underlying parenchymal injury but not to escape. Increasing

intrapleural pressure causes collapse of the ipsilateral lung, compression of

118 Surgery

the contralateral lung due to mediastinal shift toward the opposite

hemithorax, and diminished venous return. Treatment consists of relieving

the pneumothorax. This is best accomplished by tube thoracostomy.

Open pneumothorax occurs when a traumatic defect in the chest wall

permits free communication of the pleural space with atmospheric pressure.

If the defect is larger than two-thirds of the tracheal diameter, respiratory

efforts will move air in and out through the defect in the chest wall

rather than through the trachea. The immediate treatment is placement of

an occlusive dressing over the defect; subsequent interventions include

placement of a thoracostomy tube (preferably through a separate incision),

formal closure of the chest wall, and ventilatory assistance if needed.

Trauma and Shock Answers 119

This page intentionally left blank.

TRANSPLANTS,

IMMUNOLOGY,AND

ONCOLOGY

Questions

DIRECTIONS: Each item below contains a question or incomplete

statement followed by suggested responses. Select the one best response to

each question.

121

186. Tissue injury or infection

results in the release of tumor

necrosis factor (TNF) by which of

the following cells?

a. Fibroblasts

b. Damaged vascular endothelial cells

c. Monocytes/macrophages

d. Activated T lymphocytes

e. Activated killer lymphocytes

187. A cross-match is performed

by incubating

a. Donor serum with recipient lymphocytes

and complement

b. Donor lymphocytes with recipient

serum and complement

c. Donor lymphocytes with recipient

lymphocytes

d. Recipient serum with a known

panel of multiple donor lymphocytes

e. Recipient serum with donor red

blood cells and complement

188. In order to activate

helper/inducer T (CD41) lymphocytes,

macrophages release

a. Interleukin 1

b. Interleukin 2

c. Interleukin 3

d. Interleukin 4

e. Interferon

189. Which of the following cells

cause immunologically restricted

tumor cell lysis?

a. Macrophages

b. Cytotoxic T lymphocytes

c. Natural killer cells

d. Polymorphonuclear leukocytes

e. Helper T lymphocytes

Terms of Use

190. The primary mechanism of

action of cyclosporine A is inhibition

of

a. Macrophage function

b. Antibody production

c. Interleukin 1 production

d. Interleukin 2 production

e. Cytotoxic T-cell effectiveness

Items 191-192

A 24-year-old woman presents

with lethargy, anorexia, tachypnea,

and weakness. Laboratory studies

reveal a BUN of 150 mg/dL, serum

creatinine of 16 mg/dL, and potassium

of 6.2 meq/L. Chest x-ray

shows increased pulmonary vascularity

and a dilated heart.

191. Management of this patient

would include

a. Emergency kidney transplantation

b. Creation and immediate use of a

forearm arteriovenous fistula

c. Sodium polystyrene sulfonate

(Kayexalate) enemas

d. A 100-g protein diet

e. Cardiac biopsy via femoral vein

catheterization

192. In the course of 3 mo of treatment,

the patient's congestive heart

failure resolves, the lethargy and

weakness diminish markedly, and

she is able to return to work parttime.

Family immune profile studies

reveal that her mother and her

father both are haplotype identical

with regard to HLA antigens and

that her sister is a six-antigen

match. The patient at this time

should be urged to

a. Continue hemodialysis three times

a week

b. Undergo cadaveric renal transplantation

c. Accept a kidney transplant from

her sister

d. Accept a kidney transplant from

her father

e. Accept a kidney transplant from

her mother

193. After the first postoperative

year of cardiac transplantation, the

most common cause of death is

a. Infection

b. Arrhythmia

c. Accelerated graft arteriosclerosis

d. Acute rejection episode

e. Cancer

122 Surgery

194. Which of the following precludes cadaveric renal transplantation?

a. Positive cross-match

b. Donor blood type O

c. Two-antigen HLA match with donor

d. Blood pressure of 180/100 mm Hg

e. Hemoglobin level of 8.2 g/dL

195. Which of the following statements regarding hyperacute rejection of

a transplanted kidney is true?

Transplants,Immunology,and Oncology 123

a. It is mediated by preformed donor antibodies against recipient HLA antigens

b. It can be prevented by performing lymphocytotoxicity cross-match testing

c. It is manifest grossly by a swollen, pale kidney at the time of transplant surgery

d. This form of rejection is associated with disseminated intravascular coagulation

(DIC)

e. The rejection process can be treated with a steroid bolus and OKT3

196. Which of the following statements

regarding heart transplantation

is true?

a. Heart transplants are matched by

size and ABO blood type rather

than tissue typing

b. Cadaveric graft survival is significantly

lower with heart transplants

as compared with renal transplants

c. Cold ischemia time for donor hearts

should not be more than 48 h

d. The upper age limit for heart transplant

eligibility is 55 years

e. The leading cause of death after the

first year of cardiac transplantation

is chronic rejection

197. A 47-year-old man with

hypertensive nephropathy develops

fever, graft tenderness, and

oliguria 4 wk following cadaveric

renal transplantation. Serum creatinine

is 3.1 mg/dL. A renal ultrasound

reveals mild edema of the

renal papillae but normal flow in

both the renal artery and renal vein.

Nuclear scan demonstrates sluggish

uptake and excretion. The

next most appropriate step is

a. Performing an angiogram

b. Decreasing steroid and cyclosporine

dose

c. Beginning intravenous antibiotics

d. Performing renal biopsy, steroid

boost, and immunoglobulin therapy

e. Beginning FK 506

198. Posttransplant cytomegalovirus

infection may cause

a. Plyelonephritis

b. GI ulceration and hemorrhage

c. Cholecystitis

d. Intraabdominal abscess

e. Parotitis

199. In centers with experienced

personnel, 1-year liver transplant

survival is now approximately

a. 95%

b. 80%

c. 65%

d. 50%

e. 35%

200. Graft-versus-host disease has

occurred with the transplantation

of which of the following?

a. Kidney

b. Lung

c. Heart

d. Bone marrow

e. Pancreas

201. Which of the following diseases

is appropriately treated with

combined heart-lung transplantation?

a. Primary pulmonary hypertension

b. Cystic fibrosis

c. End-stage emphysema

d. Idiopathic dilated cardiomyopathy

with long-standing secondary pulmonary

hypertension

e. End-stage pulmonary fibrosis secondary

to sarcoidosis

124 Surgery

202. Which of the following is

true regarding successful wholeorgan

pancreas transplantation in

type I diabetes?

a. It results in maintenance of normal

serum glucose levels

b. Recurrence of diabetic nephropathy

in simultaneously transplanted

kidneys is not prevented

c. Oral glucose tolerance tests remain

abnormal

d. The pathologic changes of diabetic

retinopathy are reversed

e. The rate of diabetic ulcers and

amputations in the lower extremities

is reduced

203. Which of the following is

true regarding bone marrow transplantation?

a. Marrow is highly immunogenic

and easily rejected by the nonimmunosuppressed

host

b. Marrow transplantation has not

been successful in the treatment of

aplastic anemias

c. Marrow transplantation has not

been successful in the treatment of

congenital immunodeficiency diseases

d. Marrow transplantation can be

used as a successful therapy for

stage IV breast cancer following

high-dose chemotherapy

e. Marrow transplantation must be

performed with low-level immunosuppression

to enhance the degree

of chimerism

204. Which of the following statements

is true of the major histocompatibility

complex (MHC)

proteins?

a. Only nonnucleated cells express

MHC class I proteins

b. B lymphocytes, antigen-presenting

cells, and vascular endothelium

express only MHC class II proteins

c. MHC class I proteins are encoded

by the HLA-D locus (DR, DP, and

DQ)

d. MHC class I proteins act as the

major targets for antibody-mediated

rejection of organ allografts and are

detected by cross-matching techniques

e. B cells recognize antigens bound to

MHC class II proteins

205. The most useful serum

marker for detecting recurrent disease

after treatment of nonseminomatous

testicular cancer is

a. Carcinoembryonic antigen (CEA)

b. α-fetoprotein (AFP)

c. Prostate-specific antigen (PSA)

d. CA125

e. p53 oncogene

Transplants,Immunology,and Oncology 125

206. An edentulous 72-year-old

man with a 50-year history of cigarette

smoking presents with a nontender,

hard mass in the lateral

neck. The simplest way to establish

an accurate histological diagnosis

of a neck mass suspected to be cancerous

a. Fine needle aspiration cytology

b. Bone marrow biopsy

c. Nasopharyngoscopy

d. CT scan of the head and neck

e. Sinus x-ray

207. Which of the following is

true regarding intravenous administration

of chemotherapy?

a. Subcutaneous extravasation of carmustine

(BCNU) or 5-fluorouracil

(5-FU) usually causes ulceration

b. Extravasation of doxorubicin rarely

causes serious ulceration because

the agent binds quickly to tissue

nucleic acid

c. Serious and progressive ulceration

can be expected following extravasation

of vincristine or vinblastine

d. Problems of wound healing should

be anticipated if systemic 5-FU

therapy is begun less than 2 wk

postoperatively

e. Administration of folinic acid prevents

most of the toxicity of

methotrexate, but does not help to

normalize wound healing

208. For which of the following

malignancies does histologic grade

best correlate with prognosis?

a. Lung cancer

b. Melanoma

c. Colonic adenocarcinoma

d. Hepatocellular carcinoma

e. Soft tissue sarcoma

126 Surgery

209. A mother notices an abdominal mass in her 3-year-old son while giving

him a bath. There is no history of any symptoms, but the boy's blood

pressure is elevated at 105/85 mm Hg. Metastatic workup is negative and

the patient is explored. The mass below is found within the left kidney.

Which of the following statements concerning this disease is correct?

Transplants,Immunology,and Oncology 127

a. This tumor is associated with aniridia, hemihypertrophy, and cryptochidism

b. The majority of patients present with an asymptomatic abdominal mass and

hematuria

c. Treatment with surgical excision, radiation, and chemotherapy results in survival

of less than 60% even in histologically low-grade tumors

d. Surgical excision is curative and no further treatment is ordinarily advised

e. This tumor is the most common malignancy in childhood

210. An 11-year-old girl presents to your office because of a family history

of medullary carcinoma of the thyroid. Physical examination is normal.

Which of the following tests would you perform?

a. Urine vanillylmandelic acid (VMA) level

b. Serum insulin level

c. Serum gastrin level

d. Serum glucagon level

e. Serum somatostatin level

211. A 37-year-old woman has

developed a 6-cm mass on her

anterior thigh over the past 10 mo.

The mass appears to be fixed to the

underlying muscle, but the overlying

skin is movable. The next most

appropriate step in management is

a. Above-knee amputation

b. Excisional biopsy

c. Incisional biopsy

d. Bone scan

e. Abdominal CT scan

212. A 50-year-old man is incidentally

discovered to have non-

Hodgkin's lymphoma confined to

the submucosa of the stomach during

esophagogastroduodenoscopy

for dyspepsia. Which of the following

statements is true regarding his

condition?

a. Surgery alone cannot be considered

adequate treatment

b. Combined chemotherapy and radiation

therapy, without prior resecton,

are not effective

c. Combined chemotherapy and radiation

therapy, without prior resection,

result in a high risk of severe

hemorrhage and perforation

d. Outcome (freedom from progression

and overall survival) is related

to the histological grade of the

tumor

e. The stomach is the most common

site for non-Hodgkin's lymphoma

of the gastrointestinal tract

213. Interferons are correctly

characterized by which of the following

statements?

a. They are a group of complex phospholipids

b. They are produced by virus-infected

cells

c. They enhance viral replication

d. They cause Burkitt's lymphoma cell

lines to divide

e. They have not been effective in the

treatment of hairy cell leukemias

214. Which of the following statements

regarding malignant parotid

tumors is correct?

a. Acinar carcinoma is a highly aggressive

malignant tumor of the

parotid gland

b. Squamous carcinoma of the parotid

gland exhibits only moderately

malignant behavior

c. Regional node dissection for occult

metastases is not indicated for

malignant parotid tumors because

of their low incidence and the morbidity

of lymphadenectomy

d. Facial nerve preservation should be

attempted when the surgical margins

of resection are free of tumor

e. Total parotidectomy (superficial

and deep portions of the gland) is

indicated for malignant tumors

128 Surgery

215. Which of the following

potentially operable complications

is a common occurrence among

patients receiving systemic chemotherapy?

a. Acute cholecystitis

b. Perirectal abscess

c. Appendicitis

d. Incarcerated femoral hernia

e. Diverticulitis

216. Which of the following statements

regarding testicular cancer is

true?

a. Lymph node dissection after radical

orchiectomy is useful for staging

but does not increase survival

b. Seminomas and choriocarcinomas

are best treated with orchiectomy

and retroperitoneal lymph node

dissection

c. Seminomas are extremely resistant

to radiotherapy

d. Orchiectomy for a testicular mass is

approached via the scrotum

e. Cryptorchidism is associated with

an increased risk of testicular cancer

217. Advantages of dialysis over

renal transplantation include

a. Less expense if the treatment continues

for less than 2 years

b. Increased number of pregnancies

in female dialysis patients

c. Return of normal menses in female

dialysis patients

d. Less anemia in dialysis patients

e. Increased 1-year survival of dialysis

patients

Items 218-219

A 30-year-old primigravida

complains of headaches, restlessness,

sweating, and tachycardia.

She is 18 wk pregnant and her

blood pressure is 200/120 mm Hg.

218. Appropriate workup might

include

a. Exploratory laparotomy

b. Mesenteric angiography

c. Head CT scan

d. Abdominal CT scan

e. Abdominal ultrasonogram

219. Appropriate treatment might

consist of

a. Therapeutic abortion

b. Urgent excision of the tumor and a

therapeutic abortion

c. Phenoxybenzamine and propranolol

followed by a combined

cesarean section and excision of the

tumor

d. Metyrosine (Demser) blockade followed

by a combined cesarean section

and excision of the tumor

e. Phenoxybenzamine and propranolol

followed by a combined vaginal

delivery at term and excision of

the tumor

Transplants,Immunology,and Oncology 129

220. Which of the following statements

regarding radiation therapy

is true?

a. Damage to DNA occurs primarily

by the direct effect of ionizing radiation

b. Cellular hypoxia decreases sensitivity

to radiation

c. Cells in the S phase of the cell cycle

are most radiosensitive

d. Radiation therapy following lumpectomy

of a breast cancer provides

rates of local control equal to those

of mastectomy

e. Skin, gastrointestinal mucosa, and

bone marrow are relatively insensitive

to radiotherapy

221. Which of the following statements

regarding cancer therapy

with interleukin 2 (IL-2) is true?

a. It is a B-cell growth factor

b. It induces a major response in

patients with metastatic breast cancer

c. It induces a major response in

patients with metastatic colon cancer

d. It induces a major response in

patients with metastatic melanoma

e. It induces a major response in

patients with lymphoma

222. Which statement concerning

cancer and nutrition is correct?

a. Levels of nitrates in food and drinking

water are positively correlated

with the incidence of bladder cancer

b. Regular ingestion of vitamin D

from childhood probably inhibits

formation of carcinogens

c. Consumption of excessive amounts

of animal dietary fats is associated

with increased incidences of colon

cancer

d. Nutritional support of cancer

patients improves response of the

tumor to chemotherapy

e. Alcohol ingestion is associated with

pancreatic cancer

223. How do cardiac allografts differ

from renal allografts?

a. Cardiac allografts are matched by

HLA tissue typing and renal allografts

are not

b. Cardiac allografts can tolerate a

longer period of cold ischemia than

renal allografts

c. One-year graft survival for cardiac

allografts is substantially lower

than that for renal allografts

d. Cardiac allografts are matched only

by size and ABO blood type

e. Cyclosporine is a critical component

of the immunosuppressive

regimen for cardiac allografts but

not renal allografts

130 Surgery

Transplants,Immunology,and Oncology 131

224. Five-year survival rates in

excess of 20% may be expected following

resection of pulmonary

metastases if

a. Other organ metastases are present

b. Lung lesions are solitary

c. Local tumor recurrence is found

d. The tumor doubling time is less

than 20 days

e. The patient has received prior

chemotherapy

225. Which statement about

transmission of HIV in the health

care setting is true?

a. A freshly prepared solution of

dilute chlorine bleach will not adequately

decontaminate clothing

b. All needles should be capped

immediately after use

c. Cuts and other open skin wounds

are believed to act as portals of

entry for HIV

d. Double gloving reduces the risk of

intraoperative needle sticks

e. The risk of seroconversion following

a needle stick with a contaminated

needle is greater for HIV than

for hepatitis B

226. Regarding the risk of breast

cancer, which of the following

statements is true?

a. Breast cancer occurs more commonly

among women of the lower

social classes

b. A history of breast cancer in a firstdegree

family relative is associated

with a fourfold increase in risk

c. Women with a first birth after age

30 years have approximately twice

the risk of those with a first birth

before age 18

d. Cigarette smoking increases the

risk of breast cancer

e. Hair dyes have been shown to

increase the risk of breast cancer

227. Human immunodeficiency

virus (HIV) has been isolated from

many body fluids. Which of the following

is a major source of transmission?

a. Tears

b. Sweat

c. Semen

d. Urine

e. Breast milk

228. What is the primary toxicity

of doxorubicin (Adriamycin)?

a. Cardiomyopathy

b. Pulmonary fibrosis

c. Peripheral neuropathy

d. Uric acid nephropathy

e. Hepatic dysfunction

229. What is the most common

cause of cancer death among

women?

a. Breast cancer

b. Ovarian cancer

c. Colon cancer

d. Endometrial cancer

e. Lung cancer

230. Which of the following

agents causes hemorrhagic cystitis?

a. Bleomycin

b. 5-fluorouracil

c. Cisplatin

d. Vincristine

e. Cyclophosphamide

231. What is the major barrier to

successful transplantation across

animal species (xenotransplantation)?

a. Acute rejection

b. Chronic rejection

c. Hyperacute rejection

d. Infection

e. ABO incompatibility

232. Which of the following are

efficient antigen-presenting cells

found in the epidermis?

a. Macrophages

b. T cells

c. Langerhans cells

d. Dendritic cells

e. B cells

132 Surgery

233. Which of the following statements

is true regarding carcinoembryonic

antigen (CEA)?

a. CEA is an accurate screening test

for primary colorectal cancer

b. CEA levels have not been helpful in

the diagnosis of recurrent colorectal

cancer

c. CEA levels, when elevated, are

highly specific for colon cancer

d. CEA is present in normal colonic

mucosa

e. Postoperative CEA assay is 70%

accurate in predicting the appearance

of liver metastases within 1

year

DIRECTIONS: Each group of questions below consists of lettered

options followed by numbered items. For each numbered item, select the

appropriate lettered option(s). Each lettered option may be used once,

more than once, or not at all. Choose exactly the number of options

indicated following each item.

Items 234-236

For each stage in the treatment

of the patient below, select the

appropriate next step.

a. Left hemicolectomy

b. Right hemicolectomy

c. Subtotal colectomy

d. Total colectomy

e. Hepatic wedge resection

f. External beam irradiation

g. 5-fluorouracil and leucovorin

h. External beam irradiation and

chemotherapy

i. Abdominal MRI

j. No further treatment

234. A 65-year-old man is admitted

to the hospital with complaints

of intermittent constipation and

microcytic anemia. Barium enema

reveals a nonobstructing "applecore"

lesion of the proximal sigmoid

colon. Colonoscopy confirms

the location of the mass and reveals

no other synchronous lesions.

(SELECT 1 STEP)

235. The patient undergoes

surgery and recovers uneventfully.

Pathology of the resected specimen

is reported as Dukes C with negative

surgical margins. (SELECT 1

STEP)

236. In 6-mo follow-up an abdominal

CT scan shows a 2-cm isolated

lesion in the right lobe of the

liver. Repeat colonoscopy shows no

evidence of recurrent or metachronous

lesions. Chest x-ray and bone

scan are normal. (SELECT 1

STEP)

Transplants,Immunology,and Oncology 133

Items 237-240

A 32-year-old man with diabetic

nephropathy undergoes an

uneventful renal transplant from

his sister (two-haplotype match).

His immunosuppressive regimen

includes azathioprine, steroids, and

cyclosporine. For each development

in the postoperative period, select

the most appropriate next step.

a. Begin gancyclovir

b. Administer steroid boost

c. Withhold steroids

d. Decrease cyclosporine

e. Increase cyclosporine

f. Decrease azathioprine

g. Obtain renal ultrasound

h. Begin broad-spectrum antibiotics

i. Administer filgrastim (Neupogen)

j. Administer FK50

237. On postoperative day 3 the

patient is doing well, but you

notice on his routine laboratory

tests that his white blood cell count

is 2.0. (SELECT 1 STEP)

238. The patient's WBC count

gradually returns to normal, but on

postoperative day 7 he develops a

fever of 39.44°C (103°F) and a

nonproductive cough. A chest xray

reveals diffuse interstitial infiltrates,

and a "buffy coat" is positive

for viral inclusions. (SELECT 1

STEP)

239. The patient recovers from the

above illness and is discharged

home on postoperative day 18. At

3-mo follow-up he is doing well,

but you notice that his creatinine is

2.8 mg/dL. He has no fever, his

graft is not tender, and his renal

ultrasound is normal. (SELECT 1

STEP)

240. Six months following his

transplant, the patient begins to

develop fever, malaise, and pain of

the right lower quadrant. Upon palpation,

the graft is tender. Chest xray

and urine and blood cultures

are normal. Renal ultrasound shows

an edematous graft. (SELECT 1

STEP)

134 Surgery

TRANSPLANTS,

IMMUNOLOGY,AND

ONCOLOGY

Answers

186. The answer is c. (Greenfield, 2/e, pp 113-114.) Tumor necrosis factor

(TNF) is a peptide hormone produced by endotoxin-activated monocytes/

macrophages and has been postulated to be the principal cytokine

mediator in gram-negative shock and sepsis-related organ damage. Biologic

actions of TNF include polymorphonuclear neutrophil (PMN) activation

and degranulation; increased nonspecific host resistance; increased

vascular permeability; lymphopenia; promotion of interleukins 1, 2, and 6;

capillary leak syndrome; microvascular thrombosis; anorexia and cachexia;

and numerous other protective and adverse effects in sepsis. Its role in sepsis

is providing a fertile field for research in critical care.

187. The answer is b. (Greenfield, 2/e, pp 553-554.) The purpose of a

cross-match is to determine whether the recipient has circulating antibodies

against donor HLA antigens. Such antibodies do not occur naturally, but

rather are the result of prior sensitization during pregnancy, blood transfusions,

or previous transplantation. A complement-dependent lymphocytotoxicity

cross-match is performed by adding recipient serum and

complement to donor cells (T cells, B cells, or monocytes). If specific antidonor

antibodies are present, antibody binding results in complement fixation

and cell lysis. This is detected by addition of a vital dye, which is

taken up by the damaged cell membrane, resulting in a positive crossmatch.

If a positive cross-match is detected to donor T cells (HLA class I),

transplantation will result in hyperacute rejection.

188. The answer is a. (Greenfield, 2/e, pp 114-115.) Interleukin 1 (IL-1)

is a thymocyte mitogen produced by activated macrophages as well as

many other types of cells (e.g., monocytes, dendritic cells, Langerhans

cells, neutrophils, microglial cells). It induces interleukin 2 production by

135

the helper T cell, which initiates a cascade of immunoregulatory and

inflammatory functions.

189. The answer is b. (Greenfield, 2/e, pp 529-546.) Unlike the granulocyte

line, T lymphocytes express the T-cell receptor. This receptor imports

antigen specificity to T cells. The helper T cell, when stimulated by interleukin

1 and antigens, produces various lymphokines that ultimately produce

effector cells. One of these effector cells is the cytotoxic T cell, which

kills cells that express specific antigens, including viral, tumor, and nonbiologic

antigens. Macrophages and natural killer cells have some tumoricidal

activity; however, this is not specific for tumors.

190. The answer is d. (Greenfield, 2/e, pp 548-549.) Cyclosporine is a

highly effective immunosuppressive agent produced by fungi. It is more

specific than the anti-inflammatory agents such as steroids or the antiproliferative

agents such as azathioprine. The effectiveness of cyclosporine in

preventing allograft rejection is related to its ability to inhibit interleukin 2

production. Without interleukin 2 from helper T cells, there is no clonal

expansion of alloantigen-directed cytotoxic T cells and no stimulation of

antibody production by B cells.

191-192. The answers are 191-c, 192-c. (Greenfield, 2/e, pp 571-581.)

Hemodialysis, rather than management by dietary manipulation alone,

should be instituted in patients with end-stage renal failure whose serum

creatinine is over 15 mg/dL or whose creatinine clearance is less than 3

mL/min. It is important that hemodialysis be initiated prior to the onset of

uremic complications. These complications include hyperkalemia, congestive

heart failure, peripheral neuropathy, severe hypertension, pericarditis,

bleeding, and severe anemia. The uremic hyperkalemic patient in congestive

heart failure may require emergency dialysis in addition to the standard

conservative measures, which include (1) limitation of protein intake

to less than 60 g/day and restriction of fluid intake and (2) reduction of elevated

serum potassium levels by insulin-glucose or sodium polystyrene

sulfonate (Kayexalate) enema treatment. Arteriovenous fistulas require

about 2 wk to develop adequate size and flow. While awaiting maturation,

temporary dialysis can be satisfactorily performed using either an external

arteriovenous shunt or the peritoneal cavity. Renal biopsy would be performed

in an attempt to obtain a diagnosis of the underlying renal disease.

136 Surgery

Patients who are acceptable candidates for kidney transplantation usually

should undergo this form of treatment, after they are stabilized, rather than

chronic hemodialysis, the mortality for which is now higher than for transplantation.

Despite adequate dialysis, problems of neuropathy, bone disease,

anemia, and hypertension remain difficult to manage. Compared with

chronic dialysis, transplantation restores more patients to happier and

more productive lives. It had been conjectured that, all other issues being

equal, sex matching was important in the graft survival and that a motherdaughter

graft was preferred to a father-daughter graft. Review of the current

data does not support such a conclusion. The best graft survival rates

for living related transplants-over 90% at 5 years-are obtained when all

six histocompatibility loci are identical. All family members of potential

transplant recipients should be tissue typed and the donor should be

selected on the basis of closest match, if psychological and medical evaluation

makes this feasible. With the development of cyclosporine-based

immunosuppression, cadaveric kidney graft survival has approached that

of living-related transplantation. There are some transplanters who believe

that the slight improvement with living-related kidneys does not justify the

risk to the donor and that these transplantations should no longer be performed.

193. The answer is c. (Greenfield, 2/e, pp 602-606.) Chronic graft rejection

is manifested in cardiac allografts as chronic vascular rejection of main

and intramuscular coronary arteries. Myointimal proliferation and medial

scarring result in diffuse and eccentric arterial narrowing referred to as

accelerated graft atherosclerosis. Infection remains the primary cause of

death within the first year of cardiac transplant, but accelerated graft arteriosclerosis

is the most common cause of mortality thereafter. Percutaneous

transluminal coronary angioplasty, coronary artery bypass grafting, and

retransplantation are the current options for combating this problem.

194. The answer is a. (Greenfield, 2/e, pp 553-554.) A positive crossmatch

means that the recipient has circulating antibodies that are cytotoxic

to donor-strain lymphocytes. This incompatibility, which almost always

leads to an acute humoral rejection of the graft, precludes transplantation.

Blood type matching prior to organ allograft is similar to cross-matching

prior to transfusion; O is the universal donor and AB the universal recipient.

Minor blood group factors do not appear to act as histocompatibility anti-

Transplants,Immunology,and Oncology Answers 137

gens. Matching of HLA antigens in cadaveric renal transplants may improve

graft survival, but the impact is relatively minor. While attempts are made to

pair recipient and donor by tissue typing, a two-antigen match is perfectly

acceptable and even zero-antigen matches can be transplanted with good

results. Neither hypertension nor anemia is a contraindication to transplantation;

indeed, hypertension may be cured or ameliorated following successful

transplantation. Patients with end-stage renal failure generally are

anemic and can be transfused, if necessary, intra- or postoperatively. Anemia

generally also improves following transplantation because of increased

erythropoietin production by the graft.

195. The answer is c. (Greenfield, 2/e, pp 578-581.) Hyperacute rejection

is mediated by cytotoxic antibodies with subsequent triggering of the complement,

coagulation, and kinin systems. It can occur during surgery after

the clamps are released from the vascular anastomosis and the recipient's

antibodies are exposed to the donor's passenger lymphocytes and kidney

tissue. Typically, the kidney will become swollen and pale. Hyperacute

rejection is the cause of immediate and early oliguria and biopsies should

be performed intraoperatively or early postoperatively. Hyperacute rejection

is characterized pathologically by fibrin and platelet thrombosis and

necrosis of the glomerular tufts, renal arterioles, and small arteries. Massive

polymorphonuclear infiltrate with tubular necrosis occurs 24-36 h after

transplantation. The intravascular coagulation rarely results in a systemic

coagulopathy. Careful cross-matching can test for cytotoxic antibodies.

Although plasmapheresis and cyclophosphamide can transiently decrease

the preformed antibody load, to date there exists no adequate prevention

or treatment for hyperacute rejection.

196. The answer is a. (Greenfield, 2/e, pp 599-606.) Cardiac transplantation

has become an acceptable clinical treatment modality for selected

patients with end-stage cardiac failure. Allograft survivals are now comparable

to those of cadaveric renal transplants-approximately 70% at 1 year

and 50% at 5 years as reported by the Stanford group. Although kidneys

can be safely preserved by either hypothermic storage or hypothermic perfusion

for periods up to 48 h, donor hearts protected by simple hypothermia

should be transplanted within 4 h. For this reason the usual

tissue-typing procedures used in kidney transplantation are impractical in

cardiac transplantation, and indeed there is no correlation between match

138 Surgery

and outcome. In pairing donor and recipient for heart transplants there

must be at least ABO blood group compatibility. Cyclosporine has

improved results in both cardiac and renal transplantation despite its major

drawback of dose-related nephrotoxicity. Eligibility for cardiac transplantation

has evolved from strict age criteria to more flexible guidelines based on

a patient's likelihood of surviving and resuming a normally functional life

after transplantation. Many centers, however, observe age 65 as the upper

limit for transplantation. The leading cause of death in patients surviving

more than 1 year after transplantation is infection, followed by graft atherosclerosis.

197. The answer is d. (Greenfield, 2/e, pp 578-581.) The patient is experiencing

an acute rejection episode. Seventy-four percent of all acute rejection

episodes occur between 1 and 6 mo after transplantation. For cadaveric renal

transplant recipients, 63% of patients will never have an acute rejection

episode, 17% will have only one rejection episode, and 19% will have two or

more rejections. In order to grade the rejection as well as to follow the

response to treatment, a percutaneous renal biopsy should be performed.

The three treatment modalities used for acute rejection are high-dose steroids

alone, high-dose steroids plus antilymphocyte globulin (equine serum

hyperimmunized to human lymphocytes), or high-dose steroids plus OKT3

(murine monoclonal antibody to the human CD3 complex).

198. The answer is b. (Greenfield, 2/e, pp 552-553, 560.) Overall, 30% of

all infections contracted in the posttransplant period are viral. The most

common viral infections are DNA viruses of the herpesvirus family and

include cytomegalovirus (CMV), Epstein-Barr virus, herpes simplex virus,

and varicella zoster virus. CMV infections may occur as either primary or

reactive infections and have a peak incidence at about 6 wk post transplant.

The classic signs include fever, malaise, myalgia, arthralgia, and leukopenia.

CMV infection can affect several organ systems and result in pneumonitis;

ulceration and hemorrhage in the stomach, duodenum, or colon;

hepatitis; esophagitis; retinitis; encephalitis; or pancreatitis. The risk of

developing posttransplant CMV depends on donor-recipient serology, with

the greatest risk in seronegative patients who receive organs from seropositive

donors. Pyelonephritis, cholecystitis, intraabdominal abscesses, and

parotitis are caused by bacterial infections or GI perforation and not primarily

by CMV infection.

Transplants,Immunology,and Oncology Answers 139

199. The answer is b. (Greenfield, 2/e, pp 548-553.) With the introduction

of cyclosporine in the early 1980s and the rapidly accumulated experience

with liver transplantation, graft and patient survivals have improved

markedly. In the azathioprine and steroid era, 1-year graft survival was in

the range of 25%. More recently, most centers are experiencing 1-year graft

survival rates of approximately 80%.

200. The answer is d. (Greenfield, 2/e, p 553.) Donor-type lymphoid cells

transplanted within a graft may recognize the host's tissue as foreign and

mount an immune response against the host. This response, termed graftversus-

host disease (GVHD), is common in bone marrow transplantation

and is an important source of morbidity and mortality. Treatment requires

more aggressive immunosuppression. Current clinical practice includes

depletion of lymphocytes from the marrow graft in order to prevent the

development of GVHD. GVHD has been documented following liver transplantation,

presumably because of the large amount of lymphoid tissue in

the donor liver. GVHD has not been described following heart, lung, pancreas,

or kidney transplantation.

201. The answer is d. (Greenfield, 2/e, pp 606-615.) Many causes of endstage

lung disease have been appropriately treated with lung transplantation.

Whether one lung or both lungs are replaced at the time of

transplantation depends on recipient factors. Patients with restrictive

processes like primary pulmonary fibrosis do well with a single lung transplant.

For patients with primary pulmonary hypertension, unloading of

the right ventricle with single lung transplantation has been adequate and

replacement of both lungs has not been necessary in most cases. Cystic

fibrosis patients do well after lung transplantation but double lung transplant

is frequently necessary because of chronic infections. Secondary pulmonary

hypertension is due to left ventricular failure with concomitant

increases in pulmonary pressures secondary to increases in left ventricular

end-diastolic pressures. Reactive secondary pulmonary hypertension is

best treated with heart transplantation. Long-standing secondary pulmonary

hypertension that is chiefly fixed is best treated with combined

heart-lung transplantation.

202. The answer is d. (Greenfield, 2/e, pp 615-627.) Whole-organ pancreas

transplantation is the only therapy for type I insulin-dependent dia-

140 Surgery

betes that maintains normal serum glucose levels and normal glucose tolerance

tests. When the pancreas is transplanted along with a kidney, the tight

glucose control generally prevents the recurrence of diabetic nephropathy.

No series has shown the reversal of diabetic retinopathy or reduction in the

rate of diabetic ulcers or of amputations, although some parameters of diabetic

retinopathy may improve after pancreas transplantation.

203. The answer is a. (Sabiston, 15/e, pp 501-502.) Bone marrow cells

are highly immunogenic. Successful engraftment requires the use of powerful

immunosuppressants that permit the transplanted cells not only to

survive the host's immune response, but also to mount a graft-versus-host

response against recipient tissues. The graft-versus-host response is the

major impediment to more widespread clinical use of this technique.

Despite these barriers, human bone marrow transplantation has had

important clinical application in the treatment of aplastic anemias and congenital

immunodeficiency diseases and several hematologic malignancies.

Stem cell transplantation involves harvesting of a patient's own pleuripotent

bone marrow cells and subsequent reestablishment of the marrow following

high-dose, toxic chemotherapy for advanced cancer. This modality

has been used in the treatment of recurrent breast cancer, but recent metaanalyses

of the results have failed to show any significant survival benefit.

In experimental models, work with bone marrow transplantation for the

induction of tolerance to organ allografts has proved highly promising.

This may provide a key for the development of treatment protocols in

organ transplant recipients that would avoid or reduce the need for toxic

systemic immunosuppressants.

204. The answer is d. (Schwartz, 7/e, pp 366-368.) Major histocompatibility

complex (MHC) proteins are polymorphic cell surface molecules that

are important in lymphocyte-lymphocyte and lymphocyte-target interactions.

All nucleated cells express MHC class I proteins. B lymphocytes,

macrophages, antigen-presenting cells, vascular endothelium, and activated

T lymphocytes express both MHC class I and class II. MHC class I

proteins are encoded by the HLA-A, B, and C loci, and MHC class II proteins

are encoded by the HLA-D locus. Classically, MHC class I molecules

with a bound antigen are recognized by the T-cell receptor on CD81 cells,

and MHC class II molecules with a bound antigen are recognized by the

T-cell receptor on CD41 cells.

Transplants,Immunology,and Oncology Answers 141

205. The answer is b. (Schwartz, 7/e, pp 323-324.) In following patients

with nonseminomatous testicular tumors, elevated serum levels of the β

subunit of human chorionic gonadotropin (hCG), α-fetoprotein, and lactic

dehydrogenase have been found to be useful indicators of tumor activity or

recurrence. The discovery of prostate-specific antigen has recently been

touted as a major breakthrough in screening for prostate cancer, though

some clinicians feel that early diagnosis may have no impact on survival in

this disease. CA125 has been used to follow ovarian cancers; it is fairly

nonspecific but can alert the physician to the need for a more aggressive

search for persistent disease when relative increases are noted in a patient

after therapy. The p53 oncogenes have been found in soft tissue sarcomas,

osteogenic sarcomas, and colon cancers. Their significance is unknown.

206. The answer is a. (Schwartz, 7/e, pp 329-331.) Isolated enlarged cervical

lymph nodes in the adult are malignant nearly 80% of the time

(excluding benign tumors of the thyroid gland). They are usually metastatic

squamous cell carcinomas arising from primary sources above the clavicles

in the aerodigestive tract. Fine-needle aspiration cytology is commonly used

to obtain histological confirmation of suspected cancer. Aspiration cytology

can usually diagnose carcinoma accurately, but lymphoma may be difficult

to identify by this method, and open biopsy is often necessary. Bone marrow

biopsy is not indicated prior to lymph node biopsy. It is done as part of the

staging process after a diagnosis of lymphoma has been made. Endoscopy

and scanning of the oro- and nasopharynx are part of the diagnostic workup

of a suspected malignant cervical lymph node, but do not provide histological

proof of cancer.

207. The answer is d. (Schwartz, 7/e, pp 277-278, 348.) Since

chemotherapy is generally most effective in killing rapidly dividing cells,

the rapidly dividing cells of a fresh surgical wound should be in jeopardy

when chemotherapy is given in the early postoperative period. Each of the

phases of normal wound healing is theoretically at risk from one or another

class of chemotherapeutic agents. Immediately following wounding, inflammation

and vascular permeability lead to fibrin deposition and polymorphonuclear

neutrophil (PMN), monocyte, and platelet influx. Macrophages

are attracted by the activated complement system. By the fourth day the

proliferative phase begins, and for the next 20 days fibroblasts produce

mucopolysaccharides and collagen. Cross-linking of the collagen fibers

142 Surgery

then continues for several months in the maturation phase. It seems logical

to delay antineoplastic agents for 10-14 days unless there are compelling

clinical indications (e.g., superior vena cava syndrome) for more urgent

treatment. Administration of folinic acid simultaneously with methotrexate

normalizes wound healing. Extravasation of chemotherapeutic agents

during intravenous administration may result in severe ulceration and

sloughing. The nature of the injury is largely related to the nucleic-acidbinding

characteristics of the agent. Those agents that do not bind to tissue

nucleic acid (vincristine, vinblastine, nitrogen mustard, BCNU, 5-FU) generally

cause only local damage from the immediate injury. These substances

are quickly metabolized or inactivated, and usual patterns of wound healing

can be expected. On the other hand, agents that bind the nucleic acid

(doxorubicin, dactinomycin, mitomycin C, mithramycin, and daunorubicin)

cause not only immediate toxic reaction in the tissues but, unless

excised, continuing and progressive tissue damage. Though some authors

have reported success with elevation and ice packs, most recommend surgical

excision if there is severe pain, any sign of early necrosis, or significant

blistering.

208. The answer is e. (Schwartz, 7/e, pp 320-324.) The management of

malignant tumors may be guided by knowledge obtained by grading and

staging the tumors. Histologic grading reflects the degree of anaplasia of

tumor cells. Tumors in which histologic grading seems to have prognostic

value include soft tissue sarcoma, transitional cell cancers of the bladder,

astrocytoma, and chondrosarcoma. Grading has been of little predictive

value in melanoma, hepatocellular carcinoma, or osteosarcoma. Staging is

based on the extent of spread rather than histologic appearance and is more

relevant in predicting the course of lung and colorectal cancers.

209. The answer is a. (Schwartz, 7/e, pp 1747-1748.) This is a nephroblastoma

(Wilms tumor) adherent to the left kidney. These tumors are associated

with aniridia (rarely) and with hemihypertrophy, cryptorchidism, or

hypospadias in about 10% of cases. Most patients present with an asymptomatic

mass found by a parent. Less than one-third of patients experience

hematuria. As would be expected in over half such cases, this child is

hypertensive, probably due to compression of the renal artery by the mass.

Treatment with excision, radiation, vincristine, and actinomycin D results

in survival rates of over 90% in stage I and II tumors. While computed

Transplants,Immunology,and Oncology Answers 143

tomography (CT) or magnetic resonance imaging (MRI) evaluates metastatic

disease, intravenous pyelography (IVP) is better at differentiating this

tumor from polycystic kidney or neuroblastoma. Wilms tumor is the most

common abdominal malignancy of childhood, but represents only about

10% of childhood malignant tumors.

210. The answer is c. (Schwartz, 7/e, pp 1686-1688.) Medullary carcinomas

occur in families as part of syndromes called multiple endocrine neoplasia

(MEN) type 2A and type 2B. MEN 2A consists of multicentric

medullary thyroid cancer, pheochromocytomas or adrenal medullary

hyperplasia, and hyperparathyroidism. MEN 2B consists of medullary cancer,

pheochromocytoma and mucosal neuromas, gangliomas, and a Marfanlike

habitus. These patients may develop medullary carcinoma at a very

young age, and any patient with MEN 2B should be assumed to have

medullary cancer until proved otherwise. Patients are followed carefully for

pheochromocytoma with urine VMA, for hyperparathyroidism with serum

calcium, and for medullary carcinoma with serum calcitonin. However, as

some patients have a normal basal calcitonin, a pentagastrin or provocative

calcium infusion test should be performed in these high-risk patients.

Patients thought to have MEN 1 syndrome (pituitary, parathyroid, and

pancreatic tumors) or Zollinger-Ellison syndrome should be assayed for

serum gastrin, insulin, glucagon, and somatostatin. These assays may

prove to be inappropriately high in MEN 1 syndrome due to pancreatic

islet cell tumors.

211. The answer is c. (Schwartz, 7/e, pp 334-335.) Benign soft tissue

tumors far outnumber their malignant counterparts. Because of this, prolonged

delays are common before definitive treatment of soft tissue sarcomas

is instituted. Risk for malignancy is increased for tumors greater than

5 cm in largest diameter, as well as for those lesions that are symptomatic

or have enlarged rapidly over a short period of time. Properly performed

biopsy is critical in the initial treatment of any soft tissue mass. Improperly

performed biopsies can complicate the care of the sarcoma patient, and in

rare circumstances even eliminate certain surgical options. Excisional biopsies

should be reserved for small masses for which complete excision

would not jeopardize subsequent treatment should a sarcoma be found.

For all other masses incisional biopsy should be performed. The incision

144 Surgery

should be placed directly over the mass and should be oriented along the

long axis of the extremity.

212. The answer is e. [Tondini, Ann Oncol 4(10):831-837, 1993. Gobbi,

Cancer 65(11):2528-2536, 1990.] The stomach is the most common site in

the gastrointestinal tract for non-Hodgkin's lymphoma, followed by the

small intestine and the colon. Lymphomas constitute 3% of all malignant

gastric tumors. Ninety percent of these lymphomas are of the non-

Hodgkin's type. Surgery alone can be considered adequate treatment for

patients with non-Hodgkin's lymphoma that does not infiltrate beyond the

submucosa. However, gastric resection is not considered mandatory, and

there are no substantial differences in response to therapy and survival

when resection is compared with combined chemotherapy and radiation

therapy, including in advanced cases. Moreover, chemotherapy and radiation

therapy have been shown to be effective even in unresected bulky

cases, and provide minimal risk of hemorrhage and perforation even in this

setting.

213. The answer is b. (Schwartz, 7/e, pp 349-350.) The interferons are a

group of glycoproteins first found as products of virus-infected cells that

inhibited viral replication. Subsequently, they have been shown to have a

variety of effects both on cells of the immune system and on malignant

cells. Interferons cause Burkitt's lymphoma cell lines to differentiate and

lose the capacity to divide. Hematologic malignancies are very responsive

to interferons; up to 100% of hairy cell leukemias show some degree of

remission. Interferon α has been used in the treatment of chronic active

hepatitis B and C with promising results in recent clinical trials.

214. The answer is d. (Schwartz, 7/e, pp 656-662.) Acinar, adenoid cystic,

and low grades of mucoepidermoid carcinomas exhibit moderately

malignant behavior. Undifferentiated, squamous, and high grades of

mucoepidermoid carcinomas are considered highly malignant tumors.

Regional node dissection is indicated for malignant tumors because of the

high (up to 50%) incidence of occult regional metastases. Facial nerve

preservation should be attempted when the margins are adequate and the

tumor is well localized. The minimal appropriate procedure for parotid

carcinoma is a superficial parotidectomy with nerve preservation. The

Transplants,Immunology,and Oncology Answers 145

nerve must be partially or totally sacrificed if the tumor directly involves

the nerve trunk or its branches.

215. The answer is b. (Schwartz, 7/e, pp 347-348.) A surgeon is frequently

asked to evaluate patients who are receiving systemic chemotherapy. Most

complications of chemotherapy do not require surgical therapy. Perirectal

abscesses are more common in these immunosuppressed patients. Gastrointestinal

bleeding occurs secondary to mucosal irritation and thrombocytopenia.

Pancreatitis is uncommon, but is associated with L-asparaginase use. Up

to 20% of patients treated with floxuridine by continuous hepatic artery infusion

develop some degree of inflammation and obstruction of the bile duct.

Systemic chemotherapy does not increase the likelihood of acute cholecystitis,

appendicitis, incarcerated femoral hernia, or diverticulitis.

216. The answer is e. (Schwartz, 7/e, pp 1794-1795.) After radical

orchiectomy, lymph node dissection is indicated in embryonal carcinoma,

teratocarcinoma, and adult teratoma if there is no supradiaphragmatic

spread. This dissection increases the 5-year survival and helps in staging.

Seminoma is extremely radiosensitive and lymph node dissection is unnecessary.

Choriocarcinoma is associated with pulmonary metastases in 81% of

cases and is treated with chemotherapy. Orchiectomy for a testicular mass is

approached via an inguinal incision in order to perform a high ligation of

the cord and to eliminate spread of the tumor. Cryptorchidism (undescended

testicle) is associated with decreased spermatogenesis and carries a

lifelong risk of malignant degeneration even after being surgically corrected.

217. The answer is a. (Greenfield, 2/e, pp 237-238, 571-572.) Dialysis is

less expensive than renal transplantation if the graft functions for less than

2 years. Recipients with functioning grafts are less anemic because of erythropoietin

production by the graft. As more dialysis patients are treated

with recombinant erythropoietin, this advantage may disappear. Transplanted

females have more normal menses and an increased number of

successful pregnancies. The patient survival in the two groups is comparable

at 1 year.

218-219. The answers are 218-e, 219-e. (Schwartz, 7/e, pp

1646-1649.) This young pregnant woman presents with the symptoms of a

pheochromocytoma. These tumors can initially become symptomatic dur-

146 Surgery

ing pregnancy. A noninvasive workup should be performed. Ultrasonography

of the abdomen is frequently sufficient to localize the tumor to the

right or left adrenal; an abdominal CT scan with its large dose of radiation

should be avoided in pregnancy. The treatment can be early excision of the

pheochromocytoma, and in three cases in pregnant women this was done

with survival of two of the three infants. A therapeutic abortion, especially

at 18 wk, is not indicated, and cesarian section would not produce a viable

fetus. The current approach is α- and β-adrenergic blockade followed by

vaginal delivery or cesarean section with excision of the tumor at the same

time as delivery or electively after delivery. Metyrosine (Demser) inhibits

tyrosine hydroxylase and results in a decrease in endogenous levels of catecholamines.

This form of treatment, coupled with term delivery, is also

acceptable.

220. The answer is b. (Greenfield, 2/e, pp 491-495.) Only about 30% of

the biologic damage from x-rays is due to the direct effects on the target

molecule. The remainder is due to an indirect action mediated by free radicals

and can be modified by free radical scavengers such as sulfhydryl. The

percentage of cells killed by a given dose of x-rays or gamma rays is greatly

increased by molecular oxygen; cells deficient in oxygen are resistant to

radiation. Among the basic principles of radiation biology is the observation

that the sensitivity of mammalian cells to radiation varies with their

position in the cell division cycle. M phase (mitotic phase) cells are the

most radiosensitive. Radiation is frequently employed for local control of

disease. Survival rates for breast lumpectomy and radiation are equal to

those of mastectomy, but local control rates (10-15% recurrence at 10

years for stage I and II cancers treated with lumpectomy and radiation versus

approximately 5% treated with mastectomy) are nevertheless inferior.

Rapidly dividing cells of the gastrointestinal mucosa and bone marrow are

particularly sensitive to the effects of radiation.

221. The answer is d. (Schwartz, 7/e, pp 349-351.)With the availability of

recombinant interleukin-2, multiple trials of cancer therapy with this lymphokine

have been undertaken. The most successful trials have documented

complete or partial responses in patients with metastatic renal cell

carcinoma and melanoma. However, IL-2 therapy has been ineffective in the

treatment of breast cancer, colon cancer, and lymphoma. The therapy is not

innocuous. All patients exhibit a marked lymphocytosis, eosinophilia, fluid

Transplants,Immunology,and Oncology Answers 147

retention, fever, and decrease in peripheral vascular resistance, effects similar

to those of septic shock.

222. The answer is c. (Heys, Br J Surg 79:614-623, 1992.) Malignant

tumors require energy substrates to grow and ordinarily claim these substrates

from the host. In animal studies, withholding dietary proteins diminishes

the rate of tumor growth. There is no evidence in the human to suggest

acceleration of tumor growth when nutritional support is provided. There is

also no evidence that nutritional therapy improves the response of the tumor

to therapy. For nearly a century, the association of stomach cancer and diet

has been recognized. Among the wide variety of substances incriminated are

nitrates and nitrosamides in food and drinking water. There is evidence that

regular ingestion of vitamin C from childhood may reduce the formation of

carcinogens, though reduction in the incidence of cancer has not been

demonstrated. Excess amounts of dietary fat and deficiency of fiber have

been clearly associated with colon cancer. Animal fats have also been associated

with cancer of the exocrine pancreas, the prostate, and the

endometrium. Alcohol consumption, especially when combined with cigarette

smoking, increases the incidence of esophageal cancer. Consumption of

alcohol also increases the incidence of pancreatitis, but not pancreatic cancer.

223. The answer is d. (Greenfield, 2/e, pp 599-606.) Cardiac allograft has

become an accepted treatment for end-stage heart disease. One-year cardiac

allograft survival approaches 90% and is equivalent to 1-year renal

allograft survival. Cardiac allografts have a cold ischemia preservation time

of 4-5 h and therefore tissue typing is not practical. Cardiac donors are

matched to recipients only by size and ABO blood type. Tissue typing

remains an important component of cadaveric kidney allograft matching.

The mainstay of immunosuppression for both cardiac and renal allografts

continues to include cyclosporine, azathioprine (Imuran), and steroids.

224. The answer is b. (Schwartz, 7/e, pp 340-341, 352-353.) Resection of

metastases of lung, liver, and brain can result in occasional 5-year cures. In

general, surgery should be undertaken only when the primary tumor is

controlled, diffuse metastatic disease has been ruled out, and the affected

patient's condition and the location of the metastasis permit safe resection.

Five-year survival rates as high as 18% have been reported for selected

patients with liver metastases from colorectal primary tumors. However,

148 Surgery

the best results have come from resection of pulmonary metastases, in

which 5-year survival rates exceed those of resection for primary bronchogenic

carcinoma. Autopsy reviews have demonstrated that many

patients with pulmonary metastases have no other evidence of tumor,

which suggests that resectional treatment may be justified even when the

lung foci are not solitary. Selection of patients for pulmonary resections

may be aided by measurement of tumor doubling times; patients with doubling

times greater than 40 days appear to benefit most, while those with

doubling times less than 20 days are not significantly helped.

225. The answer is c. (Rhame, Postgrad Med 91:141-152, 1992. Wilson,

Postgrad Med 88:193-201, 1990.) The risk of contracting HIV is much less

than the risk of contracting hepatitis B from a patient. Although the risk of

transmission of HIV in the health care setting is very low, there are reported

cases of seroconversion after parenteral exposure. Particular precautions

should be taken in operating upon patients who are known to be seropositive

for HIV or who have known risk factors. Recommendations include

elimination of inexperienced personnel or personnel with open lesions on

body surfaces from the operating room. Disposable gowns, drapes, masks,

and eye shields should be used. Contaminated clothing should be soaked

in a dilute solution (1:10) of chlorine bleach prior to washing. Double

gloving does not reduce the major intraoperative risk of needle puncture,

which is the primary source of risk to the operating team. Needles should

never be capped; an uncapped needle is less dangerous than are the

maneuvers to recap needles.

226. The answer is c. (Harris et al, pp 159-167.) Risk factors for breast

cancer include family history, nulliparity, previous breast cancer, early

menarche, and late menopause. A late age at first birth (after age 30) doubles

the risk of breast cancer compared with early parity (age 18 or earlier).

Having one first-degree relative (mother, sister, or daughter) with breast

cancer also doubles the risk. Women of the upper social classes, as measured

by either education or income, have been found to have the highest

incidence of breast cancer. Neither cigarette smoking nor the use of hair

dye has been correlated with breast cancer.

227. The answer is c. (Recommendations for prevention of HIV transmission

in health care settings. NY State J Med 88:25-31, 1988. Rhame, Postgrad

Transplants,Immunology,and Oncology Answers 149

Med 91:141-144, 1992.) HIV has been isolated from blood, semen, vaginal

secretions, saliva, tears, breast milk, CSF, amniotic fluid, and urine. It is an

extremely fastidious virus that ordinarily is transmitted only after repeated

admixture of body fluids. Blood and semen are by far the major transmission

fluids.

228. The answer is a. (Greenfield, 2/e, p 502.) Doxorubicin, an antibiotic

derived from Streptomyces species, has activity against sarcomas and carcinomas

of the breast, liver, bladder, prostate, head and neck, esophagus,

and lung. Its major side effect is production of a dilated cardiomyopathy.

Patients receiving this agent should have echocardiography before and

after treatment in order to monitor potential cardiac toxicity.

229. The answer is e. (Greenfield, 2/e, pp 455-459.) Cancer remains the

second most common cause of mortality in the United States after heart

disease, accounting for 22% of all deaths. Both sexes have demonstrated

dramatic increases in the death rate observed from lung cancer from 1930

to 1990 owing to increases in cigarette smoking. Lung cancer is the leading

cause of cancer death in both men and women.

230. The answer is e. (Greenfield, 2/e, p 501.) Cyclophosphamide is an

alkylating agent used in the treatment of a variety of solid tumors. Its major

side effect is hemorrhagic cystitis. Bleomycin can cause pulmonary fibrosis.

Vincristine is an alkaloid that can cause peripheral and central neuropathies.

Cisplatin is an alkylating agent that can lead to ototoxity, neurotoxicity,

and nephrotoxicity. 5-fluorouracil is an antimetabolite that can

cause mucositis, dermatitis, and cerebellar dysfunction.

231. The answer is c. (Greenfield, 2/e, p 555.) The major barrier to successful

xenotransplantation has been hyperacute rejection, which refers to

the binding of preformed human antibodies to donor endothelial cells.

This results in the activation of complement, cell lysis, and eventually vascular

thrombosis.

232. The answer is c. (Greenfield, 2/e, p 537.) Processing and presentation

of antigen in association with class II molecules is critical for activation

of T cells. Langerhans cells are potent antigen-presenting cells (APCs)

150 Surgery

found in skin. Macrophages are the major APCs in the body. Dendritic cells

are APCs found in lymphoid tissue.

233. The answer is e. (Greenfield, 2/e, pp 1138, 1144.) CEA is a glycoprotein

that is present in early embryonic and fetal cells (an oncofetal antigen)

and in colon cancer. It is not found in normal colon mucosa. It is not

tumor specific and may be elevated in a variety of benign and malignant

conditions, including cirrhosis, ulcerative colitis, renal failure, pancreatitis,

pancreatic cancer, stomach cancer, breast cancer, and lung cancer. The

CEA assay is, however, a sensitive serologic tool for identifying recurrent

disease. In about two-thirds of patients with recurrent disease, an increased

CEA level is the first indicator of tumor reappearance. A rising CEA following

colon cancer surgery, in the absence of other conditions associated

with an elevated CEA, predicts the appearance of liver metastases within 1

year with an accuracy approaching 70%.

234-236. The answers are 234-a, 235-g, 236-e. (Greenfield, 2/e, pp

1137-1144.) The patient has a left colon cancer. In order to resect the

tumor with a margin of 3-5 cm on its proximal and distal ends as well as

to remove the draining lymph node basin, a left hemicolectomy should be

performed. A Dukes C tumor is one that extends through the bowel wall

and involves adjacent lymph nodes. In a study of 1166 patients with stage

B and C colon cancer, the National Surgical Adjuvant Breast and Bowel

Project (NSABP) reported an improved survival in patients randomized to

receive adjuvant chemotherapy compared with no further treatment after

resection. Adjuvant radiation therapy has only been useful in preventing

local recurrence in rectal cancers with positive surgical margins.

The liver is the most common site of bloodborne metastases from primary

colorectal cancers. In a subgroup of patients, the liver may be the

only site of disease. Overall, surgical resection is associated with a 25-30%

5-year survival rate.

237-240. The answers are 237-f, 238-a, 239-d, 240-b. (Greenfield,

2/e, pp 577-581.) Routine postoperative immunosuppression for a renal

transplant recipient includes cyclosporine, azathioprine, and steroids.

Cyclosporine is nephrotoxic and is frequently withheld in the postoperative

period until the creatinine returns to normal following transplantation.

Transplants,Immunology,and Oncology Answers 151

Azathioprine has bone marrow toxicity as its major side effect and both

WBC and platelet counts need to be monitored in the immediate posttransplant

period. The patient's decrease in WBCs is secondary to azathioprine

toxicity, and the most appropriate step is to decrease the dose of

azathioprine.

Viral infections are a serious cause of morbidity following transplantation.

A "buffy coat" is the supernatant of a centrifuged blood sample that

contains the WBCs. Viral cultures from this supernatant as well as localization

of inclusion bodies can identify transplant patients infected with

cytomegalovirus (CMV). This patient has CMV pneumonitis and needs to

be treated with high-dose gancyclovir.

An elevation in creatinine at 3-mo follow-up can be secondary to

rejection, anastomotic problems, urologic complications, infection, or

nephrotoxicity of various medications. With a normal ultrasound, no fever,

and no graft tenderness, the most likely cause is cyclosporine-induced

nephrotoxicity and the most appropriate step is a reduction in the

cyclosporine dose.

Finally, at 6 mo with graft tenderness, fever, and an edematous kidney

on ultrasound, rejection must be suspected. Negative cultures make infection

unlikely, and a steroid boost is appropriate. Addition of monoclonal

antibodies to CD3 (OKT3) or pooled antibodies against lymphocytes

(ALG) is also appropriate in the treatment of a first rejection.

152 Surgery

ENDOCRINE PROBLEMS

AND BREAST

Questions

DIRECTIONS: Each item below contains a question or incomplete

statement followed by suggested responses. Select the one best response to

each question.

153

241. Which of the following statements

regarding adrenal cortical

insufficiency is true?

a. Treatment with exogenous steroids

is usually ineffective

b. It is commonly seen as a consequence

of metastasis of distant cancers,

such as lung or breast, to the

adrenal glands

c. Chronic adrenal insufficiency

(Addison's disease) in the preoperative

patient should be recognizable

by a constellation of findings,

including hyperglycemia, hypernatremia,

and hypokalemia

d. Death from untreated chronic

adrenal insufficiency may occur

within hours of surgery

e. The most common underlying

cause today is infection with resistant

tuberculosis

242. The thyroid scan shown

below exhibits a pattern that is

most consistent with which of the

following disorders?

a. Hypersecreting adenoma

b. Graves' disease

c. Lateral aberrant thyroid

d. Papillary carcinoma of thyroid

e. Medullary carcinoma of thyroid

Terms of Use

243. A 17-year-old girl presents

with an anterior neck mass. Her

thyroid scan, shown below, is most

consistent with which of the following

disorders?

245. Estrogen receptor activity is

clinically useful in predicting

a. The presence of ovarian cancer

b. The presence of metastatic disease

c. Response to chemotherapy

d. Response to hormonal manipulation

e. The likelihood of development of

246. When galactorrhea occurs in

a high school student, a diagnostic

associated finding would be

a. Gonadal atrophy

b. Bitemporal hemianopia

c. Exophthalmos and lid lag

d. Episodic hypertension

e. "Buffalo hump"

247. The diagnosis of primary hyperparathyroidism

is most strongly

suggested by

a. Serum acid phosphatase above 120

IU/L

b. Serum alkaline phosphatase above

120 IU/L

c. Serum calcium above 11 mg/dL

d. Urinary calcium below 100 mg/day

e. Parathyroid hormone levels below

5 pmol/L

248. Somatostatin contributes to

which of the following processes?

a. Inhibition of adrenocortical cells

b. Inhibition of pancreatic α cells

c. Stimulation of antral gastrin cells

d. Stimulation of secretin-producing

cells in the duodenum

e. Stimulation of GI motility

154 Surgery

a. Hypersecreting adenoma

b. Parathyroid adenoma

c. Thyroglossal duct cyst

d. Graves' disease

e. Carcinoma

244. A 35-year-old woman undergoes

her first screening mammogram.

Which of the following

mammographic findings would

require a breast biopsy?

a. Breast calcifications larger than 2

mm in diameter

b. Five or more clustered breast

microcalcifications per square centimeter

c. A density that effaces with compression

d. Saucer-shaped microcalcifications

e. Multiple round well-circumscribed

breast densities

249. Which of the following

statements concerning Cushing syndrome

secondary to adrenal adenoma

is true?

a. Adrenal adenomas cause 40-60%

of all cases of Cushing syndrome

b. Biochemical and x-ray procedures

are generally unsuccessful in lateralizing

the tumors preoperatively

c. Exploration of both adrenal glands

is indicated

d. For uncomplicated tumors, an open

transperitoneal surgical approach is

usually employed

e. Postoperative corticoid therapy is

required to prevent hypoadrenalism

250. A 40-year-old woman is

found to have a 1- to 2-cm, slightly

tender cystic mass in her breast; she

has no perceptible axillary adenopathy.

What course would you follow?

a. Reassurance and reexamination in

the immediate postmenstrual period

b. Immediate excisional biopsy

c. Aspiration of the mass with cytologic

analysis

d. Fluoroscopically guided needle

localization biopsy

e. Mammography and reevaluation of

options with new information

251. Which statement concerning

radiation-induced thyroid cancer is

true?

a. It usually follows high-dose radiation

to the head and neck

b. A patient with a history of radiation

is safe if no cancer has been found

20 years after exposure

c. Approximately 25% of patients

with a history of head and neck

irradiation develop thyroid cancer

d. Most radiation-induced thyroid

cancers are follicular

e. The treatment of choice is a neartotal

(or total) thyroidectomy

252. The course of papillary carcinoma

of the thyroid is best

described by which of the following

statements?

a. Metastases are rare; local growth is

rapid; erosion into the trachea and

large blood vessels is frequent

b. Local invasion and metastases

almost never occur, which makes

the term carcinoma misleading

c. Bony metastases are frequent and

produce an osteolytic pattern particularly

in vertebrae

d. Metastases frequently occur to cervical

lymph nodes; distant metastases

and local invasion are rare

e. Rapid, widespread metastatic involvement

of the liver, lungs, and

bone marrow results in a 5-year

survival rate of approximately 10%

Endocrine Problems and Breast 155

253. Fibrocystic disease of the

breast has been associated with elevated

blood levels of

a. Testosterone

b. Progesterone

c. Estrogen

d. Luteinizing hormone

e. Aldosterone

254. A 14-year-old black girl had

her right breast removed because of

a large mass. The tumor weighed

1400 g and was found to have a

bulging, very firm, lobulated surface

with a whorl-like pattern, as

illustrated below. This neoplasm is

most likely

255. As an incidental finding during

an upper abdominal CT scan, a

3-cm mass in the adrenal gland is

noted. The appropriate next step in

analysis and management of this

finding would be

a. Observation

b. CT-guided needle biopsy

c. Excision of the mass

d. Measurement of urine catecholamine

excretion

e. Cortisol provocation test

Items 256-257

A 53-year-old woman presents

with complaints of weakness, anorexia,

malaise, constipation, and

back pain. While being evaluated,

she becomes somewhat lethargic.

Laboratory studies include a normal

chest x-ray; serum albumin 3.2

mg/dL; serum calcium 14 mg/dL;

serum phosphorus 2.6 mg/dL;

serum chloride 108 mg/dL; BUN 32

mg/dL; and creatinine 2.0 mg/dL.

256. Appropriate initial management

would include

a. Intravenous normal saline infusion

b. Administration of thiazide diuretics

c. Administration of intravenous phosphorus

d. Use of mithramycin

e. Neck exploration and parathyroidectomy

156 Surgery

a. Cystosarcoma phylloides

b. Intraductal carcinoma

c. Malignant lymphoma

d. Fibroadenoma

e. Juvenile hypertrophy

257. After appropriate immediate

management, the patient's symptoms

resolve. Diagnostic tests to

perform at this point would include

which of the following?

a. Abdominal angiogram

b. Measurement of serum gastrin hormone

levels

c. Kveim test

d. Serum and urine protein electrophoresis

e. Neck exploration

258. A woman sustains an injury

to her chest after striking the steering

wheel of her automobile during

a collision. Which of the following

statements concerning fat necrosis

of the breast is true?

a. Most patients report a history of

trauma

b. The lesion is usually nontender and

diffuse

c. It predisposes patients to the development

of breast cancer

d. It is difficult to distinguish from

breast cancer

e. Excision exacerbates the process

Items 259-260

259. The most likely diagnosis in a

patient with hypertension, hypokalemia,

and a 7-cm suprarenal

mass is

a. Hypernephroma

b. Cushing's disease

c. Adrenocortical carcinoma

d. Pheochromocytoma

e. Carcinoid

260. Appropriate treatment of this

condition would include which of

the following?

a. Embolization of the arterial blood

supply, including the suprarenal

artery

b. Metronidazole

c. Mitotane

d. Phentolamine

e. Phenoxybenzamine

261. For pregnant women who

are found to have breast cancer

a. Termination of a first-trimester

pregnancy is mandatory

b. Carcinoma of the breast behaves

more aggressively in pregnant

women owing to hormonal stimulation

c. Breast conservation is inappropriate

for third-trimester pregnancies

d. Most have hormonally sensitive

tumors

e. Administration of adjuvant

chemotherapy is safe for the fetus

during the second and third

262. True statements regarding

Paget's disease of the breast include

that it

a. Usually precedes development of

Paget's disease of bone

b. Presents with nipple-areolar eczematous

changes

c. Does not involve axillary lymph

nodes because it is a manifestation

of intraductal carcinoma only

d. Accounts for 10-15% of all newly

diagnosed breast cancers

e. Is adequately treated with wide

excision when it presents as a mass

Endocrine Problems and Breast 157

263. A 40-year-old man who has a

long history of peptic ulcer disease

that has not responded to medical

therapy is admitted to the hospital.

His serum gastrin levels are

markedly elevated; at celiotomy, a

small, firm mass is palpated in the tail

of the pancreas. Correct statements

concerning this patient's condition

include which of the following?

a. Histamine or a protein meal will

markedly increase basal acid secretion

b. Secretin administration will suppress

acid secretion

c. The pancreatic mass will probably

d. Distal pancreatectomy is the treatment

of choice

e. H2 receptor antagonists have not

been beneficial in the treatment of

this condition

264. Of the common complications

of thyroidectomy, the one that

may be avoided through prophylaxis

a. Injury to the recurrent laryngeal

nerve

b. Injury to the superior laryngeal

nerve

c. Symptomatic hypocalcemia

d. Thyroid storm

e. Postoperative hemorrhage and

wound hematoma

Items 265-266

265. Following correction of

the patient's hypercalcemia with

hydration and gentle diuresis with

furosemide, the most likely therapeutic

approach would be

a. Administration of maintenance

doses of steroids

b. Radiation treatment for bony

metastases

c. Neck exploration and resection of

three out of four parathyroid glands

d. Neck exploration and resection of a

parathyroid adenoma

e. Avoidance of sunlight, vitamin D,

and calcium-containing dairy products

266. This 30-year-old woman presented

with weakness, bone pain,

an elevated parathormone level,

and a serum calcium level of 15.2

mg/dL. Skeletal survey films were

taken, including the hand films and

chest x-ray shown. The most likely

cause of these findings is

a. Sarcoidosis

b. Vitamin D intoxication

c. Paget's disease

d. Metastatic carcinoma

e. Primary hyperparathyroidism

158 Surgery

159

267. A 25-year-old woman is

found to have an anterior neck

mass. Her thyroid scan, shown

below, exhibits findings that are

consistent with which of the following

disorders?

269. A 36-year-old woman, 20 wk

pregnant, presents with a 1.5-cm

right thyroid mass. Fine-needle

aspiration is consistent with a papillary

neoplasm. The mass is "cold"

by scan and solid by ultrasound.

Which method of treatment would

be contraindicated?

a. Right thyroid lobectomy

b. Subtotal thyroidectomy

c. Total thyroidectomy

d. Total thyroidectomy with lymph

node dissection

e. 131I radioactive ablation of the thyroid

gland

270. Correct statements concerning

Hürthle-cell carcinoma of the

thyroid include which of the following?

a. It is a form of anaplastic thyroid

cancer

b. It metastasizes via the lymphatics to

regional lymph node basins

c. Treatment consists of a near-total

(or total) thyroidectomy

d. Microscopically, it consists of clusters

of cells separated by areas of

collagen and amyloid

e. Once treated appropriately, it has a

low rate of recurrence

160 Surgery

a. Carcinoma

b. Toxic adenoma

c. Toxic multinodular goiter

d. Graves' disease

e. de Quervain's (subacute) thyroiditis

268. Incisional biopsy of a breast

mass in a 35-year-old woman

demonstrates a hypercellular

fibroadenoma (cystosarcoma phylloides)

at the time of frozen section.

Appropriate management of this

lesion could include

a. Wide local excision with a rim of

normal tissue

b. Lumpectomy and axillary lymphadenectomy

c. Modified radical mastectomy

d. Excision and postoperative radiotherapy

e. Excision, postoperative radiotherapy,

and systemic chemotherapy

271. A 28-year-old man presents

with a 2.5-cm mass in the anterior

triangle of the left neck. The mass

moves with swallowing and has

slowly enlarged over the past 1-2

years. The patient's past medical

history is notable for high-dose

irradiation to the chest and

abdomen for Hodgkin's lymphoma

8 years prior to presentation. Thyroid

scan shows a "cold" lesion.

Fine-needle aspiration cytology is

"suspicious." Core-needle biopsy

shows features suggestive of a follicular

neoplasm. True statements

regarding this patient's condition

include

a. Thyroid nodules in men are rarely

malignant

b. Prior radiation to the chest, if anything,

would diminish the risk of

subsequent thyroid cancer

c. In the setting of abnormal cytology,

an initial course of TSH suppression

by thyroid hormone is recommended

d. In the setting of a possible follicular

neoplasm, radioactive iodine (131I)

ablation is recommended

e. Total thyroidectomy is an acceptable

treatment for this patient

272. True statements about discharge

from the nipple include

a. Intermittent thin or milky discharge

can be physiologic

b. Expressible nipple discharge is an

indication for open biopsy

c. Bloody discharge is indicative of an

underlying malignancy

d. Galactorrhea is indicative of an

underlying malignancy

e. Pathologic discharge is usually bilateral

273. True statements regarding

Cushing's disease and Cushing syndrome

include which of the following?

a. Adrenocortical hyperplasia is the

most common cause of Cushing's

disease

b. Overproduction of ACTH is pathognomonic

of Cushing syndrome

c. Clinical manifestations of Cushing's

disease and Cushing syndrome are

identical

d. Cushing syndrome is caused only

by neoplasms of either the pituitary

or adrenal glands

e. Cushing's disease is incurable

Endocrine Problems and Breast 161

274. A 34-year-old woman has

recurrent fainting spells induced by

fasting. Her serum insulin levels

during these episodes are markedly

elevated. Correct statements regarding

this patient's condition

include which of the following?

a. The underlying lesion is probably

an α-cell tumor of the pancreas

b. The underlying lesion is usually

multifocal

c. These lesions are usually malignant

d. Serum calcium levels may be elevated

e. She should be screened for a coexistent

pheochromocytoma

275. The incidence of breast cancer

a. Increases with increasing age

b. Has declined since the 1940s

c. Is related to dietary fat intake

d. Is related to coffee intake

e. Is related to vitamin C intake

162 Surgery

DIRECTIONS: Each group of questions below consists of lettered

options followed by numbered items. For each numbered item, select the

appropriate lettered option(s). Each lettered option may be used once,

more than once, or not at all. Choose exactly the number of options

indicated following each item.

163

Items 276-280

For each clinical description

select the appropriate stage of

breast cancer.

a. Stage I

b. Stage II

c. Stage III

d. Stage IV

e. Inflammatory carcinoma

276. Tumor not palpable, clinically

positive lymph nodes fixed to

one another, no evidence of metastases

(SELECT 1 STAGE)

277. Tumor 5.0 cm; clinically positive,

movable ipsilateral lymph

nodes; no evidence of metastases

(SELECT 1 STAGE)

278. Tumor 2.1 cm, clinically negative

lymph nodes, no evidence of

metastases (SELECT 1 STAGE)

279. Tumor not palpable but

breast diffusely enlarged and erythematous,

clinically positive supraclavicular

nodes, and evidence of

metastases (SELECT 1 STAGE)

280. Tumor 0.5 cm, clinically negative

lymph nodes, pathological rib

fracture (SELECT 1 STAGE)

Items 281-285

A 43-year-old man presents

with signs and symptoms of peritonitis

in the right lower quadrant.

The clinical impression and supportive

data suggest acute appendicitis.

At exploration, however, a

tumor is found; frozen section suggests

carcinoid features. For each

tumor described, choose the most

appropriate surgical procedure.

a. Appendectomy

b. Segmental ileal resection

c. Cecectomy

d. Right hemicolectomy

e. Hepatic wedge resection and

appropriate bowel resection

281. A 2.5-cm tumor at the base

of the appendix (SELECT 1 PROCEDURE)

282. A 1.0-cm tumor at the tip of

the appendix (SELECT 1 PROCEDURE)

Endocrine Problems and Breast 163

283. A 0.5-cm tumor with serosal

umbilication in the ileum (SELECT

1 PROCEDURE)

284. A 1.0-cm tumor of the

midappendix; 1-cm firm, pale

lesion at the periphery of the right

lobe of the liver (SELECT 1 PROCEDURE)

285. A 3.5-cm tumor encroaching

onto the cecum and extensive liver

metastases (SELECT 1 PROCEDURE)

Items 286-290

For each clinical problem outlined,

select acceptable treatment

options.

a. No further surgical intervention

b. Wide local excision

c. Wide local excision with adjuvant

radiation therapy

d. Wide local excision with axillary

lymph node dissection and radiation

therapy

e. Simple mastectomy (without axillary

lymph node dissection)

f. Modified radical mastectomy (simple

mastectomy with in-continuity

axillary lymph node dissection)

g. Radical mastectomy

h. Bilateral prophylactic simple mastectomies

286. A 49-year-old woman undergoes

biopsy of a 1.0-cm breast

mass. Pathology shows extensive

comedo ductal carcinoma in situ.

(SELECT 2 CHOICES)

287. A 42-year-old woman with a

familial breast cancer (mother, four

sisters, and additional relatives)

undergoes her fifth breast biopsy

for a palpable mass. Pathology

shows ductal hyperplasia with

severe atypia. (SELECT 2

CHOICES)

288. A 51-year-old (premenopausal)

woman undergoes

needle localization biopsy for

microcalcifications. Pathology

reveals sclerosing adenosis with

microcalcifications and extensive

lobular carcinoma in situ.

(SELECT 1 CHOICE)

289. A 35-year-old woman presents

with a palpable 1.5-cm tumor

in the upper outer quadrant of her

left breast. Biopsy reveals invasive

ductal carcinoma with 10% intraductal

carcinoma. (SELECT 2

CHOICES)

290. A neglected 82-year-old

woman presents with a locally

advanced breast cancer that is

invading the pectoralis major muscle

over a broad base. She is otherwise

in good health. (SELECT 1

CHOICE)

164 Surgery

ENDOCRINE PROBLEMS

AND BREAST

Answers

241. The answer is d. (Greenfield, 2/e, pp 204-205.) Failure to recognize

adrenal cortical insufficiency, particularly in the postoperative patient, may

be a fatal error. This error is especially regrettable because therapy (exogenous

steroids) is effective and easy to administer. Adrenal insufficiency may

occur in a host of settings including tuberculosis (formerly the most common

cause), autoimmune states, severe infections (classically, meningococcal

septicemia), pituitary insufficiency, after burns, during anticoagulant

therapy, and-most commonly today-after interruption of chronically

administered exogenous steroids. Although the adrenal gland is an occasional

site for distant metastases, such as from lung or breast, it is rare for

there to be enough destruction of the glands to produce clinical adrenal

insufficiency. Chronic adrenal insufficiency (classic Addison's disease)

should be recognizable preoperatively by the constellation of skin pigmentation,

weakness, weight loss, hypotension, nausea, vomiting, abdominal

pain, hypoglycemia, hyponatremia, and hyperkalemia. Death may occur

within hours of surgery if a patient with Addison's disease is operated on

without cognizance of adrenal insufficiency and pretreatment with exogenous

steroids. Patients who have adrenal insufficiency as a result of interruption

of chronically administered exogenous steroids may not develop the

classic electrolyte abnormalities until the preterminal period. Adrenal insufficiency

may also develop insidiously in the postoperative period, progressing

over a course of several days. This insidious course is seen when adrenal

injury occurs in the perioperative period, as would be the case with adrenal

damage from hemorrhage into the gland in a patient receiving postoperative

anticoagulant therapy. Measurement of blood corticosteroid levels, urinary

corticosteroid secretion, urinary sodium levels, and the response to exogenous

steroids is helpful in establishing the diagnosis of adrenal insufficiency.

242. The answer is a. (Schwartz, 7/e, pp 1672, 1680.) The thyroid scan

illustrated in the question shows a single focus of increased isotope uptake,

165

often referred to as a "hot" nodule; the remainder of the thyroid gland has

not taken up radioactive iodine. Hyperfunctioning adenomas become

independent of thyroid stimulating hormone (TSH) control and secrete

thyroid hormone autonomously, which results in clinical hyperthyroidism.

The elevated thyroid hormone levels ultimately diminish TSH levels

severely and thus depress function of the remaining normal thyroid gland.

An isolated focus of increased uptake on a thyroid scan is virtually diagnostic

of a hyperfunctioning adenoma. Carcinomas usually display diminished

uptake and are called "cold" nodules. Graves' disease would probably

manifest as a diffusely hyperactive gland without nodularity. Multinodular

goiter would display many nodules with varying activity.

243. The answer is c. (Schwartz, 7/e, pp 601-602, 1717.) The thyroid

gland originates embryologically from the foramen cecum at the base of the

tongue. Normally, the thyroglossal duct becomes obliterated and resorbed,

but portions may remain patent and become filled with serous fluid, which

produces a midline cervical mass. Observe that in the scan of the patient

described in the question, the mass is central and appears not to be part of

the gland itself.

244. The answer is b. (Schwartz, 7/e, pp 543-546.) Breast biopsies have

traditionally been performed to obtain histology for clinically suspicious

palpable masses. In more recent years the advent of screening mammography

has led to the discovery of nonpalpable but radiographically suspicious

breast lesions that have a strong correlation with breast cancer. These

nonpalpable, mammographically detected lesions are (1) breast calcifications

that are (a) smaller than 2 mm, (b) punctate, microlinear, or branching,

and (c) clustered along ducts or concentrated in clusters greater than

five calcifications per square centimeter; (2) stellate-shaped lesions; (3)

masses with ill-defined borders or nodular contours; (4) solitary dominant

masses that are significantly larger than any other mass in either breast; and

(5) areas of increased noneffacing tissue density or distorted breast architecture.

A parenchymal density that effaces with compression represents

normal glandular tissue. Saucer-shaped microcalcifications are seen in

patients with microscopic cystic disease, a benign condition. Multiple

round well-circumscribed densities are usually cysts, whose nature may be

confirmed with breast sonography.

166 Surgery

245. The answer is d. (Schwartz, 7/e, pp 586-588.) The likelihood of

response of a breast cancer to hormonal therapy is dependent on the presence

of hormone receptors in the cytoplasm of the breast cancer cells.

Receptors for corticosteroids, progesterone, prolactin, and estrogen have

been identified. Eighty percent of patients with tumors that exhibit receptors

to both estrogen and progesterone respond favorably to hormonal

manipulation. Estrogen receptor activity has no predictive value in diagnosing

ovarian cancer or metastatic disease, forecasting the development of

osteoporosis, or determining the likelihood of a beneficial response to

chemotherapy.

246. The answer is b. (Schwartz, 7/e, pp 1620-1628.) Prolactin-secreting

tumors in the pituitary gland (previously called chromophobe adenomas)

may grow to large size and cause bitemporal hemianopia because of proximity

to the optic chiasm. They are typically associated with amenorrhea

and galactorrhea (the "A/G syndrome") in women. In both sexes lack of

libido and impotence or infertility may be noted. Sexual vigor is usually

restored after removal of the adenomas. These tumors are not life threatening;

if their physical size is not an issue or the relative sexual dysfunction is

not a problem, benign neglect is sometimes recommended.

247. The answer is c. (Norton, Ann Surg 215:297-299, 1992. Potts, Ann

Intern Med 114:593-597, 1991.) Primary hyperparathyroidism is a common

disease, with over 100,000 new cases diagnosed each year in the United

States, usually in women. Essential to the diagnosis of hyperparathyroidism

is the finding of hypercalcemia. Though there are many causes of

hypercalcemia, hyperparathyroidism is by far the most prevalent. With rare

exceptions, operations for primary hyperparathyroidism should not be

performed unless the patient is hypercalcemic. Parathyroid hormone

(PTH) is not invariably elevated, but it should be elevated relative to the

serum calcium level. Ordinarily, high serum calcium levels suppress

parathyroid secretion. Therefore, in the presence of hypercalcemia, normal

levels of PTH are "abnormal." Patients with primary hyperparathyroidism

have either normal or elevated urinary calcium. As the name suggests,

patients with familial hypocalciuric hypercalcemia (FHH) have hypercalcemia.

They also usually have elevated PTH, but surgery is not indicated in

this relatively rare setting of hypercalcemia.

Endocrine Problems and Breast Answers 167

248. The answer is b. (Greenfield, 2/e, pp 751-752, 869-870.) Somatostatin

is produced by D cells in the pancreatic islets and in a variety of other

tissue sites in the central nervous system, gut, and elsewhere. It is a potent

inhibitory regulator of intestinal hormones and motility. Because it was

originally found in the hypothalamus, somatostatin earned its name

because it was believed to be a major inhibitor of secretion of growth hormone.

It has now been shown to inhibit the secretion of most GI hormones,

particularly insulin and glucagon, as well as gastrin, secretin, VIP,

PP, gastric acid, pepsin, pancreatic enzymes, thyroid-stimulating hormone,

renin, and calcitonin. It also inhibits intestinal, biliary, and gastric motility,

and is occasionally of value in controlling bowel fistulas by sharply reducing

the amount of drainage. It has no known effect on adrenocortical cells.

249. The answer is e. (Greenfield, 2/e, pp 1344-1345.) Primary adrenal

pathology causes 10-20% of all cases of Cushing syndrome. A hyperfunctioning

adrenal adenoma can usually be lateralized by preoperative radiologic

studies, eliminating the need to explore both adrenal glands. In

10-15% of cases, adenomas are bilateral. The favored surgical approach

today is via transabdominal laparoscopy or by a posterior unilateral flank

route. The anterior transperitoneal approach should be reserved for complicated

cases such as large or obviously malignant lesions. After tumor

excision, corticosteroid therapy to correct postoperative hypoadrenalism is

necessary.

250. The answer is c. (Greenfield, 2/e, p 1373.) Most clinicians would

recommend aspiration and cytologic examination of the cyst fluid in this

situation. Cysts are common lesions in the breasts of women in their thirties

and forties; malignancies are relatively rare. All such lesions justify

attention, however, and physicians must not underestimate the fear associated

with the discovery of a mass in the breast, even in low-risk situations.

If the lesion does not completely disappear after aspiration, excision is

advised. In young women the breast parenchyma is dense, which limits the

diagnostic value of mammography. The American Cancer Society (ACS)

does not suggest a baseline mammographic examination until age 35

unless a suspicious lesion exists.

251. The answer is e. (Greenfield, 2/e, pp 1301-1303.) Radiation-induced

thyroid cancer was first recognized in 1950 by Duffy and Fitzgerald. It usu-

168 Surgery

ally follows low-dose external radiation. Most cancers occur after exposure

to 1500 rads or less to the neck, but an increase in thyroid cancer has been

noted after as little as 6 rads. Salivary gland tumors and possibly parathyroid

adenomas are also associated with radiation. The latent period for

these tumors is 30 years or longer. Of all patients who have low-dose radiation,

about 9% have been found to have thyroid cancer, usually of the

papillary type. Treatment consists of a near-total thyroidectomy because

there is a high incidence of bilaterality and because there is a greater incidence

of complications if a second operation is necessary.

252. The answer is d. (Greenfield, 2/e, pp 1301-1303.) Papillary carcinoma

of the thyroid frequently metastasizes to cervical lymph nodes, but

distant metastasis is uncommon. The nonaggressive nature of this tumor

locally and the infrequency of distant metastases combine to produce an

80-95% 5-year survival rate. A contributing factor to the success of thyroid

surgery for papillary carcinoma is the easy accessibility of cervical nodes for

examination and dissection. Slow growth and a predilection for local

extension are characteristics of this tumor that contribute to a high survival

rate in affected persons. This is true even of patients who have limited

surgery, which has led to considerable controversy regarding the extent of

the indicated surgical procedure.

253. The answer is c. (Greenfield, 2/e, pp 1372-1374.) Fibrocystic disease

(chronic cystic mastitis) is a common disorder of the adult female

breast. It is rare after cessation of ovarian function, either natural or

induced. Its association with estrogens is inferential. In postmenopausal

women it only occurs when replacement estrogen therapy is in use. Its

main clinical significance relates to the need to differentiate irregular breast

tissue from cancer. Patients afflicted with this disorder are often frustrated

by the repeated biopsies that may be recommended.

254. The answer is d. (Schwartz, 7/e, pp 552-553.) Fibroadenomas occur

infrequently before puberty but are the most common breast tumors

between puberty and the early thirties. They usually are well demarcated

and firm. Although most fibroadenomas are no larger than 3 cm in diameter,

giant or juvenile fibroadenomas frequently are very large. The bigger

fibroadenomas (greater than 5 cm) occur predominantly in adolescent

black girls. The average age at onset of juvenile mammary hypertrophy is

Endocrine Problems and Breast Answers 169

16 years. This disorder involves a diffuse change in the entire breast and

does not usually manifest clinically as a discrete mass; it may be unilateral

or bilateral and can cause an enormous and incapacitating increase in

breast size. Regression may be spontaneous and sometimes coincides with

puberty or pregnancy. Cystosarcoma phylloides may also cause a large

lesion. Together with intraductal carcinoma, it characteristically occurs in

older women. Lymphomas are less firm than fibroadenomas and do not

have a whorl-like pattern. They display a characteristic fish-flesh texture.

255. The answer is a. (Gajraj, Br J Surg 80:422-426, 1993.) With the

increasing use of CT and MRI scans for other purposes, small "incidentalomas"

of the adrenal gland are becoming a frequent finding. In the absence

of any clinical signs or symptoms of endocrine dysfunction, most experts

now recommend observation and a search for evidence of endocrine dysfunction

for lesions less than 5 cm in diameter. Lesions below that size are

common and are usually asymptomatic, nonfunctional adenomas or

adrenal cysts. Functional neoplasms secrete an excess of hormones, which

produces clinical signs and symptoms. All functional tumors and solid

tumors greater than 5.0 cm in diameter should be removed. Cystic masses

greater than 5 cm may be aspirated with a fine needle. Clear fluid suggests

a benign lesion; if the fluid is bloody or aspiration produces solid tissue,

then the lesion should be resected.

Cystic tumors ranging from 3.5 to 5.0 cm may also be aspirated. If

bloody fluid is obtained or if the lesion is solid, then resection should be

considered in a patient who is otherwise a healthy surgical candidate. Both

solid and cystic masses less than 3.5 cm may be followed and can be considered

benign if they do not increase in size or become functional.

256. The answer is a. (Schwartz, 7/e, pp 1679-1707.) The patient

described is exhibiting classic signs and symptoms of hyperparathyroidism.

In addition, if a history is obtainable, frequently the patient will

relate a history of renal calculi and bone pain-the syndrome characterized

as "groans, stones, and bones." The acute management of the hypercalcemic

state includes vigorous hydration to restore intravascular volume,

which is invariably diminished. This will establish renal perfusion and thus

promote urinary calcium excretion. Thiazide diuretics are contraindicated

because they frequently cause patients to become hypercalcemic. Instead,

diuresis should be promoted with the use of "loop" diuretics such as

170 Surgery

furosemide (Lasix). The use of intravenous phosphorus infusion is no

longer recommended because precipitation in the lungs, heart, or kidney

can lead to serious morbidity. Mithramycin is an antineoplastic agent that

in low doses inhibits bone resorption and thus diminishes serum calcium

levels; it is used only when other maneuvers fail to decrease the calcium

level. Calcitonin is useful at times. Bisphosphonates are newer agents particularly

useful for lowering calcium levels in resistant cases, such as those

associated with humoral malignancy. Finally, "emergency" neck exploration

is seldom warranted. In unprepared patients, the morbidity is unacceptably

high.

257. The answer is d. (Schwartz, 7/e, pp 64, 1698.) The mechanism of

hypercalcemia of malignancy is thought to be due to either elaboration of

a "PTH-like" humoral factor or, many times, direct bone destruction by

metastatic disease. Breast, prostatic, pulmonary, and hematologic malignancy

all may give rise to hypercalcemia. Serum and urine electrophoresis

may identify a malignancy that causes bone destruction, such as

multiple myeloma. Sarcoidosis may produce hypercalcemia, but the presence

of the normal chest x-ray essentially rules out this possibility. Thus,

a Kveim test is not indicated. An abdominal angiogram would not be

expected to identify a likely cause of hypercalcemia. Serum gastrin is not

implicated in the differential diagnosis of hypercalcemia. A neck exploration

would not be indicated unless a parathyroid adenoma or carcinoma

was suspected.

258. The answer is d. (Schwartz, 7/e, p 552.) Injury to breast tissue may

cause necrosis of mammary adipose tissue and lead to the formation of a

tender, localized, firm mass. A history of trauma is often elicited from

affected patients, but less apparent factors, such as prolonged pressure,

may also produce fat necrosis. Half the patients in whom the diagnosis is

made do not recall a history of trauma. The pathophysiology of this lesion

seems to involve early development of liquefaction of mammary fat with

the formation of a cystic mass. Through a process of fibrosis, this lesion

evolves into a firm, sometimes calcified lump that may be difficult to distinguish

from carcinoma. There is, however, no relation between fat necrosis

and the subsequent development of breast cancer. Excisional biopsy is

usually required for definitive diagnosis; if the diagnosis of fat necrosis is

confirmed, simple excision removes and terminates the process.

Endocrine Problems and Breast Answers 171

259-260. The answers are 259-c, 260-c. (Schwartz, 7/e, pp

1642-1645.) The constellation of symptoms in this patient is typical of a

functional adrenocortical tumor. Masculinization in females is also a common

finding. Elevated urine 17-ketosteroids will be found in this patient.

Any adrenocortical tumor larger than 6 cm should be considered a carcinoma

rather than an adenoma. Treatment should include resection, not

embolization, of as much tumor as possible. This would include invaded

adjacent organs such as the kidney or the tail of the pancreas. Symptoms

related to hormone production can be minimized by complete resection

despite the inability to cure advanced disease. The most effective adjuvant

therapy is mitotane, which is toxic for functional adrenocortical cells.

When mitotane is used, therefore, glucocorticoids must be administered.

Ketoconazole (not metronidazole) has been found to inhibit the production

of various steroid hormones and may be useful in the treatment of

hormone-related symptoms. The overall 5-year survival of patients with

adrenocortical carcinoma treated with resection and mitotane is 20%.

Phentolamine and phenoxybenzamine are α-adrenergic blockers that are

sometimes useful in the preoperative management of pheochromocytomas.

261. The answer is e. (Barnavon, Surg Gynecol Obstet 171:347-352, 1990.)

Approximately 2% of American women who develop carcinoma of the breast

are pregnant at the time of diagnosis. The therapeutic approach to these

patients has changed considerably in recent years. Though changes in the

breast that occur during pregnancy often lead to a delay in diagnosis of breast

carcinoma, there is no convincing evidence that breast carcinoma in pregnant

women behaves differently or is histologically different from that in

nonpregnant women. Furthermore, when patients are matched for age and

stage of disease, no significant differences in survival rates are found. The

majority of breast cancers in these patients, as with most premenopausal

patients, are estrogen-receptor negative and not hormonally sensitive. Therefore,

elective termination of pregnancy is generally no longer indicated to

decrease estrogen stimulation of the tumor. Since radiation exposure endangers

the fetus and there is no evidence that general anesthesia and nonabdominal

surgery increase premature labor, modified radical mastectomy is

recommended for stage I or II carcinoma (tumor less than 4 cm in diameter).

Patients in later stages of pregnancy, however, can start radiation therapy

shortly after delivery, and some may be candidates for breast-conserving

surgery and adjuvant radiotherapy. Chemotherapy does not appear to

172 Surgery

increase the risk of congenital malformation when given in the second or

third trimester of pregnancy. Patients who require adjuvant chemotherapy

during the first trimester may opt for a therapeutic abortion, however, since

there is a slightly increased risk of fetal malformation in that circumstance.

262. The answer is b. (Harris, pp 870-876.) Paget's disease of the breast

is unrelated to Paget's bone disease. It represents a small percentage (1-3%)

of all breast cancers and is thought to originate in the retroareolar lactiferous

ducts. It progresses toward the nipple-areola complex in most patients,

where it causes the typical clinical finding of nipple eczema and erosion.

Up to 20% of patients with Paget's disease have an associated breast mass,

and these patients are more likely to have involvement of axillary nodes.

Nipple-areolar disease alone usually represents in situ cancer; these

patients have a 10-year survival rate of over 80%. In contrast, if Paget's disease

presents with a mass, the mass is likely to be an infiltrating ductal carcinoma.

The generally recommended surgical procedure for Paget's disease

is currently a modified radical mastectomy. The validity of breast-saving

surgery and adjuvant radiation therapy for patients without an associated

mass is under investigation.

263. The answer is d. (Schwartz, 7/e, pp 1190-1196.) The syndrome of a

gastrin-secreting non-β-cell pancreatic tumor is a rare entity first described

by Zollinger and Ellison. They originally described a triad of (1) fulminant,

complicated peptic ulceration; (2) extreme gastric hypersecretion; and (3) a

non-β-cell tumor of pancreatic islets. Over 50% of the tumors are malignant,

and 40% have metastases at the time of surgery. Until recently, total

gastrectomy was the primary operation for this tumor; however, it is now

believed that operative exploration of the patient with resection of the

tumor should be done if possible. H2 receptor antagonists have also proved

very promising in the management of these patients. Patients with Zollinger-

Ellison tumors have very high basal gastric acid (greater than 35 meq/h) and

serum gastrin levels (usually greater than 200 pg/mL). A protein meal or histamine

usually does not increase acid and gastrin levels as it would in conventional

duodenal ulcer patients. A paradoxical rise in serum gastrin after

intravenous secretin is diagnostic of Zollinger-Ellison syndrome.

264. The answer is d. (Schwartz, 7/e, pp 1692-1693.) The incidence of

complications with thyroidectomy or parathyroidectomy is relatively low in

Endocrine Problems and Breast Answers 173

most series. Thyroid storm, a manifestation of severe thyrotoxicosis, is

avoided by prophylactic treatment with propylthiouracil or methimazole

prior to surgery. The remaining complications listed are complications of

technique. The likelihood of serious complications increases with the extent

of resection ("total thyroidectomy" versus "subtotal thyroidectomy") and

with the number of neck explorations (initial exploration versus reexploration).

Injury to the recurrent laryngeal nerve can compromise the airway,

as can hemorrhage into the wound. Superior laryngeal nerve injury causes

annoying voice "fatigue," but is rarely of significant consequence. Hypocalcemia

is usually transient, but can at times necessitate permanent calcium

supplementation. Perforation of hollow neck structures very seldom occurs,

and, unless it is massive or not appreciated, usually causes no morbidity.

265-266. The answers are 265-d, 266-e. (Schwartz, 7/e, pp

1697-1707.) This patient's presentation and films are consistent with primary

hyperparathyroidism. The elevated parathormone level (PTH) confirms

the diagnosis. Her chest film demonstrates marked osteopenia and

the hand films are classic for this disease with severe demineralization and

periosteal bone resorption most prominent in the middle phalanges. The

films show no evidence of malignant lesions or mediastinal adenopathy

consistent with sarcoidosis, and an elevated PTH level is not found in

Paget's disease or vitamin D intoxication.

Treatment for primary hyperparathyroidism in this setting is resection

of the diseased parathyroid glands after initial correction of the severe

hypercalcemia. A neck exploration would yield a single parathyroid adenoma

in about 85% of cases. Two adenomata are found less often (approximately

5%) and hyperplasia of all four glands occurs in about 10-15% of

patients. If hyperplasia is found, treatment includes resection of three and

one-half glands. The remnant of the fourth gland can be identified with a

metal clip in case reexploration becomes necessary. Alternatively, all four

glands can be removed with autotransplantation of a small piece of parathyroid

tissue into the forearm or sternocleidomastoid muscle. Subsequent

hyperfunction, should it develop, can then be treated by removal of this tissue.

A patient with osteopenia this severe will need calcium supplementation

postoperatively. Vitamin D supplementation may also be necessary if

hypocalcemia develops and persists despite treatment with oral calcium.

267. The answer is a. (Schwartz, 7/e, pp 1678-1689.) The thyroid scan of

the patient discussed in the question shows a discrete area of decreased

174 Surgery

radioactive iodine uptake with the remainder of the gland accepting iodine

normally. This means the tissue that composes the nodule is not endocrinologically

active for thyroid hormone. The two major mass lesions of the

thyroid that can produce this pattern are a nonfunctioning follicular adenoma

and a carcinoma. Carcinomas seldom produce thyroid hormone.

Adenomas may be very active (toxic) and suppress the remaining gland.

Most thyroid adenomas, however, are not hormone producing and appear

as "cold" nodules on a thyroid scan. Graves' disease produces a diffusely

hyperactive gland without nodularity. de Quervain's thyroiditis presents as

a painful, swollen thyroid gland rather than as a discrete nodule. A large

parathyroid adenoma could conceivably displace the thyroid gland and

produce a pattern similar to the one shown, but it would be unusual. A

localized infectious process also could produce such a pattern. The essential

point is that a "cold" thyroid nodule may represent a carcinoma, and

needle biopsy or surgical excision is indicated to rule out this possibility.

268. The answer is a. (Schwartz, 7/e, pp 552-553.) Cystosarcoma phylloides

is a tumor most often seen in younger women. It can grow to enormous

size and at times ulcerate through the skin. Still, it is a lesion with

low propensity toward metastasis. Local recurrence is common, especially

if the initial resection was inadequate. Simple reexcision with adequate

margins is curative. Very large lesions may necessitate simple mastectomy

to achieve clear margins. Axillary lymphadenectomy, however, is seldom

indicated without biopsy-positive demonstration of tumor in the nodes.

The low incidence of metastatic disease suggests that adjunctive therapy is

indicated only for known metastatic disease, even when the tumors are

quite large and ulcerated.

269. The answer is e. (Schwartz, 7/e, pp 1681-1684.) This patient has cytologic

evidence of a papillary lesion, possibly papillary carcinoma. Papillary

carcinoma is a relatively nonaggressive lesion with long-term survival (>20

years) of more than 90%. The lesion is frequently multicentric, which argues

for more complete resection. Metastases, when they occur, are usually responsive

to surgical resection or radioablation therapy. Removal of the involved

lobe, and possibly the entire thyroid gland, is appropriate. Central and lateral

lymph node dissection is performed for clinically suspect lymph nodes. Papillary

carcinoma is frequently multifocal. Bilateral disease mandates total thyroidectomy.

The use of radioactive 131I, however, is contraindicated in

pregnancy and should be used with caution in women of childbearing age.

Endocrine Problems and Breast Answers 175

270. The answer is c. (Schwartz, 7/e, pp 1685-1686.) Hürthle-cell cancer

is a type of follicular cancer, but it tends to recur more often than other

types. Follicular cancer spreads hematogenously to distant sites. This is

unlike papillary cancer, which metastasizes via the lymphatics. Amyloid

deposits in the stroma of a thyroid tumor are diagnostic of medullary carcinoma.

The treatment of choice is a near-total thyroidectomy to facilitate

later body scanning for metastases and treatment with 131I.

271. The answer is e. (Schwartz, 7/e, pp 1681-1689.) Thyroid nodules

are somewhat less common in men and should always suggest malignancy.

The history of irradiation to the chest and the findings on biopsy mandate

resection of the lesion in this patient, since prior exposure to radiation,

even at low dosage, is a strong risk factor for the subsequent development

of thyroid cancer. The optimum management of thyroid carcinoma

remains controversial. Thyroid lobectomy, subtotal thyroidectomy, and

total thyroidectomy are all acceptable techniques for treatment. Removal of

the gland permits more accurate histologic diagnosis, particularly with

regard to the relatively radioresistant Hürthle-cell follicular variant.

Removal of the gland also makes subsequent treatment of metastases with

radioactive iodine more effective. Suppression with thyroid hormone (Synthroid)

in the setting of abnormal cytology is not recommended.

272. The answer is a. (Harris, pp 106-110.) Nipple discharge from the

breast may be classified as pathologic, physiologic, or galactorrhea. Galactorrhea

may be due to hormonal imbalance (hyperprolactinemia,

hypothyroidism), drugs (oral contraceptives, phenothiazines, antihypertensives,

tranquilizers), or trauma to the chest. Physiologic nipple discharge

is intermittent, nonlactational (usually serous), and due to stimulation

of the nipple or to drugs (estrogens, tranquilizers). Both galactorrhea

and physiologic discharge are frequently bilateral and arise from multiple

ducts. Pathologic nipple discharge may be caused by benign lesions

of the breast (duct ectasia, papilloma, fibrocystic disease) or by cancer. It

may be bloody, serous, or gray-green. It is spontaneous and unilateral and

can often be localized to a single nipple duct. When pathologic discharge

is diagnosed, an effort should be made to identify the source. If an associated

mass is present, it should be biopsied. If no mass is found, a terminal

duct excision of the involved duct(s) should be performed. Only 10 percent

of patients with pathologic nipple discharge are found to have breast

cancer.

176 Surgery

273. The answer is c. (Schwartz, 7/e, pp 1622-1623, 1635-1639.) Cushing's

disease is caused by hypersecretion of ACTH by the pituitary gland.

This hypersecretion, in turn, is caused by either a pituitary adenoma (90%

of cases) or diffuse pituitary corticotrope hyperplasia (10% of cases) due to

hypersecretion of CRH (corticotropin-releasing hormone) by the hypothalamus.

A high cure rate is achieved with surgery, occasionally followed by

adjuvant radiotherapy for large pituitary adenomas. Cushing syndrome

refers to the clinical manifestations of glucocorticoid excess due to any

cause (Cushing's disease, administration of exogenous glucocorticoids,

adrenocortical hyperplasia, adrenal adenoma, adrenal carcinoma, ectopic

ACTH-secreting tumors) and includes truncal obesity, hypertension, hirsutism,

moon facies, proximal muscle wasting, ecchymoses, skin striae,

osteoporosis, diabetes mellitus, amenorrhea, growth retardation, and

immunosuppression. The most common cause of Cushing syndrome is

iatrogenic, via administration of synthetic corticosteroids.

274. The answer is d. (Schwartz, 7/e, pp 1493-1494, 1686-1688.) Insulinsecreting

β-cell tumors of the pancreas produce paroxysmal nervous system

manifestations that may be a consequence of hypoglycemia, although the

blood glucose level may bear little relation to the severity of the symptoms,

even in the same patient from episode to episode. Most insulinomas are single

discrete tumors. Patients with insulinoma in the setting of the MEN 1

syndrome (synchronous islet cell tumors of the pancreas, pituitary hyperplasia

or adenomas, and parathyroid chief cell hyperplasia), however, are

more likely to have multiple tumors throughout the pancreas. If a careful

examination of the pancreas reveals one or more specific adenomas, these

can be locally excised. Excision of these tumors may be difficult in MEN 1,

when the tumors are small and multiple (10-15% of cases). The finding of

an elevated serum calcium level would raise the suspicion of MEN 1 and

parathyroid hyperplasia. Insulinomas are not associated with MEN 2, which

comprises coexistent medullary thyroid cancer, parathyroid hyperplasia,

and pheochromocytoma. About one in seven of these tumors is malignant.

Streptozotocin, a potent antibiotic that selectively destroys islet cells, can be

useful in controlling symptoms from unresectable malignant tumors of the

islet cells but probably has little to offer in the definitive management of the

typical benign islet cell insulinoma.

275. The answer is a. (Harris, pp 159-167.) Breast cancer is rarely seen

before the age of 20, but thereafter its incidence increases inexorably. While

Endocrine Problems and Breast Answers 177

the prevalence of breast cancer (the raw number of patients alive with disease)

is greatest among perimenopausal women, the incidence of breast cancer

(the number of new cases per 100,000 population) rises so sharply that it

is twice as common among women between 80 and 85 years of age as among

those 60 to 65. In addition, the age-adjusted incidence has increased steadily

since the mid-1940s. No data is presently available consistently linking the

incidence of breast cancer to dietary factors. A possible linkage between

breast cancer and alcohol consumption at an early age is being studied.

276-280. The answers are 276-c, 277-b, 278-b, 279-e, 280-d.

(Schwartz, 7/e, p 321.) The American Joint Committee on Cancer has

defined a four-tiered staging system for breast cancer based on the clinical

criteria of tumor size, involvement of lymph nodes, and metastatic disease.

In one version of this system, a separate category is reserved for inflammatory

breast cancer. While the grouping of breast cancers into stages provides

a useful shorthand for expressing a patient's survival probability, it is

noteworthy that considerable heterogeneity exists both with respect to

tumor size and nodal characteristics among tumors that are classified

within a given stage.

The TNM stage of breast cancer is assigned by measuring the greatest

diameter of the tumor ("T"), assessing the axillary and clavicular lymph

nodes for enlargement and fixation ("N"), and judging whether metastatic

disease is present ("M"). In general, the worst of the three TNM parameters

will determine the stage assignment.

Tumors that are not palpable are classified T0; tumors 2 cm or less, T1;

tumors greater than 2 but not more than 5 cm, T2; tumors greater than 5

cm, T3; and tumors with extension into the chest wall or skin, T4.

Clinically negative lymph nodes are classified N0; positive, movable

ipsilateral axillary nodes, N1; fixed ipsilateral axillary nodes, N2; and clavicular

nodes, N3.

Absence of evidence of metastatic disease is classified M0; distant

metastatic disease, M1.

The patient in question 276 has a T0, N2, M0 lesion. This is stage III

(fixed or matted nodes are a poor prognostic sign).

The patient in question 277 has a T2, N1, M0 lesion. This is stage II.

The patient in question 278 has a T2, N0, M0 lesion. Though smaller

than the tumor in question 277 and without clinically involved nodes, this

tumor is also stage II.

178 Surgery

The patient in question 279 has findings compatible with inflammatory

breast cancer. A biopsy of the involved skin and a mammogram would

confirm the diagnosis.

The patient in question 280 has a T1, N0, M1 lesion. This is stage IV

(stage IV is any T, any N, M1).

281-285. The answers are 281-d, 282-a, 283-b, 284-e, 285-c.

(Schwartz, 7/e, pp 1244-1246.) Carcinoid tumors are most commonly found

in the appendix and small bowel, where they may be multiple. They have a

tendency to metastasize, which varies with the size of the tumor. Tumors

< 1 cm uncommonly metastasize. Tumors > 2.0 cm are more often found to

be metastatic. Metastasis to the liver and beyond may give rise to the carcinoid

syndrome. The tumors cause an intense desmoplastic reaction. Spread

into the serosal lymphatics does not imply metastatic disease; local resection

is potentially curative. When metastatic lesions are found in the liver, they

should be resected when technically feasible to limit the symptoms of the

carcinoid syndrome. When extensive hepatic metastases are found, the disease

is not curable. Resection of the appendix and cecum may be performed

to prevent an early intestinal obstruction by locally encroaching tumor.

286-290. The answers are 286-c, f; 287-a, h; 288-a; 289-d, e; 290-

g. (Schwartz, 7/e, pp 572-586.) Generally accepted treatment for stage I

breast cancer in premenopausal women includes lumpectomy (wide excision,

partial mastectomy, quadrantectomy) combined with axillary lymph

node dissection and adjuvant radiation therapy, and modified radical mastectomy.

Both approaches offer equivalent chances of cure; there is a higher

incidence of local recurrence with lumpectomy, axillary dissection, and

radiation, but this observation has not been found to affect the overall cure

rate in comparison with mastectomy.

Patients with familial breast cancer (multiple first-degree relatives and

penetrance of breast cancer through several familial generations) have

extremely high risks of developing breast cancer in the course of their lifetimes.

A subset of patients with familial breast cancer has been identified

by a specific gene mutation (BRCA1); however, the genetic basis of most

cases of familial breast cancer has yet to be elucidated. A patient with a history

of familial breast cancer and multiple biopsies showing atypia may reasonably

request bilateral prophylactic simple mastectomies. Alternatively,

she may continue with routine surveillance.

Endocrine Problems and Breast Answers 179

Lobular carcinoma in situ is a histologic marker that identifies patients

who are at increased risk for the development of breast cancer. It is not a

precancerous lesion in itself, and there is no benefit to widely excising it

because the risk of subsequent cancer is equal for both breasts. As the risk

for the future development of breast cancer is now estimated to be approximately

1% per year, prophylactic mastectomy is no longer recommended.

Proper management would consist of close surveillance for cancer by twice

yearly examinations and yearly mammography. Sclerosing adenosis is a

benign lesion.

Ductal carcinoma in situ is the precursor of invasive ductal carcinoma.

It is described in four histologic variants (papillary, cribriform, solid, and

comedo type), of which the comedo subtype shows the greatest tendency

to recur after wide excision alone. For years, ductal carcinoma in situ was

treated by simple mastectomy. In recent years, studies have shown equally

good results with wide excision alone (for small noncomedo lesions) or

wide excision plus radiation therapy. For a 1.0-cm comedocarcinoma

(which may extend microscopically wider still), most experts would favor

simple mastectomy or wide excision with radiation therapy.

There are few indications for radical mastectomy as it is both more

traumatic and disfiguring than any other method of local control of breast

cancer and offers no greater survival benefit. One indication for radical

mastectomy, however, is locally advanced breast cancer with wide invasion

of the pectoralis major in a patient who is physiologically able to tolerate

general anesthesia.

180 Surgery

GASTROINTESTINAL

TRACT, LIVER,AND

PANCREAS

Questions

DIRECTIONS: Each item below contains a question or incomplete

statement followed by suggested responses. Select the one best response to

each question.

181

291. Omeprazole has been added

to the H2 antagonists as a therapeutic

approach to the management of

acute gastric and duodenal ulcers.

It acts by

a. Blocking breakdown of mucosaldamaging

metabolites of NSAIDs

b. Providing a direct cytoprotective

effect

c. Buffering gastric acids

d. Inhibiting parietal cell hydrogenpotassium-

ATPase

e. Inhibiting gastrin release and parietal

cell acid production

292. Evidence that a splenectomy

might benefit a patient with

immune (idiopathic) thrombocytopenic

purpura (ITP) includes

a. A significant enlargement of the

spleen

b. A high reticulocyte count

c. Megakaryocytic elements in the

bone marrow

d. An increase in the platelet count on

cortisone therapy

e. Patient age of less than 5 years

Terms of Use

293. An 18-year-old woman presents with abdominal pain, fever, and

leukocytosis. With the presumptive diagnosis of appendicitis, a right lower

quadrant (McBurney) incision is made and the lesion pictured below is

delivered. The process is 50 cm proximal to the ileocecal valve. This lesion

182 Surgery

a. Can best be diagnosed by preoperative angiogram, which should be done

whenever the diagnosis is suspected

b. Should routinely be removed when incidentally discovered during celiotomy

c. Is embryologically derived from a persistent vitelline duct (omphalomesenteric

duct)

d. Often contains ectopic adrenal tissue

e. Is frequently associated with cutaneous flushing and episodic tachycardia

294. A 41-year-old man complains of regurgitation of saliva and of

ingested but undigested food. An esophagram reveals a "bird's beak" deformity.

Which of the following statements is true about this condition?

a. Chest pain is common in the advanced stages of this disease

b. More patients are improved by forceful dilation than by surgical intervention

c. Manometry can be expected to show high resting pressures of the lower

esophageal sphincter

d. Surgical treatment primarily consists of resection of the distal esophagus with

reanastomosis to the stomach above the diaphragm

e. Patients with this disease are at no increased risk for the development of carcinoma

295. Which of the following statements

concerning imperforate anus

is true?

a. Imperforate anus affects males

more frequently than females

b. In 90% of males, but only 50% of

females, the rectum ends below the

level of the levator ani complex

c. The rectum usually ends in a blind

pouch

d. The chance for eventual continence

is greater when the rectum has

descended to below the levator ani

muscles

e. Immediate definitive repair of the

anatomic defect is required to maximize

the chance of eventual continence

Items 296-297

A previously healthy 80-yearold

woman presents with early satiety

and abdominal fullness. The CT

scan shown below is obtained.

Gastrointestinal Tract, Liver, and Pancreas 183

296. The lesion is most likely a

a. Pancreatic pseudocyst

b. Pancreatic adenocarcinoma

c. Pancreatic cystadenocarcinoma

d. Retroperitoneal lymphoma

e. Pancreatic serous cystadenoma

297. Which of the following statements

about this lesion is true?

a. Clinical and laboratory findings

together establish a preoperative

diagnosis

b. Significant weight loss and back

pain are the typical presentation

c. The lesion may be multilocular or

calcified

d. It is unlikely to be cured by resection

if large

e. It is associated with a history of

pancreatitis

298. A patient with a history of

familial polyposis undergoes a

diagnostic polypectomy. Which of

the following types of polyps is

most likely to be found?

a. Villous adenoma

b. Hyperplastic polyp

c. Adenomatous polyp

d. Retention polyp

e. Pseudopolyp

299. What is the most common

serious complication of an end

colostomy?

a. Bleeding

b. Skin breakdown

c. Parastomal hernia

d. Colonic perforation during irrigation

e. Stomal prolapse

300. Which of the following statements

regarding pancreatic carcinoma

is true?

a. The majority of cases present with

jaundice alone

b. CT scan, angiography, and laparoscopy

have been unsuccessful in

predicting resectability

c. If a patient is jaundiced, the

resectability rate is less than 5%

d. 99% of patients with pancreatic

cancer have metastatic disease at

the time of diagnosis

e. The 5-year survival rate after a

Whipple procedure (pancreaticoduodenectomy)

performed for cure

is 30-40%

Items 301-302

A 45-year-old woman is

explored for a perforated duodenal

ulcer 6 h after onset of symptoms.

She has a history of chronic peptic

ulcer disease treated medically with

minimal symptoms.

301. The procedure of choice is

a. Simple closure with omental patch

b. Truncal vagotomy and pyloroplasty

c. Antrectomy and truncal vagotomy

d. Highly selective vagotomy

e. Hemigastrectomy

302. Six weeks after surgery, the

patient returns complaining of

postprandial weakness, sweating,

light-headedness, crampy abdominal

pain, and diarrhea. The best

management would be

a. Antispasmodic medications (e.g.,

Lomotil)

b. Dietary advice and counseling that

symptoms will probably abate

within 3 mo of surgery

c. Dietary advice and counseling that

symptoms will probably not abate

but are not dangerous

d. Workup for neuroendocrine tumor

(e.g., carcinoid)

e. Preparation for revision to Rouxen-

Y gastrojejunostomy

184 Surgery

Items 303-304

A 60-year-old male alcoholic is

admitted to the hospital with

hematemesis. His blood pressure is

100/60 mm Hg, the physical examination

reveals splenomegaly and

ascites, and the initial hematocrit is

25%. Nasogastric suction yields

300 mL of fresh blood.

303. A 55-year-old man complains

of chronic intermittent epigastric

pain, and gastroscopy

demonstrates a 2-cm ulcer of the

distal lesser curvature. Endoscopic

biopsy yields no malignant tissue.

After a 6-wk trial of H2 blockade

and antacid therapy, the ulcer is

unchanged. Proper therapy at this

a. Repeat trial of medical therapy

b. Local excision of the ulcer

c. Billroth I partial gastrectomy

d. Billroth I partial gastrectomy with

vagotomy

e. Vagotomy and pyloroplasty

304. After initial resuscitation, this

man should undergo

a. Esophageal balloon tamponade

b. Barium swallow

c. Selective angiography

d. Esophagogastroscopy

e. Exploratory celiotomy

305. A diagnosis of bleeding

esophageal varices is made in this

patient. Appropriate initial therapy

would be

a. Intravenous vasopressin

b. Endoscopic sclerotherapy

c. Emergency portacaval shunt

d. Emergency esophageal transection

e. Esophageal balloon tamponade

306. During an operation for carcinoma

of the hepatic flexure of the

colon, an unexpected discontinuous

3-cm metastasis is discovered

in the edge of the right lobe of the

liver. The surgeon should

a. Terminate the operation, screen the

patient for evidence of other metastases,

and plan further therapy after

the reevaluation

b. Perform a right hemicolectomy and

a right hepatic lobectomy

c. Perform a right hemicolectomy and

a wedge resection of the metastasis

d. Perform a cecostomy and schedule

reoperation after a course of systemic

chemotherapy

e. Perform local resection of the primary

colon cancer and plan radiation

therapy for the lesion on the

liver

Gastrointestinal Tract, Liver, and Pancreas 185

307. A 42-year-old man with no

history of use of nonsteroidal antiinflammatory

drugs (NSAIDs) presents

with recurrent gastritis.

Infection with Helicobacter pylori is

suspected. Which of the following

statements is true?

a. Morphologically, the bacteria is a

gram-positive, tennis-racket-shaped

organism

b. Diagnosis can be made by serologic

testing or urea breath tests

c. Diagnosis is most routinely

achieved via culturing endoscopic

scrapings

d. The most effective way to treat and

prevent recurrence of this patient's

gastritis is through the use of singledrug

therapy aimed at eradicating

H. pylori

e. The organism is easily eradicated

308. Which of the following hernias

follows the path of the spermatic

cord within the cremaster

muscle?

a. Femoral

b. Direct inguinal

c. Indirect inguinal

d. Spigelian

e. Interparietal

309. A spry octogenarian who has

never before been hospitalized is

admitted with signs and symptoms

typical of a small bowel obstruction.

Which of the following clinical

findings would give the most

help in ascertaining the diagnosis?

a. Coffee-grounds aspirate from the

stomach

b. Aerobilia

c. A leukocyte count of 40,000/μL

d. A pH of 7.5, PCO2 of 50 kPa, and

paradoxically acid urine

e. A palpable mass in the pelvis

310. Which of the following colonic

pathologies is thought to have

no malignant potential?

a. Ulcerative colitis

b. Villous adenomas

c. Familial polyposis

d. Peutz-Jeghers syndrome

e. Crohn's colitis

311. A 70-year-old woman has

nausea, vomiting, abdominal distention,

and episodic, crampy midabdominal

pain. She has no history

of previous surgery but has a long

history of cholelithiasis for which

she has refused surgery. Her abdominal

radiograph reveals a spherical

density in the right lower quadrant.

Correct treatment should consist of

a. Ileocolectomy

b. Cholecystectomy

c. Ileotomy and extraction

d. Nasogastric tube decompression

e. Intravenous antibiotics

186 Surgery

312. Which of the following statements

concerning Hirschsprung's

disease is true?

a. It is initially treated by colostomy

b. It is best diagnosed in the newborn

period by barium enema

c. It is characterized by the absence of

ganglion cells in the transverse

colon

d. It is associated with a high incidence

of genitourinary tract anomalies

e. It is the congenital disease that

most commonly leads to subsequent

fecal incontinence

313. Spontaneous closure of

which of the following congenital

abnormalities of the abdominal wall

generally occurs by the age of 4?

a. Umbilical hernia

b. Patent urachus

c. Patent omphalomesenteric duct

d. Omphalocele

e. Gastroschisis

314. Laparoscopic cholecystectomy

is indicated for symptomatic

gallstones in which of the following

conditions?

a. Cirrhosis

b. Prior upper abdominal surgery

c. Suspected carcinoma of the gallbladder

d. Morbid obesity

e. Coagulopathy

Gastrointestinal Tract, Liver, and Pancreas 187

315. Infants with anorectal anomalies

tend to have other congenital

anomalies. Associated abnormalities

include which of the following?

a. Abnormalities of the cervical spine

b. Hydrocephalus

c. Duodenal atresia

d. Heart disease

e. Corneal opacities

316. A 48-year-old woman develops

pain of the right lower quadrant

while playing tennis. The pain

progresses and the patient presents

to the emergency room later that

day with a low-grade fever, a white

blood count of 13,000, and complaints

of anorexia and nausea as

well as persistent, sharp pain of the

right lower quadrant. On examination

she is tender in the right lower

quadrant with muscular spasm and

there is a suggestion of a mass

effect. An ultrasound is ordered

and shows an apparent mass in the

abdominal wall. Which of the following

is the most likely diagnosis?

a. Acute appendicitis

b. Cecal carcinoma

c. Hematoma of the rectus sheath

d. Torsion of an ovarian cyst

e. Cholecystitis

317. A 36-h-old infant presents with bilious vomiting and an increasingly

distended abdomen. At exploration the segment below is found as the

point of obstruction. Which of the following statements regarding this

finding is true?

188 Surgery

a. Resection with primary anastomosis should not be performed

b. Gentle, persistent traction on the specimen usually corrects the defect and

removes the need for a resection

c. The lesion is much more common in the jejunum than in the ileum in this age

group

d. This problem is probably related to mesenteric vascular insufficiency

e. A properly monitored barium enema might have corrected this defect and

removed the need for an operation

318. In determining the proper

treatment for a sliding hiatal hernia,

the most useful step would be

a. Barium swallow with cinefluoroscopy

during Valsalva maneuver

b. Flexible endoscopy

c. 24-h monitoring of esophageal pH

d. Measuring the size of the hernia

e. Assessing the patient's smoking and

drinking history

319. Which of the following statements

regarding the etiology of

obstructive jaundice is true?

a. A markedly elevated SGOT and

SGPT are usually associated with

obstructive jaundice

b. When extrahepatic biliary obstruction

is suspected, the first test

should be endoscopic ultrasonography

(EUS)

c. A Klatskin tumor will result in

extrahepatic ductal dilation only

d. A liver-spleen scan will add significantly

to the diagnostic workup for

obstructive jaundice

e. Carcinoma of the head of the pancreas

can cause deep epigastric or

back pain in as many as 80% of

patients

320. A previously healthy 9-yearold

child comes to the emergency

room because of fulminant upper

gastrointestinal bleeding. The hemorrhage

is most likely to be the

result of

a. Esophageal varices

b. Mallory-Weiss syndrome

c. Gastritis

d. A gastric ulcer

e. A duodenal ulcer

321. Intragastric pressure remains

steady near 2-5 mm Hg during

slow gastric filling, but rises rapidly

to high levels after reaching a volume

of

a. 400-600 mL

b. 700-900 mL

c. 1000-1200 mL

d. 1300-1500 mL

e. 1600-1800 mL

Gastrointestinal Tract, Liver, and Pancreas 189

322. Which of the following statements

is true regarding the effects

of colon resection?

a. Net absorption of water by the rectum

has been demonstrated in

humans

b. Patients who undergo major colon

resections suffer little change in

their bowel habits following operation

c. The left colon is better adapted for

water absorption than the right

colon

d. The right colon is better adapted

for electrolyte absorption than the

left colon

e. The role of the ileocecal valve in

normal fluid homeostasis is well

established

323. Operative planning and preoperative

counseling for a patient

with a rectal carcinoma can be best

provided if the patient is staged

before surgery by

a. Rigid proctoscopy

b. Barium enema

c. MRI of the pelvis

d. CT scanning of the pelvis

e. Rectal endosonography

324. Which statement regarding

absorption by the small intestine is

true?

a. All but the fat in milk is digested

and absorbed in humans by the

end of the duodenum

b. Complete absorption of carbohydrates

in a normal meal occurs in

the ileum

c. In short gut syndrome, much of the

dietary carbohydrate appears in the

stool

d. Aldosterone markedly decreases

sodium transport across the gut

mucosa

e. Enzymes of the brush border of the

small intestine can digest and

absorb less than 5% of an average

protein meal in the absence of the

pancreas

325. Local stimuli that inhibit the

release of gastrin from the gastric

mucosa include which of the following?

a. Small proteins

b. 20-proof alcohol

c. Caffeine

d. Acidic antral contents

e. Antral distention

190 Surgery

326. Which statement regarding

fat absorption is true?

a. Half of neutral fat can be absorbed

in the complete absence of bile and

pancreatic lipase

b. Fifty percent of the total bile salt

pool is lost in the stool and replaced

daily by synthesis in the liver

c. Glycerol, short-chain fatty acids,

and medium-chain triglycerides

exit the mucosal cell in chylomicrons

d. Conjugated bile salts are actively

resorbed in the colon and returned

to the liver via the portal vein

e. Water-insoluble dietary lipid is rendered

into soluble micelles through

mixing with pancreatic amylase

327. For a symptomatic partial

duodenal obstruction secondary to

an annular pancreas, the operative

treatment of choice is

a. A Whipple procedure

b. Gastrojejunostomy

c. Vagotomy and gastrojejunostomy

d. Partial resection of the annular pancreas

e. Duodenojejunostomy

328. A previously healthy 15-yearold

boy is brought to the emergency

room with complaints of about 12 h

of progressive anorexia, nausea, and

pain of the right lower quadrant.

On physical examination, he is

found to have a rectal temperature

of 38.18°C (100.58°F) and has

direct and rebound abdominal tenderness

localizing to McBurney's

point as well as involuntary guarding

in the right lower quadrant. At

operation through a McBurney-type

incision, the appendix and cecum

are found to be normal, but the surgeon

is impressed with the marked

edema of the terminal ileum, which

also has an overlying fibrinopurulent

exudate. The correct procedure

a. Close the abdomen after culturing

the exudate

b. Perform a standard appendectomy

c. Resect the involved terminal ileum

d. Perform the ileocolic resection

e. Perform an ileocolostomy to bypass

the involved terminal ileum

Gastrointestinal Tract, Liver, and Pancreas 191

329. A 32-year-old woman undergoes

a cholecystectomy for acute

cholecystitis and is discharged home

on the sixth postoperative day. She

returns to the clinic 8 mo after the

operation for a routine visit and is

noted by the surgeon to be jaundiced.

Laboratory values on readmission

show total bilirubin 5.6

mg/dL; direct bilirubin 4.8 mg/dL;

alkaline phosphatase 250 IU (normal

21-91 IU); SGOT 52 KU

(normal 10-40 KU); SGPT 51 KU

(normal 10-40 KU). An ultrasonogram

shows dilated intrahepatic

ducts. The patient undergoes the

transhepatic cholangiogram seen below.

Appropriate management is

a. Choledochoplasty with insertion of

a T tube

b. End-to-end choledochocholedochal

anastomosis

c. Roux-en-Y choledochojejunostomy

d. Percutaneous transhepatic dilatation

e. Choledochoduodenostomy

192 Surgery

330. After complete removal of a

sessile polyp of 2.0 × 1.5 cm found

one fingerlength above the anal

mucocutaneous margin, the pathologist

reports it to have been a

villous adenoma that contained

carcinoma in situ. You would recommend

that this patient undergo

a. Reexcision of the biopsy site with

wider margins

b. Abdominoperineal rectosigmoid

resection

c. Anterior resection of the rectum

d. External radiation therapy to the

rectum

e. No further therapy

331. A 55-year-old woman with

cancer of the cervix undergoes hysterectomy

and is found to have

pelvic lymph nodes involved with

cancer. She then receives a course

of external beam radiation (4500

rads). When the physician counsels

her prior to her radiation treatment,

she should be told of all the

possible complications of radiation

enteritis. Which of the following is

generally not associated with radiation

injury?

a. Malabsorption

b. Intussusception

c. Ulceration

d. Fistulization

e. Perforation

332. Which of the following

would be expected to stimulate

intestinal motility?

a. Fear

b. Gastrin

c. Secretin

d. Acetylcholine

e. Cholecystokinin

333. Which of the following statements

concerning carcinoma of the

esophagus is true?

a. Alcohol has been implicated as a

precipitating factor

b. Squamous carcinoma is the most

common type at the cardioesophageal

junction

c. It has a higher incidence in males

d. It occurs more commonly in patients

with corrosive esophagitis

e. Surgical excision is the only effective

treatment

Gastrointestinal Tract, Liver, and Pancreas 193

Items 334-335

334. A 30-year-old man with a

duodenal ulcer is being considered

for surgery because of intractable

pain and a previous bleeding episode.

Serum gastrin levels are found

to be over 1000 pg/mL (normal

40-150) on three separate determinations.

The patient should be told

that the operation of choice is

a. Vagotomy and pyloroplasty

b. Highly selective vagotomy and

tumor resection

c. Subtotal gastrectomy

d. Total gastrectomy

e. Partial pancreatectomy

335. Another 30-year-old man

with the identical clinical situation

presented in the previous question

is being considered for surgery. His

serum gastrin level, however, is 150

10 pg/mL on three determinations.

The surgeon should perform

a. An arteriogram

b. A secretin stimulation test

c. A total gastrectomy

d. A subtotal gastrectomy

e. A highly selective vagotomy

336. The most common clinical

presentation of idiopathic retroperitoneal

fibrosis is

a. Ureteral obstruction

b. Leg edema

c. Calf claudication

d. Jaundice

e. Intestinal obstruction

337. A 55-year-old man who is

extremely obese reports weakness,

sweating, tachycardia, confusion,

and headache whenever he fasts for

more than a few hours. He has

prompt relief of symptoms when he

eats. These symptoms are most

suggestive of which of the following

disorders?

a. Diabetes mellitus

b. Insulinoma

c. Zollinger-Ellison syndrome

d. Carcinoid syndrome

e. Multiple endocrine neoplasia, type

II

338. In planning the management

of a 2.8-cm epidermoid carcinoma

of the anus, the first therapeutic

approach should be

a. Abdominoperineal resection

b. Wide local resection with bilateral

inguinal node dissection

c. Local radiation therapy

d. Systemic chemotherapy

e. Combined radiation therapy and

chemotherapy

194 Surgery

339. An 80-year-old man is admitted

to the hospital complaining of

nausea, abdominal pain, distention,

and diarrhea. A cautiously

performed transanal contrast study

reveals an "apple core" configuration

in the rectosigmoid. Appropriate

management at this time would

include

a. Colonoscopic decompression and

rectal tube placement

b. Saline enemas and digital disimpaction

of fecal matter from the rectum

c. Colon resection and proximal

colostomy

d. Oral administration of metronidazole

and checking a Clostridium difficile

e. Evaluation of an electrocardiogram

and obtaining an angiogram to

evaluate for colonic mesenteric

ischemia

340. Indications for operation in

Crohn's disease include which of

the following?

a. Intestinal obstruction

b. Enterovesical fistula

c. Ileum-ascending colon fistula

d. Enterovaginal fistula

e. Free perforation

341. A 50-year-old man presents

to the emergency room with a 6-h

history of excruciating abdominal

pain and distention. The abdominal

film shown below is obtained.

The next diagnostic maneuver

should be

Gastrointestinal Tract, Liver, and Pancreas 195

a. Emergency celiotomy

b. Upper gastrointestinal series with

small-bowel follow-through

c. CT scan of the abdomen

d. Barium enema

e. Sigmoidoscopy

342. Which of the following

organisms is most closely associated

with gastric and duodenal

ulcer disease?

a. Campylobacter

b. Cytomegalovirus

c. Helicobacter

d. Mycobacterium avium-intracellulare

e. Yersinia enterocolitica

343. On Monday morning, a septuagenarian

man has a moderatesized

abdominal aneurysm

resected. On Friday, he is noted to

be markedly distended with an

abdominal radiograph on which

the cecum is measured as 12 cm

across. Proper management at this

time would be

a. Decompression of the large bowel

via colonoscopy

b. Replacement of the nasogastric

tube and administration of lowdose

cholinergic drugs

c. Continued nothing-by-mouth

orders, administration of a gentle

saline enema, and encouragement

of ambulation

d. Immediate return to the operating

room for operative decompression

by transverse colostomy

e. Right hemicolectomy

344. In the management of echinococcal

liver cysts

a. A large cyst should be treated by

percutaneous aspiration of its contents

b. Medical treatment with albendazole

usually preempts the need for

surgical drainage

c. Negative serologic tests suggest that

the cyst is chronic and inactive and

that no treatment is indicated

d. Leakage of cyst fluid puts the

patient at risk for anaphylactic reaction

e. Coexistent extrahepatic cysts are

uncommon

345. Which of the following statements

regarding appendicitis during

pregnancy is correct?

a. Appendicitis is the most prevalent

extrauterine indication for celiotomy

during pregnancy

b. Appendicitis occurs more commonly

in pregnant women than in

nonpregnant women of comparable

age

c. Suspected appendicitis in a pregnant

woman should be managed with a

period of observation of due to the

risks of laparotomy to the fetus

d. Noncomplicated appendicitis results

in a 20% fetal mortality and

premature labor rate

e. The severity of appendicitis correlates

with increased gestational age

of the fetus

196 Surgery

346. Which of the following is most likely to require surgical correction?

a. Large sliding esophageal hiatal hernia

b. Paraesophageal hiatal hernia

c. Traction diverticulum of esophagus

d. Schatzki's ring of distal esophagus

e. Esophageal web

347. A 65-year-old man who is hospitalized with pancreatic carcinoma

develops abdominal distention and obstipation. The following abdominal

radiograph is obtained. Appropriate management would best be achieved by

a. Urgent colostomy or cecostomy

b. Discontinuation of anticholinergic medications and narcotics and correction of

metabolic disorders

c. Digital disimpaction of a fecal mass in the rectum

d. Diagnostic and therapeutic colonoscopy

e. Detorsion of the volvulus and colopexy or resection

Gastrointestinal Tract, Liver, and Pancreas 197

348. True statements regarding

Zenker's diverticulum include

a. Aspiration pneumonitis is unlikely

b. It is a congenital abnormality

c. The most common symptom is a

sensation of high obstruction on

swallowing

d. It is a traction-type diverticulum

e. Treatment is restriction of certain

foods

349. True statements regarding

hemobilia include which of the following?

a. The classic presentation includes

biliary colic, jaundice, and gastrointestinal

bleeding

b. Spontaneous bleeding secondary to

hematologic disorders is the major

cause of this disorder

c. Percutaneous transhepatic catheter

placement of an absorbable gelatin

sponge (Gelfoam) is the preferred

treatment in cases of significant

intrahepatic bleeding

d. Angiography and endoscopy have

no role in the treatment of intrahepatic

bleeding

e. Arterial embolization is advocated

for hemobilia from the extrahepatic

bile ducts

Items 350-351

350. A 30-year-old female patient

who presents with bleeding per rectum

is found at colonoscopy to have

colitis confined to the transverse

and descending colon. A biopsy is

performed. Which of the following

statements is true about this patient?

a. The inflammatory process is likely

to be confined to the mucosa and

submucosa

b. The inflammatory reaction is likely

to be continuous

c. Superficial as opposed to linear

ulcerations can be expected

d. Noncaseating granulomata can be

expected in up to 50% of patients

with similar disease

e. Microabcesses within crypts are

common

351. Regarding potential complications

in this patient, which of the

following statements is true?

a. The occurrence of toxic megacolon

is common

b. Perforation occurs in about 25% of

patients with similar disease

c. Fistulas between the colon and segments

of intestine, bladder, vagina,

urethra, and skin may develop

d. Extraintestinal manifestations including

uveitis and erythema

nodosum would be exceedingly

rare in this patient

e. This patient would be at no

increased risk for the development

of cancer of the colon as compared

with an age-matched population

198 Surgery

352. An upper GI series is performed on a 71-year-old woman who presented

with several months of chest pain that occurred when she was eating.

The film below is obtained. Investigation reveals a microcytic anemia

and erosive gastritis on upper endoscopy. Which of the following statements

about the patient's condition is true?

Gastrointestinal Tract, Liver, and Pancreas 199

a. It is congenital

b. The gastroesophageal junction is above the diaphragm

c. Ulceration, gastritis, and anemia are common

d. It usually is controlled by medical therapy

e. Surgical treatment, if indicated, should be delayed up to 3 mo to allow inflammation

around the gastroesophageal junction to subside

353. Which statement regarding

adenocarcinoma of the pancreas is

true?

a. It occurs most frequently in the

body of the gland

b. It carries a 1-2% 5-year survival

rate

c. It is nonresectable if it presents as

painless jaundice

d. It can usually be resected if it presents

in the body or tail of the pancreas

and does not involve the

common bile duct

e. It is associated with diabetes insipidus

354. Correct statements concerning

intussusception in infants

include which of the following?

a. Recurrence rates following treatment

are high

b. It is frequently preceded by a gastrointestinal

viral illness

c. A 1- to 2-wk period of parenteral

alimentation should precede surgical

reduction when surgery is

required

d. Hydrostatic reduction without

surgery rarely provides successful

treatment

e. The most common type occurs at

the junction of the descending

colon and sigmoid colon

355. A 32-year-old woman presents

to the hospital with a 24-h

history of abdominal pain of the

right lower quadrant. She undergoes

an uncomplicated appendectomy

for acute appendicitis and is

discharged home on the fourth

postoperative day. The pathologist

notes the presence of a carcinoid

tumor (1.2 cm) in the tip of the

appendix. Which of the following

statements is true?

a. The patient should be advised to

undergo ileocolectomy

b. The most common location of carcinoids

is in the appendix

c. The carcinoid syndrome occurs in

more than half the patients with

carcinoid tumors

d. The tumor is an apudoma

e. Carcinoid syndrome is seen only

when the tumor is drained by the

portal venous system

356. Which of the following statements

regarding direct inguinal

hernias is true?

a. They are the most common

inguinal hernias in women

b. They protrude medially to the inferior

epigastric vessels

c. They should be opened and ligated

at the internal ring

d. They commonly protrude into the

scrotal sac in men

e. They incarcerate more commonly

than indirect hernias

200 Surgery

357. A 35-year-old woman presents with pancreatitis. Subsequent endoscopic

retrograde cholangiopancreatography (ERCP) reveals the congenital

cystic anomaly of her biliary system illustrated in the film below. Which of

the following statements regarding this problem is true?

Gastrointestinal Tract, Liver, and Pancreas 201

a. Treatment consists of internal drainage via choledochoduodenostomy

b. Malignant changes may occur within this structure

c. Most patients present with the classic triad of epigastric pain, an abdominal

mass, and jaundice

d. Cystic dilation of the intrahepatic biliary tree may coexist and is managed in a

similar fashion

e. Surgery should be reserved for symptomatic patients

358. Which of the following statements

regarding stress ulceration is

true?

a. It is true ulceration, extending into

and through the muscularis mucosa

b. It classically involves the antrum

c. Increased secretion of gastric acid

has been shown to play a causative

role

d. It frequently involves multiple sites

e. It is seen following shock or sepsis,

but for some unknown reason does

not occur following major surgery,

trauma, or burns

359. Which statement concerning

cholangitis is correct?

a. The most common infecting organism

is Staphylococcus aureus

b. The diagnosis is suggested by the

Charcot triad

c. The disease occurs primarily in

young, immunocompromised patients

d. Cholecystostomy is the procedure

of choice in affected patients

e. Surgery is indicated once the diagnosis

of cholangitis is made

360. An 88-year-old man with a

history of end-stage renal failure,

severe coronary artery disease, and

brain metastases from lung cancer

presents with acute cholecystitis.

His family wants "everything

done." The best management option

in this patient would be

a. Tube cholecystostomy

b. Open cholecystectomy

c. Laparoscopic cholecystectomy

d. Intravenous antibiotics followed by

elective cholecystectomy

e. Lithotripsy followed by long-term

bile acid therapy

202 Surgery

361. After a weekend drinking binge, a 45-year-old alcoholic man presents

to the hospital with abdominal pain, nausea, and vomiting. On physical

examination the patient is afebrile and is noted to have a palpable

tender mass in the epigastrium. Laboratory tests reveal an amylase of 250

U/dL (normal < 180). A CT scan done on the second hospital day is pictured

below. Which of the following statements concerning this patient's

condition is true?

a. The mass may cause gastric outlet or extrahepatic biliary obstruction

b. Spontaneous resolution almost never occurs

c. The mass is seen only with acute pancreatitis

d. The mass has an epithelial lining

e. Malignant degeneration occurs in about 25% of cases if left untreated

362. Dieulafoy's lesion of the stomach is characterized by

a. A large mucosal defect with underlying, friable vascular plexus

b. Frequent rebleeding after endoscopic treatment

c. Massive bleeding that requires subtotal gastrectomy

d. Location in the proximal stomach

e. Acid-peptic changes of the gastric mucosa

Gastrointestinal Tract, Liver, and Pancreas 203

363. During an appendectomy for

acute appendicitis, a 4-cm mass is

found in the midportion of the

appendix. Frozen section reveals

this lesion to be a carcinoid tumor.

Which of the following statements

is true?

a. No further surgery is indicated

b. A right hemicolectomy should be

performed

c. There is about a 50% chance that

this patient will develop the carcinoid

syndrome

d. Carcinoid tumors arise from islet

cells

e. Carcinoid syndrome can occur only

in the presence of liver metastases

364. Correct statements regarding

rectal carcinoid tumors include

a. Endoscopic resection is sufficient

for tumors smaller than 2 cm

b. Patients frequently present with the

carcinoid syndrome

c. They are rapidly growing tumors

d. Local recurrence is rare with complete

resection of the primary

lesion

e. They can develop the carcinoid

syndrome even in the absence of

liver metastases

365. Indications for surgical removal

of polypoid lesions of the

gallbladder include

a. Size greater than 0.5 cm

b. Presence of clinical symptoms

c. Patient age of over 25 years

d. Presence of multiple small lesions

e. Absence of shadowing on ultrasound

366. A patient who has a total

pancreatectomy might be expected

to develop which of the following

complications?

a. Diabetes mellitus

b. Hypercalcemia

c. Hyperphosphatemia

d. Constipation

e. Weight gain

204 Surgery

367. A 28-year-old previously

healthy woman arrives in the emergency

room complaining of 24 h of

anorexia and nausea and lower

abdominal pain that is more

intense in the right lower quadrant

than elsewhere. On examination

she has peritoneal signs of the right

lower quadrant and a rectal temperature

of 38.38°C (101.8°F). At

exploration through incision of the

right lower quadrant, she is found

to have a small, contained perforation

of a cecal diverticulum. Which

of the following statements regarding

this situation is true?

a. Cecal diverticula are acquired disorders

b. Cecal diverticula are usually multiple

c. Cecal diverticula are mucosal herniations

through the muscularis

propria

d. Diverticulectomy, closure of the

cecal defect, and appendectomy

may be indicated

e. An ileocolectomy is indicated even

with well-localized inflammation

368. True statements regarding

cavernous hemangiomata of the

liver in adults include

a. The majority become symptomatic

b. They may undergo malignant

transformation

c. They enlarge under hormonal stimulation

d. They should be resected to avoid

spontaneous rupture and lifethreatening

hemorrhage

e. A liver/spleen radionucleotide scan

is the most sensitive and specific

way to make the diagnosis

369. Correct statements regarding

carcinoembryonic antigen (CEA)

and colorectal tumors include

which of the following?

a. Elevated CEA is indicative of a

tumor of gastrointestinal origin

b. A low CEA level after resection of a

colon tumor is a poor marker of

disease control

c. Ninety percent of colorectal tumors

produce CEA

d. There is a high likelihood of liver

involvement if the CEA level is high

(greater than 100 ng/mL)

e. CEA levels are unusually low in

cigarette smokers

Gastrointestinal Tract, Liver, and Pancreas 205

Items 370-373

Select the appropriate surgical

procedure for each patient.

a. Vagotomy and antrectomy

b. Antrectomy alone

c. Vagotomy and pyloroplasty

d. Vagotomy and gastrojejunostomy

e. Proximal gastric vagotomy

370. A 72-year-old patient with an

intractable type I ulcer along the

incisura with a significant amount

of scarring along the entire length

of the lesser curvature (SELECT 1

PROCEDURE)

371. A 46-year-old patient with

gastric outlet obstruction secondary

to ulcer disease and severe

inflammation around the pylorus

and first and second portions of the

duodenum (SELECT 1 PROCEDURE)

372. A 90-year-old patient with a

bleeding duodenal ulcer (SELECT

1 PROCEDURE)

373. A 36-year-old patient with a

type III (pyloric) ulcer that is

refractory to medical treatment

(SELECT 1 PROCEDURE)

Items 374-376

Match each description with

the correct abnormality.

a. Rupture of the diaphragm

b. Paraesophageal hiatal hernia

c. Sliding hiatal hernia

d. Foramen of Bochdalek hernia

e. Foramen of Morgagni hernia

374. The most common congenital

diaphragmatic hernia in infants

(SELECT 1 ABNORMALITY)

375. The hernia most likely to

cause acute respiratory distress in

infants (SELECT 1 ABNORMALITY)

376. A congenital hernia that is

most frequently discovered as an

incidental finding in adults (SELECT

1 ABNORMALITY)

206 Surgery

DIRECTIONS: Each group of questions below consists of lettered

options followed by numbered items. For each numbered item, select the

appropriate lettered option(s). Each lettered option may be used once,

more than once, or not at all. Choose exactly the number of options

indicated following each item.

Items 377-378

For each patient listed below,

select the likely diagnosis.

a. Spontaneous bacterial peritonitis

b. Perforated diverticulum

c. Perforated gastric ulcer

d. Ruptured spleen

e. Ruptured echinococcal liver cyst

f. Sigmoid volvulus

g. Cecal volvulus

h. Perforated transverse colon carcinoma

i. Strangulated hernia with necrotic

bowel

377. A 65-year-old previously

healthy man presents with severe

abdominal pain that came on suddenly.

He has abdominal tenderness

and guarding in all four

quadrants on physical examination.

A radiograph is obtained and

demonstrates a radiolucency under

the right hemidiaphragm.

(SELECT 4 DIAGNOSES)

378. An 82-year-old nursing

home patient presents to the emergency

room with vomiting, abdominal

pain, and distention. A

radiograph is obtained and demonstrates

a grossly dilated loop of

intestine overlying the sacrum in

the shape of an upside down U.

(SELECT 1 DIAGNOSIS)

Items 379-380

For each patient, select the

best course of action.

a. Administration of intravenous vasopressin

b. Administration of intraarterial vasopressin

c. Left thoracotomy, full-thickness

suture ligation, and drainage of the

pleural cavity

d. Balloon tamponade

e. Endoscopic control of bleeding

f. Gastrotomy and suture ligation

g. Insertion of a chest tube

h. Pulmonary arteriogram and streptokinase

infusion

i. Cardiac catheterization and intraarterial

infusion of tissue plasminogen

activator

379. A 72-year-old man with

severe coronary artery disease presents

with painless hematemesis

following a prolonged bout of vomiting.

Upper endoscopy reveals a

tear just below the gastroesophageal

junction, which is actively bleeding.

(SELECT 3 ACTIONS)

380. A 56-year-old man complains

of the onset of severe substernal

pain after a night of heavy

drinking followed by uncontrolled

retching. He states that there was a

small amount of blood in his vomit.

A chest x-ray shows a moderatesized

left pleural effusion. (SELECT

1 ACTION)

Gastrointestinal Tract, Liver, and Pancreas 207

208

GASTROINTESTINAL

TRACT, LIVER,AND

PANCREAS

Answers

291. The answer is d. (McQuaid, Surg Clin North Am 72:285-316, 1992.)

Omeprazole (Prilosec) irreversibly inhibits the hydrogen-potassium-ATPase

(proton pump) in the secretory canaliculus of the gastric parietal cell. This

blocks the last step in the acid-secretory process. Omeprazole's duration of

action exceeds 24 h and doses of 20-30 mg per day inhibit more than 90%

of 24-h acid secretion. Omeprazole provides excellent suppression of mealstimulated

and nocturnal acid secretion. It seems very safe for short-term

therapy. However, its safety for long-term use is uncertain since it produces

significant hypergastrinemia, hyperplasia of enterochromaffin-like cells, and

carcinoid tumors in laboratory animals with prolonged administration.

292. The answer is d. (Schwartz, 7/e, pp 1507-1508.) Patients with ITP

who have mild symptoms need no therapy, but they are usually advised to

avoid contact sports and elective surgery. When symptoms (e.g., easy

bruising, menorrhagia, bleeding gums) are troublesome, the bleeding time

will be prolonged, capillary fragility greatly increased, and clot retraction

poor. Corticosteroid therapy will increase the platelet count in over 75% of

cases and provides the best indication that splenectomy will be of lasting

benefit. The platelet count can be expected to rise shortly after splenectomy

and prolonged remissions are anticipated in 80% of cases. The size of

the spleen and the state of function in the bone marrow have no predictive

value in assessing the likelihood of response to splenectomy. In children,

complete spontaneous remissions are common (80% of cases) and surgical

intervention should be avoided.

293. The answer is c. (Schwartz, 7/e, pp 1249, 1387.) This is an inflamed

Meckel's diverticulum. This common lesion is often clinically indistinguishable

from acute appendicitis. It is the remnant of the vitelline duct.

Meckel's diverticula are usually located 50-75 cm proximal to the ileocecal

valve, are antimesenteric, and may contain either gastric and pancreatic or

pancreatic tissue. Hemorrhage or obstruction is a more common presentation

than inflammation. 99mTc pertechnetate has affinity for gastric mucosa

and a scan with this isotope can aid in the diagnosis of this anomaly as a

cause of lower gastrointestinal hemorrhage in a child. Angiography is more

useful when looking for arteriovenous malformations. Since complications

are relatively rare, most authors do not recommend removing asymptomatic

diverticula when they are incidentally discovered during abdominal

procedures. Those diverticula with a narrow neck, palpable heterotopic tissue,

or nodularity are prone to obstruction and should be excised. In addition,

patients explored for abdominal pain of unknown etiology should

also undergo diverticulectomy, as should those operated on for appendicitis

who are to be left with a scar of the right lower quadrant.

294. The answer is c. (Schwartz, 7/e, pp 1126-1137.) Patients with achalasia

typically present with distal esophageal obstruction, which leads to

regurgitation of saliva and undigested food. The characteristic appearance

of the esophagram is the tapered "bird's beak" deformity at the level of the

esophagogastric junction. Chest pain may be seen in the early stages of the

disease. Manometry yields high resting pressures of the lower esophageal

sphincter, which fails to relax or only partially relaxes. The absence of peristaltic

deglutitory contractions in the body of the esophagus is also noted

during manometry. Although both surgical intervention and forceful dilation

have been used to treat this disease, surgery results in improvement in

over 90% of patients, compared with only 70% of patients treated by forceful

dilation. Surgical treatment is an esophagomyotomy. Patients with achalasia

have seven times the risk of developing squamous cell carcinoma as

compared with the general population. This dreaded complication can

occur even after successful treatment for the disease.

295. The answer is d. (Schwartz, 7/e, pp 1736-1737, 1768.) Imperforate

anus affects males and females with equal frequency, occurring in 1 of each

20,000 live births. It is due to failure of descent of the urorectal septum.

Imperforate anus may be broadly classified into "high" or "low," depending

on whether the rectum ends above or below the level of the levator ani

complex. In 90% of females, but only 50% of males, the lesion is of the low

variety. The rectal fistula may end in the prostatic urethra or vagina in the

Gastrointestinal Tract, Liver, and Pancreas Answers 209

high cases, while the low cases terminate in a perineal fistula. For the low

cases, only a perineal operation may be required and these children will be

expected to be continent. A pull-through procedure will be required for the

high imperforate anus and the likelihood of continence is smaller. If there

is doubt about the level or location of the termination of the rectum, it is

better to perform a temporary colostomy than to compromise the ultimate

chances of continence by an injudicious perineal approach.

296-297. The answers are 296-c, 297-e. (Greenfield, 2/e, p 916.) This

woman has a cystadenocarcinoma arising from the pancreatic body and tail,

which was successfully resected. About 90% of primary malignant neoplasms

of the exocrine pancreas are adenocarcinomas of duct cell origin.

The remaining neoplasms include adenosquamous carcinoma, mucinous

carcinomas, microadenocarcinoma, giant cell carcinoma, and cystadenocarcinoma

of uncertain histogenesis. The clinical presentation is usually quite

subtle, with symptoms related primarily to the enlarging mass. There are no

diagnostic laboratory findings and definitive preoperative diagnosis is rare.

An elderly patient with no history of pancreatitis is unlikely to have a

pseudocyst and a benign neoplasm is also less likely in this age group. These

less common carcinomas are often several times the size of typical ductal

cancers and often arise in the body or tail of the pancreas. They may become

very large without invading adjacent viscera and do not generally cause significant

pain or weight loss. Therefore, even large tumors may be cured by

resection, and aggressive surgical management is indicated.

298. The answer is c. (Greenfield, 2/e, pp 1109-1127.) Varying types of

colonic polyps can be distinguished on pathologic examination. Adenomatous

polyps are distributed throughout the entire large bowel, more commonly

in the right and left colon than the rectum. They are often

pedunculated and show an increased number of glands compared with

normal mucosa. Although polyps that appear in familial polyposis are

indistinguishable from single adenomatous polyps, they are manifested

much earlier in life. Carcinomatous changes in patients who have familial

polyposis occur approximately 20 years before carcinomatous changes of

the bowel occur among patients in the general population.

299. The answer is c. (Schwartz, 7/e, pp 472-473.) According to the

United Ostomy Association Data Registry, the most frequent serious com-

210 Surgery

plication of end colostomies is parastomal herniation, which commonly

occurs when the stoma is placed lateral to, rather than through, the rectus

muscle. Symptomatic herniation requires operative relocation of the stoma

or mesh herniorrhaphy. Minor problems are frequently encountered with

colostomies. They include irregularity of function, irritation of the skin due

to leakage of enteric contents, or bleeding from the exposed mucosa following

trauma. Prolapse occurs most frequently with transverse loop

colostomies and is likely due to the use of the transverse loop to decompress

distal colon obstructions. As the intestine decompresses, it retracts

from the edge of the surrounding fascia, which allows prolapse or herniation

of the mobile transverse colon. Optimal treatment of stomal prolapse

is restoration of intestinal continuity or conversion to an end colostomy.

Perforation of a stoma is usually due to careless instrumentation with an

irrigation catheter. Perforations that cause minimal peritoneal contamination

may be treated with observation and antibiotics, while more extensive

leaks require operative closure.

300. The answer is d. (Greenfield, 2/e, pp 901-915.) The prognosis for a

patient with carcinoma of the pancreas is dismal. The plurality of cases

(46%) present as pain without jaundice; 34% present as pain with jaundice;

and only 13% present with jaundice alone. Tumors over 1-2 cm may

be seen by ultrasonography, computed tomography, or magnetic resonance

imaging, but none of these methods can visualize smaller tumors. Endoscopic

retrograde pancreaticocholangiography is helpful in distinguishing

the more favorable tumors of the duodenum, ampulla, and common bile

duct and lymphomas from cancer of the head of the pancreas. A combination

of techniques including CT, angiography, and laparoscopy will accurately

determine resectability in 97% of cases. Overall, the rate of

resectability for possible cure is dismal: 5-10% of all patients and 10-25%

of patients who present with jaundice alone, the latter due to earlier diagnosis

of small tumors obstructing the common bile duct in the head of the

pancreas. Ninety-nine percent of patients have metastatic disease at the

time of diagnosis, and only 5-20% will be alive at 5 years following a pancreaticoduodenectomy.

301. The answer is c. (Greenfield, 2/e, pp 759-773.) Perforation of a duodenal

ulcer is an indication for emergency celiotomy and closure of the perforation.

In patients with no prior history of peptic ulcer disease, simple

Gastrointestinal Tract, Liver, and Pancreas Answers 211

closure with an omental patch is recommended. Seventy-two percent of

patients who are asymptomatic preoperatively will remain so postoperatively.

Patients with long-standing ulcer disease require a definitive acidreducing

procedure, except in high-risk situations. The choice of

procedure is made by weighing the risk of recurrence against the incidence

of undesirable side effects of the procedure, and considerable controversy

persists about this issue. Antrectomy and truncal vagotomy offers a recurrence

rate of 1%, but carries a 15-25% incidence of sequelae such as diarrhea,

dumping syndrome, bloating, and gastric stasis. Highly selective

vagotomy, if technically feasible, offers a 1-5% incidence of side effects but

carries a recurrence rate of 10-13% in some series, although results are better

when gastric and prepyloric ulcers are excluded. In general, definitive

acid-reducing procedures should be postponed if the perforation is more

than 12 h old or if there is extensive peritoneal soilage. Pyloroplasty and

truncal vagotomy carries intermediate rates of recurrence and side effects,

but has the advantage of speed in the setting of very ill patients with acute

perforation.

302. The answer is b. (Sawyers, Am J Surg 159:8-14, 1990.) Though

reminiscent of the carcinoid syndrome, this patient's complaints in the context

of recent gastric surgery are highly suggestive of the "dumping syndrome,"

seen after gastroenteric bypass such as antrectomy and

gastrojejunostomy. Dumping syndrome presents as vasomotor symptoms

(weakness, sweating, syncope) and intestinal symptoms (bloating, cramping,

diarrhea). The etiology of dumping has best been attributed to the

rapid influx of fluid with a high osmotic gradient into the small intestine

from the gastric remnant. Medical management consists of reassurance and

frequent small meals that are low in carbohydrates (to limit the osmotic

load). Antispasmodic medications are sometimes used if dietary adjustments

are unsuccessful. The majority of cases will resolve within 3 mo of

operation on this regimen. Surgery for intractable dumping consists of creation

of an antiperistaltic limb of jejunum distal to the gastrojejunostomy.

303. The answer is c. (Greenfield, 2/e, pp 779-787.) Benign gastric ulcers

have a peak incidence in the fifth decade, with male predominance. About

95% of gastric ulcers are located near the lesser curvature. It should be recognized

that up to 16% of patients with gastric carcinoma pass a 12-wk

healing trial and that benign ulcers may enlarge during medical therapy.

212 Surgery

Therefore, the possibility of malignancy must be assessed by biopsy despite

a 5-10% false negative rate. Six weeks of medical therapy will heal many

gastric ulcers, but a recurrence rate as high as 63% and the serious consequence

of complications in this older group of patients warrant surgery for

recurrent or nonhealing ulcers. A distal gastrectomy with gastroduodenostomy

is usually feasible in the absence of duodenal disease. Vagotomy,

while advocated by some, is generally not included. Local excision with

definitive distal resection or vagotomy and pyloroplasty is appropriate for

a proximal ulcer that would otherwise require a subtotal gastrectomy.

304-305. The answers are 304-d, 305-b. (Greenfield, 2/e, pp

986-1005.) The diagnosis of bleeding esophageal varices is aided in the

adult by stigmata of portal hypertension. Upper gastrointestinal hemorrhage

in cirrhotics is due to esophageal varices in less than half of patients.

Gastritis and peptic ulcer disease account for the majority of cases.

Esophagoscopy is the single most reliable means of establishing the source

of bleeding, though variations in transvariceal blood flow may result in

nonvisualization of the varices. In addition, endoscopic sclerotherapy is

reported to control acute variceal hemorrhage in 80-90% of cases and carries

an acute mortality lower than that of other procedures. Barium swallow

has a high false negative rate and offers no therapeutic advantage.

Celiac angiography will rule out arterial hemorrhage and will demonstrate

venous collateral circulation, but will not demonstrate variceal bleeding.

Parenteral vasopressin controls variceal hemorrhage by constriction of the

splanchnic arteriolar bed and a resultant drop in portal pressure. Intraarterial

vasopressin offers no advantage over intravenous administration and

requires a mesenteric catheter. The reported control rate is 50-70%.

Esophageal balloon tamponade controls variceal hemorrhage in two-thirds

of patients, but may also control bleeding ulcers and thereby obscure the

diagnosis. Although balloon tamponade has reduced the mortality and

morbidity from variceal hemorrhage in good-risk patients, an increased

awareness of associated complications (aspiration, asphyxiation, and ulceration

at the tamponade site), as well as a rebleeding rate of 40%, has

reduced its use. It is indicated as a temporary measure when vasopressin

and sclerotherapy fail. Emergency portacaval shunt is advised in good-risk

cirrhotic patients whose bleeding is not controlled with vasopressin or sclerosis.

The mortality for patients with bleeding varices not subjected to

shunting is between 66 and 73%, whereas operative mortality of emer-

Gastrointestinal Tract, Liver, and Pancreas Answers 213

gency shunts ranges from 20 to 50%. Esophageal transection with the

autostapler carries the same mortality as shunt procedures and the rebleeding

rate is estimated to be 50% at 1 year.

306. The answer is c. (Greenfield, 2/e, pp 1019-1021.) Because approximately

5% of colorectal cancers are associated with resectable hepatic

metastases, appropriate preoperative discussion should include obtaining

permission for removal of synchronous peripheral hepatic lesions if they

are found. If gross tumor is removed, a 25% "cure" rate can be anticipated.

Adequate local resection, either by wedge or by limited partial hepatectomy,

may be carried out whenever no extrahepatic disease is found and

the hepatic lesion is technically removable. Any option that leaves the

potentially obstructing primary cancer unremoved would be unacceptable.

Radiation therapy has little to offer in colon cancer or its hepatic metastases.

Local infusion of floxuridine (FUDR) via an implantable Infusaid

pump for 14 days at 0.3 mg/kg/day has been reported to provide some

acceptable palliation in selected patients with unresectable hepatic lesions.

307. The answer is b. (Schwesinger, Am J Surg 172:411-417, 1996.) Helicobacter

pylori infections have become extremely common. Nearly a third

of all American adults are now infected. Morphologically, the organism is a

gram-negative, corkscrew-shaped, motile bacillus with three-seven flagella.

Noninvasive approaches with simple, relatively inexpensive serologic

and urea breath tests can establish the diagnosis of H. pylori infection. Culturing

endoscopic scrapings or biopsy specimens has proved to be impractical

because of the need for special media and elaborate growth

conditions. A rapid urease test is used when endoscopy provides a specimen

for analysis. Therapy is problematic because the organism is not easily

eradicated. Monotherapy is largely ineffective. However, dual- and

triple-drug therapy can achieve eradication in 80-90% of patients. Unfortunately,

compliance rates with multidrug therapy are low.

308. The answer is c. (Zinner, 10/e, pp 479-572.) An indirect inguinal

hernia leaves the abdominal cavity by entering the dilated internal inguinal

ring and passing along the anteromedial aspect of the spermatic cord. The

internal inguinal ring is an opening in the transversalis fascia for the passage

of the spermatic cord; an indirect inguinal hernia, therefore, lies

within the fibers of the cremaster muscle. Repair consists of removing the

214 Surgery

hernia sac and tightening the internal inguinal ring. A femoral hernia

passes directly beneath the inguinal ligament at a point medial to the

femoral vessels, and a direct inguinal hernia passes through a weakness in

the floor of the inguinal canal medial to the inferior epigastric artery. Each

is dependent on defects in Hesselbach's triangle of transversalis fascia and

neither lies within the cremaster muscle fibers. Repair consists of reconstructing

the floor of the inguinal canal. Spigelian hernias, which are rare,

protrude through an anatomic defect that can occur along the lateral border

of the rectus muscle at its junction with the linea semilunaris. An interparietal

hernia is one in which the hernia sac, instead of protruding in the

usual fashion, makes its way between the fascial layers of the abdominal

wall. These unusual hernias may be preperitoneal (between the peritoneum

and transversalis fascia), interstitial (between muscle layers), or

superficial (between the external oblique aponeurosis and the skin).

309. The answer is b. (Zinner, 10/e, pp 581-591.) The finding of air in

the biliary tract of a nonseptic patient is diagnostic of a biliary enteric fistula.

When the clinical findings also include small bowel obstruction in an

elderly patient without a history of prior abdominal surgery (a "virgin"

abdomen), the diagnosis of gallstone ileus can be made with a high degree

of certainty. In this condition, a large chronic gallstone mechanically erodes

through the wall of the gallbladder into adjacent stomach or duodenum. As

the stone moves down the small intestine, mild cramping symptoms are

common. When the gallstone arrives in the distal ileum, the caliber of the

bowel no longer allows passage and obstruction develops. Surgical removal

of the gallstone is necessary. The diseases suggested by each of the other

response items (bleeding ulcer, peritoneal infection, pyloric outlet obstruction,

pelvic neoplasm) are common in elderly patients, but each of them

would probably present with symptoms other than those of small bowel

obstruction.

310. The answer is d. (Zinner, 10/e, pp 1286-1300.) Cancer of the colon

in patients with chronic ulcerative colitis is 10 times more frequent than in

the general population. Duration of disease is very important; the risk of

developing cancer is low in the first 10 years but thereafter rises about 4%

per year. The average age of cancer development in patients with chronic

ulcerative colitis is 37 years; idiopathic carcinoma of the colon, however,

develops at an average age of 65 years. Crohn's colitis is currently felt to be

Gastrointestinal Tract, Liver, and Pancreas Answers 215

a precancerous condition as well. The chance of development of carcinoma

of the colon in patients with familial polyposis is essentially 100%. Treatment

of the patient with familial polyposis generally consists of subtotal

colectomy with ileoproctostomy and regular proctoscopic examination of

the rectal stump. Villous adenomas have been demonstrated to contain

malignant portions in about one-third of affected persons and invasive

malignancy in another one-third of removed specimens. Anterior resection

is performed for large lesions or those containing invasive carcinomas

when the lesion is above the peritoneal reflection. Abdominoperineal

resection is indicated for low-lying rectal villous adenomas when they have

demonstrated invasive carcinomas. Transrectal excision with regular

follow-up examinations is sufficient for lesions without invasive carcinomas.

Peutz-Jeghers syndrome is characterized by intestinal polyposis and

melanin spots of the oral mucosa. Unlike the adenomatous polyps seen in

familial polyposis, the lesions in this condition are hamartomas, which

have no malignant potential.

311. The answer is c. (Greenfield, 2/e, pp 825-826.) Gallstone ileus is due

to erosion of a stone from the gallbladder into the gastrointestinal tract

(most commonly into the duodenum). The stone becomes lodged in the

small bowel (usually in the terminal ileum) and causes small-bowel

obstruction. Plain films of the abdomen that demonstrate small-bowel

obstruction and air in the biliary tract are diagnostic of the condition.

Treatment consists of ileotomy, removal of the stone, and cholecystectomy

if it is technically safe. If there is significant inflammation of the right upper

quadrant, ileotomy for stone extraction followed by an interval cholecystectomy

is often a safer alternative. Operating on the biliary fistula doubles

the mortality rate compared with simple removal of the gallstone from the

intestine.

312. The answer is a. (Greenfield, 2/e, pp 2057-2066.) Hirschsprung's

disease, which is the congenital absence of ganglion cells in the rectum or

rectosigmoid colon, is definitively diagnosed by rectal biopsy. The typical

findings on barium enema, a distal narrow segment of bowel with

markedly distended colon proximally, may not be seen early in life. Symptoms

may go unrecognized in the newborn period with consequent development

of malnutrition or enterocolitis. Initial treatment is colostomy

decompression. Definitive repair is best delayed until nutritional status is

216 Surgery

adequate and the chronically distended bowel has returned to normal size.

Unlike the situation with imperforate anus, which is associated with a high

incidence of genitourinary tract anomalies and a 50% incidence of longterm

fecal incontinence, in Hirschsprung's disease repair leads to satisfactory

bowel function in most affected patients.

313. The answer is a. (Zinner, 10/e, pp 529-531.) Omphalocele and gastroschisis

result in evisceration of bowel and require emergency surgical

treatment to effect immediate or staged reduction and abdominal wall closure.

Patent urachal or omphalomesenteric ducts result from incomplete

closure of embryonic connections from the bladder and ileum, respectively,

to the abdominal wall. They are appropriately treated by excision of the

tracts and closure of the bladder or ileum. In most children, umbilical hernias

close spontaneously by the age of 4 and need not be repaired unless

incarceration or marbled enlargement and distortion of the umbilicus occur.

314. The answer is d. (Zinner, 10/e, pp 1855-1865.) Laparoscopic cholecystectomy

is now viewed as the treatment of choice for most patients with

symptomatic gallstones. This procedure has frequently been performed in

obese patients with the same efficiency, morbidity and mortality rates, and

length of hospitalization as in the average-weight population. The other

conditions listed represent currently accepted relative contraindications,

but as experience increases and techniques improve, the safe indications

for laparoscopic cholecystectomy are likely to expand.

315. The answer is d. (Zinner, 10/e, pp 2097-2115.) Congenital anorectal

anomalies are frequently associated with other congenital anomalies

including heart disease, esophageal atresia, abnormalities of the lumbosacral

spine, double urinary collecting systems, hydronephrosis, and

communication between the rectum and the urinary tract, vagina, or perineum.

They occur in approximately 1 in 2000 live births. Depending on

the type of anomaly (whether the rectum ends above or below the level of

the levator ani complex), a variety of surgical procedures has been devised

to treat the problem. However, even when anatomic integrity is established,

the prognosis for effective toilet training is poor. In 50% of cases

continence is never achieved. Cervical spine abnormalities, hydrocephalus,

duodenal atresia, and corneal opacities have no significant association with

congenital anorectal anomalies.

Gastrointestinal Tract, Liver, and Pancreas Answers 217

316. The answer is c. (Greenfield, 2/e, p 1236.) Hematomas of the rectus

sheath are more common in women and present most often in the fifth

decade. A history of trauma, sudden muscular exertion, or anticoagulation

can usually be elicited. The pain is of sudden onset and is sharp in nature.

The hematoma is most common in the right lower quadrant and irritation

of the peritoneum leads to fever, leukocytosis, anorexia, and nausea. Preoperatively

the diagnosis can be established with an ultrasound or CT scan

showing a mass within the rectus sheath. Management is conservative

unless symptoms are severe and bleeding persists, in which case surgical

evacuation of the hematoma and ligation of bleeding vessels is required.

317. The answer is d. (Zinner, 10/e, pp 2083-2087.) This is an example

of an ileal atresia. Whether the atresia is jejunal or ileal does not affect treatment

and there is no predilection for one site over the other. Resection and

primary anastomosis should be performed if possible, but the bowel

should be exteriorized if there is a question of viability or there is a large

size discrepancy between two segments. Plain films will reveal a small

bowel obstruction with no gas beyond the lesion. A carefully administered

meglumine diatrizoate (Gastrografin) enema can help in the differential

diagnosis. Midgut volvulus and meconium ileus can be apparent on an

enema, which is important as meconium ileus should be managed nonoperatively.

The basis of jejunoileal atresia is probably a mesenteric vascular

accident during intrauterine growth.

318. The answer is b. (Greenfield, 2/e, pp 680-694.) Surgical treatment

for sliding esophageal hernias should only be considered in symptomatic

patients with objectively documented esophagitis or stenosis. The overwhelming

majority of sliding hiatal hernias are totally asymptomatic, even

many of those with demonstrable reflux. Even in the presence of reflux,

esophageal inflammation rarely develops because the esophagus is so efficient

at clearing the refluxed acid. Symptomatic hernias should be treated

vigorously by the variety of medical measures that have been found helpful.

Patients who do have symptoms of episodic reflux and who remain

untreated can expect their disease to progress to intolerable esophagitis or

fibrosis and stenosis. Neither the presence of the hernia nor its size is

important in deciding on surgical therapy. Once esophagitis has been documented

to persist under adequate medical therapy, manometric or pH

studies may help determine the optimum surgical treatment.

218 Surgery

319. The answer is b. (Zinner, 10/e, pp 1739-1751.) While elevation of

SGOT and SGPT are indicative of hepatocellular disease, elevated alkaline

phosphatase is indicative of biliary obstruction. Based on safety and cost,

ultrasonography is the initial diagnostic procedure. Once ductal dilation is

identified, a percutaneous transhepatic cholangiogram or ERCP may be

performed to localize and characterize the obstruction. If a distal common

bile duct obstruction is noted, a CT scan is recommended to image the

head of the pancreas. In most instances, a liver-spleen scan adds little to the

diagnostic workup. This also applies to the upper gastrointestinal series.

Cancer of the head of the pancreas is associated with painless jaundice.

320. The answer is a. (Schwartz, 7/e, pp 1417-1420.) Massive hematemesis

in children is almost always due to variceal bleeding. The varices

usually result from extrahepatic portal vein obstruction consequent to bacterial

infection transmitted via a patent umbilical vein during infancy. In

spite of this common cause, a history of neonatal omphalitis is infrequently

obtainable. Bleeding can be massive but is usually self-limited, and

esophageal tamponade or vasopressin is usually not necessary. Elective

portal-systemic decompression is recommended for recurrent bleeding

episodes.

321. The answer is c. (Schwartz, 7/e, pp 1181, 1187-1188.) The proximal

stomach can distend or accommodate a large volume without any increase

in intragastric pressure. This phenomenon permits solid food to settle

along the greater curvature while liquids are propelled along the lesser curvature

by slow tonic contractions of the upper stomach. In the normal

state, once a volume of 1000-1200 mL is reached, intragastric pressure

rises to high levels. While the stomach's ability to accommodate large volumes

is necessary for normal gastric motor activity, a potentially deleterious

effect is seen in patients with gastric atony. These patients may

accumulate several liters of gastric juice in the stomach without sensing

fullness, and this often leads to massive emesis and aspiration.

322. The answer is b. (Schwartz, 7/e, pp 1265-1274.) Because the reserve

capacity of the colon for water absorption greatly exceeds the normal

requirements for maintaining stable bowel function, patients may undergo

resection of a large fraction of the colon and suffer little change in bowel

habits. Neither the right nor the left colon appears to be a site of preferential

Gastrointestinal Tract, Liver, and Pancreas Answers 219

water and electrolyte absorption, nor does the ileocecal valve play a noticeable

role in fluid homeostasis. However, in diseases characterized by increased

fluid secretion of the small bowel, the colon is more likely to be

overwhelmed by the absorptive demand following partial colectomy than in

the intact state. The rectum does not appear to play a role in fluid absorption.

323. The answer is e. (Zinner, 10/e, pp 1455-1500.) Workup of a patient

with a diagnosed rectal cancer should include CT scan of the upper

abdomen in search of liver metastases and assessment of the depth of local

invasion by transanal ultrasound. Sonographic staging of the rectal wall

and pararectal lymph nodes has become crucial in planning the magnitude

of the resection and choice of preoperative treatment. The survival advantages

of neoadjuvant radiation therapy now seem clear. Administering radiation

preoperatively to large or deeply invasive tumors often reduces the

tumor mass and permits clean resection of previously bulky disease. In

addition, the cytoreductive effect of preoperative radiation therapy now

allows many patients to undergo sphincter-saving procedures and avoid

the morbidity of proctectomy and colostomy.

324. The answer is a. (Greenfield, 2/e, pp 812-816.) Digestion and

absorption of dietary carbohydrate by the duodenum and small intestine

are so avid that complete absorption has already occurred by the time

ingested food has traversed 200 cm of jejunum. Simple fluids that

require minimal digestion, such as milk, are entirely absorbed, save for

their fat content, within the duodenum. Even in the short gut syndrome,

virtually all dietary carbohydrate is absorbed within the residual

jejunum. While pancreatic peptidases are important to protein digestion,

redundant digestive enzymes are so widely distributed within the duodenal

and jejunal brush border that 95% of a protein meal can be absorbed

in the absence of the pancreas. Salt and water flux in the small intestine

is influenced by a variety of hormones; aldosterone markedly increases

sodium uptake, while prostaglandins stimulate fluid and electrolyte

secretion.

325. The answer is d. (Greenfield, 2/e, pp 749-751.) Gastrin, an aqueous

extract of the antral G cell, stimulates acid and pepsin secretion. A variety

of local stimuli cause the release of gastrin. The most potent of these are

small proteins, 20-proof alcohol, and caffeine. Acidic antral contents

220 Surgery

inhibit gastrin secretion; alkalinization of the antrum is stimulatory.

Mechanical distention of the antrum also stimulates gastrin secretion.

326. The answer is a. (Greenfield, 2/e, p 816.) As it does with carbohydrate

digestion, the gastrointestinal tract exhibits remarkable redundancy

and alternative pathways to facilitate fat uptake. In the normal state,

water-insoluble dietary lipid is rendered into soluble micelles through

mixing with pancreatic and intestinal lipase and with bile. However,

lipases of the stomach and small intestine permit absorption of approximately

half of neutral dietary fat in the absence of bile and pancreatic

secretion. Small breakdown products of complex fats-such as glycerol,

short-chain fatty acids, and medium-chain triglycerides-can be transported

directly from the jejunal mucosal cell into the portal venous system,

whereas larger triglycerides, resynthesized by the mucosal cells from

fatty acids, are deposited in chylomicrons and released into the lymphatic

system. Enterohepatic recirculation of bile with active resorption in the

ileum and secretion into the portal venous system yields an effective bile

salt pool 6-8 times its actual volume. Normal daily losses of bile into the

stool represent 10-15% of the total bile salt pool; these losses can usually

be replaced by new synthesis in the liver. However, bile salt-wasting

states, such as inflammatory bowel disease or ileal resection, may exceed

the liver's capacity to maintain an adequate volume of bile.

327. The answer is e. (Schwartz, 7/e, pp 1728-1729.) A bypass procedure

is the operation of choice for obstruction secondary to an annular pancreas.

A Whipple procedure is too radical a therapy for this benign disease, and a

partial resection of the annular pancreas often is complicated by fistula.

Duodenojejunostomy is much more physiologic than gastrojejunostomy

and does not require a vagotomy to prevent marginal ulceration; it is therefore

the procedure of choice.

328. The answer is b. (Schwartz, 7/e, pp 1231-1232, 1387.) Patients

with regional enteritis usually have a chronic and slowly progressive

course with intermittent symptom-free periods. The usual symptoms are

anorexia, abdominal pain, diarrhea, fever, and weight loss. There are

extraintestinal syndromes that may be seen, such as ankylosing spondylitis;

polyarthritis; erythema nodosum; pyoderma gangrenosum; gallstones;

hepatic fatty infiltration; and fibrosis of the biliary tract, pancreas, and

Gastrointestinal Tract, Liver, and Pancreas Answers 221

retroperitoneum. However, in about 10% of patients, especially those who

are young, the onset of the disease is abrupt and may be mistaken for

acute appendicitis. Appendectomy is indicated in such patients as long as

the cecum at the base of the appendix is not involved; otherwise the risk

of fecal fistula must be considered. Interestingly, about 90% of patients

who present with the acute appendicitis-like form of regional enteritis will

not progress to development of the full-blown chronic disease. Thus,

resection or bypass of the involved areas is not indicated at this time.

329. The answer is c. (Moosa, Arch Surg 125:1028-1031, 1990. Schwartz,

7/e, p 1446.) The scenario in the question is a typical course of a patient

with iatrogenic injury of the common bile duct. These injuries commonly

occur in the proximal portion of the extrahepatic biliary system. The transhepatic

cholangiogram documents a biliary stricture, which in this clinical

setting is best dealt with surgically. Choledochoduodenostomy generally

cannot be performed because of the proximal location of the stricture. The

best results are achieved with end-to-side choledochojejunostomy (Rouxen-

Y) performed over a stent. Percutaneous transhepatic dilation has been

attempted in select cases, but follow-up is too short to make an adequate

assessment of this technique. Primary repair of the common bile duct may

result in recurrent stricture.

330. The answer is e. (Schwartz, 7/e, pp 1341-1346.) Many authorities

now recommend abandonment of the phrase carcinoma in situ because it

gives a misleading impression to the patient and family regarding the true

implications of severe dysplasia. Almost all agree that no further treatment

is indicated when a polyp has been adequately removed and such changes

are found. Only when malignant cells penetrate the muscularis mucosae is

there any potential for metastases, and only when that depth of penetration

is seen should the term carcinoma be used. Even then resection is probably

not indicated if the gross and microscopic margins are clear, the tumor is

well differentiated, and the stalk is not invaded.

331. The answer is b. (Schwartz, 7/e, pp 1253-1255.) The effects of radiation

on the intestine depend on a variety of factors, which include the age

of the patient, temperature, degree of oxygenation, and metabolic activity.

Acute intestinal radiation injury is manifested in the bowel by the cessation

of viable cell production and is seen clinically as diarrhea or gastrointesti-

222 Surgery

nal bleeding. Progressive vasculitis and fibrosis are seen in the latter stages

of radiation injury and may result in malabsorption, ulceration, fistulization,

or perforation. Intussusception is generally not associated with radiation

injury.

332. The answer is d. (Greenfield, 2/e, pp 757-758, 807-808.) Drugs,

hormones, or emotional states (e.g., fear) that stimulate or simulate sympathetic

activity inhibit intestinal motility. Those factors that arouse

parasympathetic activity (acetylcholine) stimulate motility. Gastrin has specific

delaying effects on gastric emptying. Secretin and cholecystokinin are

potent regulators of intestinal and digestive activities but probably have no

effect on motility per se.

333. The answer is d. (Greenfield, 2/e, pp 698-712.) Carcinoma of the

esophagus occurs primarily in the sixth and seventh decades of life in a

male:female ratio of 3:1. Although the cause is unknown, alcohol, tobacco,

and dietary factors have been implicated as causative agents. A high incidence

is reported in patients with corrosive esophagitis. The malignant

tumors arising in the esophagus are usually squamous cell carcinomas except

those involving the esophagogastric junction, which are usually adenocarcinomas.

Even though squamous cell carcinomas are weakly radiosensitive,

surgical extirpation affords reasonable, if short-term, palliation. Some

authorities recommend radiotherapy for palliation alone or in combination

with surgery to treat this lesion. Adenocarcinomas are not particularly

radiosensitive and surgical treatment is generally employed. Following

resection for esophageal carcinoma among the highly select group of

patients whose tumors are still resectable when the diagnosis is made, survival

is only about 14% at 5 years. The overall 5-year survival is under 5%.

334-335. The answers are 334-b, 335-b. (Greenfield, 2/e, pp 919-925.)

Total gastrectomy was formerly the procedure of choice for patients with

Zollinger-Ellison syndrome (ZES). However, with the knowledge that most

patients will die of metastatic disease and that the symptoms can often be

controlled with H2 receptor antagonists, the role for surgery has changed. Initial

surgical exploration is aimed at curative resection of the tumor. Unfortunately

metastatic disease is often present or will develop at a later date despite

tumor resection. Therefore, highly selective vagotomy is also added to the

procedure to reduce the required dose of H2 receptor antagonists.

Gastrointestinal Tract, Liver, and Pancreas Answers 223

The second patient has a gastrin level suggestive of but not diagnostic

of ZES. A secretin stimulation test will cause a significant rise in serum gastrin

levels in patients with ZES.

336. The answer is a. (Greenfield, 2/e, pp 1243-1244.) Idiopathic

retroperitoneal fibrosis is a nonsuppurative inflammatory process of the

retroperitoneum that causes problems by extrinsic compression of

retroperitoneal structures. The ureters, aorta, and inferior vena cava are

most at risk; however, the aorta is quite resistant to compression and the

inferior vena cava has multiple collaterals, so that ureteral obstruction is

the most common presentation of this disease process. The common bile

duct and duodenum may be compressed and obstructed, but this occurs

much less frequently. Treatment of ureteral obstruction includes conservative

therapy with steroids. Surgical intervention is often required and

ureterolysis with intraperitoneal transplantation is the current procedure of

choice. Biopsies must also be taken to exclude a malignant process as the

cause of the fibrosis.

337. The answer is b. (Greenfield, 2/e, pp 919-923.) Tumors arising from

the pancreatic β cells give rise to hyperinsulinism. Seventy-five percent of

these tumors are benign adenomas and in 15% of affected patients the adenomas

are multiple. Symptoms relate to a rapidly falling blood glucose

level and are due to epinephrine release triggered by hypoglycemia (sweating,

weakness, tachycardia). Cerebral symptoms of headache, confusion,

visual disturbances, convulsions, and coma are due to glucose deprivation

of the brain. Whipple's triad summarizes the clinical findings in patients

with insulinomas: (1) attacks precipitated by fasting or exertion; (2) fasting

blood glucose concentrations below 50 mg/dL; (3) symptoms relieved by

oral or intravenous glucose administration. These tumors are treated surgically

and simple excision of an adenoma is curative in the majority of cases.

338. The answer is e. (Greenfield, 2/e, pp 1148-1150.) Epidermoid cancers

of the anal canal metastasize to inguinal nodes as well as to the perirectal and

mesenteric nodes. The results of local radical surgery have been disappointing.

Combined external radiation (dose range 3500-5000 cG) with synchronous

chemotherapy (fluorouracil and mitomycin) is now recommended as

the means for controlling the disease. Radical surgical approaches are now

generally reserved for treatment failures and recurrences.

224 Surgery

339. The answer is c. (Greenfield, 2/e, pp 1137-1141.) A markedly distended

colon could have many causes in this 80-year-old man. The contrast

study, however, reveals a classic "apple core" lesion in the distal colon,

which is diagnostic of colon cancer. No further diagnostic studies are

appropriate prior to relief of this large bowel obstruction. After medical

preparation (e.g., hydration, normalization of electrolytes), this patient

should undergo prompt surgical management of his mechanical obstruction;

conservative management by resection and proximal colostomy

would generally be preferred in this elderly patient with an obstructed,

unprepared bowel.

340. The answer is e. (Greenfield, 2/e, pp 831-843.) Surgical treatment of

Crohn's disease is aimed at correcting complications that are causing symptoms.

Intestinal obstruction is usually partial and secondary to a fixed stricture

that is not responsive to anti-inflammatory agents. When the

obstruction causes symptoms that compromise nutritional status, surgery

is warranted. Fistula formation in itself is not an indication for surgery. Fistulas

between the intestine and the bladder and the intestine and the

vagina, however, generally cause significant symptoms and warrant surgical

intervention, while an ileum-ascending colon fistula is very common

yet rarely symptomatic. Perforation of bowel into the free abdominal cavity

is obviously a surgical emergency.

341. The answer is e. (Greenfield, 2/e, pp 827, 1092-1093.) The film

shows a markedly distended colon. The differential diagnosis includes

tumor, foreign body, and colitis, but far more likely is either cecal or sigmoid

volvulus. Sigmoid volvulus may be ruled out quickly by proctosigmoidoscopy,

which is preferable to barium enema, since sigmoid volvulus

may be treated successfully by rectal tube decompression via the sigmoidoscope.

If sigmoidoscopy is negative, the working diagnosis, based on

this classic film, must be cecal volvulus; barium enema would clinch the

diagnosis, but the colon might rupture in the intervening 1-2 h. Emergency

celiotomy should be done.

342. The answer is c. (Graham, Gastroenterology 105:279-282, 1993.)

Helicobacter pylori is a spiral-shaped, gram-negative bacterium that is found

in the viscous gastric mucus layer and has an affinity for epithelial cells. It

was originally classified as a form of Campylobacter, but its genomic and

Gastrointestinal Tract, Liver, and Pancreas Answers 225

phenotypic characteristics were subsequently found to be unique and it

was given a new genus name. Urease and other peptides released by H.

pylori may be toxic and cause direct gastroduodenal injury. Evidence is

strong that H. pylori plays a role in the etiology of ulcer disease. There is an

almost 100% association between gastric H. pylori infection and duodenal

ulcer disease, and about 70% of patients with gastric ulcers are also

infected with H. pylori. Furthermore, colonization with H. pylori increases

the risk of developing a duodenal ulcer by up to 20-fold. Eradication of H.

pylori from the stomach markedly decreases the rate of ulcer recurrence.

This generally requires "triple therapy" with colloidal bismuth (Pepto-

Bismol), an antibiotic (amoxicillin or ampicillin), and a nitroimidazole

such as metronidazole. Recent studies have also demonstrated a possible

association between H. pylori infection and gastric carcinoma.

343. The answer is e. (Schwartz, 7/e, pp 1275-1277.) The history, x-ray,

and clinical findings are typical of a postoperative cecal volvulus, a condition

in which the cecum is twisted on its mesentery (often, after aneurysm

resection, a neomesentery) and becomes acutely obstructed. At 12 cm, the

cecum is in imminent danger of perforation. Particularly in the presence of

a prosthetic graft, cecal perforation is a catastrophe. Urgent decompression

is needed. To attempt colonoscopic decompression would necessitate

insufflation of additional air and increase the stress on the already compromised

cecal wall. A transverse colostomy "decompression" would not

decompress the cecum nor would it provide detorsion of the cecal mesentery

to allow restoration of adequate blood supply to the right colon. While

untwisting the cecum and fixing it to the lateral abdominal wall (to inhibit

recurrence) by a decompressing cecostomy might be advocated in some

settings, the risk of contaminating the aortic graft would be excessive.

Resection of the offending organ with ileotransverse colostomy would be

the procedure of choice.

344. The answer is d. (Case Records, N Engl J Med 317:1209-1218, 1987.)

Hydatid cysts secondary to echinococcal infection are most common in the

liver in adults. Up to 25% of patients with hepatic cysts also have cysts in

their lungs. In general, serologic tests are more likely to be positive the

longer the lesion has been present, but false negativity occurs with sufficient

frequency that results should not influence the decision to treat

hepatic hydatid cysts. Spontaneous rupture of the cyst or leakage of cyst

226 Surgery

fluid during diagnostic or therapeutic aspiration may cause anaphylactic

reactions or peritoneal dissemination of the disease. Definitive treatment

requires surgical resection, enucleation, or evacuation of the cysts. Agents

such as 0.5% silver nitrate or hypertonic saline are introduced into the cyst

at the time of surgery, and efforts are made to avoid spillage and contamination

of the peritoneal cavity. Treatment of patients with liver cysts with

mebendazole or albendazole has not been effective enough to replace the

need for surgery.

345. The answer is a. (Mahmoodian, South Med J 85:19-24, 1992.)

Appendicitis complicates approximately 1 in 1700 pregnancies at an incidence

comparable with that in nonpregnant women matched for age. It is

the most prevalent extrauterine indication for laparotomy in pregnancy.

The duration of gestation does not influence the severity of the disease, but

the diagnosis does become more difficult as the pregnancy progresses. By

the twentieth week of gestation the appendix often lies at the level of the

umbilicus and more lateral than usual. Pregnancy should not delay surgery

if appendicitis is suspected; appendiceal perforation greatly increases the

chance of premature labor and fetal mortality (approximately 20% for

each). In contrast, negative laparotomy under general anesthesia and nonperforated

appendicitis are associated with very low risk to both the fetus

and mother (less than 1% and 5%, respectively).

346. The answer is b. (Schwartz, 7/e, pp 1161-1169.) Normal respiration

creates negative pressure in the thoracic cavity. As a result of the pressure

gradient, blood enters the chest via the venae cava and air via the trachea;

both are life-sustaining results of this pressure gradient. The pathophysiologic

consequence of a hole in the diaphragm is that eventually abdominal

viscera will be aspirated into the thorax. The sliding hernia, contained in

the lower mediastinum by intact pleura, may rarely cause symptoms of

reflux that would justify surgical attention, but such patients are in no danger

of vascular compromise or of obstructive displacement of hollow viscera.

The paraesophageal hernia, on the other hand, leaves the patient at

substantial risk for both strangulation and obstruction. Either result would

be a surgical catastrophe; with rare exceptions, paraesophageal hernias

should be surgically repaired whenever diagnosed. A traction diverticulum

is usually caused by inflammatory contraction around mediastinal nodes,

is rarely of any symptomatic consequence, and need not be repaired. Nei-

Gastrointestinal Tract, Liver, and Pancreas Answers 227

ther the Schatzki's ring nor the esophageal web justifies esophageal surgery.

They can be ignored or dilated as symptoms demand.

347. The answer is d. (Greenfield, 2/e, pp 1092-1093.) As classically

described, Olgilvie syndrome was associated with the rare occurrence of

malignant infiltration of the colonic sympathetic nerve supply in the region

of the celiac plexus. The eponym is now applied to the condition in which

massive cecal and colonic dilation is seen in the absence of mechanical

obstruction. Other terms used to describe this condition are acute colonic

pseudo-obstruction, colonic ileus, and functional colonic obstruction. It tends to

occur in elderly patients in the setting of cardiopulmonary insufficiency, in

other systemic disorders that require prolonged bed rest, and in the postoperative

state. The diagnosis of Olgilvie syndrome cannot be confirmed until

mechanical obstruction of the distal colon is excluded by colonoscopy or

contrast enema. Anticholinergic agents and narcotics need to be discontinued,

but any delay in decompressing the dilated cecum is inappropriate

since colonic ischemia and perforation become a distinct hazard as the

cecum reaches this degree of dilation. Cautious endoscopic colonic decompression

has been demonstrated recently to be a safe and effective form of

treatment. Endoscopy should be combined with rectal tube placement, correction

of metabolic abnormalities, and the discontinuation of medications

that diminish gastrointestinal motility. The high complication rate in this

population notwithstanding, a direct surgical approach to decompression

becomes necessary when colonoscopic decompression fails; a perforated

cecum is a catastrophic event in such patients.

348. The answer is c. (Schwartz, 7/e, p 1125.) Zenker's diverticulum is an

acquired abnormality. Premature contraction of the cricopharyngeus muscle

on swallowing, which leads to partial obstruction, is believed to be the cause

of this pulsion-type diverticulum of the pharyngoesophageal junction. High

intraluminal pressure results in an outpouching of mucosa through the

oblique fibers of the pharyngeal constrictors. Dysphagia is common and is

the usual presenting symptom. The diagnosis is established by barium swallow.

Treatment is surgical: diverticulectomy or suspension of the diverticulum

is usually recommended. Because the diverticulum is located above the

superior esophageal sphincter, no mechanism exists to prevent aspiration of

the contents of the diverticulum. Pulmonary complications are common.

228 Surgery

349. The answer is a. (Merrell, West J Med 155:621-625, 1991.) The classic

Quincke triad of abdominal pain in the right upper quadrant, jaundice,

and gastrointestinal bleeding is present in 30-40% of patients with hemobilia.

With more frequent use of percutaneous liver procedures (e.g., transhepatic

cholangiogram, transhepatic catheter drainage), iatrogenic injury

has replaced other trauma as the most common cause of bloody bile. Other

causes include spontaneous bleeding during anticoagulation, gallstones,

parasitic infections/abscesses, and neoplastic lesions. Angiography and

endoscopy are useful diagnostic studies and intrahepatic bleeding can be

controlled by angiographic embolization in up to 95% of cases. Surgical

treatment is advocated for bleeding from extrahepatic bile ducts or the gallbladder

or in cases of penetrating trauma in which associated injuries

might need attention.

350. The answer is d. (Podolosky, N Engl J Med 325:928-937, 1991.) The

patient depicted in this question has Crohn's disease of the colon (Crohn's

colitis). Crohn's colitis is characterized by linear mucosal ulcerations, discontinuous

("skip") lesions, a transmural inflammatory process, and noncaseating

granulomata in up to 50% of patients. Because their clinical

features and management differ, Crohn's colitis must be distinguished from

ulcerative colitis. Ulcerative colitis is usually found in the rectum, although

in rare cases the rectum is spared involvement. The entire colon, from

cecum to rectum, may be involved ("pancolitis"). Ulcerative colitis typically

presents as a grossly continuous inflammatory process (without skip

lesions) that microscopically is confined to the mucosa and submucosa of

the colon. In addition, crypt abscesses and superficial ulcerations are common

in ulcerative colitis.

351. The answer is c. (Podolosky, N Engl J Med 325:928-937, 1991.)

Patients with Crohn's disease can develop fistulas between the colon and

other segments of intestine, the bladder, urethra, vagina, skin, and prostate

in the male. Intestinal perforation can occur in about 5% of patients. Toxic

megacolon can occur in patients with Crohn's disease, ulcerative colitis, or

any severe inflammatory process of the large intestine. Extraintestinal manifestations

are usually associated with active disease. Finally, patients with

Crohn's colitis have a 5.6-fold increased risk of colon cancer relative to an

age-matched population.

Gastrointestinal Tract, Liver, and Pancreas Answers 229

352. The answer is c. (Schwartz, 7/e, pp 1161-1169.) The condition

demonstrated is a paraesophageal hernia. It is encountered much less frequently

(approximately 5%) than is the sliding hiatal hernia and it has

completely different therapeutic implications. Paraesophageal hernias are

acquired, rarely present before middle age, and are most common in

patients in their seventh decade. The position of the gastroesophageal junction

distinguishes the two types of hernias, which occur near the

esophageal hiatus of the diaphragm. In the more common sliding hernia,

the gastroesophageal junction protrudes above the diaphragm; in the

paraesophageal hernia, the anatomic junction between the esophagus and

the stomach is anchored in its normal position below the diaphragm. The

gastric cardia or fundus and occasionally other viscera herniate into the

thorax within a true peritoneal sac alongside the gastroesophageal junction.

Surgical repair is indicated as soon as the patient can be properly prepared

for the procedure, as bleeding, ulceration, obstruction, necrosis of

the stomach wall, and perforation are common.

353. The answer is b. (Schwartz, 7/e, pp 1488-1492.) The vast majority

of pancreatic carcinomas are located in the head of the gland. Patients may

present with painless jaundice by virtue of the carcinoma's obstruction of

the intrapancreatic portion of the common bile duct. It is in this group of

patients that resection is possible, although most tumors will be unresectable.

Tumors in the body or tail of the gland are universally unresectable.

The cause of pancreatic cancer is not known. There is a very

strong association with diabetes mellitus (but not diabetes insipidus), but

the nature of this relationship is not known. Prognosis is uniformly dismal

whether resection is done or not, and only an anecdotal survivor will be

alive at 5-year follow-up.

354. The answer is b. (Schwartz, 7/e, pp 1732-1734.) Intussusception is

the result of invagination of a segment of bowel into distal bowel lumen.

The most common type is ileocolic, which typically appears as a "coiled

spring" on barium enema. Ileoileal and colocolic intussusceptions occur

less commonly and are not easily diagnosed on barium enema. If bloody

mucus, peritonitis, or systemic toxicity have not developed, hydrostatic

reduction by barium enema is the appropriate initial treatment. Most

patients are successfully managed this way and do not require surgical

intervention. Immediate treatment should be instituted to avert the danger

230 Surgery

of bowel infarction. Recurrence is surprisingly uncommon after either surgical

or nonsurgical treatment.

355. The answer is d. (Schwartz, 7/e, pp 1244-1246.) Carcinoid tumors

arise from the neuroectoderm and are a type of apudoma. The most common

site of carcinoid tumors is the small bowel, although appendiceal carcinoids

are also common. Carcinoid syndrome, which is characterized by

flushing, diarrhea, and cardiac valvular disease, occurs in a small percentage

of patients with carcinoid tumors; it is rarely seen with appendiceal carcinoids.

It occurs when serotonin is released into the systemic circulation

and thus avoids breakdown by the liver. The appropriate therapy for a

small carcinoid (less than 2 cm) of the appendix is simple appendectomy.

356. The answer is b. (Schwartz, 7/e, pp 1586-1604.) Direct inguinal

hernias occur medial to the inferior epigastric vessels and are best repaired

by reapproximating the transversalis fascia to Cooper's ligament and thus

reconstructing the floor of the inguinal canal or by a tension-free Lichtenstein-

type repair. The hernia sac is opened and ligated routinely during

indirect hernia repair but not during direct hernia repair. The most common

inguinal hernia in women is an indirect hernia. Direct hernias rarely

present with a scrotal component and are less likely to present with incarceration

than indirect hernias.

357. The answer is b. (Cosentino, Surgery 112:740-748, 1992.) Choledochal

cysts are congenital cystic dilations of the extrahepatic biliary ducts.

Intrahepatic cystic dilation can coexist (Caroli's disease), but it represents a

distinct problem and is managed differently. Patients may present with

symptoms at any age, but the classic triad of epigastric pain, abdominal

mass, and jaundice is not frequently seen. Rather, most patients present

with other conditions such as cholecystitis, cholangitis, or pancreatitis.

Ultrasonography or endoscopic retrograde cholangiopancreatography

(ERCP) is helpful in demonstrating cysts. Nonsurgical treatment of these

cysts results in high morbidity and mortality, and therefore surgery is

advised in all cases. The present recommendation is for complete resection

of the cyst and Roux-en-Y choledochojejunostomy. Since malignant

changes in choledochal cysts have been frequently described, complete

resection rather than the performance of an internal drainage procedure is

preferred whenever the resection can be done safely.

Gastrointestinal Tract, Liver, and Pancreas Answers 231

358. The answer is d. (Schwartz, 7/e, pp 1062-1065.) Stress ulceration

refers to acute gastric or duodenal erosive lesions that occur following

shock, sepsis, major surgery, trauma, or burns. These lesions tend to be

superficial and can involve multiple sites. McClelland and associates

showed that patients subjected to trauma and subsequent hemorrhagic

shock do not have increased gastric secretion, but rather show decreased

splanchnic blood flow. Ischemic damage to the mucosa may therefore play

a role. Unlike chronic benign gastric ulcers, which are generally found

along the lesser curvature and in the antrum, acute erosive lesions usually

involve the body and fundus and spare the antrum.

359. The answer is b. (Schwartz, 7/e, pp 1454-1455.) Cholangitis is suggested

by the presence of the Charcot triad: fever, jaundice, and pain in the

right upper quadrant. These symptoms are usually caused by choledocholithiasis,

but they can also occur in association with obstructing neoplasms

and choledochal cysts. The disease occurs primarily in the elderly.

Therapy is aimed at decompression of the common bile duct. In patients

with suppurative cholangitis who fail to respond to intravenous antibiotics

initially and fluid resuscitation, the nonoperative approach is the preferred

intervention either via percutaneous or endoscopic drainage of the

obstructed common bile duct. If the nonoperative approach fails, surgery

is indicated. This is usually best accomplished by surgical placement of a T

tube into the duct. Percutaneous transhepatic catheter drainage is an

acceptable alternative in select patients. This procedure can often provide

effective decompression during the acute septic phase of the disease.

Cholecystostomy will be effective only if there is free flow of bile into the

gallbladder via the cystic duct and in general should not be depended on

to secure drainage of the common bile duct.

360. The answer is a. (Schwartz, 7/e, pp 1459-1460.) High-risk, critically

ill patients with multisystem disease and cholecystitis experience a significant

increase in morbidity and mortality following operative intervention.

Tube cholecystostomy can be performed under local anesthesia in the

operating room or via a percutaneous approach in the radiology suite.

Open or laparoscopic procedures would carry the same general anesthetic

risk whether done urgently or in a delayed (elective) fashion. Lithotripsy

has no role in the treatment of acute cholecystitis.

232 Surgery

361. The answer is a. (Schwartz, 7/e, pp 1485-1487.) Pancreatic pseudocysts

can develop in the setting of acute and chronic pancreatitis. They are

cystic collections that do not have an epithelial lining and therefore have no

malignant potential. Most pseudocysts spontaneously resolve. Therapy

should not be considered for 6 wk to allow for the possibility of spontaneous

resolution as well as to allow for maturation of the cyst wall if the

cyst persists. Complications of pseudocysts include gastric outlet and

extrahepatic biliary obstructions as well as spontaneous rupture and hemorrhage.

Pseudocysts can be excised, externally drained, or internally

drained into the gastrointestinal tract (most commonly the stomach or a

Roux-en-Y limb of jejunum).

362. The answer is d. (Reilly, Dig Dis Sci 36:1702-1707, 1991.) Dieulafoy's

lesion has been identified more frequently recently as a source of

gastrointestinal bleeding. It is characteristically located within 6 cm distal

to the gastroesophageal junction. Dieulafoy's lesion typically consists of an

abnormally large submucosal artery that protrudes through a small, solitary

mucosal defect. The lesions may bleed spontaneously and massively

for unclear reasons, in which case they require emergency intervention.

Upper endoscopy is usually successful in localizing the lesion, and permanent

hemostasis can be obtained endoscopically in most cases with injection

sclerotherapy, electrocoagulation, or heater probe. If surgery is

required, a gastrotomy and simple ligation or wedge resection of the lesion

may be adequate. No large series have yet established the optimal surgical

treatment for Dieulafoy's lesion; however, acid-reducing procedures have

not been successful in preventing further bleeding.

363. The answer is b. (Schwartz, 7/e, pp 1244-1246.) Carcinoid tumors

arise from enterochromaffin cells in the crypts of Lieberkühn. When they

are encountered in the appendix and are less than 2 cm in size, simple

appendectomy is the procedure of choice. When the tumors are larger

than 2 cm, a right hemicolectomy should be performed. Carcinoid syndrome

(hepatomegaly, diarrhea, cutaneous flushing, right heart valvular

disease, and asthma) usually occurs in the presence of liver metastases

but can also be seen when there are metastases to sites drained by systemic

(as opposed to portal) veins or from primary carcinoids outside the

portal system. Carcinoid syndrome is rare in patients with carcinoid of

Gastrointestinal Tract, Liver, and Pancreas Answers 233

the appendix because the tumors are usually discovered before metastases

occur.

364. The answer is d. (Schwartz, 7/e, p 1374.) Rectal carcinoids are

slowly growing tumors, but they can be locally invasive and metastasize in

up to 15% of patients. Patients manifest systemic signs of the carcinoid

syndrome only in the rare circumstance where hepatic metastases have

occurred. The malignant potential is low in carcinoid tumors when they

are less than 2 cm in diameter, as is typically the case when diagnosed. The

tumors are curable by wide, local transanal resection that includes the

muscle layer. Endoscopic treatment leaves tumor cells near the margin of

resection and is felt to increase the risk of recurrence. Whether more

aggressive resection (abdominoperineal or low anterior resection)

improves the prognosis in larger tumors remains controversial. The prognosis

is excellent for patients with local disease.

365. The answer is b. (Reilly, Dig Dis Sci 36:1702-1707, 1991.) Polypoid

lesions of the gallbladder are found most often in the third through fifth

decades of life and are increasingly being detected by ultrasonography.

These are generally small lesions that typically do not show a shadow on

ultrasound. Ninety percent are benign lesions, such as cholesterol polyps

(pseudotumors). True adenomas constitute about 10% of these benign

lesions, but they can undergo malignant transformation. The indications

for operative intervention remain controversial. Recent reviews suggest

that the vast majority of malignant polypoid lesions are solitary, larger than

1.0 cm, and much more common in patients greater than 50 years of age.

There is also an increased incidence of malignancy if the lesions are associated

with gallstones. Symptomatic lesions should be removed regardless of

their size. Asymptomatic small lesions can probably be safely followed by

ultrasonography.

366. The answer is a. (Greenfield, 2/e, p 912.) The metabolic consequences

of total pancreatectomy are manifold. They include weight loss,

malabsorption attended by hypocalcemia and hypophosphatemia, diabetes

mellitus, diarrhea, and both iron deficiency and pernicious anemia. In theory,

total pancreatectomy should provide good surgical treatment for pancreatic

carcinoma; in reality, the severe metabolic problems that result from

total removal of the pancreas make partial pancreaticoduodenectomy a fre-

234 Surgery

quently preferred treatment for most cases of pancreatic carcinoma that are

resectable. Because of the frequently multicentric nature of pancreatic cancers,

however, some surgeons would rather perform a total pancreatectomy

and accept the more complicated postoperative metabolic management

entailed by the loss of pancreatic endocrine function.

367. The answer is d. (Greenfield, 2/e, pp 1156-1157.) Cecal diverticula

must be differentiated from the more common variety of diverticula that

are usually found in the left colon. Cecal diverticula are thought to be a

congenital entity. The cecal diverticulum is often solitary and involves all

layers of the bowel wall; therefore, cecal diverticula are true diverticula.

Diverticula elsewhere in the colon are almost always multiple and are

thought to be an acquired disorder. These acquired diverticula are really

herniations of mucosa through weakened areas of the muscularis propria of

the colon wall. The preoperative diagnosis in the case of cecal diverticulitis

is "acute appendicitis" about 80% of the time. If there is extensive inflammation

involving much of the cecum, an ileocolectomy is indicated. If the

inflammation is well localized to the area of the diverticulum, a simple

diverticulectomy with closure of the defect is the procedure of choice. To

avoid diagnostic confusion in the future, the appendix should be removed

whenever an incision is made in the right lower quadrant, unless operatively

contraindicated.

368. The answer is c. (Schwartz, 7/e, pp 1407-1408.) Hepatic hemangiomata

are the most common of all liver tumors. The infantile forms are

highly vascular and occasionally cause hepatomegaly or congestive cardiac

failure that requires angiographic or surgical interruption. The diagnostic

incidence of incidental cavernous hemangiomata in adults has increased in

this era of noninvasive imaging of organs with MRI, ultrasonography, and

CT. When this lesion is suspected, the diagnosis can be confirmed with

sensitive and more specific imaging techniques such as labeled red blood

cell scanning (not liver/spleen scans). The mean age of presentation in

adults is about 50 years and the vast majority of these lesions are asymptomatic.

There is no evidence that they undergo malignant transformation.

They may enlarge and become symptomatic more readily in women after

multiple pregnancies or during the use of estrogen or oral contraceptives.

The risk of rupture and severe hemorrhage into or from hemangiomata is

extremely low; when it does occur, it is usually iatrogenic (following

Gastrointestinal Tract, Liver, and Pancreas Answers 235

attempted biopsy). Given the typically benign and static nature of these

lesions, management by angiographic embolization or resection should be

reserved for the rare patient with symptomatic or complicated hemangioma.

369. The answer is d. (Greenfield, 2/e, pp 1138, 1144.) CEA is a tumor

marker that was described in 1965 by Gold and Freedman. It is a nonspecific

tumor marker that is elevated in only about one-half of patients with

colorectal tumors and is often elevated in patients with lung, pancreatic,

gastric, and gynecologic malignancies. CEA is also elevated in cigarette

smokers. Patients in whom the primary colon tumor produced CEA and in

whom the level falls below 2-3 ng/mL after resection have an excellent

prognosis for disease control. In such patients, a subsequent rise in CEA

has been demonstrated to be a very sensitive marker of the presence and

extent of recurrent disease. Many surgeons follow CEA levels and perform

"second-look" operations to resect local disease or possibly isolated

metastatic disease if the levels become elevated postoperatively. Some surgeons

recommend exploration in that circumstance even in the absence of

other evidence (CT scan, colonoscopy) of recurrence. The long-term survival

seems to be improved following this aggressive approach in some

patients. Very high elevations of CEA, however, suggest extensive liver disease

or peritoneal spread, which is unresectable.

370-373. The answers are 370-b, 371-d, 372-c, 373-a. (Greenfield,

2/e, pp 785-787.) Gastric ulcers have been classified as type I (incisura or

most inferior portion of lesser curvature), type II (gastric and duodenal),

type III (pyloric and prepyloric), and type IV (juxtacardial). Indications for

surgery are intractability, perforation, obstruction, and bleeding. A patient

with an intractable type I ulcer can be treated with an antrectomy alone or

with a proximal gastric vagotomy. If done properly, antrectomy offers

slightly lower recurrence rates and a higher incidence of postoperative

sequelae as compared with proximal gastric vagotomy. However, significant

scarring along the lesser curvature makes a proximal gastric vagotomy

technically unfeasible.

Gastric outlet obstruction and severe inflammation around the pylorus

and duodenum make resection a difficult and dangerous option. Similarly,

pyloroplasty is often not adequate in the setting of gastric outlet obstruction

to provide adequate drainage. Vagotomy and gastrojejunostomy,

236 Surgery

although associated with the highest recurrence rate, offers the best choice

in the described setting.

In an elderly patient with a bleeding duodenal ulcer, recurrence rates

are less of a consideration and thus the simplest and most expedient operation

offers the best surgical outcome. Vagotomy and pyloroplasty with

oversewing of the ulcer is the best choice in this setting.

Finally, in a young patient with intractable type III ulcers, antrectomy

with vagotomy offers the best long-term outcome. Recurrence rates following

this procedure are about 2-3%, as compared with 7.4% for vagotomy

and drainage and 10-31% in patients receiving a proximal gastric vagotomy

only.

374-376. The answers are 374-d, 375-d, 376-e. (Schwartz, 7/e, pp

1161-1167.) Paraesophageal hernias, generally thought to be acquired,

involve herniation of any portion or all of the stomach into the thoracic cavity

via the esophageal hiatus. These hernias are usually repaired electively

because of a high incidence of complications. In these dangerous hernias, the

cardioesophageal junction is in its normal position below the diaphragm.

Diaphragmatic ruptures usually affect adults and result from blunt

trauma to the abdomen. Unless such ruptures are repaired, the negative

intrathoracic pressure associated with each respiratory effort tends to such

the abdominal contents into the chest with consequent loss of necessary

space for lung expansion and substantial risk of damage to the intrathoracic

bowel.

Sliding hiatal hernias, the most frequent type of hernia found in

adults, are generally acquired. The significance of this type of hernia rests

in its association with gastroesophageal reflux, a condition that may lead to

reflux esophagitis. Because sliding hiatal hernias frequently do not exhibit

significant gastroesophageal reflux, it is likely that other factors may be

more important in the pathophysiology of that disorder.

The foramen of Bochdalek hernia is a congenital hernia of the posterolateral

aspect of the diaphragm in which abdominal viscera enter the

thorax and cause acute respiratory distress in infants. This hernia requires

emergency repair.

The foramen of Morgagni hernia, although also congenital, is not usually

detected until adulthood. It is usually an incidental finding on chest xray,

where it appears as a low anterior mediastinal mass. However, on rare

occasions it can produce acute respiratory distress in infants.

Gastrointestinal Tract, Liver, and Pancreas Answers 237

377-378. The answers are 377-b, c, h, i; 378-f. (Schwartz, 7/e, pp

1529-1530, 1275-1277.) The radiograph demonstrates pneumoperitoneum.

Only a perforated viscus can produce this radiographic appearance

in conjunction with diffuse peritonitis. A perforated gastric ulcer,

perforated diverticulum, perforated transverse colon carcinoma, or strangulated

hernia with necrotic bowel would all produce this clinical picture.

A sigmoid volvulus appears radiographically on plain film of the

abdomen as an upside-down U or "bent inner tube." Acute sigmoid volvulus

presents in the elderly with nausea, vomiting, abdominal distention,

colicky abdominal pain, and obstipation. The first diagnostic and often

therapeutic maneuver should be a sigmoidoscopy.

379-380. The answers are 379-d, e, f; 380-c. (Schwartz, 7/e, pp 1168,

1156-1158.) Patients with Mallory-Weiss syndrome typically present with

a massive, painless hematemesis after severe vomiting or retching. The

majority of tears (87%) occur just below the gastroesophageal junction.

These tears occur 3 times more commonly in cirrhotics than in the normal

population. Most of the time (90%), bleeding will stop without any intervention.

When bleeding persists, balloon tamponade, endoscopic control

of the bleeding, and surgical intervention with gastrotomy and oversewing

of the tear have all been successful. Both intravenous and intraarterial infusion

of vasopressin are also useful in controlling bleeding but are contraindicated

in patients with coronary artery disease.

The patient presents with Boerhaave syndrome (spontaneous perforation

of the esophagus following sudden increase in intraabdominal pressure).

Unlike Mallory-Weiss tears, these tears are transmural perforations.

Typical presentation is that of severe retrosternal or left chest or shoulder

pain following an episode of retching; therefore, the symptoms can sometimes

mimic a myocardial or pulmonary infarction. However, a good history

can usually distinguish a Boerhaave perforation from these other

entities. A Gastrografin swallow is helpful in cases that are diagnostically

challenging. Treatment consists of left thoracotomy, repair of the transmural

tear, and adequate drainage.

238 Surgery

CARDIOTHORACIC

PROBLEMS

Questions

DIRECTIONS: Each item below contains a question or incomplete

statement followed by suggested responses. Select the one best response to

each question.

239

381. Among the cardiovascular

anomalies of newborns, the one

most likely to present with cyanosis

a. Patent ductus arteriosus

b. Coarctation of the aorta

c. Atrial septal defect

d. Ventricular septal defect

e. Transposition of the great vessels

382. The superior vena cava syndrome

is most frequently seen in

association with

a. Histoplasmosis (sclerosing mediastinitis)

b. Substernal thyroid

c. Thoracic aortic aneurysm

d. Constrictive pericarditis

e. Bronchogenic carcinoma

383. During endoscopic biopsy of

a distal esophageal cancer, perforation

of the esophagus is suspected

when the patient complains of significant

new substernal pain. An

immediate chest film reveals air in

the mediastinum. You would recommend

a. Placement of a nasogastric tube to

the level of perforation, antibiotics,

close observation

b. Spit fistula (cervical pharyngostomy),

gastrostomy

c. Left thoracotomy, pleural patch

oversewing of perforation, drainage

of mediastinum

d. Esophagogastrectomy via celiotomy

and right thoracotomy

e. Transhiatal esophagogastrectomy

with cervical esophagogastrostomy

Terms of Use

Items 384-385

384. A noncyanotic 2-day-old child

has a systolic murmur along the left

sternal border; the examination is

otherwise normal. Chest x-ray and

electrocardiogram are normal. These

findings are most closely associated

with which of the following congenital

cardiac anomalies?

a. Tetralogy of Fallot

b. Ventricular septal defect

c. Tricuspid atresia

d. Transposition of the great vessels

e. Patent ductus arteriosus

385. A 3-year-old child with congenital

cyanosis is most probably

suffering from

a. Tetralogy of Fallot

b. Ventricular septal defect

c. Tricuspid atresia

d. Transposition of the great vessels

e. Patent ductus arteriosus

386. A stockbroker in his midforties

consults you with complaints

of episodes of severe, often

incapacitating chest pain on swallowing.

The diagnostic studies on

the esophagus you have ordered

yield the following: endoscopic

examination and biopsy-mild

inflammation distally; manometry-

prolonged high-amplitude

contractions from the arch of the

aorta distally, lower esophageal

sphincter (LES) pressure 20 mm

Hg with relaxation on swallowing;

barium swallow-2-cm epiphrenic

diverticulum. You would recommend

a. Myotomy from level of aortic arch

to distal sphincter; no disruption of

LES

b. Diverticulectomy, myotomy from

level of aortic arch to fundus, fundoplication

c. Diverticulectomy, cardiomyotomy

of distal 3 cm of esophagus and

proximal 2 cm of stomach with

antireflux fundoplication

d. A trial of calcium channel blockers

e. Pneumatic dilation of LES

240 Surgery

387. A 4-year-old boy is seen 1 h

after ingestion of a lye drain

cleaner. No oropharyngeal burns

are noted, but the patient's voice is

hoarse. Chest x-ray is normal. Of

the following, which is the most

appropriate therapy?

a. Immediate esophagoscopy

b. Parenteral steroids and antibiotics

c. Administration of an oral neutralizing

agent

d. Induction of vomiting

e. Rapid administration of a quart of

water to clear remaining lye from

the esophagus and dilute material

in stomach

388. A previously healthy 20-yearold

man is admitted to a hospital

with acute onset of left-sided chest

pain. The electrocardiographic

findings are normal but chest x-ray

shows a 40% left pneumothorax.

Treatment consists of which of the

following procedures?

a. Observation

b. Barium swallow

c. Thoracotomy

d. Tube thoracostomy

e. Thoracostomy and intubation

389. A 50-year-old salesman is on

a yacht with a client when he has a

severe vomiting and retching spell

punctuated by a sharp substernal

pain. He arrives in your emergency

room 4 h later and has a chest film

in which the left descending aorta

is outlined by air density. Optimum

strategy for care would be

a. Immediate thoracotomy

b. Serial ECGs and CPKs to rule out

myocardial ischemia

c. Left chest tube and spit fistula (cervical

esophagostomy)

d. Flexible esophagogastroscopy to

establish diagnosis

e. Nasogastric tube, antibiotics, close

Cardiothoracic Problems 241

Items 390-391

A 26-year-old man is brought to the emergency room after being extricated

from the driver's seat of a car involved in a head-on collision in which

the patient was not wearing his seat belt. His ECG is shown below.

242 Surgery

390. The ECG is most consistent with

a. Preexisting disease

b. Myocardial ischemia that caused the accident

c. Myocardial contusion that resulted from the accident

d. Chagas disease

e. Normal variant

391. The best test for establishing the diagnosis and the degree of myocardial

dysfunction is

a. Serial ECGs

b. Creatine phosphokinase (CPK-MB) fractionation

c. Echocardiography

d. Radionuclide angiography

e. Coronary angiography

Items 392-393

Several days following esophagectomy

a patient complains of

dyspnea and chest tightness. A

large pleural effusion is noted on

chest radiograph and thoracentesis

yields milky fluid consistent with

chyle.

392. Initial management of this

patient consists of which of the following

procedures?

a. Immediate operation to repair the

thoracic duct

b. Immediate operation to ligate the

thoracic duct

c. Tube thoracostomy and low-fat diet

d. Observation and low-fat diet

e. Observation and antibiotics

393. Two weeks following the initial

management of this patient's

chylothorax there is persistent

accumulation of chyle in the

pleural space. Appropriate management

at this time includes which of

the following procedures?

a. Neck exploration and ligation of

the thoracic duct

b. Subdiaphragmatic ligation of the

thoracic duct

c. Thoracotomy and repair of the thoracic

duct

d. Thoracotomy and ligation of the

thoracic duct

e. Thoracotomy and abrasion of the

pleural space

394. A 56-year-old woman was

treated for 3 years for wheezing on

exertion, which was diagnosed as

asthma. The chest radiograph

below is obtained, which reveals a

midline mass compressing the trachea.

The most likely diagnosis is

Cardiothoracic Problems 243

a. Lymphoma

b. Neurogenic tumor

c. Lung carcinoma

d. Goiter

e. Pericardial cyst

395. A full-term male newborn

experiences respiratory distress

immediately after birth. A prenatal

sonogram had been read as normal.

An emergency radiograph is shown

below. The patient was intubated

and placed on 100% O2. The arterial

blood gas revealed pH 7.24; PO2

60 kPa; PCO2 52 kPa. The baby has

sternal retractions and a scaphoid

abdomen. Which of the following

statements correctly refers to this

condition?

a. The most likely cause of this problem

is in utero traumatic rupture of

the diaphragm

b. The most important aspect in management

would be immediate

exploration and repair of the defect

c. The size of the defect directly correlates

with severity of the disease

d. The defect is usually anteromedial

in location

e. Any abdominal organ can be

involved

244 Surgery

396. An 89-year-old man has lost 30 lb over the past 2 years. He reports

that food frequently sticks when he swallows. He also complains of a

chronic cough. Pulmonary function tests show a vital capacity of 60% of

expected, and forced expiratory volume is 50% of predicted. Barium swallow

is shown below. Which of the following statements is true?

Cardiothoracic Problems 245

a. Radiation therapy and stenting can be expected to produce the same long-term

survival as would surgery

b. Esophagoscopy and biopsy should be performed to confirm the x-ray findings

c. This patient is atypical in that the lesion usually appears in the second or third

decade of life

d. The patient should be treated with antituberculous medications before any surgical

intervention is considered

e. The carotid bifurcation lies adjacent to the lesion

397. Which of the following statements

is true concerning aortocoronary

bypass grafting?

a. It is indicated for crescendo (preinfarction)

angina

b. It is indicated for congestive heart

failure

c. It is not indicated for chronic disabling

angina

d. It is associated with a 10% operative

mortality

e. It is only indicated if significant

triple vessel disease is documented

angiographically

398. Which of the following statements

is true regarding the thoracic

outlet syndrome?

a. It is associated with cervical spine

disk disease

b. It is reliably diagnosed by positional

obliteration of the radial

pulse

c. If conservative measures fail, it is

best treated by surgical decompression

of the brachial plexus

d. It most commonly affects the

median nerve

e. It can be reliably ruled out by

angiography

399. A 35-year-old man presents

with a history of 4 days of severe

substernal pain and fever to

38.89°C (102°F). He has a past

medical history of peptic ulcer disease

that resulted in a Billroth II

procedure 5 years earlier. On

admission, the chest film below is

obtained. A true statement regarding

this patient's case is which of

the following?

246 Surgery

a. Pericardial effusion is present

b. The condition may be managed

with antibiotics and close observation

if the patient remains hemodynamically

stable

c. The condition could have resulted

from recurrent peptic ulcer disease

d. The condition could have resulted

from a myocardial infarction

e. The previous Billroth II procedure

effectively rules out peptic ulcer as

the cause of the condition

400. Superior pulmonary sulcus

carcinomas (Pancoast tumors) are

bronchogenic carcinomas that typically

produce which of the following

clinical features?

a. Atelectasis of the involved apical

b. Horner syndrome

c. Pain in the T4 and T5 dermatomes

d. Nonproductive cough

e. Hemoptysis

401. A 2-year-old asymptomatic

child is noted to have a systolic

murmur, hypertension, and diminished

femoral pulses. Which of the

following is true about this child's

disorder?

a. The life expectancy without surgery

is about 5 years

b. Immediate surgery is indicated

c. Rib notching is often seen on x-ray

d. Claudication is frequently noted

e. Operative mortality approaches

10%

402. A correct statement concerning

bronchial carcinoid tumors is

that

a. They frequently metastasize

b. They most commonly arise in

peripheral terminal bronchioles

c. They rarely produce the carcinoid

syndrome

d. They are radiosensitive

e. Five-year survival is less than 50%

Items 403-404

Six months ago at the time of

lumpectomy for breast cancer, a

60-year-old female attorney quit a

30-year smoking habit of two

packs per day. She had the chest

radiograph below as part of her

routine follow-up examination.

Cardiothoracic Problems 247

403. True statements about the

lesion visualized on the film

include which of the following?

a. It is more apt to be metastatic

breast carcinoma than primary

lung carcinoma

b. There is a 90% chance that this

mass is malignant

c. Since the diagnosis can only be

established with certainty by resection,

the mass should be excised

d. If the mass is malignant, the possibility

for cure with excision is

e. The mass is most likely benign

404. At the time of operation on

the patient in the preceding question,

a firm, rubbery lesion in the

periphery of the lung is discovered.

It is sectioned in the operating

room to reveal tissue that looks like

cartilage and smooth muscle. The

most likely diagnosis is

a. Fibroma

b. Chondroma

c. Osteochondroma

d. Hamartoma

e. Aspergilloma

405. The condition shown in the

x-rays below is compatible with

which of the following manifestations?

a. Difficulty swallowing solids but not

liquids

b. Higher-than-normal incidence of

esophageal carcinoma

c. Failure of the upper esophageal

sphincter to relax in response to

swallowing

d. Normal pressure in the body of the

esophagus

e. Normal esophageal motility

248 Surgery

249

DIRECTIONS: Each group of questions below consists of lettered

options followed by numbered items. For each numbered item, select the

appropriate lettered option(s). Each lettered option may be used once,

more than once, or not at all. Choose exactly the number of options

indicated following each item.

Items 406-410

For each physical finding or

group of physical findings below,

select the cardiovascular disorder

with which it is most likely to be

associated.

a. Massive tricuspid regurgitation

b. Aortic regurgitation

c. Coarctation of the aorta

d. Thoracic aortic aneurysm

e. Myocarditis

406. Argyll Robertson pupil

(SELECT 1 DISORDER)

407. Exophthalmos (SELECT 1

DISORDER)

408. Quincke pulse (SELECT 1

DISORDER)

409. Conjunctivitis, urethral discharge,

and arthralgia (SELECT 1

DISORDER)

410. Short stature, webbed neck,

low-set ears, and epicanthal folds

(SELECT 1 DISORDER)

Items 411-415

For each pathologic sign

below, select the mediastinal tumor

with which it is most likely to be

associated.

a. Thymoma

b. Hodgkin's disease

c. Neuroblastoma

d. Parathyroid adenoma

e. Cystic teratoma

411. Increased urinary catecholamine

level (SELECT 1

TUMOR)

412. Red blood cell aplasia

(SELECT 1 TUMOR)

413. Renal stones (SELECT 1

TUMOR)

414. T-cell deficiency (SELECT 1

TUMOR)

415. Ectopic hair (SELECT 1

TUMOR)

250 Surgery

Items 416-420

Match the appropriate pharmacologic

agent with each description.

a. Epinephrine

b. Norepinephrine

c. Isoproterenol

d. Dopamine

e. Dobutamine

f. Amrinone

g. Digitalis

h. Nitroprusside

i. Nitroglycerin

j. Milrinone

416. Balanced arterial and venous

dilation (SELECT 1 AGENT)

417. Action as an inotrope and

vasodilator by inhibiting endogenous

phosphodiesterase (SELECT

2 AGENTS)

418. Pure beta agonist with profound

chronotropic properties

(SELECT 1 AGENT)

419. Endogenous catecholamine

secreted into the circulation under

normal conditions (SELECT 2

AGENTS)

420. Inotropic and antiarrhythmic

properties (SELECT 1 AGENT)

Cardiothoracic Problems 251

CARDIOTHORACIC

PROBLEMS

Answers

381. The answer is e. (Schwartz, 7/e, pp 812-826.) With the exception of

coarctation, in which no shunt (or cyanosis) exists, the anomalies listed cause

a shunting of blood between the systemic and lower-pressure pulmonary circulation.

Transposition of the great vessels is a right-to-left shunt that leads to

cyanosis. Except where there is persistent congenital pulmonary hypertension,

patent ductus arteriosus and atrial septal defects cause a shunting of

oxygenated blood from the aorta and left atrium, respectively, back into the

pulmonary artery and right atrium. These anomalies cause "recirculation" of

oxygenated blood within the cardiopulmonary circuit but not cyanosis.

When a ventricular septal defect is combined with pulmonary artery atresia

(tetralogy of Fallot), the resulting undercirculation in the pulmonary system

joins transposition as a cause of cyanosis. Other less common congenital

lesions in which the pulmonary arterial blood flow is relatively decreased

include tricuspid atresia, Ebstein's anomaly, and hypoplastic right ventricle.

382. The answer is e. (Schwartz, 7/e, p 784.) Superior vena cava obstruction

is almost always due to malignancy and, in three out of four cases,

results from invasion of the vena cava by bronchogenic carcinoma. Lymphomas

account for most of the remaining cases of the superior vena cava

syndrome. Fibrosing mediastinitis as a complication of histoplasmosis or

ingestion of methysergide may occur but is rare. Rarely a substernal thyroid

or thoracic aortic aneurysm may be responsible for the obstruction.

Although constrictive pericarditis may decrease venous return to the heart,

it does not produce obstruction of the superior vena cava. Whatever the

cause of the superior vena cava syndrome, the resultant increased venous

pressure produces edema of the upper body, cyanosis, dilated subcutaneous

collateral vessels in the chest, and headache. Cervical lymphadenopathy

may also be present as a result of either stasis or metastatic involvement.

When carcinoma is the cause of the superior vena cava syndrome, the treatment

is usually palliative and consists of diuretics and radiation.

252

383. The answer is d. (Schwartz, 7/e, pp 1156-1158.) Perforation of the

esophagus in the chest is a surgical catastrophe that requires aggressive

intervention in virtually all circumstances. While that intervention can

usually consist of efforts to patch the perforation and drain the mediastinum,

concomitant obstructive esophageal disease, whether inflammatory

stenosis or cancer, mandates removal or bypass of the obstruction if

control of the leak and its consequent persisting mediastinal and pleural

contamination is to be accomplished. For distal esophageal cancers, many

thoracic surgeons would use the classic Ivor-Lewis operation, which consists

of mobilizing the stomach in the abdomen and then performing a

right thoracotomy with mediastinal cleanout, esophagectomy, and esophagogastrostomy.

In some circumstances, and by some surgeons' preference,

a left thoracotomy approach might be used. The transhiatal approach

would probably be avoided in this situation where an unknown amount of

mediastinal contamination has taken place.

384-385. The answers are 384-b, 385-a. (Schwartz, 7/e, pp 812-826.)

Ventricular septal defect accounts for 20-30% of all congenital cardiac

anomalies. It may lead to cardiac failure and pulmonary hypertension if the

defect is larger than 1 cm; or it may be asymptomatic if the defect is small.

Surgery is not indicated for the asymptomatic patient with a small defect

since a substantial number of these anomalies close spontaneously during

the first few years of life. Operation is indicated in infants with congestive

heart failure or rising pulmonary vascular resistance (owing to the left-toright

shunt). When symptoms are mild and can be controlled medically,

operation is usually delayed until age 4-6 years. Operative mortality ranges

from less than 5% to more than 20% depending on the degree of pulmonary

vascular resistance.

Tetralogy of Fallot, transposition, and tricuspid atresia are cyanotic

lesions. Congenital cyanosis that persists beyond the age of 2 years is associated,

in the vast majority of cases, with a tetralogy of Fallot. Patent ductus

arteriosus is associated with the characteristic continuous machinery

murmur.

386. The answer is a. (Schwartz, 7/e, pp 1103-1121.) The diagnostic studies

listed reveal minimal reflux esophagitis, normal LES relaxation and pressure,

and an incidental small epiphrenic diverticulum. None of these

findings justifies treatment and none explains the patient's symptoms. On the

Cardiothoracic Problems Answers 253

other hand, the finding of prolonged high-amplitude contractions in the

body of the esophagus in a highly symptomatic patient is diagnostic of diffuse

esophageal spasm. The cause of this hypermotility disorder is unknown,

but its symptoms can be disabling. The recommended treatment for this relatively

rare disorder is a long myotomy guided by the manometric evidence.

If the LES is functioning properly, most surgeons would now recommend

stopping the myotomy short of the normal lower sphincter. It should continue

upward at least to the level of the aortic arch-higher if manometric

findings of spasm are noted above that level. Eighty to 90% of patients

treated in this fashion will experience acceptable relief of symptoms.

387. The answer is b. (Schwartz, 7/e, pp 1158-1161.) Corrosive injuries

of the esophagus most frequently occur in young children due to accidental

ingestion of strong alkaline cleaning agents. Significant esophageal

injury occurs in 15% of patients with no oropharyngeal injury, while 70%

of patients with oropharyngeal injury have no esophageal damage. Signs of

airway injury or imminent obstruction warrant close observation and possibly

tracheostomy. The risk of adding injury, particularly in a child, makes

esophagoscopy contraindicated in the opinion of most surgeons. Administration

of oral "antidotes" is ineffective unless given within moments of

ingestion; even then, the additional damage potentially caused by the

chemical reactions of neutralization often makes use of them unwise. A

barium esophagogram is usually done within 24 h unless evidence of perforation

is present. In most reports, steroids in conjunction with antibiotics

reduce the incidence of formation of strictures from about 70% to about

15%. Vomiting should be avoided, if possible, to prevent further corrosive

injury and possible aspiration. It is probably wise to avoid all oral intake

until the full extent of injury is ascertained.

The most helpful ECG finding is the presence of a new right bundle

branch block, which occurs because of damage to the anterior portion of

the interventricular septum; ST-segment and T-wave changes and even the

development of new Q waves may be seen. CPK-MB fractions are useful if

they are positive; however, frequent false negatives may be seen because of

the release of CPK-MM from other contused organs, such as the pectoralis

muscles, which can dilute the cardiac CPK-MB to nondiagnostic levels.

Echocardiography may be helpful, but the right ventricle is often poorly

visualized. Radionuclide angiography is most useful because it suggests the

degree of myocardial impairment caused by decreased compliance.

254 Surgery

Therapy of myocardial contusion is directed at inotropic support of the

ventricle; usually, the coronary arteries are intact after the injury and so there is

little role for coronary vasodilators and less for coronary artery bypass grafting.

388. The answer is d. (Schwartz, 7/e, pp 711-713, 781.) Spontaneous

pneumothorax usually results from the rupture of subpleural blebs in

young men (age 20-40), which is often signaled by a sudden onset of chest

and shoulder pain. Pneumothorax of more than 25% requires placement of

a chest tube; thoracotomy with bleb excision and pleural abrasion is generally

recommended if spontaneous pneumothorax is recurrent. Small

pneumothoraxes in patients with minimal symptoms usually resolve and

therefore can be observed. A spontaneous perforation of the esophagus

(Boerhaave syndrome) can result in hydropneumothorax as well as the

more usual pneumomediastinum, but would not present with an isolated

40% pneumothorax. Barium swallow is an appropriate diagnostic test for

evaluation of a suspected leaking esophagus.

389. The answer is a. (Henderson, Am J Med 86:559-567, 1989.) The presence

of air in the mediastinum after an episode of vomiting and retching is

virtually pathognomonic of spontaneous rupture of the esophagus (Boerhaave

syndrome). The evidence is overwhelming that without prompt surgical

exploration of the mediastinum by left thoracotomy, the patient has little

chance for a short-term outcome of low morbidity. The aspiration of highly

acidic gastric contents into the mediastinum creates havoc in the tissues

exposed to it. The surgical procedure must include extensive opening of the

mediastinal pleura and removal of any particulate debris that might have

been aspirated into the thorax from the stomach. Closure of the esophageal

laceration with reinforcement by a pleural flap and secure chest tube drainage

of the pleural space are mandatory. If the operation is delayed beyond the first

8-24 h, the mortality rises sharply and survival will only follow prolonged

intensive care and multiple operations. This catastrophic event is one of the

few in which prompt diagnosis and intervention are crucial to success.

Because the findings are classic and the diagnosis is so important, Boerhaave

syndrome justifiably receives emphasis in educational programs for emergency

physicians, internists, radiologists, and surgeons alike.

390-391. The answers are 388-c, 389-d. (Schwartz, 7/e, pp 159-161.)

The incidence of myocardial contusion is about 25% in patients with severe

Cardiothoracic Problems Answers 255

blunt injury to the chest. The injury occurs as a result of direct compression

of the heart between the sternum and the vertebral column. The right ventricle,

being the most anterior portion of the heart, is the most commonly

injured portion. The blow causes extravasation of blood into the

myocardium and results in a progressive loss of ventricular compliance and

decreased cardiac output, which usually peaks by 8-24 h after the injury.

392-393. The answers are 392-c, 393-b. (Schwartz, 7/e, pp 706-709.)

Chylothorax may occur after intrathoracic surgery, or it may follow malignant

invasion or compression of the thoracic duct. Intraoperative recognition

of a thoracic duct injury is managed by double ligation of the duct.

Direct repair is impractical owing to the extreme friability of the thoracic

duct. Injuries not recognized until several days after intrathoracic surgery

frequently heal following the institution of a low-fat diet and either

repeated thoracentesis or tube thoracostomy drainage. A low-fat, mediumchain

triglyceride diet often reduces the flow of chyle. Failure of this treatment

modality requires direct surgical ligation of the thoracic duct. This is

best approached from below the diaphragm, regardless of the site of

intrathoracic injury.

394. The answer is d. (Schwartz, 7/e, pp 771-780.) The boundaries of

the mediastinum are the thoracic inlet, the diaphragm, the sternum, the

vertebral column, and the pleura bilaterally. The mediastinum itself is

divided into three portions delineated by the pericardial sac: the anterosuperior

and posterosuperior regions are in front of and behind the sac,

respectively, while the middle region designates the contents of the pericardium.

Mediastinal masses occur most frequently in the anterosuperior

region (54%) and less often in the posterosuperior (26%) and middle

(20%) regions. Cysts (either pericardial, bronchogenic, or enteric) are the

most common tumors of the middle region; neurogenic tumors are the

most common (40%) of the primary tumors of the posterior mediastinum.

The primary neoplasms of the mediastinum in the anteroposterior region

are thymomas (31%), lymphomas (23%), and germ-cell tumors (17%).

More commonly, though, a mass in this area represents the substernal

extension of a benign substernal goiter. Diagnosis may be made by visualization

of an enhancing structure on CT; radioactive iodine scanning is useful

in management because it may make the diagnosis if the mediastinal

tissue is functional and will also document the presence of functioning cer-

256 Surgery

vical thyroid tissue to prevent removal of all functional thyroid tissue during

mediastinal excision.

395. The answer is e. (Schwartz, 7/e, pp 1719-1721.) This radiograph of

a child with a scaphoid abdomen and respiratory disease is characteristic of

a congenital diaphragmatic hernia. These defects are posterolateral and

occur from failure of the embryologic diaphragm to fuse between the

eighth and twelfth weeks of intrauterine life. The size of the defect does not

correlate with the symptoms. Even a large diaphragmatic hernia can be

missed on prenatal sonogram if the abdominal contents have slipped back

into the abdomen at the time of the study. Hernias of Morgagni are anteromedial

and do not present as emergencies at birth. Any abdominal organ-

pancreas, kidney, small and large intestine, stomach, liver, or spleen-can

herniate into the chest. The abdominal organ acts as a space-occupying

lesion and retards growth of the lung, which results in pulmonary hypoplasia.

Respiratory problems at birth stem from primary pulmonary hypertension,

the consequence of hypoplasia, rather than from compression of the

lung by abdominal contents. Most experts recommend stabilizing the pulmonary

hypertensive crisis medically or with extracorporeal membrane

oxygenation (ECMO) prior to attempting repair.

396. The answer is e. (Schwartz, 7/e, p 1125.) Pharyngoesophageal

(Zenker's) diverticulum is an outpouching of mucosa between the lower

pharyngeal constrictor and the cricopharyngeus muscles. It is thought to

result from an incoordination of cricopharyngeal relaxation with swallowing.

These diverticula occur in elderly patients and more commonly on the

left. The typical patient presents with complaints of dysphagia, weight loss,

and choking. Other patients present with the effects of repeated aspiration,

pneumonia, or chronic cough. A mass is sometimes palpable and a gurgle

may be heard. Treatment is excision and division of the cricopharyngeus

muscle, which can be done under local anesthesia in a cooperative patient.

Esophagoscopy is dangerous because the blind pouch is easily perforated.

Even though the pouch may extend down into the mediastinum, the origin

of the diverticulum is at the cricopharyngeus muscle near the level of the

bifurcation of the carotid artery.

397. The answer is a. (Schwartz, 7/e, pp 865-867.) Coronary artery

bypass surgery was developed in the late 1960s and is now being regularly

Cardiothoracic Problems Answers 257

performed. Indications for surgery include chronic disabling angina and

crescendo (or preinfarction) angina. Cardiac catheterization with selective

coronary angiography defines the extent of disease, which generally is

localized to the proximal segments of the vessels. Operative mortality is

about 2%, and relief of angina is obtained in most affected patients.

Patients with left main coronary artery disease as well as those with triple

vessel disease and ventricular dysfunction have an increased longevity following

successful bypass. Data regarding extension of life in other groups

is conflicting. Coronary artery bypass is not indicated for congestive heart

failure unless this condition is ischemic in origin and angiography identifies

disease amenable to surgical revascularization.

398. The answer is c. (Schwartz, 7/e, pp 977-980.) The thoracic outlet

syndrome designates a symptom complex whose precise cause is

unknown. It is felt to result from compression of the brachial plexus or

subclavian vessels, or both, in the anatomic space bounded by the first rib,

the clavicle, and scalene muscles. Since objective determinants of disease

may be lacking or imprecise, the diagnosis often is established by resectional

surgery. Carpal tunnel syndrome (compression of the median nerve

as it passes through the carpal tunnel of the wrist) and cervical disk disease

are the two entities most commonly confused with the thoracic outlet syndrome,

whose symptoms and signs include pain, paresthesias, edema,

venous congestion, and digital vasospastic changes. Positional dampening

or obliteration of the radial pulse is an unreliable finding since it is present

in up to 70% of the normal population. Neurologic abnormalities may be

documented by nerve conduction studies. Angiographic studies are often

negative. Conservative management, which generally should precede

surgery, consists of an exercise program to strengthen shoulder girdle muscles

and decrease shoulder droop. Operative treatment includes division of

the scalenus anticus and medius muscles, first rib resection, cervical rib

resection, or a combination of all three.

399. The answer is c. (Cummings, Ann Thorac Surg 37:511-518, 1984.)

This x-ray demonstrates an air-fluid level in the pericardium. Pneumopericardium

can result from penetrating or blunt chest trauma, spontaneous formation

of gas from anaerobic bacteria, iatrogenic causes, or direct extension

into the pericardium by diseased adjacent organs. In this case, a patient with

a high gastrojejunostomy developed a recurrent ulcer that eroded through

258 Surgery

the diaphragm and into the pericardium and thus caused a pneumopyopericardium.

Often these patients have an unrecognized gastrinoma (Zollinger-

Ellison syndrome) and therefore continue to have peptic ulcer disease

despite aggressive surgical therapy. The presence of pneumopyopericardium

as seen in this chest film should be treated as a surgical emergency in this setting.

Inability to demonstrate a fistula on roentgenographic investigation

should not preclude the diagnosis of this entity. If the cause of the pericardial

fluid is not clearly diagnosed by available means, then a pericardial window

should be performed for diagnostic as well as therapeutic reasons. The pericardial

sac should be irrigated and adequate continuing drainage should be

ensured. Although myocardial infarction may result in pericardial effusion or

(rarely) tamponade, it does not cause pneumopericardium.

400. The answer is b. (Schwartz, 7/e, p 1913.) Pancoast tumors are

peripheral bronchogenic carcinomas that produce symptoms by involvement

of extrapulmonary structures adjacent to the cupula. These structures

include the nerve roots of C8 and T1, as well as the sympathetic trunk.

Interruption of the cervical sympathetic trunk leads to miosis, ptosis, and

anhidrosis, the triad of signs that constitutes Horner syndrome. Involvement

of the nerve roots causes pain along the corresponding dermatomes.

The peripheral location of the neoplasm makes pulmonary signs, such as

atelectasis, cough, and hemoptysis, unlikely.

401. The answer is c. (Schwartz, 7/e, pp 802-805.) Coarctation of the

aorta is a congenital anomaly that usually causes aortic stenosis just distal

to the left subclavian artery in the area of the ligamentum arteriosum. Collateral

circulation develops around the obstruction by way of intercostal

vessels and accounts for the classic x-ray appearance of rib notching. Without

surgery, the average life span is about 30-40 years with eventual death

from cardiac failure, rupture of aortic aneurysms or of a cerebral artery, and

bacterial endocarditis. Surgery can be accomplished with less than a 1%

mortality and should be performed around 5 years of age, when the aorta

is sufficiently large to be operable but before it becomes fibrotic and calcified,

conditions that increase the technical difficulty of the operation. Claudication

is not a common feature of this disorder.

402. The answer is c. (Schwartz, 7/e, p 759-761.) Bronchial carcinoid

tumors rarely produce the carcinoid syndrome. They are slow-growing,

Cardiothoracic Problems Answers 259

infrequently metastatic tumors that histologically resemble the carcinoid

tumors of the small intestine. Over 80% arise in the major proximal

bronchi, and their intraluminal growth is responsible for the frequent presentation

of bronchial obstruction. The only therapy for this lesion is operative

resection, because neither the primary tumor nor the infrequent

lymph node metastasis is radiosensitive. The low malignant potential for

this lesion is reflected by a long-term survival rate that approaches 90%.

403. The answer is c. (Schwartz, 7/e, pp 758-759.) "Coin lesions" have

been defined as densities within the lung field of up to 4 cm, usually round,

and free of signs of infections such as cavitation or surrounding infiltrates.

Malignant solitary lesions may contain flecks of calcification, but heavy calcification

or concentric rings of calcium generally suggest a benign etiology.

The differential diagnosis for coin lesions includes primary pulmonary carcinomas,

metastatic carcinomas to the lung, benign lung neoplasms such as

chondromas and other benign lung processes such as granulomas, or vascular

abnormalities such as arteriovenous malformations. The likelihood

that a coin lesion is a primary lung malignancy increases linearly with age:

15% at age 40, 40% at age 55, 70% at age 75. With the diminishing frequency

of granulomatous disease and the continued rise in lung cancers,

such lesions should be removed because there is an excellent chance of

cure if the lesion is a primary lung malignancy. If the patient has had a previous

malignancy of tissue other than lung, the likelihood that the lesion

represents a metastatic lesion depends on the tissue of origin of the previous

malignancy. If all patients with a history of prior cancer are considered

together, a lung nodule will be a new lung primary in 60%, a metastatic

lesion in 25%, and a benign process in 15% of cases. However, 80% of solitary

lesions in patients with melanoma represent metastatic disease, while

only 40% of lesions in patients with breast cancer represent metastasis, and

solitary lesions in patients with colon carcinoma are equally likely to be

metastatic or primary lung cancers.

404. The answer is d. (Schwartz, 7/e, pp 759-764.) The term hamartoma

denotes a tumor that arises from the disorganized arrangement of tissues normally

found in an organ. Pulmonary hamartomas are solitary lesions of the

pulmonary parenchyma and generally appear as asymptomatic peripheral

nodules; they represent the most common benign epithelial and mesodermal

elements. Pulmonary chondromas consist of mesodermal elements alone

260 Surgery

and arise centrally in major bronchi, where they produce signs and symptoms

of bronchial obstruction. Fibromas are the most common benign mesodermal

tumors found in the lung; they may occur either within the lung

parenchyma or, more commonly, within the tracheobronchial tree. Osteochondromas

are lesions of bone and are not found in the lung. Aspergillomas

are due to infection with the fungus Aspergillus and most commonly appear

in the upper lobes as oval, friable, necrotic gray or yellow masses often surrounded

by evidence of preexisting parenchymal lung disease.

405. The answer is b. (Schwartz, 7/e, pp 1126-1127.) The x-rays presented

in the question are consistent with a diagnosis of achalasia, a motility

disorder of the esophagus that usually affects persons between 30 and

50 years of age. The x-rays show a classic beaklike narrowing of the distal

esophagus and a large, dilated esophagus proximal to the narrowing. The

diagnosis of achalasia is generally suspected on the basis of barium x-rays,

but, because other esophageal disorders may mimic the condition, an

esophageal motility study is usually required to confirm the diagnosis. The

characteristic findings on a motility study are small-amplitude, repetitive,

simultaneous postdeglutition contractions in the body of the esophagus,

failure of the lower esophageal sphincter to relax after deglutition, and a

higher-than-normal pressure in the body of the esophagus. Carcinoma of

the esophagus is approximately 7 times more frequent in persons who have

achalasia than in the general population. Patients usually describe difficulty

in swallowing solids and liquids.

406-410. The answers are 406-d, 407-a, 408-b, 409-e, 410-c.

(Greenfield, 2/e, pp 1575-1577, 1506. Sabiston, 15/e, p 2139.) Myocarditis,

aortitis, and pericarditis all have been described in association with Reiter

syndrome; the original description included conjunctivitis, urethritis, and

arthralgias. Although its cause is unknown, Reiter syndrome is associated

with HLA-B27 antigen, as are aortic regurgitation, pericarditis, and ankylosing

spondylitis.

Short stature, webbed neck, low-set ears, and epicanthal folds are the

classic features of patients who have Turner syndrome. Persons affected by

the syndrome, which is commonly linked with aortic coarctation, are genotypically

XO. However, females and males have been described with normal

sex chromosome constitutions (XX, XY) but with the phenotypic

abnormalities of Turner syndrome. Additional cardiac lesions associated

Cardiothoracic Problems Answers 261

with Turner syndrome include septal defects, valvular stenosis, and anomalies

of the great vessels.

The Argyll Robertson pupil, a pupil that constricts with accommodation

but not in response to light, is characteristic of central nervous system

syphilis and is associated with vascular system manifestations of this disease.

Treponema pallidum invades the vasa vasorum and causes an obliterative

endarteritis and necrosis. The resulting aortitis gradually weakens the

aortic wall and predisposes it to aneurysm formation. Once an aneurysm

has formed, the prognosis is grave.

Massive isolated tricuspid regurgitation produces a markedly elevated

venous pressure, usually manifested by a severely engorged (often pulsating)

liver. If the venous pressure is sufficiently elevated, exophthalmos may

result. Tricuspid regurgitation of rheumatic origin is almost never an isolated

lesion, and the major symptoms of patients who have rheumatic heart

disease are usually attributable to concurrent left heart lesions. Bacterial

endocarditis from intravenous drug abuse is becoming an increasingly

important cause of isolated tricuspid regurgitation.

A Quincke pulse, which consists of alternate flushing and paling of the

skin or nail beds, is associated with aortic regurgitation. Other characteristic

features of the peripheral pulse in aortic regurgitation include the waterhammer

pulse (Corrigan pulse, caused by a rapid systolic upstroke) and

pulsus bisferiens, which describes a double systolic hump in the pulse contour.

The finding of a wide pulse pressure provides an additional diagnostic

clue to aortic regurgitation.

411-415. The answers are 411-c, 412-a, 413-d, 414-b, 415-e.

(Schwartz, 7/e, pp 771-780.) Neuroblastoma, a highly malignant tumor of

children, occurs along the distribution of the sympathetic nervous system.

It is derived from ganglion cell precursors and thus usually causes an

increased excretion of catecholamines and their metabolites. Because of its

propensity to metastasize to bone and its histological resemblance to

Ewing's sarcoma, its association with elevated catecholamine levels is a

major factor in differential diagnosis.

Renal stones occur in about half the cases of hyperparathyroidism.

Other disorders sometimes associated with hyperparathyroidism include

peptic ulcers, pancreatitis, and bone disease; central nervous system symptoms

may also arise in connection with hyperparathyroidism. Occasionally,

parathyroid adenomas occur in conjunction with neoplasms of other

endocrine organs, a condition known as multiple endocrine adenomatosis.

262 Surgery

Cystic teratomas, or dermoid cysts, include endodermal, ectodermal,

and mesodermal elements. They are characteristically cystic and contain

poorly pigmented hair, sebaceous material, and occasionally teeth. Dermoid

cysts occur in the gonads and central nervous system, as well as in

the mediastinum. With rare exceptions, these lesions are benign.

Thymomas are associated with myasthenia gravis, agammaglobulinemia,

and red blood cell aplasia. These tumors are typically cystic and occur

in the anterior mediastinum. Most thymic lesions associated with myasthenia

gravis are hyperplastic rather than neoplastic.

Persons afflicted with Hodgkin's disease have impaired cell-mediated

immunity and are particularly susceptible to mycotic infections and tuberculosis.

The severity of the immune deficiency correlates with the extent of

the disease. The nodular sclerosing variant of primary mediastinal

Hodgkin's disease is the most common type.

416-420. The answers are 416-h; 417-f, j; 418-c; 419-a, d; 420-g.

(Schwartz, 7/e, pp 103-105, 114.) Epinephrine is a circulating endogenous

catecholamine, released mainly from the adrenal medulla, whose effects are

mediated by binding of free circulating hormone to β1- and β2-receptors,

with lesser effects on α-adrenoreceptors. Norepinephrine is also endogenously

produced, but acts locally through release at nerve synapses. Isoproterenol

is a synthetic sympathomimetic that acts as a pure beta agonist,

resulting in profound vasodilator and chronotropic effects. Dopamine is an

endogenous catecholamine that is released into the circulation and acts by

binding to β1-receptors as well as specific dopamine receptors in the renal,

mesenteric, coronary, and intracerebral vascular beds, causing vasodilation.

Dobutamine is a synthetic sympathomimetic structurally related to

dopamine and is a potent inotrope but possesses only small chronotropic

properties. Amrinone and milrinone are bipyridine derivatives that induce

vasodilation and inotropy via inhibition of phosphodiesterase, thereby

enhancing intracellular concentrations of cyclic AMP. Digitalis exerts positive

inotropic effects by inhibition of Na-K-activated ATPase, resulting in

increased intracellular sodium concentrations, which lead to increased

intracellular calcium concentrations. Digitalis is also used in the management

of arrhythmias, most commonly atrial fibrillation. Nitroprusside and

nitroglycerin are systemic vasodilators, and while nitroprusside causes balanced

arterial and venous dilation, the effects of nitroglycerin are less pronounced

in the arterial than the venous system, often resulting in venous

pooling.

Cardiothoracic Problems Answers 263

This page intentionally left blank.

PERIPHERALVASCULAR

PROBLEMS

Questions

DIRECTIONS: Each item below contains a question or incomplete

statement followed by suggested responses. Select the one best response to

each question.

265

421. Patients with phlebographically

confirmed deep vein thrombosis

of the calf

a. Can expect asymptomatic recovery

if treated promptly with anticoagulants

b. May be effectively treated with lowdose

heparin

c. May be effectively treated with

pneumatic compression stockings

d. May be effectively treated with

acetylsalicylic acid

e. Are at risk for significant pulmonary

embolism

422. For the first 6 h following

surgical repair of a leaking abdominal

aortic aneurysm in a 70-yearold

man, oliguria (total urinary

output of 25 mL since the operation)

has become a concern. Of

most diagnostic help would be

a. Renal scan

b. Aortogram

c. Left heart preload pressures

d. Urinary sodium concentration

e. Creatinine clearance

423. Following aortic reconstruction,

the viability of the sigmoid

colon can most reliably be evaluated

by

a. Intraoperative measurement of

inferior mesenteric artery stump

pressure

b. Intraoperative Doppler arterial signal

in the sigmoid mesentery

c. Intraoperative observation of bowel

peristalsis

d. Postoperative sigmoidoscopy

e. Postoperative barium enema

Terms of Use

424. A 25-year-old woman presents to the emergency room complaining

of redness and pain in her right foot up to the level of the midcalf. She

reports that her right leg has been swollen for at least 15 years, but her left

leg has been normal. On physical examination she has a temperature of

39°C (102.2°F). The left leg is normal. The right leg is not tender, but it is

swollen from the inguinal ligament down and there is an obvious cellulitis

of the right foot. The patient's underlying problem is

a. Popliteal entrapment syndrome

b. Acute arterial insufficiency

c. Primary lymphedema

d. Deep venous thrombosis

e. None of the above

425. A 76-year-old woman is admitted with back pain and hypotension.

A CT scan (shown below) is obtained, and the patient is taken to the operating

room. Three days after resection of a ruptured abdominal aortic

aneurysm, she complains of severe, dull left flank pain and passes bloody

mucus per rectum. The diagnosis that must be immediately considered is

a. Staphylococcal enterocolitis

b. Diverticulitis

c. Bleeding AV malformation

d. Ischemia of the left colon

e. Bleeding colonic carcinoma

266 Surgery

426. The angiogram depicted below is most typical of the patient whose history

includes

a. Cigarette smoking

b. Alcoholism

c. Hypertension

d. Diabetes

e. Type I hyperlipoproteinemia

Peripheral Vascular Problems 267

427. An 80-year-old man is found

to have an asymptomatic abdominal

mass. An arteriogram is obtained,

which is pictured below.

This patient should be advised that

Items 428-429

428. A 75-year-old man is found

by his internist to have an asymptomatic

carotid bruit. The best initial

diagnostic examination would be

a. Transcranial Doppler studies

b. Doppler ultrasonography (duplex)

c. Spiral CT angiography

d. Arch aortogram with selective

carotid artery injections

e. Magnetic resonance arteriogram

(MRA)

268 Surgery

a. Surgery should be performed, but a

mortality of 20% is to be anticipated

b. Surgery should be performed only

if symptoms develop

c. Surgery will improve his 5-year

survival

d. Surgery this extensive should not

be performed in a patient of his age

e. Surgery should be performed only

if follow-up ultrasound demonstrates

increasing size

429. An arteriogram on the above

patient is shown below. The patient

has mild hypertension and mild

COPD. The current recommendation

for this man would be

430. A 55-year-old man with

recent onset of atrial fibrillation presents

with a cold, pulseless left

lower extremity. He complains of

left leg paresthesia and is unable to

dorsiflex his toes. Following a successful

popliteal embolectomy, with

restoration of palpable pedal pulses,

the patient is still unable to dorsiflex

his toes. The next step in management

should be

a. Electromyography (EMG)

b. Measurement of anterior compartment

pressure

c. Elevation of the left leg

d. Immediate fasciotomy

e. Application of a posterior splint

431. Conservative management

rather than reconstructive arterial

surgery is generally recommended

for patients with which of the following

symptoms or signs of arterial

insufficiency?

a. Ischemic ulceration

b. Ischemic neuropathy

c. Claudication

d. Nocturnal foot pain

e. Toe gangrene

Peripheral Vascular Problems 269

a. Medical therapy with aspirin 325

mg/day and medical risk factor

management

b. Medical therapy with warfarin

c. Angioplasty of the carotid lesion

followed by carotid endarterectomy

if the angioplasty is unsuccessful

d. Carotid endarterectomy

e. Medical risk factor management

and carotid endarterectomy if neurologic

symptoms develop

432. Correct statements concerning

antiplatelet therapy include

a. Aspirin has been shown to be an

effective antiplatelet agent

b. Most antiplatelet agents work by

enhancing prostaglandin synthesis

c. Antiplatelet agents have not been

shown to increase patency rates of

coronary artery bypass grafts

d. Aspirin can be used to treat deep

venous thrombophlebitis

e. The antiplatelet effect of aspirin

will last for the life of the platelet,

which is generally 20-25 days

433. The subclavian steal syndrome

is associated with which of

the following hemodynamic abnormalities?

a. Antegrade flow through a vertebral

artery

b. Venous congestion of upper

extremities

c. Occlusion of the carotid artery

d. Occlusion of the vertebral artery

e. Occlusion of the subclavian artery

434. Symptoms or signs of atherosclerotic

occlusive disease of the

bifurcation of the abdominal aorta

(Leriche syndrome) include

a. Claudication of the buttock and

thigh

b. Causalgia of the lower leg

c. Retrograde ejaculation

d. Gangrene of the feet

e. Dependent rubor of the feet

435. Among patients with suspected

(occult) coronary artery

disease, the occurrence of postoperative

ischemic cardiac events

following peripheral vascular

surgery correlates closely with

abnormal preoperative

a. Exercise stress testing

b. Gated blood pool studies that

demonstrate an ejection fraction of

50% or less

c. Coronary angiography

d. Dipyridamole-thallium imaging

e. Transesophageal echocardiography

436. A 64-year-old man is admitted

14 mo following a femoropopliteal

bypass graft procedure with a

cold foot and no graft pulse. Urokinase

infusion is begun. Which of

the following statements regarding

management is true?

a. Clot lysis is accomplished in 25%

of patients

b. After successful clot lysis, surgical

revision of the opened graft should

be considered only if early reocclusion

occurs

c. With optimal treatment, a 20%

reocclusion rate is expected within

1 year

d. Urokinase is less successful in

lysing acute thromboses of prosthetic

grafts than those of vein

grafts

e. Streptokinase is the preferred

thrombolytic agent when treating

graft occlusions

270 Surgery

Peripheral Vascular Problems 271

437. A 60-year-old man is admitted

to the coronary care unit with a

large anterior wall myocardial

infarction. On his second hospital

day he begins to complain of the

sudden onset of numbness in his

right foot and an inability to move

his right foot. On physical examination,

the right femoral, popliteal,

and pedal pulses are no longer palpable.

Vascular consultation is

obtained. Diagnosis of acute arterial

embolus is made. Which of the

following statements concerning

this condition is true?

a. Appropriate management would be

embolectomy of the right femoral

artery under general anesthesia

b. Noninvasive hemodynamic testing

is required

c. Prophylactic exploration of the

contralateral femoral artery should

be done despite the presence of a

normal pulse

d. The source of the embolus is most

likely the left ventricle

e. Arteriography is mandatory prior

to operative intervention

438. Which of the following statements

concerning the condition

depicted on the arteriogram shown

below is true?

a. Surgery should be performed only

if the patient is symptomatic

b. Limb loss is a definite risk in the

untreated patient

c. The contralateral limb is affected in a

similar fashion in over 75% of cases

d. Embolization is unlikely

e. Bleeding into the leg is the most

common presentation

439. A 65-year-old male cigarette smoker reports onset of claudication of

his right lower extremity approximately 3 wk previously. His walking

radius is limited to three blocks before the onset of claudication. Physical

examination reveals palpable pulses in the entire left lower extremity, but

no pulses are palpable below the right groin level. Noninvasive flow studies

are obtained, which are pictured below. Which of the following statements

regarding this patient's condition is true?

a. Femoropopliteal bypass is indicated on a relatively urgent basis in order to salvage

the right leg

b. The occlusive process is in the right superficial femoral artery, with flow to the

right foot supplied by the profunda femoris artery

c. About one-half of patients with similar symptoms will ultimately require amputation

d. The occlusive process is most likely caused by embolic disease

e. The noninvasive studies suggest iliac as well as superficial femoral occlusive

disease on the right side

272 Surgery

440. Indications for placement of the device pictured in the abdominal

x-ray shown below include

a. Recurrent pulmonary embolus despite adequate anticoagulation therapy

b. Axillary vein thrombosis

c. Pulmonary embolus in a patient with a perforated duodenal ulcer

d. Pulmonary embolus due to deep vein thrombosis of the lower extremity that

occurs 2 wk postoperatively

e. Pulmonary embolus in a patient with metastatic pancreatic carcinoma

Peripheral Vascular Problems 273

441. Two days after admission to

the hospital for a myocardial infarction,

a 65-year-old man complains

of severe, unremitting midabdominal

pain. His cardiac index is 1.6.

Physical examination is remarkable

for an absence of peritoneal irritation

or distention despite the

patient's persistent complaint of

severe pain. Serum lactate is 9 (normal

less than 3). In managing this

problem you should

a. Perform computed tomography

b. Perform mesenteric angiography

c. Perform laparoscopy

d. Perform flexible sigmoidoscopy to

assess the distal colon and rectum

e. Defer decision to explore the

abdomen until the arterial lactate is

greater than 10

442. During evaluation for the

repair of an expanding abdominal

aortic aneurysm, a patient is

discovered to have a horseshoe

kidney. The optimum surgical approach

would be

a. Midline abdominal incision, preservation

of the renal isthmus

b. Midline abdominal incision, division

of the renal isthmus

c. Retroperitoneal approach, implantation

of anomalous renal arteries

d. Nephrectomy, repair of aneurysm,

chronic dialysis

e. Repair of aneurysm after autotransplantation

of the kidney into the

iliac fossa

443. Which statement regarding

contrast venography is true?

a. It is more accurate than Doppler

analysis and B-mode ultrasound

(duplex scan) at detecting thrombi

in the deep veins responsible for

pulmonary emboli

b. It identifies incompetent deep,

superficial, and perforating veins

c. It is totally noninvasive, painless,

and safe

d. It is easily performed in a vascular

laboratory or radiology suite or at

the bedside

e. It is particularly sensitive in identifying

the proximal extent of an

iliofemoral thrombus

274 Surgery

DIRECTIONS: The group of questions below consists of lettered

options followed by numbered items. For each numbered item, select the

appropriate lettered option(s). Each lettered option may be used once,

more than once, or not at all. Choose exactly the number of options

indicated following each item.

Items 444-445

The arteriogram below applies to both patients described on the following

page. For each patient, select the appropriate options.

a. Femorofemoral bypass

b. Axillofemoral bypass

c. Femoropopliteal bypass

d. Common femoral and profunda femoral endarterectomies

e. Aorto-left-iliac bypass

f. Aortobifemoral bypass

Peripheral Vascular Problems 275

444. A 52-year-old man presents

with severe pain in his left hip and

buttocks while walking about 50

yd. The pain is relieved shortly

after resting. The patient is otherwise

healthy. He claims to have

stopped smoking, after a long history

of cigarette abuse, approximately

1 year prior to presentation.

(SELECT 3 PROCEDURES)

445. A 72-year-old woman with

severe COPD that requires home

oxygen is unable to ambulate

inside her home without experiencing

severe left hip pain. She was

hospitalized 1 year ago for a viral

pneumonia and was ventilator

dependent at that time for 6 wk.

(SELECT 2 PROCEDURES)

276 Surgery

PERIPHERALVASCULAR

PROBLEMS

Answers

421. The answer is e. (Schwartz, 7/e, pp 1007-1014.) Low-dose heparin

and pneumatic compression stockings have been shown to be effective

prophylaxis against deep vein thrombosis; however, they are not effective

against established thrombosis, the treatment for which is therapeutic

heparinization. Salicylate has not been convincingly shown to have either a

prophylactic or therapeutic role in the treatment of deep vein thrombosis.

Even following prompt, aggressive treatment of deep vein thrombosis of

the calf, as many as half of affected patients will develop symptoms of

chronic venous hypertension, and a larger number will have abnormal

venous hemodynamic findings. Untreated vein thrombosis of the calf may

propagate into the larger popliteal veins and cause life-threatening pulmonary

embolism.

422. The answer is c. (Schwartz, 7/e, pp 947-948.) By far the most likely

cause of the oliguria observed in this patient is hypovolemia. Volume status

would be best assessed by floating a Swan-Ganz catheter to measure the

preload pressures in the left atrium (by inference from the pulmonary capillary

wedge pressures). Patients who have had a leaking aneurysm and

then a long, usually difficult operation with large surgical fields that collect

"third-space" fluids may be intravascularly depleted despite large volumes

of intravenous fluid and blood replacement. The proper management usually

involves titrating the cardiac output by providing as much fluid as necessary

to keep the wedge pressures near 15 mm Hg. The other studies

listed might become useful if urinary flow remains depressed after optimal

cardiac output has been achieved, but in view of the probability of hypovolemia,

they are not indicated as a first diagnostic study.

423. The answer is d. (Schroeder, Surg Gynecol Obstet 160:299-303, 1985.

Schwartz, 7/e, p 948.) Viability of the colon can be evaluated intraoperatively

by Doppler auscultation of the bowel mesentery and serosa, observa-

277

tion of bowel peristalsis, and measurement of the IMA stump pressure. A

strong, pulsatile Doppler signal in the mesentery; active sigmoid peristalsis;

a chronically occluded IMA; or a patent IMA with stump pressure greater

than 40 mm Hg presage viability of the sigmoid colon postoperatively.

However, none of these observations excludes the possibility of late sigmoid

ischemia. Serial postoperative sigmoidoscopic examination is the

best predictor of ischemic colitis and in experienced hands allows assessment

of the depth of ischemic injury before frank perforation has occurred.

Barium enema is not as accurate as sigmoidoscopy in determining depth of

injury and carries grave risks of contamination by barium and feces if perforation

occurs.

424. The answer is c. (Schwartz, 7/e, pp 1028-1030.) This patient is at

high risk for developing cellulitis of her right foot because her underlying

problem is unilateral primary lymphedema. Hypoplasia of the lymphatic

system of the lower extremity accounts for greater than 90% of patients

with primary lymphedema. If edema is present at birth it is referred to as

congenital; if it starts early in life (as in this woman) it is called praecox;

and if it appears after age 35 it is tarda. The inadequacy of the lymphatic

system accounts for the repeated episodes of cellulitis that these patients

experience. Swelling is not seen with acute arterial insufficiency or with

popliteal entrapment syndrome. Deep venous thrombophlebitis will result

in tenderness and is generally not a predisposing factor for cellulitis of the

foot.

425. The answer is d. (Brewster, Surgery 109:447-457, 1991.) The CT

scan reveals a fractured ring of calcification in the abdominal aorta with significant

density in the paraaortic area. The inferior mesenteric artery (IMA)

is always at risk in patients with the changes in the vessel wall characteristic

of abdominal aneurysms, but particularly so in the presence of rupture

and retroperitoneal dissection of blood under systemic arterial pressures.

The incidence of ischemic colitis following abdominal aortic resection is

about 2%. Blood flow to the left colon normally derives from the IMA with

collateral flow from the middle and inferior hemorrhoidal vessels. The

superior mesenteric artery (SMA) may also contribute via the marginal

artery of Drummond. If the SMA is stenotic or occluded, flow to the left

colon will be primarily dependent on an intact IMA. The IMA is usually ligated

at the time of aneurysmorrhaphy. Those patients at highest risk for

278 Surgery

diminished flow through collateral vessels are those with a history of visceral

angina, those found to have a patent IMA at the time of operation,

patients who have suffered an episode of hypotension following rupture of

an aneurysm, those in whom preoperative angiograms reveal occlusion of

the SMA, and those in whom Doppler flow signals along the mesenteric

border cease following occlusion of the IMA. Recognition of bowel

ischemia at the time of operation should be treated by reimplantation of the

IMA into the graft to restore flow.

426. The answer is a. (Schwartz, 7/e, pp 957-964.) The angiogram presented

in the question demonstrates an isolated segment of atherosclerotic

occlusion of the superficial femoral artery. Patients who have isolated

femoropopliteal disease tend to be smokers, whereas those who have isolated

tibioperoneal disease frequently are diabetic. Hypertension and

hyperlipidemia predispose to accelerated atherosclerosis. On the other

hand, type I hyperlipoproteinemia (hyperchylomicronemia), which is associated

with dramatic levels of plasma triglyceride and formation of xanthomas,

does not cause accelerated vascular disease.

427. The answer is c. (Schwartz, 7/e, pp 941-944.) Most abdominal aortic

aneurysms are asymptomatic and are discovered on palpation by a

physician. A radiograph of the abdomen is useful in demonstrating the

aneurysm if there is calcification in the walls. Ultrasound is generally the

first diagnostic procedure in confirming the presence of an aneurysm, with

arteriography being performed if the aneurysm is considered large enough

to require resection (greater than 5 cm in diameter). Recently CT scan has

been found to be useful as a preoperative study in patients suspected of

having aneurysms. Surgery should be performed despite the absence of

symptoms and can be carried out with a mortality of less than 5%. With

leaking or ruptured aneurysms, the operative mortality associated with this

emergency situation is upward of 75%. The patient's age is not a contraindication

to surgery, because several studies have demonstrated a low

mortality (less than 5%) and satisfactory long-term survival and quality of

life in elderly, even octogenarian, patients.

428. The answer is b. (Greenfield, 2/e, pp 1751-1752.) Doppler ultrasonography

(duplex) has become the best initial test for screening patients

with carotid disease. It has become a highly accurate test, often obviating

Peripheral Vascular Problems Answers 279

the need for carotid arteriography prior to carotid endarterectomy. Carotid

arteriography remains the "gold standard" when quantifying the degree of

carotid stenosis, but it is usually performed after noninvasive testing suggests

significant stenosis. Spiral CT angiography is a new noninvasive

modality that has been used to evaluate many segments of the vascular tree,

but as yet its accuracy does not approach that of standard arteriography

and it would certainly not be used in the initial evaluation of a patient with

an asymptomatic bruit. Magnetic resonance arteriography (MRA) is also a

relatively new modality that has enjoyed moderate success in the investigation

of carotid disease. Although not quite as accurate as standard arteriography,

it has been used in conjunction with the duplex as a complementary

study. Once again, because of its cost, MRA would not be used as the primary

screening modality. Transcranial Doppler studies are used to assess

the intracranial vasculature.

429. The answer is d. (Executive Committee, JAMA 273:1421-1428,

1995.) In a recent prospective, randomized, multicenter trial involving

1662 patients in a study known as the Asymptomatic Carotid Atherosclerosis

Study, patients with asymptomatic carotid artery stenosis of 60% or

greater reduction in diameter and whose general health made them good

candidates for elective surgery were found to have a significant reduction

in the 5-year risk for ipsilateral stroke with surgery compared with medically

treated cohorts (5.1 vs. 11.0%). Medically treated patients were

treated with aspirin on a daily basis. Warfarin has not been shown to be

effective in the management of patients with carotid disease. Angioplasty of

carotid stenoses is being performed in some institutions on a purely investigational

basis and to date has not replaced surgery as the treatment for

high-grade carotid stenoses.

430. The answer is d. (Greenfield, 2/e, pp 1640-1642.) This case illustrates

two (among many) conditions that lead to the anterior compartment

syndrome, namely, acute arterial occlusion without collateral inflow and

rapid reperfusion of ischemic muscle. Treatment for a compartment syndrome

is prompt fasciotomy. Assessing a compartment syndrome and proceeding

with fasciotomy are generally based on clinical judgment. Inability

to dorsiflex the toes is a grave sign of anterior compartment ischemia. EMG

studies and compartment pressure measurements would probably be

abnormal, but are unnecessary in view of the known findings and would

280 Surgery

delay treatment. Mere elevation of the leg would be an ineffective means of

relieving compartment pressure, although elevation should accompany

fasciotomy. Application of a splint has no role in the acute management of

this problem.

431. The answer is c. (Greenfield, 2/e, pp 1715-1718.) The major threat

to patients with arterial occlusive disease is limb loss. Ischemic ulceration,

neuropathy, rest pain, and gangrene represent advanced stages of arterial

insufficiency and warrant reconstructive surgery whenever clinically feasible.

Claudication, in most cases, reflects mild ischemia; the majority of

affected patients are successfully managed without surgery (only 2.5%

develop gangrene). Most will stabilize or improve with development of

increased collateral blood flow following institution of a program of daily

exercise, cessation of smoking, and weight loss. Vasodilator drugs have

been shown to have little benefit in the conservative management of intermittent

claudication.

432. The answer is a. (Willerson, Am J Cardiol 67:12A-18A, 1991.)

Aspirin exerts an antiplatelet effect that will last for the life of the platelet

(approximately 7-10 days). Patients who take aspirin will experience its

effect for 7-10 days after stopping the medication. Aspirin interferes with

platelet function by inhibiting the synthesis of thromboxane A2 and the

subsequent production of prostaglandins. The platelet does not have a

nucleus and thus cannot remanufacture the prostaglandins necessary for its

functioning. Antiplatelet agents are generally used to prevent thrombotic

and embolic events on the arterial side of the circulation. The Canadian

Cooperative Study has shown antiplatelet therapy to be effective in preventing

strokes in men with carotid artery disease, but it is not used to treat

thrombophlebitis in the deep venous system. Antiplatelet therapy has been

shown to increase graft patency rates following coronary artery bypass

grafting if the medication is started preoperatively and continued postoperatively.

433. The answer is e. (Schwartz, 7/e, pp 965, 971.) Atherosclerotic occlusion

of the subclavian artery proximal to the vertebral artery is the

anatomic situation that results in the subclavian steal syndrome. On being

subjected to exercise, the involved extremity (usually left) develops relative

ischemia, which gives rise to reversal of flow through the vertebral artery

Peripheral Vascular Problems Answers 281

with consequent diminished flow to the brain. The upper extremity symptom

is intermittent claudication. Venous occlusive disease is not a feature

of the syndrome. The operative procedure for treating the subclavian steal

syndrome consists of delivering blood to the extremity by creating either a

carotid-subclavian bypass or a subclavian-carotid transposition.

434. The answer is a. (Schwartz, 7/e, pp 957-961.) The slow progression

of aortoiliac atherosclerotic occlusive disease is usually associated with the

development of collateral flow through the lumbar branches of the aorta,

anastomosing via retroperitoneal branches of the gluteal arteries with the

profunda femoris arteries in the legs. This network of collateral vessels provides

sufficient blood flow to nourish the extremities at rest but cannot prevent

claudication of the upper and lower muscle groups of the leg during

exercise. Sexual impotence, also part of the Leriche syndrome, is believed

to be a result of bilateral stenosis or occlusion of the hypogastric (internal

iliac) arteries. Retrograde ejaculation can occur after disruption of the sympathetic

chain overlying the distal aorta and left iliac and can occur after

dissection around these vessels during vascular reconstructions. Gangrene

of the feet or toes is rarely seen unless distal embolization of atherosclerotic

material from the aorta occludes the pedal or digital arteries. Dependent

rubor is usually a sign of significant ischemia resulting from lower extremity

occlusive and not aortoiliac disease. Causalgia or reflex sympathetic

dystrophy is a disorder of the sympathetic nervous system that can affect

the upper or lower extremities.

435. The answer is d. (Boucher, N Engl J Med 312:389-394, 1985. Pasternack,

Circulation 72:13-17, 1985.) The occurrence of perioperative ischemic

cardiac events among patients undergoing peripheral vascular reconstruction

has been found to correlate with gated blood pool ejection fractions of

35% or less and with reversible perfusion defects (thallium redistribution)

on dipyridamole-thallium imaging. Ischemic rest pain or early onset of

claudication after minimal exercise limits the effectiveness of stress testing

as a screening procedure for occult coronary artery disease in this group of

patients. Screening coronary angiography, followed by angioplasty or

bypass of asymptomatic lesions, had an adverse effect on patient survival in

a large prospective study of patients who had peripheral vascular surgery.

Transesophageal echocardiography has no role in the preoperative screening

of peripheral vascular patients.

282 Surgery

436. The answer is c. (Belkin, Surgery 212:769-773, 1986. Eisbud, Am J

Surg 160:160-165, 1990.) Management of acute graft occlusion must

include both reestablishment of peripheral perfusion and correction of any

underlying hemodynamic problem. Urokinase is associated with fewer

allergic reactions than streptokinase and is the preferred thrombolytic

agent. Treatment results in total clot lysis in 75% of patients. However, high

reocclusion rates are observed (20% within 1 year) even if angioplasty or

anastomotic revision is performed after successful lysis. Without surgical

revision following clot lysis, a 50% reocclusion rate is expected within 3

mo. Urokinase has proved equally successful in opening both vein and

prosthetic graft thromboses.

437. The answer is d. (Schwartz, 7/e, pp 953-954.) The heart is the most

common source of arterial emboli and accounts for 90% of cases. Within

the heart, sources include diseased valves, endocarditis, the left atrium in

patients with unstable atrial arrhythmias, and mural thrombus on the wall

of the left ventricle in patients with a myocardial infarction. The diagnosis

in this patient is clear, and therefore neither noninvasive testing nor arteriography

is indicated. Arteriography in fact may also prove to be too stressful

for a patient undergoing an acute myocardial infarction. Embolectomy

of the femoral artery can be performed under local anesthesia with minimal

risk to the patient. Emboli typically lodge in one femoral artery; contralateral

exploration is not indicated in the absence of signs or symptoms. One

should always prepare the contralateral groin in case flow is not restored

via simple thrombectomy and femoral-femoral bypass is needed to provide

inflow to the affected limb.

438. The answer is b. (Schwartz, 7/e, pp 949-950.) Popliteal aneurysms

are usually due to atherosclerosis, are bilateral 25% of the time, and require

excision even if asymptomatic. Because of the risk of embolization

(60-70%) and thrombosis with resultant gangrene, as well as the lesser risk

of rupture, all of which lead to substantial likelihood of limb loss, even relatively

small, asymptomatic aneurysms should be excised when discovered.

Rupture of the aneurysm can occur but is an uncommon presentation

compared with embolization.

439. The answer is b. (Schwartz, 7/e, pp 961-964.) This patient has occlusion

of the right superficial femoral artery caused by atherosclerosis, and

Peripheral Vascular Problems Answers 283

this is confirmed by both the physical examination and the flow study findings,

which indicate a sharp decrease in the blood pressure below the level

of the common femoral artery. Fewer than 10% of patients with claudication

progress to gangrene and the need for amputation. Operative therapy would

not be suggested at this time because it is quite likely that with cessation of

cigarette smoking and adherence to an exercise program, the patient could

markedly improve his walking radius as collateral vessels enlarge to deliver

more blood to the affected tissues. Operative therapy (femoropopliteal

bypass) would be indicated at this time in this patient only if symptoms of

rest pain or ischemic ulceration were present. Physical examination and

flow studies indicate disease distal to the aortoiliac distribution.

440. The answer is a. (Schwartz, 7/e, p 1014.) The Greenfield filter pictured

on the x-ray is used to interrupt migration of emboli to the lungs

from the veins below the level of the filter. It is indicated in patients who

sustain a recurrent pulmonary embolus despite adequate anticoagulant

therapy or in patients with pulmonary emboli who cannot receive anticoagulants

because of a contraindication (e.g., bleeding ulcer, intracranial

hemorrhage). The filter is not used in patients who sustain a single pulmonary

embolus. It is placed in the inferior vena cava just below the renal

veins and therefore would not be effective for emboli that arise cephalad to

its position. Despite the hypercoagulable state seen in some patients with

metastatic pancreatic cancer, anticoagulation can still be used as a first-line

defense.

441. The answer is b. (Schwartz, 7/e, pp 966-968.) Abdominal pain out

of proportion to findings on physical examination is characteristic of

intestinal ischemia. The etiology of ischemia may be embolic or thrombotic

occlusion of the mesenteric vessels or nonocclusive ischemia due to a low

cardiac index or mesenteric vasospasm. Differentiation among these etiologies

is best made by mesenteric angiography. While not without serious

risks, angiography also offers the possibility of direct infusion of vasodilators

into the mesenteric vasculature in the setting of nonocclusive

ischemia. This patient, with a recent myocardial infarction and a low cardiac

index, is at risk for embolism of clot from a left ventricle mural thrombus

as well as "low-flow" mesenteric ischemia. If embolism or thrombosis

is found angiographically (usually involving the superior mesenteric

artery), operative embolectomy or vascular bypass is indicated to restore

284 Surgery

flow. If occlusive disease cannot be demonstrated, efforts should be made

to simultaneously increase cardiac output with inotropic agents and dilate

the mesenteric vascular bed by angiographic instillation of papaverine,

nitrates, or calcium channel blockers. Computed tomography is not helpful

in delineating the cause of intestinal ischemia because it does not provide

a sufficiently detailed image of the mesenteric vessels. Laparoscopy

might secure the diagnosis of intestinal ischemia, but requires administering

general anesthesia and would shed no light on the etiology of this

patient's problem. Flexible sigmoidoscopy, while useful in patients with

ischemic colitis, has no role in the workup of mesenteric ischemia, which

primarily involves the small intestine and right colon. Serum lactate is

helpful in raising the suspicion of intestinal ischemia, but no absolute level

should be used to decide whether or not to explore a patient.

442. The answer is c. (O'Hara, J Vasc Surg 17:940-947, 1993.) A horseshoe

kidney is a fused kidney that occupies space on both sides of the vertebral

column. The fusion is ordinarily at the lower poles with the isthmus

anterior to the aorta. The ureters run anterior to the isthmus and the kidney

frequently has an anomalous blood supply. The arterial supply to the

kidney is highly variable with vessels arising not only from the normal

position in the aorta but also from a variable number of accessory segmental

end-arteries from the lower aorta and iliac arteries. Most cases of

abdominal aortic aneurysm associated with a horseshoe kidney can be successfully

resected, but these anomalies make the repair challenging. When

the horseshoe kidney is recognized preoperatively, an arteriogram helps to

define the vascular anatomy. The preferred operative approach is then via a

retroperitoneal dissection. This allows the kidney and its collecting system

to be swept anteromedially and provides relatively unobstructed access to

the aneurysm. All anomalous renal arteries should be implanted into the

graft after the aneurysm sac is opened since the proportionate contribution

from each may be hard to determine.

The renal isthmus and collecting system restrict access to the

aneurysm and make the anterior approach less desirable. Though division

of the isthmus can be accomplished, there is high risk of calyceal or

ureteral injury. Given the numerous arterial, venous, and collecting system

anomalies, autotransplantation of the kidney is not a good option. The

presence of the fresh intravascular foreign body (aortic graft) contraindicates

dialysis because of the excessive risk of infecting the graft.

Peripheral Vascular Problems Answers 285

443. The answer is b. (Schwartz, 7/e, p 1009.) Doppler analysis and

B-mode ultrasonography (duplex scan) has virtually replaced venography

as the first diagnostic test in the evaluation of deep venous thrombosis. The

duplex scanning device is portable and therefore the study is easily performed

at the bedside, in a vascular laboratory, or in a radiology suite. It is

completely noninvasive, painless, and safe. Venography, however, must be

performed in a radiology suite, and requires the use of an intravenous contrast

medium that is painful upon injection and is itself thrombogenic.

Incompetent deep, superficial, and perforating veins can be accurately

identified by either venography or duplex scan. Both venography and

duplex scan are highly accurate in diagnosing deep venous thrombi that

may result in pulmonary embolism. But, significantly, contrast venography

does not provide information regarding the proximal extent of an

iliofemoral thrombus when it fails to fill the deep femoral system due to

total occlusion by blood clot.

444-445. The answers are 444-a, e, f; 445-a, b. (Schwartz, 7/e, pp

957-961.) The arteriogram shown demonstrates a left iliac artery occlusion.

In a patient with severe symptoms that are interfering with his lifestyle,

intervention is indicated. In the young healthy patient with iliac artery

occlusive disease, when angioplasty is not a treatment option, femorofemoral

and aortoiliac bypasses offer excellent long-term relief. Femorofemoral

bypass offers the additional benefit of not disturbing sexual

function. Both bypasses provide similar long-term patencies. Aortobifemoral

bypass, while clearly the most risky of the treatment options

offered, provides the best long-term patency.

In the elderly patient with severe COPD, so-called extraanatomic

bypasses (femorofemoral or axillofemoral bypasses) offer fair long-term

patencies while not subjecting the patient to the risks of general anesthesia.

286 Surgery

UROLOGY

Questions

DIRECTIONS: Each item below contains a question or incomplete

statement followed by suggested responses. Select the one best response to

each question.

287

446. Initial management of a patient

who has a flaccid neurogenic

bladder may include which of the

following measures?

a. Surgical bladder augmentation

b. Self-catheterization

c. Supravesical urinary diversion

d. Limiting fluid intake to less than

300 mL/day

e. Transurethral resection of the bladder

neck

447. Which of the following statements

regarding hypospadias is

correct?

a. It is often associated with chordee

(ventral curvature of the penis)

b. It is associated with undescended

testes in more than 50% of cases

c. It is a rare fusion defect of the posterior

male urethra

d. It occurs sporadically, without evidence

of familial inheritance

e. The most common location is

penoscrotal

448. The recommended treatment

for stage A (superficial and submucosal)

transitional cell carcinoma of

the bladder is

a. Local excision

b. Radical cystectomy

c. Radiation therapy

d. Topical (intravesicular) chemotherapy

e. Systemic chemotherapy

449. A 36-year-old man presents

to the emergency room with renal

colic. A radiograph reveals a 1.5-

cm stone. Which of the following

statements regarding this disorder

is correct?

a. Conservative treatment including

hydration and analgesics will not

result in a satisfactory outcome

b. Serial kidney, ureter, bladder (KUB)

radiographs should be used to follow

this patient

c. The urinalysis will nearly always

reveal microhematuria

d. When the acute event is correctly

treated, this disease seldom recurs

e. Elevated BUN and creatinine are

expected

Terms of Use

450. Optimal management of bilateral

undescended testicles in an

infant is

a. Immediate surgical placement into

the scrotum

b. Chorionic gonadotropin therapy

for 1 mo; operative placement into

the scrotum before age 1 if descent

has not occurred

c. Observation until the child is 2

years old because delayed descent

is common

d. Observation until age 5; if no

descent by then, plastic surgical

scrotal prostheses before the child

enters school

e. No therapy; reassurance of the parent

that full masculinization and

normal spermatogenesis are likely

even if the testicle does not fully

descend

451. Seminoma is accurately described

by which of the following

statements?

a. It is the most common type of testicular

cancer

b. Metastases to liver and bone are frequently

found

c. It does not respond to radiation

d. The 5-year survival rate approaches

50%

e. Common presentation is that of a

painful lump that transilluminates

452. A 10-year-old boy presents to

the emergency room with testicular

pain of 5 h duration. The pain was

of acute onset and woke the patient

from sleep. On physical examination,

he is noted to have a highriding,

indurated, and markedly

tender left testis. Pain is not diminished

by elevation. Urinalysis is

unremarkable. Which of the following

statements regarding the

patient's diagnosis and treatment is

true?

a. There is a strong likelihood that

this patient's father or brother has

had or will have a similar event

b. Operation should be delayed until

a technetium scan clarifies the diagnosis

c. The majority of testicles that have

undergone torsion can be salvaged

if surgery is performed within 24 h

d. If torsion is found, both testes

should undergo orchiopexy

e. The differential diagnosis includes

spermatocele

453. Genitourinary tuberculosis

in a male patient is suggested by

which of the following findings?

a. Microscopic hematuria

b. Bacteriuria without pyuria

c. Unilateral renal cysts

d. Painful swelling of the epididymis

e. Pneumaturia

288 Surgery

454. Which of the following statements

regarding carcinoma of the

prostate is true?

a. It has a higher incidence among

American blacks than other American

ethnic groups

b. A single microscopic focus of prostate

cancer discovered on transurethral

resection of the prostate

(TURP) is an indication for radical

prostatectomy

c. It arises initially in the gland's central

d. It commonly produces osteoclastic

bony metastases

e. Screening for prostate-specific antigen,

although easily done, offers

no advantage over simple rectal

examination in the detection of the

disease

455. Which of the following statements

regarding benign prostatic

hyperplasia (BPH) is true?

a. The fibrostromal proliferation of

BPH occurs mainly in the outer

portion of the gland

b. Assuming a voided volume greater

than 100 mL, a peak urine flow rate

of 30 mL/s or less is good evidence

of outflow obstruction

c. Suprapubic prostatectomy for BPH

involves enucleation of the entire

prostate and eliminates the risk of

future prostate cancer

d. Indications for surgery include

acute urinary retention and recurrent

urinary tract infections (UTIs)

e. BPH is a risk factor for the development

of prostatic cancer

456. During the course of an operation

on an unstable, critically ill

patient, the left ureter is lacerated

through 50% of its circumference.

If the patient's condition is felt to be

too serious to allow time for definitive

repair, alternative methods of

management include

a. Ligation of the injured ureter and

ipsilateral nephrostomy

b. Ipsilateral nephrectomy

c. Placement of a catheter from the

distal ureter through an abdominal

wall stab wound

d. Placement of a suction drain adjacent

to the injury without further

manipulation that might convert

the partial laceration into a complete

disruption

e. Bringing the proximal ureter up to

the skin as a ureterostomy

Urology 289

457. A pedestrian is hit by a

speeding car. Radiologic studies

obtained in the emergency room,

including a retrograde urethrogram,

are consistent with a pelvic

fracture with a rupture of the urethra

superior to the urogenital

diaphragm. Management should

consist of

a. Immediate percutaneous nephrostomy

b. Immediate placement of a Foley

catheter through the urethra into

the bladder to align and stent the

injured portions

c. Immediate reconstruction of the

ruptured urethra after initial stabilization

of the patient

d. Immediate exploration of the pelvis

for control of hemorrhage from

pelvic fracture and drainage of the

pelvic hematoma

e. Immediate placement of a suprapubic

cystostomy tube

290 Surgery

UROLOGY

Answers

446. The answer is b. (Schwartz, 7/e, pp 1759, 1768-1769.) Patients who

have a lower motor neuron lesion (flaccid neurogenic bladder) can usually

be managed by conservative measures that prevent the development of a

large residual urine volume in the bladder. These measures include intermittent

self-catheterization and scheduled voiding with increased abdominal

pressure provided by the Valsalva maneuver or manual pressure on the

abdomen. Detrusor contractions can sometimes be strengthened by

parasympathomimetic agents such as bethanechol chloride (Urecholine).

Bladder augmentation to increase capacitance, bladder neck resection to

reduce outlet obstruction, and supravesicle ureteral diversion are indicated

only in the presence of deterioration of bladder compliance or gross ureterocalyxectasis

that resists the foregoing measures and threatens the loss of

renal function or debilitating urinary incontinence. Severely restricting

fluid intake is impractical and may promote formation of calculi.

447. The answer is a. (Schwartz, 7/e, pp 1813-1815.) Hypospadias is a

common congenital anomaly of the penis resulting from incomplete development

of the anterior urethra. It occurs in about 1 in 300 live births and

is believed to have a multifactorial genetic mode of inheritance. Of those

with hypospadias, about 7% have a father with the disorder, 14% a brother,

and 20% a second family member. Hypospadias occurs in the corona in

about 75% of cases, where it is often accompanied by chordee. Undescended

testes occur in about 10% of cases of hypospadias, as do inguinal

hernias. Hypospadias in the scrotal area is associated with bilateral undescended

testes and infertility and must be differentiated from pseudohermaphroditism

and adrenogenital syndrome.

448. The answer is d. (Schwartz, 7/e, pp 1792-1793.) Bladder cancer

represents 2% of all cancers, and 90% of bladder cancers are of transitional

cell origin. It is most prevalent among men with a heavy smoking history

and is usually multifocal and superficial, even when recurrent. When the

disease is still superficial, transurethral resection of visible lesions and

291

intravesicular chemotherapy are most often recommended. More radical

surgical extirpation is reserved for advanced stages of the disease.

449. The answer is a. (Schwartz, 7/e, pp 1774-1784.) Initial management

should include hydration and analgesics. However, as the stone is larger

than 1 cm, it is unlikely to pass spontaneously, though stones smaller than

0.5 cm usually do pass spontaneously. The size of the stone also makes a

high-grade obstruction more likely; therefore an intravenous pyelogram

(IVP) must be urgently performed. A high-grade obstruction will require

nephrostomy or the passage of a ureteral stent. If the stone is completely

occluding the lumen of the ureter, the urinalysis may not show microhematuria

and thus may be misleading. Approximately 15% of patients

will have a recurrence within 1 year, and almost 50% may have a recurrence

within 4 years. Elevated BUN and creatinine are expected only in the

setting of an obstructed single functioning kidney.

450. The answer is b. (Schwartz, 7/e, pp 1744-1745, 1810-1811.) By the

second year, a testicle not in the cooler environment of the scrotal sac will

begin to undergo histologic changes characterized by reduced spermatogonia.

Testicles left longer in the undescended state not only have a higher

incidence of malignant degeneration, but are inaccessible for examination.

If a malignancy should occur, diagnosis will be delayed. There is also a

substantial psychological burden when children reach school age or are

otherwise subjected to exposure of their deformed genitalia. Gel-filled

prostheses are generally inserted when a testicle cannot be placed in the

scrotum. Close follow-up by a physician until the late teens is indicated in

all patients who have had an undescended testicle. Since these patients

may be at increased risk for malignancy throughout life, careful training

should be given in self-examination.

451. The answer is a. (Schwartz, 7/e, pp 1794-1795.) Seminomas tend to

grow slowly and metastasize late. They usually present as a nonpainful

lump that does not transilluminate. They represent about 40% of malignant

testicular tumors; embryonal cell carcinoma and teratocarcinoma

each represent about 25%. Because most tumors have mixed elements,

they are usually classified according to the most malignant cell type

encountered, whatever the predominant cell type. When metastases occur,

they are usually along the regional lymphatic drainage pathways to the

292 Surgery

iliac, aortic, and renal lymph nodes. Because of their slow growth and

radiosensitivity, seminomas are associated with a 90% 5-year survival rate.

Therapy generally consists of removing the affected testis and sampling the

lymph nodes (usually external iliac) for evidence of metastasis. If metastases

are present, radiation therapy is given locally to areas of known

involvement. Radiation therapy is highly effective in seminoma, and

metastatic disease may be palliated for extended periods.

452. The answer is d. (Schwartz, 7/e, pp 1812-1813.) Testicular torsion

occurs commonly in adolescents. The underlying pathology is secondary

to an abnormally narrowed testicular mesentery with tunica vaginalis surrounding

the testis and epididymis in a "bell-clapper" deformity. As the

testis twists, it comes to lie in a higher position within the scrotum. Urinalysis

is usually negative. Elevation will not provide a decrease in pain (negative

Prehn sign); a positive Prehn sign might indicate epididymitis. A 99mTc

pertechnetate scan may be helpful in clarifying a confusing case; however,

operation should not be delayed beyond 4 h from the time of onset of

symptoms in order to maximize testicular salvage. This patient's presentation

warrants immediate operation. The salvage rate for delay greater than

12 h is less than 20%. Both the affected and unaffected testes should

undergo orchiopexy. The differential diagnosis between torsion of the testicle

and epididymitis is sometimes quite difficult. On occasion, one has to

explore a patient with epididymitis just to rule out a torsion of the testicle.

Epididymitis usually occurs in sexually active males. Urinalysis is usually

positive for inflammatory cells, and urethral discharge is often present.

Spermatocele is a cyst of an efferent ductule of the rete testis. It presents as

a painless transilluminable cystic mass that is separate from the testes.

453. The answer is a. (Schwartz, 7/e, pp 1773-1774.) Genitourinary

tuberculosis develops from reactivation of foci in the renal cortex or

prostate that were hematogenously seeded during the primary (usually

asymptomatic) pulmonary infection. Local spread from the renal and prostatic

sites can lead to involvement of the calyx, ureter, bladder, vas deferens,

epididymis, and (rarely) the testis. A low-grade inflammatory response

results in hematuria or pyuria without bacteriuria. Whenever pus cells are

seen on routine urine culture without bacteria on smear or culture plate,

genitourinary tuberculosis should be considered. The end result of focal

caseation necrosis in the kidney may be scarring and dystrophic calcifica-

Urology Answers 293

tion. Genital tract infection often causes an asymptomatic swelling in the

epididymis; secondary infection or formation of a sinus tract to the scrotal

skin may cause more dramatic signs and symptoms. Epididymal tuberculosis

is usually managed by chemotherapy, with surgery reserved for refractory

cases. Pneumaturia is associated with a colovesical fistula and not with

genitourinary tuberculosis.

454. The answer is a. (Schwartz, 7/e, pp 1793-1795.) One of the most

frequent causes of male cancer deaths, prostate cancer has an incidence of

more than 75,000 new cases per year in the United States. American blacks

appear to have a 50% higher incidence and mortality. Prostate cancer (adenocarcinoma)

arises initially in the periphery of the gland. Therefore, one

of the best screening tests is careful rectal examination. However, the use of

screening for prostate-specific antigen (PSA) has increased the detection

rate fourfold. Spread is by direct local extension and by lymphatic and vascular

channels. The most common locations of distant metastases are in the

axial skeleton with osteoblastic bony lesions. A single focus of disease discovered

on TURP or simple prostatectomy is considered stage A1. Only 2%

of patients have unsuspected nodes (i.e., only 2% 5- to 10-year mortality).

Therefore, no definitive therapy is required except possibly in patients less

than 60 years old. Follow-up should be undertaken and progression of disease

may be treated as necessary. Several foci or diffuse disease is considered

stage A2 and surgery or radiation therapy is generally indicated.

455. The answer is d. (Schwartz, 7/e, pp 1784-1788.) In contrast to

prostate cancer, BPH arises first in the periurethral prostate tissue as a

fibrostromal proliferation. As the periurethral prostate grows, the outer

prostate glands are compressed against the true prostatic capsule, which

results in a thick pseudocapsule. As the prostate enlarges, it encroaches on

the urethra and causes urinary outflow obstruction. Obstructive symptoms

include decreased force of stream, hesitancy, recurrent UTIs, and occasionally

acute urinary retention; the latter two are indications for surgery. Uroflow

is the best noninvasive method of estimating the degree of outlet obstruction.

Flow less than 10 mL/s is good evidence of significant obstruction. The major

treatments for BPH are surgical. Simple prostatectomy involves shelling out

the prostate adenoma and leaving the pseudocapsule (true prostate) behind.

Therefore, these patients are still at risk of developing prostate cancer

although BPH in and of itself is not a risk factor for prostatic cancer.

294 Surgery

456. The answer is a. (Schwartz, 7/e, pp 1800-1801.) If time and the

patient's condition permit, primary ureteral reconstruction should be carried

out. In the middle third of the ureter, this will usually consist of

ureteroureterostomy using absorbable sutures over a stent. If the injury

involves the upper third, ureteropyeloplasty may be necessary. In the lower

third, ureteral implantation into the bladder using a tunneling technique is

preferred. If time does not permit definitive repair, suction drainage adjacent

to the injured segment alone is inadequate; either ligation and

nephrostomy or placement of a catheter into the proximal ureter is an

acceptable alternative that would allow reconstruction to be performed

later. The creation of a watertight seal is difficult and nephrectomy may be

required if the injury occurs during a procedure in which a vascular prosthesis

is being implanted (e.g., an aortic reconstructive procedure) and

contamination of the foreign body by urine must be avoided.

457. The answer is e. (Schwartz, 7/e, pp 1804-1807.) If a rupture of the

urethra is suspected, a retrograde urethrogram should be obtained before

any attempts are made to place a Foley catheter, as efforts to do so may

result in the creation of multiple false passages or conversion of a partial

laceration into complete rupture. Previously, treatment had included

attempts to realign the urethra immediately through the placement of

interlocking sounds and traction using either a catheter passed over the

sounds or perineal traction sutures through the bladder neck. Preferred

treatment currently avoids both dissection into the pelvic hematoma surrounding

the disruption and manipulation of the urethra; instead, only a

suprapubic tube is placed immediately with delayed reconstruction after

3-6 mo, at which time the hematoma will have resolved and the prostate

will have descended into the proximity of the urogenital diaphragm. Percutaneous

nephrostomy has no role in the management of this problem.

Urology Answers 295

This page intentionally left blank.

ORTHOPEDICS

Questions

DIRECTIONS: Each item below contains a question or incomplete

statement followed by suggested responses. Select the one best response to

each question.

297

458. Meniscal tears usually result

from which of the following circumstances?

a. Hyperextension

b. Flexion and rotation

c. Simple hyperflexion

d. Compression

e. Femoral condylar fracture

459. Volkmann's ischemic contracture

is associated with

a. Intertrochanteric femoral fracture

b. Supracondylar fracture of the humerus

c. Posterior dislocation of the knee

d. Traumatic shoulder separation

e. Colles "silver fork" fracture

460. In an uncomplicated dislocation

of the glenohumeral joint, the

humeral head usually dislocates

primarily in which of the following

directions?

a. Anteriorly

b. Superiorly

c. Posteriorly

d. Laterally

e. Medially

461. The most severe epiphyseal

growth disturbance is likely to

result from which of the following

types of fracture?

a. Fracture dislocation of a joint adjacent

to an epiphysis

b. Fracture through the articular cartilage

extending into the epiphysis

c. Transverse fracture of the bone

shaft on the metaphyseal side of the

epiphysis

d. Separation of the epiphysis at the

diaphyseal side of the growth plate

e. Crushing injury compressing the

growth plate

462. Which of the following fractures

is most commonly seen in

healthy bones subjected to violent

falls?

a. Colles fracture

b. Femoral neck fracture

c. Intertrochanteric fracture

d. Clavicular fracture

e. Vertebral compression fracture

Terms of Use

463. Which nerve is most at risk

in the injury in the accompanying

radiograph?

465. Which of the following

statements regarding compartment

syndromes following orthopedic

injuries is true?

a. The first sign is usually loss of pulse

in the extremity

b. Passive flexion of the extremity

proximal to the involved compartment

will aggravate the pain

c. Surgical decompression (fasciectomy)

is necessary only as a last

d. These syndromes are most commonly

associated with supracondylar

fractures of the humerus and

tibial shaft

e. The syndrome is often painless

466. In contrast to closed reduction,

open reduction of a fracture

a. Produces a shorter healing time

b. Decreases trauma to the fracture

c. Produces a higher incidence of

nonunion

d. Reduces the risk of infection

e. Requires longer periods of immobilization

298 Surgery

a. Median nerve

b. Radial nerve

c. Posterior interosseous nerve

d. Ulnar nerve

e. Ascending circumflex brachial nerve

464. In a failed suicide gesture, a

depressed student severs her radial

nerve at the wrist. The expected

disability is

a. Loss of ability to extend the wrist

b. Loss of ability to flex the wrist

c. Wasting of the intrinsic muscles of

the hand

d. Sensory loss over the thenar pad

and the thumb web

e. Palmar insensitivity

DIRECTIONS: Each group of questions below consists of lettered

options followed by numbered items. For each numbered item, select the

appropriate lettered option(s). Each lettered option may be used once,

more than once, or not at all. Choose exactly the number of options

indicated following each item.

Items 467-470

For each description below,

select the type of fracture or dislocation

with which it is most likely

to be associated.

a. Navicular (scaphoid) fracture

b. Monteggia's deformity

c. Greenstick fracture

d. Spiral fracture

e. Posterior shoulder dislocation

467. Epileptiform convulsion may

be a cause. (SELECT 1 INJURY)

468. Avascular necrosis is not

uncommon. (SELECT 1 INJURY)

469. The radial head is dislocated

and the proximal third of the ulna

is fractured. (SELECT 1 INJURY)

470. Tenderness in the anatomist's

snuffbox may be observed. (SELECT

1 INJURY)

Items 471-474

For each description below,

select the type of bone disease with

which it is most likely to be associated.

a. Osteogenesis imperfecta

b. Osteopetrosis

c. Osteitis fibrosa cystica

d. Osteomalacia

e. Osteitis deformans

471. Association with hyperparathyroidism

(SELECT 1 DISEASE)

472. A defect in the mineralization

of adult bone secondary to abnormalities

in vitamin D metabolism

(SELECT 1 DISEASE)

473. Genetically determined disorder

in the structure or processing

of type I collagen (SELECT 1 DISEASE)

474. Synonym for Paget's disease

(SELECT 1 DISEASE)

Orthopedics 299

Items 475-477

For each description below,

select the type of bone lesion with

which it is most likely to be associated.

a. Osteoma

b. Osteoid osteoma

c. Osteoblastoma

d. Osteosarcoma

e. Paget's disease

f. Ewing's sarcoma

475. An 11-year-old boy presents

with pain in his right leg. A radiograph

shows a "sunburst" appearance

with bone destruction, soft

tissue mass, new bone formation,

and sclerosis limited to the metaphysis

of the lower femur. (SELECT

1 LESION)

476. A 25-year-old man presents

with severe pain in the left femur.

The pain is relieved by aspirin. On

plain film, a 0.5-cm lucent lesion,

which is surrounded by marked

reactive sclerosis, is seen. (SELECT

1 LESION)

477. A 12-year-old boy complains

of pain in his left leg that is worse at

night. He has been experiencing

fevers and also has a 9-lb weight

loss. X-ray demonstrates an aggressive

lesion with a permeative pattern

of bone lysis and periosteal

reaction. There is an associated

large soft tissue mass as well.

Pathology demonstrates the tumor

to be of the round cell type.

(SELECT 1 LESION)

300 Surgery

ORTHOPEDICS

Answers

458. The answer is b. (Schwartz, 7/e, pp 1979-1980.) Most meniscal

tears are produced by flexion and rapid rotation. A classic example ("football

knee") involves a player who is hit while running. The knee, supporting

all the player's weight, usually is slightly flexed, and the foot is

anchored to the ground by cleats. Impact from an opposing player usually

causes rotation almost entirely restricted to the knee. The injury involves

rapid rotation of the flexed femoral condyles about the tibial plateau,

which most frequently tears the medial meniscus. (Less frequently, the lateral

meniscus is torn.) A tear in the inner free border of the cartilage is also

common whenever excessive rotation without flexion or extension occurs.

Early surgical removal of the displaced menisci is usually recommended to

prevent further damage to the cartilage or ligaments.

459. The answer is b. (Schwartz, 7/e, pp 1959, 2052-2053.) Compromise

of blood supply to the muscles of the forearm can lead to a compartment

syndrome and permanent serious functional deformity of the arm. Any

patient with a compressive dressing or cast of the upper extremity can

experience this potential catastrophe. Whenever a patient has increasing

pain in the presence of a circular dressing around the arm or forearm, the

dressing should be removed immediately. If there is tenderness in the forearm

on either the ulnar or dorsal aspect, a fasciotomy should be considered.

460. The answer is a. (Schwartz, 7/e, pp 1963-1964.) The glenohumeral

joint is bounded posteriorly by the teres minor and infraspinatus muscles

and partially by the long head of the triceps. It is bounded laterally by the

powerful deltoid muscle; superiorly, the acromion process precludes

upward dislocation. However, anteriorly and inferiorly the pectoralis major

and the long head of the biceps do not completely stabilize the glenohumeral

joint; in this region the articular ligaments and joint capsule provide

the major structural support. Thus, the joint is not strongly supported

in its anteroinferior aspect, and consequently anterior (or anteroinferior)

301

dislocations are the most common glenohumeral dislocations. The

humeral head is driven anteriorly, which tears the shoulder capsule,

detaches the labrum from the glenoid, and produces a compression fracture

of the humeral head. Most glenohumeral dislocations result from a

posteriorly directed force on an arm that is partially abducted. Posterior

dislocation is much rarer and should raise the possibility of a seizure as the

precipitating cause.

461. The answer is e. (Schwartz, 7/e, pp 1958-1959.) Longitudinal

growth of bone follows ossification of cartilage that forms at the epiphyseal

plate. Fractures that involve separation of the growth plate (type I) (almost

always on the diaphyseal side) may be realigned; normal growth usually

follows epiphyseal separation because the proliferative cells are still

attached to their blood supply in the bone epiphysis. Fractures that extend

perpendicular to and through the epiphysis (types II, III, IV) may result in

the formation of bony bridges across the epiphysis that can disrupt later

growth. Though all the fractures listed in the question place the epiphyseal

growth plate in some jeopardy, crushing injuries to the epiphysis (type V)

have the worst prognosis; numerous bony bridges may form and prevent

longitudinal growth.

462. The answer is d. (Schwartz, 7/e, pp 1948-1949.) Postmenopausal

osteoporosis is responsible for a large number of fractures in elderly

women. Though bone mineralization is normal in osteoporosis, total bone

mass and trabecular volume are decreased. Common fracture sites are the

vertebrae, distal radius, and hip. Vertebral compression fractures are often

sustained by elderly men and women even without trauma. A minor fall on

the outstretched hand can lead to a Colles' fracture when the distal radius

is weakened by osteoporosis. Similarly, either a femoral neck fracture or an

intertrochanteric fracture can follow a fall on the hip. Clavicular fractures

are less likely to result from osteoporosis. While these fractures occur in

both children and adults, they are common in healthy children and young

adults after violent falls onto an outstretched hand.

463. The answer is b. (Schwartz, 7/e, pp 1964-1965.) The radiograph

demonstrates a transverse fracture of the distal half of the humeral shaft.

The radial nerve runs in a groove on the posterior aspect of the humerus as

it courses into the forearm compartment and is therefore at high risk of

302 Surgery

injury. If the nerve injury is apparent before any manipulation has been

done, the fracture should be reduced; the nerve injury should be observed

since the nerve function will likely improve with time. If the nerve injury is

only present after reduction, immediate surgical exploration is warranted

because the nerve might be trapped in the fracture site. At this level of the

arm, the ulnar and median nerves are well protected by muscle. The posterior

interosseous nerve is a distal branch of the radial nerve and may be

injured in fractures near the radial head, but it is in no danger from injuries

at the level seen in this radiograph. There is no "ascending circumflex

brachial nerve."

464. The answer is d. (Schwartz, 7/e, p 2068.) An injury to the radial

nerve at the wrist would cause primarily sensory abnormalities. The dorsum

of the hand from the radial aspect of the fourth digit over the thumb,

including the thenar pad and thumb web, becomes insensate after severance

of the radial nerve at the wrist. Radial injuries more proximally would

impair extension of the wrist and digits as well as forearm supination.

465. The answer is d. (Schwartz, 7/e, pp 1959, 2052-2053.) Compartment

syndromes result from increasing pressures in the fascial compartments

of the arm or leg. When the pressure in the muscles is greater than

that of the capillaries, ischemia and necrosis of the muscles occur even

though the arterial pressure is still high enough to produce pulses; pulselessness

is an unreliable sign. Extreme pain (out of proportion to the

injury), pain on passive extension of the fingers or toes, pallor of the

extremity, motor paralysis, and paresthesias are all components of the syndrome.

The patient will usually hold the injured part in a position of flexion

to maximally relax the fascia and reduce the pain; passive extension

will usually produce severe pain. The diagnosis can be confirmed by measuring

intracompartmental pressures, but whenever physical findings or

symptoms are suspicious, immediate surgical decompression by fasciectomy

is indicated since delay is likely to lead to irreversible damage.

466. The answer is c. (Schwartz, 7/e, pp 1973-1978.) Open reduction of a

fracture involves the restoration of normal bone alignment under direct

observation at surgery. In effect, open reduction converts a simple fracture

into a compound (or open) fracture and thereby increases the risk of infection.

Operative manipulation also increases trauma at the fracture site and

Orthopedics Answers 303

may consequently add to the probability of infection. Hematomas at the site

of fracture may be important for early healing; open reduction, which usually

involves removing the clots in the field, could contribute to a delay in

bone healing and to nonunion. The major advantage of open reduction is

the shorter period of immobilization it allows, an advantage that often outweighs

all the disadvantages previously mentioned, as in the open reduction

of femoral neck fractures in the elderly. This allows these patients to get out

of bed much sooner than if they were treated with several weeks of traction.

467-470. The answers are 467-e, 468-a, 469-b, 470-a. (Schwartz,

7/e, pp 1963-1964, 1968-1971, 1981-1982.) Fractures of the navicular bone

of the wrist should be suspected in anyone, particularly a young person,

who falls on an outstretched hand. Although x-rays are mandatory, it is

important to realize that the fracture may not be seen on the initial x-ray

and that a presumptive diagnosis can and should be made on clinical

grounds alone. Typically, there will be tenderness to palpation over the

navicular tuberosity and limitation of wrist flexion and extension. Immobilization

of the wrist for about 16 wk and sometimes up to 6 mo is required.

Nonunion or avascular necrosis is not uncommon and may require bone

grafting for correction.

Dislocation of the radial head with a fracture of the proximal third of

the ulna is known as Monteggia's deformity. Usually, the radial head is dislocated

anteriorly. The injury is usually caused by forced pronation. The

injury can be treated by reduction and stabilization of the ulna followed by

reduction of the radial head via supination and direct pressure.

Anterior shoulder dislocations occur more frequently than posterior

dislocations. However, posterior dislocations are seen in special situations,

such as during an epileptiform convulsion and during electroshock therapy.

Closed reduction followed by immobilization is usually sufficient therapy.

A spiral fracture, frequently seen in the tibia in skiers, results from the

application of torque to a long bone. Greenstick fractures are common in

children. The bones of young children are able to bend to a greater degree

than those of adults; the fracture may occur only at the site of maximal cortical

stress but not at the opposite cortex, the site of maximal longitudinal

compression.

471-474. The answers are 471-c, 472-d, 473-a, 474-e. (Schwartz,

7/e, pp 1946-1951.) Osteitis fibrosa cystica is commonly associated with

304 Surgery

hyperparathyroidism. Hemorrhagic cystic lesions (brown tumors) usually

occur in the long bones. Treatment is parathyroidectomy. Osteomalacia is

defined as a defect in mineralization of adult bone that results from abnormalities

in vitamin D metabolism. Treatment generally involves vitamin D

supplementation. Osteogenesis imperfecta is a genetically determined disorder

in the structure or processing of type I collagen. Treatment is surgical

and involves orthoses to prevent fractures and correction of deformities by

multiple osteotomies. Osteitis deformans is also known as Paget's disease.

Osteopetrosis is a rare skeletal deformity associated with increased density

of the bones.

475-477. The answers are 475-d, 476-b, 477-f. (Schwartz, 7/e, pp

2008-2011, 2014.) Osteosarcoma, or osteogenic sarcoma, usually is seen in

patients between the ages of 10 and 25 years. The distal femur is the site

most frequently involved. The radiograph has a blastic, or sunburst, appearance.

The tumor is not sensitive to radiation but does respond well to combination

chemotherapy followed by surgical resection or amputation.

An osteoid osteoma typically presents with severe pain that is characteristically

relieved by aspirin. On radiograph, the lesion appears as a small

lucency (usually <1.0 cm) within the bone that is surrounded by reactive

sclerosis. These lesions gradually regress over 5-10 years, but most are

excised to relieve symptoms. Surgical extirpation is usually curative.

Ewing's sarcoma is a round cell-type tumor. This is a highly malignant

tumor that affects children (age range 5-15 years) and tends to occur in the

diaphyses of long bones. The spine and pelvis can also be primary sites.

There is a permeative pattern of bone lysis and periosteal reaction often

associated with a large soft tissue mass. Fever and weight loss are common.

The pain is often more pronounced at night. Treatment usually involves a

combination of radiation and systemic chemotherapy, with 5-year survivals

around 50%. Adjuvant surgery in combination with radiation and chemotherapy

improves the 5-year survival to about 75%.

Orthopedics Answers 305

This page intentionally left blank.

NEUROSURGERY

Questions

DIRECTIONS: Each item below contains a question or incomplete

statement followed by suggested responses. Select the one best response to

each question.

307

478. Which of the following statements

regarding the Glasgow coma

scale is true?

a. It serves as a scale to assess the

long-term sequelae of head trauma

b. A high score correlates with a high

mortality

c. It includes measurement of intracranial

pressure

d. It includes measurement of pupillary

reflexes

e. It includes measurement of verbal

479. Controlled hyperventilation

(induced hypocapnia) is frequently

recommended following head

trauma. The therapeutic consequences

of this therapy include

a. Reduction of endogenous catecholamines

b. Reduction of intracellular potassium

levels

c. Increase in cerebrovascular resistance

d. Induction of compensatory metabolic

alkalosis

e. Requirement of monitoring the

intracranial pressure

480. Which of the following statements

regarding glioblastoma multiforme

is true?

a. It is a neuronal cell tumor

b. It arises from the malignant degeneration

of an astrocytoma

c. With aggressive treatment, most

patients can live up to 10 years

with this disease

d. It is the most common childhood

intracranial neoplasm

e. With combined surgery, chemotherapy,

and radiation therapy, cure

rates now approach 50%

481. A 60-year-old woman presents

to her physician with a 3-wk

history of severe headaches. A contrast

CT scan reveals a small, circular,

hypodense lesion with ringlike

contrast enhancement. The most

likely diagnosis is

a. Brain abscess

b. High-grade astrocytoma

c. Parenchymal hemorrhage

d. Metastatic lesion

e. Toxoplasmosis

Terms of Use

482. Which of the following statements regarding skull fractures is true?

a. Depressed fractures are those in which the patient's level of consciousness is

diminished or absent

b. Compound fractures are those in which the skull is fractured and the underlying

brain is lacerated

c. Any bone fragment displaced more than 1 cm inwardly should be elevated surgically

d. Drainage of cerebrospinal fluid via the ear or nose requires prompt surgical

treatment

e. Most skull fractures require surgical treatment

483. A 39-year-old man presents to his physician with the complaint of loss

of peripheral vision. The subsequent magnetic resonance imaging (MRI) scan

below demonstrates

308 Surgery

a. Cerebral atrophy

b. Pituitary adenoma

c. Optic glioma

d. Pontine hemorrhage

e. Multiple sclerosis plaque

484. An 18-year-old man is admitted to the emergency room following a

motorcycle accident. He is alert and fully oriented, but witnesses to the

accident report an interval of unresponsiveness following the injury. Skull

films disclose a fracture of the left temporal bone. Following x-ray, the

patient suddenly loses consciousness and dilation of the left pupil is noted.

This patient should be considered to have

a. Ruptured berry aneurysm

b. Acute subdural hematoma

c. Epidural hematoma

d. Intraabdominal hemorrhage

e. Ruptured arteriovenous malformation

485. Which of the following statements regarding the cerebral angiogram

below is true?

Neurosurgery 309

a. The aneurysm arises from an arteriovenous malformation

b. The lesion is a giant aneurysm

c. There is a basilar artery lesion

d. Initial treatment includes aggressive fluid hydration

e. Surgical clipping of this lesion is curative

486. An acute increase in intracranial

pressure is characterized by

which of the following clinical findings?

a. Respiratory irregularities

b. Decreased blood pressure

c. Tachycardia

d. Papilledema

e. Compression of the fifth cranial

nerve

487. Which of the following statements

about schwannomas is true?

a. They represent central nerve tumors

b. Treatment is via excision

c. They arise most frequently in motor

nerves

d. They often degenerate to malignancy

e. The most common presentation is a

painful mass

488. Which of the following statements

about craniopharyngiomas

is true?

a. The tumors are uniformly solid

b. The tumors are usually malignant

c. Children with these tumors often

develop signs and symptoms of

acromegaly

d. The tumors may cause compression

of the optic tracts and visual

symptoms

e. The primary mode of treatment is

radiation therapy

489. Which of the following statements

regarding cerebral contusions

is true?

a. They occur most frequently in the

occipital lobes

b. They may occur opposite the point

of skull impact

c. They are rarely accompanied by

parenchymal bleeding

d. They may occur spontaneously in

patients receiving anticoagulants

e. Anticonvulsants have no role in the

early management of this disorder

490. True statements regarding

meningiomas include that they

a. Are malignant in 50% of cases

b. Occur predominantly in men

c. Are treated primarily by surgical

excision

d. Are cured, when properly treated,

in nearly 95% of cases

e. Arise from the dura

310 Surgery

DIRECTIONS: The group of questions below consists of lettered

options followed by numbered items. For each numbered item, select the

appropriate lettered option(s). Each lettered option may be used once,

more than once, or not at all. Choose exactly the number of options

indicated following each item.

Items 491-492

For each description below,

select the type of vascular event

with which it is most likely to be

associated.

a. Subdural hematoma

b. Epidural hematoma

c. Carotid dissection

d. Brain contusion

e. Ruptured intracranial aneurysm

491. While watching a golf tournament,

a 37-year-old man is

struck on the side of the head by a

golf ball. He is conscious and talkative

after the injury, but several

days later he is noted to be increasingly

lethargic, somewhat confused,

and unable to move his right

side. (SELECT 1 DIAGNOSIS)

492. A 42-year-old woman complains

of the sudden onset of a

severe headache, stiff neck, and

photophobia. She loses consciousness.

She is later noted to have a

dilated pupil. (SELECT 1 DIAGNOSIS)

Neurosurgery 311

NEUROSURGERY

Answers

478. The answer is e. (Schwartz, 7/e, pp 1880-1881.) The Glasgow coma

scale was developed to enable an initial assessment of the severity of head

trauma. It is now also used to standardize serial neurologic examinations in

the early postinjury period. It measures the level of consciousness using

three parameters: verbal response (5 points), motor response (6 points),

and eye opening (4 points). The score is the sum of the highest number

achieved in each category. The fully oriented and alert patient will receive

a maximum score of 15. A score of less than 5 is associated with a mortality

of over 50%.

479. The answer is c. (Schwartz, 7/e, pp 1878, 1880-1881.) Controlled

hyperventilation to a PaCO2 of 25 kPa raises tissue pH, increases cerebrovascular

resistance, decreases cerebral blood flow, and consequently

reduces intracerebral pressure (ICP). In the effort to avoid brain swelling by

lowering cerebral blood flow and ICP, the clinician must be wary of causing

ischemic brain damage through hypoperfusion. The metabolic compensation

to induced hypocapnia leads to normalization of the pH by loss

of bicarbonate (metabolic acidosis), and over 8-24 h the beneficial effects

of the hypocapnia will have been lost. The partial pressures of carbon dioxide

should be allowed to slowly return to normal and should be held in

reserve in case unanticipated increases in ICP require another pulse of

short-term reduction. It is important to monitor the patient while the PaCO2

is rising because untoward or rapid increases in ICP may occur in response

to the rising cerebral blood flow.

480. The answer is b. (Schwartz, 7/e, pp 1886-1887.) Glioblastoma multiforme

is the most common form of primary intracranial neuroepithelial

tumor. It represents 25% of all intracranial tumors and 50% of tumors originating

in the central nervous system. It is a heterogeneous glial cell tumor

derived from the malignant degeneration of an astrocytoma or anaplastic

astrocytoma. These tumors are most commonly found in the cerebral

hemispheres during the fifth decade of life. CT and MRI scans typically

312

reveal an irregular lesion with hypodense central necrosis, peripheral ring

enhancement of the highly cellular tumor tissue, and surrounding edema

and mass effect. Curative resections are rare. Therapy consists of diagnostic

biopsy followed by radiotherapy to slow the tumor growth. The course

of the disease progresses rapidly after presentation, with few patients living

more than 2 years.

481. The answer is d. (Schwartz, 7/e, p 1890.) The CT findings are consistent

with any of the suggested lesions. However, the most likely diagnosis

is metastatic disease. Almost 50% of intracranial neoplasms are

metastatic lesions. Roughly 20-25% of cancer patients develop intracranial

metastases during the course of their disease. Cancers of the lung and

breast and melanomas frequently metastasize to the brain parenchyma.

Leukemia shows a predilection for the leptomeninges. A large majority of

these lesions become symptomatic owing to mass effect from white matter

edema. Palliation is the primary goal for most patients and involves corticosteroids

and radiation. Surgery is employed for the 25% of patients with

a solitary brain metastasis and cured or arrested systemic disease.

482. The answer is c. (Schwartz, 7/e, pp 1879-1880.) Most skull fractures

do not require surgical treatment unless they are depressed or compound.

A general rule is that all depressed skull fractures, defined as

fractures in which the cranial vault is displaced inward, should be surgically

elevated, especially if they are depressed more than 1 cm, if a fragment

is over the motor strip, or if small, sharp fragments are seen on x-ray

(as they may tear the underlying dura). Compound fractures, defined as

fractures in which the bone and the overlying skin are broken, must be

cleansed and debrided and the wound must be closed. When a skull fracture

occurs in an area of the paranasal sinuses, the mastoid air cells, or the

middle ear, a tear in the meninges may result in cerebrospinal fluid

drainage from the ear or nose. The presence of rhinorrhea or otorrhea

requires observation and prophylactic antibiotics, because meningitis is a

serious sequel. Otorrhea usually heals within a few days. Persistent cerebrospinal

fluid from the nose or ear for more than 14 days requires surgical

repair of the torn dura.

483. The answer is b. (Schwartz, 7/e, pp 1620-1628.) This T1-weighted

sagittal MRI scan reveals a dumbbell-shaped homogeneous mass involving

Neurosurgery Answers 313

the sella turcica and the suprasellar region. This lesion is most consistent

with a pituitary adenoma, a benign tumor arising from the adenohypophysis.

Pituitary adenomas are the most common sellar lesion and constitute

10-15% of all intracranial neoplasms. Macroadenomas (>10 mm) are generally

nonsecreting tumors. Microadenomas (<10 mm) become clinically

apparent from hormonal secretion. They may secrete prolactin (amenorrhea

or galactorrhea), growth hormone (gigantism or acromegaly), or ACTH

(Cushing syndrome). The tumor pictured is a macroadenoma. Its dumbbell

shape results from impingement on the adenoma by the diaphragm of the

sella turcica. The suprasellar extension seen here makes a frontal craniotomy

rather than a transsphenoidal approach more appropriate.

484. The answer is c. (Schwartz, 7/e, p 1881.) Epidural hematomas are

typically caused by a tear of the middle meningeal artery or vein or a dural

venous sinus. Ninety percent of epidural hematomas are associated with

linear skull fractures, usually in the temporal region. Only 2% of patients

admitted with craniocerebral trauma suffer epidural hematomas. The

lesion appears as a hyperdense biconvex mass between the skull and brain

on CT scan. Clinical presentation is highly variable and outcome largely

depends on promptness of diagnosis and surgical evacuation. The typical

history is one of head trauma followed by a momentary alteration in consciousness

and then a lucid interval lasting for up to a few hours. This is

followed by a loss of consciousness, dilation of the pupil on the side of the

epidural hematoma, and then compromise of the brainstem and death.

Treatment consists of temporal craniectomy, evaluation of the hemorrhage,

and control of the bleeding vessel. The mortality of epidural hematoma is

approximately 50%.

485. The answer is e. (Schwartz, 7/e, pp 1893-1895.) This digital subtraction

cerebral angiogram is an oblique view of the anterior circulation of

the brain. Dye injected in the internal carotid reveals an aneurysm at the

bifurcation of the internal carotid and the posterior communicating artery.

A giant aneurysm is generally regarded as a lesion greater than 24 mm in

cross-section. Surgical clipping of this aneurysm would be curative. Only

after the risk of rebleeding is eliminated by clipping can the patient

undergo volume expansion if vasospasm arises. The vertebrobasilar system

is not visualized here.

314 Surgery

486. The answer is a. (Schwartz, 7/e, pp 1878, 1895-1896.) The onset of

irregular respirations, bradycardia, and finally increased blood pressure

with increasing intracranial pressure (ICP) is termed the Cushing response.

These physiologic alterations are caused by brainstem compression. Slow

rises in ICP are, by contrast, autoregulated by the brain's compensatory

mechanisms and lead to a late onset of neurologic sequelae. A mass lesion

is more apt to compromise local cerebral blood flow and increase cerebral

edema and ICP. The vector of the mass effect may lead to herniation of

brain parenchyma through the tentorial incisura or foramen magnum with

resultant brainstem compression. Herniation usually causes compression

of the third cranial nerve and thus leads to a fixed and dilated pupil on that

side. Papilledema is a finding with chronic increases in ICP.

487. The answer is b. (Schwartz, 7/e, pp 1888-1889, 1892.) Peripheral

nerve tumors include lesions of peripheral nerves, the adrenal gland nerve

tissue, and the sympathetic chain. Schwannomas are peripheral nerve

sheath tumors that arise from perineural fibroblasts (Schwann cells). They

are usually painless. Malignant schwannomas are rare. Treatment is via surgical

excision. The nerve of origin can usually be preserved. Because

schwannomas have virtually no malignant potential, if a major nerve would

have to be sacrificed in order to extirpate the tumor, the nerve is spared and

a small portion of the tumor is left in situ. Intracranial schwannomas most

frequently originate in the vestibular branch of the eighth cranial nerve and

represent 10% of all intracranial neoplasms. Symptoms include hearing

loss, tinnitus, and vertigo. Neurofibromas are also Schwann cell tumors but

are histologically distinguishable from schwannomas. Neurofibromatosis

(von Recklinghausen's disease) involves multiple peripheral nerve neoplasms.

Neuronal tumors of peripheral nerves include ganglioneuroma,

neuroblastoma, chemodectoma, and pheochromocytoma.

488. The answer is d. (Schwartz, 7/e, pp 1628-1629.) Craniopharyngiomas

are cystic tumors with areas of calcification and originate in the

epithelial remnants of Rathke's pouch. These usually benign tumors are

found in the sellar and suprasellar region and lead to compression of the

pituitary, optic tracts, and third ventricle. As a result they show up on radiographic

imaging as an area of sellar erosion with calcification within or

above the sella. Craniopharyngiomas are most commonly found in chil-

Neurosurgery Answers 315

dren but may also present in adulthood. In children they can cause growth

retardation because of hypothalamic-pituitary dysfunction. Treatment consists

of subfrontal or transsphenoidal excision with adjuvant radiotherapy

if total removal is not possible.

489. The answer is b. (Sabiston, 15/e, pp 1355-1358.) Cerebral contusions

are bruises of neural parenchyma that most commonly involve the

convex surface of a gyrus. The most frequent sites of cerebral contusion are

the orbital surfaces of the frontal lobes and the anterior portion of the temporal

lobes. The etiology of the contusion is always traumatic, and subsequent

neurologic impairment, such as epilepsy, is common if the original

injury was significant. Patients deemed to have a substantial contusion

should receive anticonvulsive medication in the early posttraumatic

period.

490. The answer is c. (Schwartz, 7/e, pp 1887-1888.) Meningiomas are

relatively benign tumors that arise from the arachnoid layer of the

meninges. They occur predominantly in women (65%) and are treated primarily

by surgical excision. Despite their relatively benign nature, the 15-

year survival rate for nonmalignant meningiomas is only 68%.

491-492. The answers are 491-a, 492-e. (Sabiston, 15/e, pp 1349-

1352, 1360.) Subdural hematomas usually arise from tears in the veins

bridging from the cerebral cortex to the dura or venous sinuses, often after

only minor head injuries. They can become apparent several days after the

initial injury. Treatment is with drainage of the hematoma through a burr

hole; a formal craniotomy may be required if the fluid reaccumulates. Significant

brain contusions due to blunt trauma are usually associated with at

least transient loss of consciousness; similarly, epidural hematomas result

in a period of unconsciousness, although a "lucid interval" may follow during

which neurologic findings are minimal.

Subarachnoid hemorrhage (SAH) in the absence of antecedent trauma

most commonly arises from a ruptured intracranial aneurysm, which typically

is found at the bifurcation of the major branches of the circle of Willis.

Other less frequent causes include hypertensive hemorrhage, trauma, and

bleeding from an arteriovenous malformation. Patients present with the

sudden onset of an excruciating headache. Complaints of a stiff neck and

photophobia are common. Loss of consciousness may be transient or

316 Surgery

evolve into frank coma. Cranial nerve palsies are seen as a consequence

both of increased intracranial pressure due to hemorrhage and pressure of

the aneurysm on adjacent cranial nerves. CT scans followed by cerebral

arteriography help to confirm the diagnosis as well as to identify the location

of the aneurysm. Treatment consists of surgical ligation of the

aneurysm by placing a clip across its neck. Early surgical intervention

(within 72 h of SAH) may prevent aneurysmal rebleeding and allow aggressive

management of posthemorrhage vasospasm.

Neurosurgery Answers 317

This page intentionally left blank.

OTOLARYNGOLOGY

Questions

DIRECTIONS: Each item below contains a question or incomplete

statement followed by suggested responses. Select the one best response to

each question.

319

493. Which of the following statements

concerning nasopharyngeal

cancer is true?

a. It has an unusually high incidence

among Chinese

b. It occurs primarily after the sixth

decade of life

c. It undergoes early metastasis to the

lungs

d. The treatment of choice is wide

surgical excision of the primary

tumor

e. Initial evaluation should involve a

biopsy of the primary tumor and

neck nodes

494. Severe maxillofacial trauma

is often the result of high-velocity

impact sustained in automobile or

motorcycle accidents. Regarding

these injuries, which of the following

statements is true?

a. Evaluation of the cervical spine

should precede that of the facial

injuries

b. Severe hemorrhage from the nasopharynx

rarely occurs with LeFort

fractures

c. Direct oral or nasotracheal intubation

should be performed promptly

to prevent airway obstruction

d. Standard facial x-ray series are

preferable to computed tomography

to assess facial fractures because

they may be obtained in the emergency

department, are performed

faster, and are equally accurate

e. Definitive management of fractures

of facial bones should not be

delayed

Terms of Use

495. Which of the following statements

regarding squamous cell carcinoma

of the head and neck is

true?

a. Squamous cancers of the head and

neck are caused by smoking tobacco

rather than chewing tobacco

b. Chemotherapy rarely produces a

response with pharyngeal carcinoma

and is not employed

c. Squamous cancers of the nasopharynx

are best treated by radiotherapy;

surgery is reserved for lymph

node metastases that have not responded

to radiation

d. Squamous cancers of the oropharynx

are best treated by radiotherapy;

surgery is not recommended

e. For squamous cancers of the hypopharynx,

radical neck dissection is

performed only if lymph nodes are

enlarged

496. Pleomorphic adenomas

(mixed tumors) of the salivary

glands are characterized by which

of the following?

a. They occur most commonly on the

lips, tongue, and palate

b. They grow rapidly

c. They rarely recur if simply enucleated

d. They present as rock-hard masses

e. They have no malignant potential

497. Which of the following statements

about branchial cleft anomalies

is true?

a. A fistula that lies between the external

auditory canal and the submandibular

region originates from

the second branchial cleft

b. The course of the first branchial

cleft fistula is through the bifurcation

of the carotid artery

c. Injury to the hypoglossal nerve

may occur during excision of a second

branchial cleft fistula

d. The internal opening of the second

branchial cleft fistula is usually

found in the maxillary sinus

e. The internal opening of the first

branchial cleft cyst is just underneath

the base of the tongue

498. Which of the following statements

regarding symptomatic thyroglossal

duct cysts is true?

a. Over 90% manifest themselves

before age 12

b. Treatment includes resection of the

hyoid bone

c. They usually present as a painful

swelling in the lateral neck

d. Approximately 10-15% contain

malignant elements

e. They rarely become infected

320 Surgery

499. Which of the following statements

regarding cancer of the

tongue is true?

a. Carcinomas at the base of the

tongue are best treated by irradiation

alone rather than surgery

b. Stage I and stage II cancers of the

mobile tongue are treated more

effectively by irradiation than by

surgery

c. Cancer of the tongue is usually

advanced to stage III by the time it

is diagnosed

d. Prophylactic irradiation of the neck

nodes is indicated in patients

whose primary cancer of the

tongue is treated by irradiation

e. Cancer of the tongue is the third

most common malignancy of the

oral cavity

500. Verrucous carcinoma of the

buccal mucosa is identified with

which of the following characteristics?

a. It is faster growing than the epidermoid

form

b. It is not associated with tobacco

chewing

c. It has a predilection for the gingivobuccal

gutter

d. It rarely extends to the mandible

e. It has a dark-brown, flat, smooth

border on presentation

Otolaryngology 321

OTOLARYNGOLOGY

Answers

493. The answer is a. (Schwartz, 7/e, pp 645-648.) There is an unusually

high incidence of carcinoma of the nasopharynx among Chinese. In the

early stages of the disease, metastases remain confined to the neck. Diagnosis

of nasopharyngeal cancer, which tends to arise in relatively young

people, should be made by biopsy of the primary tumor. Biopsy of the neck

nodes should be avoided because implantation of the tumor in skin and

subcutaneous tissue may occur. Radiation therapy is the treatment of

choice for the primary nasopharyngeal cancer. Cervical metastases that

remain clinically evident should be removed by a radical neck dissection.

494. The answer is a. (Greenfield, 2/e, pp 298-308.) In patients with

severe facial or mandibular trauma, airway difficulties may develop secondary

to the effects of massive hemorrhage, tissue swelling, or associated

laryngeal trauma. A cricothyroidotomy is preferred over direct oral or

nasotracheal intubation because it can be performed quickly without

manipulation of the cervical spine or injured parts. If prolonged postoperative

airway problems are anticipated, the cricothyroidotomy may convert

to a tracheostomy. Evaluation of the cervical spine is a top priority and

should be performed in any patient with head trauma prior to further facial

studies. Although most facial fractures can be diagnosed easily with a standard

"facial series," computed tomography (CT) is more accurate and

allows assessment of areas (e.g., intracerebral contents) that cannot be evaluated

by conventional techniques. Therefore, in most centers, CT is

presently the preferred method of evaluation for patients with severe maxillofacial

trauma. Maxillary fractures are categorized by the LeFort classification,

and, unlike other facial fractures, are frequently associated with

severe nasal and nasopharyngeal hemorrhage. This may be treated with

head elevation and ice compresses. Nasal packing also affords good control

of hemorrhage, and in extreme cases ligation or embolization of the internal

maxillary artery may be necessary. Definitive reduction and fixation of

fractures may be delayed while other injuries and medical problems are

322

addressed. In addition to control of hemorrhage, initial management of

facial fractures may include temporary stabilization, wound closure, and

oral lavage with solutions containing antibiotics.

495. The answer is c. (Greenfield, 2/e, pp 637-651.) Squamous cell cancers

of the head and neck appear to arise as a response to tobacco in general

(including chewing tobacco), rather than just to cigarette smoking,

especially when used in combination with alcohol ingestion. Chemotherapy

for squamous cell pharyngeal cancer has been used very successfully in

childhood and adolescence, although its role in adult pharyngeal cancer is

uncertain. Treatment of nasopharyngeal squamous cell carcinoma is by

radiation, followed by radical neck dissection if lymph node metastases

have not been controlled. Oropharyngeal cancers have responded equally

well to surgery and radiation, and both treatments are routinely employed.

In the hypopharynx surgery is the optimal treatment, often supplemented

by postoperative radiation therapy. Surgery for hypopharyngeal cancers

includes radical neck dissection because lymph node metastases occur frequently

and are not well controlled by radiation alone.

496. The answer is a. (Schwartz, 7/e, pp 656-662.) There are approximately

400-700 minor salivary glands in the oral cavity. Pleomorphic adenomas

(mixed tumors) can occur in any of them. These round tumors have

a rubbery consistency and are slow-growing; all are potentially malignant.

Unless adequately excised, they tend to recur locally in a high percentage

of cases. The sites most commonly affected by pleomorphic adenomas of

the salivary glands are the lips, tongue, and palate.

497. The answer is c. (Greenfield, 2/e, pp 1995-1998.) Branchial cleft

cysts, sinuses, and fistulas are remnants of the first and second branchial

pouches. The internal opening of the first is the external auditory canal; for

the second, it is the posterolateral pharynx below the tonsillar fossa. The

facial nerve may be injured during dissection of the first fistula. The second

fistula passes between the carotid bifurcation and adjacent to the hypoglossal

nerve. In childhood most branchial cleft anomalies present as a painless

nodule along the lateral border of the sternocleidomastoid muscle. In

adults, superinfection of the cyst or fistulous drainage via an orifice in the

supraclavicular region may occur. Treatment is surgical excision.

Otolaryngology Answers 323

498. The answer is b. (Greenfield, 2/e, pp 1998-1999.) Thyroglossal duct

cysts result from retention of an epithelial tract between the thyroid and its

embryologic origin in the foramen cecum at the base of the tongue. This

tract usually penetrates the hyoid bone. There is no sex predilection, and

although these cysts are more frequently detected in children, up to 25% do

not become symptomatic until adulthood. The most common presentation

is a painless swelling in the midline of the neck that moves with protrusion

of the tongue or swallowing. The cysts are prone to infection and progressive

enlargement. Although rare (less than 1%), epidermoid or papillary carcinomas

do occur within thyroglossal duct cysts. Surgical resection is the

standard therapy. The Sistrunk procedure, which involves local resection of

the cyst and the central portion of the hyoid bone, is the operation of choice.

Simple excision of the cyst results in an unacceptably high recurrence rate.

499. The answer is d. (Schwartz, 7/e, pp 635-639.) Cancer of the tongue

is the most common malignant tumor in the oral cavity and accounts for

slightly less than one-third of the malignancies in the area. About twothirds

of cases present as early lesions in the mobile anterior portion of the

tongue. Most workers in the field agree that for these stage I and stage II

lesions, surgery and irradiation give equivalent results (45% 5-year survival),

and the treatment should therefore be tailored to the patient. Failures

are almost always due to supraclavicular recurrence and many

recommend excision of the radiation scar and prophylactic irradiation of

the neck nodes, particularly in the stage II and stage IV tumors in the base

of the tongue, which have a poorer prognosis.

500. The answer is c. (Schwartz, 7/e, pp 631-632.) Verrucous carcinoma

is a less aggressive form of locally invasive buccal cancer than the usual epidermoid

form. Its frequency is increased in people who chew tobacco. The

tumor usually grows very slowly, occurs chiefly in the gingivobuccal gutter,

and has a tendency to invade bone. It is identified by its characteristic exophytic,

white, shaggy appearance. Wide excision is the best initial treatment

for this neoplasm. Even though the tumor may regress in response to

radiation, it tends to recur in a more malignant form with metastases. Cervical

metastases usually are not present when the lesion is first diagnosed;

it is only for the most highly malignant grades of verrucous carcinoma that

radical neck dissection and block excision of the cheek are indicated.

324 Surgery

BIBLIOGRAPHY

ANDERSON RJ, ET AL: Unrecognized adult salicylate intoxication. Ann Intern

Med 85:745-748, 1976.

BARNAVON Y, WALLACK MK: Management of the pregnant patient with carcinoma

of the breast. Surg Gynecol Obstet 171:347-352, 1990.

BELKIN M, ET AL: Intra-arterial fibrinolytic therapy. Efficacy of streptokinase

vs urokinase. Arch Surg 121:769-773, 1986.

BERCI: Am J Surg 161:332-335, 1991.

BOUCHER CA, ET AL: Determination of cardiac risk by dipyridamolethallium

imaging before peripheral vascular surgery. N Engl J Med

312:389-394, 1985.

BREWSTER DC, FRANKLIN DP, CAMBRIA RP, ET AL: Intestinal ischemia complicating

abdominal aortic surgery. Surgery 109:447-454, 1991.

BUNT TJ, ET AL: Frequency of vascular injury with blunt trauma-induced

extremity injury. Am J Surg 160:226-228, 1990.

CAMERON JL: Current Surgical Therapy, 5th ed. St Louis, Mosby, 1995.

CASE RECORDS OF THE MASSACHUSETTS GENERAL HOSPITAL. Weekly Clinicopathological

Exercises. Case 45-1987: A 16-year-old girl with hepatic

and pulmonary masses. N Engl J Med 317:1209-1218, 1987.

CASS AS: Renovascular injuries from external trauma. Diagnosis, treatment,

and outcome. Urol Clin North Am 16:213-220, 1989.

CHARLSON ME, ET AL: The preoperative and intraoperative hemodynamic

predictors of postoperative myocardial infarction or ischemia in

patients undergoing noncardiac surgery. Ann Surg 210:637-648,

1989.

COSENTINO CM, ET AL: Choledochal duct cyst: Resection with physiologic

reconstruction. Surgery 112:740-748, 1992.

CUMMINGS RA, ET AL: Pneumopericardium resulting in cardiac tamponade.

Ann Thorac Surg 37:511-518, 1984.

DIETTRICH NA, ET AL: A growing spectrum of surgical disease in patients

with human immunodeficiency virus/acquired immunodeficiency

syndrome. Experience with 120 major cases. Arch Surg 126:860-866,

1991.

DUBROW TJ, ET AL: Myocardial contusion in the stable patient. Surgery

106:267-273, 1989.

325

DUTKY PA, STEVENS SL, MAULL KI: Factors affecting rapid fluid resuscitation

with large-bore introducer catheters. J Trauma 29:856-860, 1989.

EISBUD DE, ET AL: Treatment of acute vascular occlusions with intraarterial

urokinase. Am J Surg 160:160-165, 1990.

ERWIN TJ, CLARK JR, WEICHSELBAUM RR: Multidisciplinary treatment of

advanced squamous carcinoma of the head and neck. Semin Oncol

12:71-82, 1985.

Executive Committee for the Asymptomatic Carotid Atherosclerosis Study:

JAMA 273:1421-1428, 1995.

FLINT L, ET AL: Definitive control of mortality from severe pelvic fracture.

Ann Surg 211:703-707, 1990.

GAJRAJ H, YOUNG AE: Adrenal incidentaloma. Br J Surg 80:422-426, 1993.

GOBBI PG: Cancer 65(11):2528-2536, 1990

GOLDMAN: J Cardiothorac Anesth 1:237, 1987.

GOODNOUGH LT, SHUCK JM: Risks, options, and informed consent for blood

transfusion in elective surgery. Am J Surg 159:602-609, 1990.

GRAHAM DY, GO MF: Helicobacter pylori: Current status. Gastroenterology

105:279-282, 1993.

GREENFIELD L, ET AL (EDS): Surgery: Scientific Principles and Practice, 2d ed.

Philadelphia, Lippincott-Raven, 1997.

HARRIS J, ET AL: Diseases of the Breast. Philadelphia, Lippincott-Raven, 1996.

HENDERSON JA, PELOQUIN AJ: Boerhaave revisited: Spontaneous esophageal

perforation as a diagnostic masquerader. Am J Med 86:559-567,

1989.

HEYS SD, ET AL: Nutrition and malignant disease: Implications for surgical

practice. Br J Surg 79:614-623, 1992.

LANDERCASPER J, ET AL: Perioperative stroke risk in 173 consecutive patients

with a past history of stroke. Arch Surg 125:986-989, 1990.

LIPSETT PA, ET AL: Pseudomembranous colitis: A surgical disease? Surgery

116:491-496, 1994.

MAHMOODIAN S: Appendicitis complicating pregnancy. South Med J

85:19-24, 1992.

MCQUAID KR, ISENBERG JI: Medical therapy of peptic ulcer disease. Surg

Clin North Am 72:285-316, 1992.

MERRELL SW, SCHNEIDER PD: Hemobilia: Evolution of current diagnosis and

treatment. West J Med 155:621-625, 1991.

MILLER FB, SHUMATE CR, RICHARDSON JD: Myocardial contusion. When can

the diagnosis be eliminated? Arch Surg 124:805-808, 1989.

326 Bibliography

MOOSA AR, MAYER AD, STABILE B: Iatrogenic injury to the bile duct: Who,

how, where? Arch Surg 125:1028-1031, 1990.

NORTON JA: Controversies and advances in primary hyperparathyroidism.

Ann Surg 215:297-299, 1992.

O'HARA PJ, HAKAIM AG, HERTZER NR, ET AL: Surgical management of aortic

aneurysm and coexistent horseshoe kidney: Review of a 31-year experience.

J Vasc Surg 17:940-947, 1993.

PASTERNACK PF, ET AL: The value of the radionuclide angiogram in the prediction

of perioperative myocardial infarction in patients undergoing

lower extremity revascularization procedures. Circulation 72:13-17,

1985.

PODOLOSKY DK: Inflammatory bowel disease. N Engl J Med 325:928-937,

1991.

POTTS JT, ET AL: Diagnosis and management of primary hyperparathyroidism:

Consensus development conference statement. Ann Intern

Med 114:593-597, 1991.

PRUITT BA JR, ET AL: Evaluation and management of patients with inhalation

injury. J Trauma 30 (suppl):S63-S68, 1990.

Recommendations for prevention of HIV transmission in health care settings.

N Y State J Med 88:25-31, 1988.

REILLY HF, AL-KAWAS FH: Dieulafoy's lesion. Diagnosis and management.

Dig Dis Sci 36:1702-1707, 1991.

RHAME FS: Preventing HIV transmission. Strategies to protect clinicians and

patients. Postgrad Med 91:144, 147-152, 1992.

ROBINSON T, ET AL: Body smuggling of illicit drugs: Two cases requiring surgical

intervention. Surgery 113:709-711, 1993.

SABISTON DC JR, ET AL (EDS): Surgery. The Biological Basis of Modern Surgical

Practice, 15th ed. Philadelphia, Saunders, 1997.

SAWYERS JL: Management of postgastrectomy syndromes. Am J Surg

159:8-14, 1990.

SCHIRMER BD, DIX J, EDGE SB, ET AL: Laparoscopic cholecystectomy in the

obese patient. Ann Surg 216:146-152, 1992.

SCHROEDER T, ET AL: Ischemic colitis complicating reconstruction of the

abdominal aorta. Surg Gynecol Obstet 160:299-303, 1985.

SCHWARTZ SI, ET AL (EDS): Principles of Surgery, 7th ed. New York, McGraw-

Hill, 1993.

SCHWESINGER, WH: Is Helicobacter pylori a myth or the missing link? Am J

Surg 172:411-416, 1996.

Bibliography 327

SHOEMAKER: 2d ed.

THOREN T, WATTWIL M: Effects on gastric emptying of thoracic epidural analgesia

with morphine or bupivacaine. Anesth Analg 67:687-694, 1988.

TONDINI C: Ann Oncol 4(10):831-837, 1993.

TRUNKEY: Can J Surg 27:479-486, 1984.

WALTERS HL, ET AL: Peritoneal lavage and the surgical resident. Surg

Gynecol Obstet 165:496-502, 1988.

WEISSMAN C: The metabolic response to stress: An overview and update.

Anesthesiology 73:308-327, 1990.

WILLERSON JT, ET AL: Role of new antiplatelet agents as adjunctive therapies

in thrombolysis. Am J Cardiol 67:12A-18A, 1991.

WILSON SE, WILLIAMS RA, ROBINSON G: Operating on HIV-positive patients.

What are the risks to healthcare workers? To patients? Postgrad Med

88:193-194, 199-201, 1990.

ZINNER, MJ, ET AL (EDS): Maingot's Abdominal Operations, 10th ed. Stamford,

CT, Appleton & Lange, 1997.

328 Bibliography

Bạn đang đọc truyện trên: Truyen2U.Pro

#surgery